You are on page 1of 323

POLITICAL LAW

DIGESTS

***CONSTITUTIONAL LAW***

*Mirasol v. Court of Appeals, G.R. No. 128448, February 1, 2001.


(Judicial Review; Notice; Solicitor General)

-Petitioners filed a case for accounting, specific performance and damages against
PNB involving a crop loan financing scheme they had availed from the said bank. PNB
argued that under P.D. No. 579, there was nothing to account since under said law,
all earnings from the export sales of sugar pertained to the National Government and
were subject to the disposition of the President of the Philippines for public purposes.

-Without notice to the Solicitor General, the RTC declared P.D. 579
unconstitutional and ruled in favor of petitioners. Upon appeal by petitioners on the
ground that the trial court failed to award them their full money claims and damages,
the Court of Appeals reversed the RTC and ruled that it was improper for the trial
court to have declared P.D. No. 579 unconstitutional since petitioners had not
complied with Rule 63, Section 3 of the Rules of Court.

-Petitioners then went to the Supreme Court via Rule 45.

-Petitioners’ contention: Rule 63, Section 3, of the Rules of Court (regarding notice to
the OSG) specifically refers only to actions for declaratory relief and not to an
ordinary action for accounting, specific performance, and damages. Hence, notice to
the Solicitor General is not required.

Issue:
Whether or not notice to the Solicitor General is mandatory in cases involving the
validity of any treaty, law, executive order or proclamation.

Ruling:
Yes.

Rule 63, Section 3 of the Rules of Court provides:

“SEC. 3. Notice to Solicitor General. - In any action which involves the validity of a statute, or executive
order or regulation, the Solicitor General shall be notified by the party attacking the statute,
executive order, or regulation, and shall be entitled to be heard upon such question.”

This should be read in relation to Section 1 [c] of P.D. No. 478,13 which states in part:

“SECTION 1. Functions and Organizations - (1) The Office of the Solicitor General shall have the following
specific powers and functions:
xxxx
"[c] Appear in any court in any action involving the validity of any treaty, law, executive order or
proclamation, rule or regulation when in his judgment his intervention is necessary or when
requested by the court.” x x x x

It is basic legal construction that where words of command such as “shall,” “must,” or
“ought” are employed, they are generally and ordinarily regarded as mandatory. Thus,
where, as in Rule 64, Section 3 of the Rules of Court, the word “shall” is used, a
mandatory duty is imposed, which the courts ought to enforce.”

The purpose of the mandatory Notice in Rule 64, Section 3 is to enable the Solicitor
General to decide whether or not his intervention in the action assailing the validity of

Page 1 of 323
a law or treaty is necessary. To deny the Solicitor General such notice would be
tantamount to depriving him of his day in court. We must stress that, contrary to
petitioners’ stand, the mandatory notice requirement is not limited to actions
involving declaratory relief and similar remedies. The rule itself provides that such
notice is required in “any action” and not just actions involving declaratory
relief. Where there is no ambiguity in the words used in the true, there is no room for
construction. In all actions assailing the validity of a statute, treaty, presidential
decree, order, or proclamation, notice to the Solicitor General is mandatory.

In this case, the Solicitor General was never notified about Civil Case No. 14725. Nor
did the trial court ever require him to appear in person or by a representative or to file
any pleading or memorandum on the constitutionality of the assailed decree. Hence,
the Court of Appeals did not err in holding that lack of the required notice made it
improper for the trial court to pass upon the constitutional validity of the questioned
presidential decrees.

*Vinuya v. Executive Secretary, G.R. No. 162230, April 28, 2010.


(Judicial Review; Political Questions)

Facts:
-Petitioners are all members of the MALAYA LOLAS, a non-stock, non-profit
organization registered with the Securities and Exchange Commission, established for
the purpose of providing aid to the victims of rape by Japanese military forces in the
Philippines during the Second World War.

-Since 1998, petitioners have approached the Executive Department through the DOJ,
DFA, and OSG, requesting assistance in filing a claim against the Japanese officials
and military officers. However, officials of the Executive Department declined to
assist the petitioners, and took the position that the individual claims of the comfort
women for compensation had already been fully satisfied by Japan’s compliance with
the Peace Treaty between the Philippines and Japan.

-In view of such refusal by the government, petitioners filed a Petition for Certiorari
under Rule 65 of the Rules of Court with an application for the issuance of a writ of
preliminary mandatory injunction against the Office of the Executive Secretary, DFA,
OSG and DOJ (collectively as respondents).

-Petitioners’ contention: The general waiver of claims made by the Philippine


government in the Treaty of Peace with Japan is void. The comfort women system
established by Japan, and the brutal rape and enslavement of petitioners constituted
a crime against humanity, sexual slavery, and torture. In waiving the claims of
Filipina comfort women and failing to espouse their complaints against Japan, the
Philippine government is in breach of its legal obligation not to afford impunity for
crimes against humanity. Also, the Philippine government’s acceptance of the
“apologies” made by Japan as well as funds from the Asian Women’s Fund (AWF) was
contrary to international law.

Issue:
-Whether or not respondents committed grave abuse of discretion in refusing to
espouse petitioners’ claims for the crimes against humanity and war crimes committed
against them by Japanese officials and military officers during the Second World War.

Ruling:
No.

In Tañada v. Cuenco, we held that political questions refer “to those questions which,
under the Constitution, are to be decided by the people in their sovereign capacity, or
in regard to which full discretionary authority has been delegated to the legislative or
executive branch of the government. It is concerned with issues dependent upon
the wisdom, not legality of a particular measure.”

Page 2 of 323
Certain types of cases often have been found to present political questions. One such
category involves questions of foreign relations. It is well-established that “the
conduct of the foreign relations of our government is committed by the
Constitution to the executive and legislative—the “political”—departments of
the government, and the propriety of what may be done in the exercise of this
political power is not subject to judicial inquiry or decision." x x x x

To be sure, not all cases implicating foreign relations present political questions, and
courts certainly possess the authority to construe or invalidate treaties and executive
agreements. However, the question whether the Philippine government should
espouse claims of its nationals against a foreign government is a foreign
relations matter, the authority for which is demonstrably committed by our
Constitution not to the courts but to the political branches. In this case, the
Executive Department has already decided that it is to the best interest of the country
to waive all claims of its nationals for reparations against Japan in the Treaty of Peace
of 1951. The wisdom of such decision is not for the courts to question. Neither
could petitioners herein assail the said determination by the Executive Department via
the instant petition for certiorari.

In the seminal case of US v. Curtiss-Wright Export Corp., the US Supreme Court held
that “the President is the sole organ of the nation in its external relations, and
its sole representative with foreign relations.” x x x x

The Executive Department has determined that taking up petitioners’ cause would be
inimical to our country’s foreign policy interests, and could disrupt our relations with
Japan, thereby creating serious implications for stability in this region. For us to
overturn the Executive Department’s determination would mean an assessment of the
foreign policy judgments by a coordinate political branch to which authority to make
that judgment has been constitutionally committed. x x x x

From a municipal law perspective, certiorari will not lie. As a general principle –
and particularly here, where such an extraordinary length of time has lapsed between
the treaty’s conclusion and our consideration – the Executive must be given ample
discretion to assess the foreign policy considerations of espousing a claim
against Japan, from the standpoint of both the interests of the petitioners and those
of the Republic, and decide on that basis if apologies are sufficient, and whether
further steps are appropriate or necessary.

*Montesclaros v. Comelec, G.R. No. 152295, July 9, 2002.


(Judicial Review; Ripeness for Judicial Determination)

Facts:
-On March 11, 2002, petitioners filed with the SC a petition for certiorari, prohibition
and mandamus with prayer for a temporary restraining order or preliminary
injunction, seeking, among others, to prevent a proposed bill reducing the age
requirement for membership in the SK from “15 but not more than 21 years old” to
“15 but less than 18 years of age.”

-On the same day, the Senate approved the Bicameral Committee's consolidated bill
and on March 13, 2002, the House of Representatives approved the same. The
President signed the approved bill into law on March 19, 2002.

Issue:
Whether or not such petition will prosper.

Ruling:
No.

Petitioners' prayer to prevent Congress from enacting into law a proposed bill lowering
the membership age in the SK does not present an actual justiciable controversy. A
proposed bill is not subject to judicial review because it is not a law. A proposed

Page 3 of 323
bill creates no right and imposes no duty legally enforceable by the Court. A
proposed bill, having no legal effect, violates no constitutional right or duty. The
Court has no power to declare a proposed bill constitutional or unconstitutional
because that would be in the nature of rendering an advisory opinion on a
proposed act of Congress. The power of judicial review cannot be exercised in vacuo.

The second paragraph of Section 1, Article VIII of the Constitution states –

“Judicial power includes the duty of the courts of justice to settle actual controversies involving rights
which are legally demandable and enforceable, and to determine whether or not there has been a grave
abuse of discretion amounting to lack or excess of jurisdiction on the part of any branch or
instrumentality of the Government.”

Thus, there can be no justiciable controversy involving the constitutionality of a


proposed bill. The Court can exercise its power of judicial review only after a law
is enacted, not before.

Under the separation of powers, the Court cannot restrain Congress from passing
any law, or from setting into motion the legislative mill according to its internal
rules. Thus, the following acts of Congress in the exercise of its legislative powers are
not subject to judicial restraint: the filing of bills by members of Congress, the
approval of bills by each chamber of Congress, the reconciliation by the Bicameral
Committee of approved bills, and the eventual approval into law of the reconciled bills
by each chamber of Congress. Absent a clear violation of specific constitutional
limitations or of constitutional rights of private parties, the Court cannot exercise its
power of judicial review over the internal processes or procedures of Congress.

The Court has also no power to dictate to Congress the object or subject of bills that
Congress should enact into law. The judicial power to review the constitutionality
of laws does not include the power to prescribe to Congress what laws to enact.
The Court has no power to compel Congress by mandamus to enact a law allowing
petitioners, regardless of their age, to vote and be voted for in the July 15, 2002 SK
elections. To do so would destroy the delicate system of checks and balances finely
crafted by the Constitution for the three co-equal, coordinate and independent
branches of government.

Under RA No. 9164, Congress merely restored the age requirement in PD No. 684, the
original charter of the SK, which fixed the maximum age for membership in the SK to
youths less than 18 years old. Petitioners do not have a vested right to the
permanence of the age requirement under Section 424 of the Local Government
Code of 1991. Every law passed by Congress is always subject to amendment or
repeal by Congress. The Court cannot restrain Congress from amending or
repealing laws, for the power to make laws includes the power to change the
laws.

The Court cannot also direct the Comelec to allow over-aged voters to vote or be voted
for in an election that is limited under RA No. 9164 to youths at least 15 but less than
18 years old. A law is needed to allow all those who have turned more than 21 years
old on or after May 6, 2002 to participate in the July 15, 2002 SK elections. Youths
from 18 to 21 years old as of May 6, 2002 are also no longer SK members, and cannot
participate in the July 15, 2002 SK elections. Congress will have to decide whether to
enact an amendatory law. Petitioners’ remedy is legislation, not judicial
intervention.

*Estarija v. G. R. No. 15931, June 26, 2006.


(Judicial Review; Earliest Possible Time; Ombudsman)

-After due proceedings, the Ombudsman found Estarija (petitioner) guilty of


dishonesty and grave misconduct and dismissed him from service.

-Estarija seasonably filed a motion for reconsideration. He claimed that his


dismissal was unconstitutional since the Ombudsman did not have direct and

Page 4 of 323
immediate power to remove government officials, whether elective or appointive, who
are not removable by impeachment. He maintained that under the 1987 Constitution,
the Ombudsman’s administrative authority is merely recommendatory.

-Estarija likewise argued that Republic Act No. 6770, otherwise known as "The
Ombudsman Act of 1989" is unconstitutional because it gives the Office of the
Ombudsman additional powers that are not provided for in the Constitution. The
Ombudsman denied petitioner’s MR.

-Estarija then went to the Court of Appeals via Petition for Review. The CA affirmed
the judgment of the Ombudsman. It held that the constitutionality issue was
belatedly raised by petitioner in the motion for reconsideration of the decision of the
Ombudsman.

-Petitioner then went to the SC via Rule 45.

Issues:
(1) Whether or not the constitutionality issue was brought by the petitioner at the
“earliest possible time”;
(2) Whether or not the power of the Ombudsman to directly remove, suspend, demote,
fine or censure erring officials violates the Constitution.

Ruling:
(1) Yes.

Unequivocally, the law requires that the question of constitutionality of a statute must
be raised at the earliest opportunity. In Matibag v. Benipayo, we held that the earliest
opportunity to raise a constitutional issue is to raise it in the pleadings before a
competent court that can resolve the same, such that, if it was not raised in the
pleadings before a competent court, it cannot be considered at the trial, and, if not
considered in the trial, it cannot be considered on appeal.

In this case, petitioner raised the issue of constitutionality of Rep. Act No. 6770 in his
motion for the reconsideration of the Ombudsman’s decision. Verily, the
Ombudsman has no jurisdiction to entertain questions on the constitutionality
of a law. Thus, when petitioner raised the issue of constitutionality of Rep. Act
No. 6770 before the Court of Appeals, which is the competent court, the
constitutional question was raised at the earliest opportune time. Furthermore,
this Court may determine, in the exercise of sound discretion, the time when a
constitutional issue may be passed upon.

(2) No.

In passing Rep. Act No. 6770, Congress deliberately endowed the Ombudsman with
the power to prosecute offenses committed by public officers and employees to make
him a more active and effective agent of the people in ensuring accountability in public
office. Moreover, the legislature has vested the Ombudsman with broad powers to
enable him to implement his own actions. x x x x

In Ledesma v. Court of Appeals, we held that Rep. Act No. 6770 is consistent with the
intent of the framers of the 1987 Constitution. They gave Congress the discretion to
give the Ombudsman powers that are not merely persuasive in character. Thus,
in addition to the power of the Ombudsman to prosecute and conduct investigations,
the lawmakers intended to provide the Ombudsman with the power to punish for
contempt and preventively suspend any officer under his authority pending an
investigation when the case so warrants. He was likewise given disciplinary
authority over all elective and appointive officials of the government and its
subdivisions, instrumentalities and agencies except members of Congress and the
Judiciary.

Page 5 of 323
It must be stressed that the Constitution gave Congress the discretion to give the
Ombudsman other powers and functions. Thus, the Constitution does not restrict
the powers of the Ombudsman in Section 13, Article XI of the 1987 Constitution, but
allows the Legislature to enact a law that would spell out the powers of the
Ombudsman. Through the enactment of Rep. Act No. 6770, specifically Section
15, par. 3, the lawmakers gave the Ombudsman such powers to sanction erring
officials and employees, except members of Congress, and the Judiciary.

To conclude, we hold that Sections 15, 21, 22 and 25 of Republic Act No. 6770 are
constitutionally sound. The powers of the Ombudsman are not merely
recommendatory. His office was given teeth to render this constitutional body not
merely functional but also effective. Thus, we hold that under Republic Act No. 6770
and the 1987 Constitution, the Ombudsman has the constitutional power to
directly remove from government service an erring public official other than a
member of Congress and the Judiciary.

*Arigo v. Swift, G.R. No. 206510, September 16, 2014.


(State Immunity; Writ of Kalikasan; VFA)

Facts:
-In 1988, Tubbataha was declared a National Marine Park by virtue of Proclamation
No. 306 issued by President Corazon C. Aquino. In 1993, Tubbataha was inscribed by
the United Nations Educational Scientific and Cultural Organization (UNESCO) as a
World Heritage Site.

-On April 6, 2010, Congress passed Republic Act (R.A.) No. 10067,3 otherwise known
as the "Tubbataha Reefs Natural Park (TRNP) Act of 2009" "to ensure the protection
and conservation of the globally significant economic, biological, sociocultural,
educational and scientific values of the Tubbataha Reefs into perpetuity.”

-The USS Guardian is an Avenger-class mine countermeasures ship of the US Navy.

-In December 2012, the US Embassy in the Philippines requested diplomatic


clearance for the said vessel “to enter and exit the territorial waters of the
Philippines and to arrive at the port of Subic Bay for the purpose of routine ship
replenishment, maintenance, and crew liberty.”

-On January 15, 2013, the USS Guardian departed Subic Bay for its next port of call
in Makassar, Indonesia. On January 17, 2013 at 2:20 a.m. while transiting the Sulu
Sea, the ship ran aground on the northwest side of South Shoal of the Tubbataha
Reefs, about 80 miles east-southeast of Palawan. No one was injured in the incident,
and there have been no reports of leaking fuel or oil.

-On January 20, 2013, U.S. 7th Fleet Commander, Vice Admiral Scott Swift,
expressed regret for the incident in a press statement. Likewise, US Ambassador to the
Philippines Harry K. Thomas, Jr., in a meeting at the Department of Foreign Affairs
(DFA) on February 4, “reiterated his regrets over the grounding incident and assured
Foreign Affairs Secretazy Albert F. del Rosario that the United States will provide
appropriate compensation for damage to the reef caused by the ship.”

-On April 17, 2013, petitioners on their behalf and in representation of their respective
sector/organization and others, including minors or generations yet unborn, filed a
petition for the issuance of a Writ of Kalikasan with prayer for the issuance of a
Temporary Environmental Protection Order (TEPO) under Rule 7 of A.M. No. 09-6-8-
SC, otherwise known as the Rules of Procedure for Environmental Cases (Rules).

-Petitioners filed the petition against SCOTT H. SWIFT in his capacity as Commander
of the US. 7th Fleet and MARK A. RICE in his capacity as Commanding Officer of the
USS Guardian. They were the US respondents. The Philippine respondents,
meanwhile, included President Aquino, the Secretary of the DFA, the Secretary of
DND, Secretary of DENR, Commandant of Philippines Coast Guard, et al.

Page 6 of 323
-Specifically, petitioners cite the following violations committed by US respondents
under R.A. No. 10067: unauthorized entry (Section 19); non-payment of conservation
fees (Section 21); obstruction of law enforcement officer (Section 30); damages to the
reef (Section 20); and destroying and disturbing resources (Section 26[g]).

-Furthermore, petitioners assail certain provisions of the Visiting Forces Agreement


(VFA) which they want the SC to nullify for being unconstitutional.

-Only the Philippine respondents filed their comment to the petition. The US
respondents did not submit any pleading or manifestation in this case.

Issues:
(1) Whether or not the Supreme Court has jurisdiction over the US respondents who
did not submit any pleading or manifestation in this case;
(2) Whether or not there is a waiver of immunity from suit found in the VFA.

Ruling:
(1) No.

The immunity of the State from suit, known also as the doctrine of sovereign immunity
or non-suability of the State, is expressly provided in Article XVI of the 1987
Constitution which states:

Section 3. The State may not be sued without its consent.

In United States of America v. Judge Guinto, we discussed the principle of state


immunity from suit, as follows:

The rule that a state may not be sued without its consent, now expressed in Article
XVI, Section 3, of the 1987 Constitution, is one of the generally accepted principles of
international law that we have adopted as part of the law of our land under Article II,
Section 2. x x x.

Even without such affirmation, we would still be bound by the generally accepted
principles of international law under the doctrine of incorporation. Under this
doctrine, as accepted by the majority of states, such principles are deemed
incorporated in the law of every civilized state as a condition and consequence of its
membership in the society of nations. Upon its admission to such society, the state is
automatically obligated to comply with these principles in its relations with other
states. x x x x

In the case of Minucher v. Court of Appeals, we further expounded on the immunity of


foreign states from the jurisdiction of local courts, as follows:

The precept that a State cannot be sued in the courts of a foreign state is a long-
standing rule of customary international law then closely identified with the personal
immunity of a foreign sovereign from suit and, with the emergence of democratic
states, made to attach not just to the person of the head of state, or his representative,
but also distinctly to the state itself in its sovereign capacity. If the acts giving rise to
a suit are those of a foreign government done by its foreign agent, although not
necessarily a diplomatic personage, but acting in his official capacity, the
complaint could be barred by the immunity of the foreign sovereign from suit
without its consent. Suing a representative of a state is believed to be, in effect,
suing the state itself. The proscription is not accorded for the benefit of an individual
but for the State, in whose service he is, under the maxim -par in parem, non habet
imperium -that all states are sovereign equals and cannot assert jurisdiction over
one another. The implication, in broad terms, is that if the judgment against an
official would require the state itself to perform an affirmative act to satisfy the
award, such as the appropriation of the amount needed to pay the damages

Page 7 of 323
decreed against him, the suit must be regarded as being against the state itself,
although it has not been formally impleaded.

In the same case we also mentioned that in the case of diplomatic immunity, the
privilege is not an immunity from the observance of the law of the territorial sovereign
or from ensuing legal liability; it is, rather, an immunity from the exercise of
territorial jurisdiction.

In United States of America v. Judge Guinto, one of the consolidated cases therein
involved a Filipino employed at Clark Air Base who was arrested following a buy-bust
operation conducted by two officers of the US Air Force, and was eventually dismissed
from his employment when he was charged in court for violation of R.A. No. 6425. In a
complaint for damages filed by the said employee against the military officers, the
latter moved to dismiss the case on the ground that the suit was against the US
Government which had not given its consent. The RTC denied the motion but on a
petition for certiorari and prohibition filed before this Court, we reversed the RTC
and dismissed the complaint. We held that petitioners US military officers were
acting in the exercise of their official functions when they conducted the buy-
bust operation against the complainant and thereafter testified against him at his
trial. It follows that for discharging their duties as agents of the United States,
they cannot be directly impleaded for acts imputable to their principal, which
has not given its consent to be sued.

This traditional rule of State immunity which exempts a State from being sued in the
courts of another State without the former's consent or waiver has evolved into a
restrictive doctrine which distinguishes sovereign and governmental acts (“Jure
imperil”) from private, commercial and proprietary acts (“Jure gestionis”). Under the
restrictive rule of State immunity, State immunity extends only to acts Jure imperii.
The restrictive application of State immunity is proper only when the proceedings arise
out of commercial transactions of the foreign sovereign, its commercial activities or
economic affairs.

However, the doctrine of immunity from suit will not apply and may not be invoked
where the public official is being sued in his private and personal capacity as an
ordinary citizen. The cloak of protection afforded the officers and agents of the
government is removed the moment they are sued in their individual capacity. This
situation usually arises where the public official acts without authority or in excess of
the powers vested in him. It is a well-settled principle of law that a public official may
be liable in his personal private capacity for whatever damage he may have caused by
his act done with malice and in bad faith, or beyond the scope of his authority or
jurisdiction.

In this case, the US respondents were sued in their official capacity as


commanding officers of the US Navy who had control and supervision over the
USS Guardian and its crew. The alleged act or omission resulting in the
unfortunate grounding of the USS Guardian on the TRNP was committed while
they were performing official military duties. Considering that the satisfaction of
a judgment against said officials will require remedial actions and appropriation
of funds by the US government, the suit is deemed to be one against the US
itself. The principle of State immunity therefore bars the exercise of jurisdiction
by this Court over the persons of respondents Swift, Rice and Robling.

(2) None.

The VFA is an agreement which defines the treatment of United States troops and
personnel visiting the Philippines to promote "common security interests" between the
US and the Philippines in the region. It provides for the guidelines to govern such
visits of military personnel, and further defines the rights of the United States and the
Philippine government in the matter of criminal jurisdiction, movement of vessel and
aircraft, importation and exportation of equipment, materials and supplies. The
invocation of US federal tort laws and even common law is thus improper considering

Page 8 of 323
that it is the VFA which governs disputes involving US military ships and crew
navigating Philippine waters in pursuance of the objectives of the agreement.

As it is, the waiver of State immunity under the VFA pertains only to criminal
jurisdiction and not to special civil actions such as the present petition for
issuance of a writ of Kalikasan. In fact, it can be inferred from Section 17, Rule 7 of
the Rules that a criminal case against a person charged with a violation of an
environmental law is to be filed separately:

SEC. 17. Institution of separate actions.-The filing of a petition for the issuance of the writ of kalikasan
shall not preclude the filing of separate civil, criminal or administrative actions.

In any case, it is our considered view that a ruling on the application or non-
application of criminal jurisdiction provisions of the VFA to US personnel who
may be found responsible for the grounding of the USS Guardian, would be
premature and beyond the province of a petition for a writ of Kalikasan. We also
find it unnecessary at this point to determine whether such waiver of State immunity
is indeed absolute. In the same vein, we cannot grant damages which have resulted
from the violation of environmental laws. The Rules allows the recovery of
damages, including the collection of administrative fines under R.A. No. 10067,
in a separate civil suit or that deemed instituted with the criminal action
charging the same violation of an environmental law.

[NOTE:
The petition for the issuance of the privilege of the Writ of Kalikasan was denied.]

*University of the Philippines v. Hon. Dizon, G.R. No. 171182, August 23, 2012.
(State Immunity; COA; Execution)

Facts:
-In 1990, the UP, through its then President Jose V. Abueva, entered into a General
Construction Agreement with respondent Stern Builders Corporation (Stern
Builders), represented by its President and General Manager Servillano dela Cruz, for
the construction of the extension building and the renovation of the College of Arts
and Sciences Building in the campus of the University of the Philippines in Los Baños
(UPLB).

-In the course of the implementation of the contract, Stern Builders submitted three
progress billings corresponding to the work accomplished, but the UP paid only two of
the billings. The third billing worth ₱ 273,729.47 was not paid due to its
disallowance by the Commission on Audit (COA).

-Despite the lifting of the disallowance, the UP failed to pay the billing, prompting
Stern Builders and dela Cruz to sue the UP and its co-respondent officials to collect
the unpaid billing and to recover various damages.

-After trial, the RTC rendered its decision in favor of the plaintiffs and against UP. UP
filed a notice of appeal, which was denied due course for having been filed out of time.
The RTC subsequently granted respondents’ motion for execution.

-The RTC issued the writ of execution and the sheriff of the RTC served the writ of
execution and notice of demand upon the UP, through its counsel. The UP filed an
urgent motion to reconsider the order, to quash the writ of execution and to restrain
the proceedings. However, the RTC denied the urgent motion.

-The UP appealed to the CA up to the SC, but the appeals were denied. The judgment,
hence, became final and executory.

-In view thereof, the sheriff served notices of garnishment on the UP’s depository
banks, namely: Land Bank of the Philippines (Buendia Branch) and the Development
Bank of the Philippines (DBP), Commonwealth Branch. The UP assailed the

Page 9 of 323
garnishment through an urgent motion to quash the notices of garnishment; and a
motion to quash the writ of execution, which were later denied by the Court.

-Eventually, the RTC authorized the release of the garnished funds of the UP.
Thereafter, Stern Builders and dela Cruz moved to cite DBP in direct contempt of
court for its non-compliance with the order of release.

Issue:
Whether or not the funds of the UP were the proper subject of garnishment in order to
satisfy the judgment award.

Ruling:
No.

UP’s funds, being government funds, are not subject to garnishment.

The UP is a government instrumentality, performing the State’s constitutional


mandate of promoting quality and accessible education. As a government
instrumentality, the UP administers special funds sourced from the fees and income
enumerated under Act No. 1870 and Section 1 of Executive Order No. 714, and from
the yearly appropriations, to achieve the purposes laid down by Section 2 of Act 1870,
as expanded in Republic Act No. 9500.

All the funds going into the possession of the UP, including any interest accruing
from the deposit of such funds in any banking institution, constitute a “special
trust fund,” the disbursement of which should always be aligned with the UP’s
mission and purpose, and should always be subject to auditing by the COA.

Presidential Decree No. 1445 defines a “trust fund” as a fund that officially comes in
the possession of an agency of the government or of a public officer as trustee, agent
or administrator, or that is received for the fulfillment of some obligation. A trust fund
may be utilized only for the "specific purpose for which the trust was created or the
funds received."

The funds of the UP are government funds that are public in character. They
include the income accruing from the use of real property ceded to the UP that may be
spent only for the attainment of its institutional objectives. Hence, the funds subject
of this action could not be validly made the subject of the RTC’s writ of
execution or garnishment. The adverse judgment rendered against the UP in a
suit to which it had impliedly consented was not immediately enforceable by
execution against the UP, because suability of the State did not necessarily
mean its liability.

A marked distinction exists between suability of the State and its liability. As the
Court succinctly stated in Municipality of San Fernando, La Union v. Firme:

“A distinction should first be made between suability and liability. "Suability depends
on the consent of the state to be sued, liability on the applicable law and the
established facts. The circumstance that a state is suable does not necessarily
mean that it is liable; on the other hand, it can never be held liable if it does not first
consent to be sued. Liability is not conceded by the mere fact that the state has
allowed itself to be sued. When the state does waive its sovereign immunity, it is only
giving the plaintiff the chance to prove, if it can, that the defendant is liable.” x x x x

Indeed, an appropriation by Congress was required before the judgment that


rendered the UP liable for moral and actual damages (including attorney’s fees)
would be satisfied considering that such monetary liabilities were not covered by
the "appropriations earmarked for the said project. The Constitution strictly
mandated that "no money shall be paid out of the Treasury except in pursuance
of an appropriation made by law." x x x x

Page 10 of 323
The execution of the monetary judgment against the UP was within the primary
jurisdiction of the COA. This was expressly provided in Section 26 of Presidential
Decree No. 1445, to wit:

“Section 26. General jurisdiction. - The authority and powers of the Commission shall extend to and
comprehend all matters relating to auditing procedures, systems and controls, the keeping of the general
accounts of the Government, the preservation of vouchers pertaining thereto for a period of ten years, the
examination and inspection of the books, records, and papers relating to those accounts; and the audit
and settlement of the accounts of all persons respecting funds or property received or held by them in an
accountable capacity, as well as the examination, audit, and settlement of all debts and claims of any sort
due from or owing to the Government or any of its subdivisions, agencies and instrumentalities. The said
jurisdiction extends to all government-owned or controlled corporations, including their subsidiaries, and
other self-governing boards, commissions, or agencies of the Government, and as herein prescribed,
including non governmental entities subsidized by the government, those funded by donations through
the government, those required to pay levies or government share, and those for which the government
has put up a counterpart fund or those partly funded by the government.”

It was of no moment that a final and executory decision already validated the
claim against the UP. The settlement of the monetary claim was still subject to
the primary jurisdiction of the COA despite the final decision of the RTC having
already validated the claim. As such, Stern Builders and dela Cruz as the claimants
had no alternative except to first seek the approval of the COA of their monetary
claim.

On its part, the RTC should have exercised utmost caution, prudence and
judiciousness in dealing with the motions for execution against the UP and the
garnishment of the UP’s funds. The RTC had no authority to direct the immediate
withdrawal of any portion of the garnished funds from the depository banks of
the UP. By eschewing utmost caution, prudence and judiciousness in dealing with the
execution and garnishment, and by authorizing the withdrawal of the garnished funds
of the UP, the RTC acted beyond its jurisdiction, and all its orders and issuances
thereon were void and of no legal effect. x x x x

All money claims against the Government must first be filed with the
Commission on Audit which must act upon it within sixty days. Rejection of the
claim will authorize the claimant to elevate the matter to the Supreme Court on
certiorari and in effect, sue the State thereby (P.D. 1445, Sections 49-50).

*Ortigas & Co. v Court of Appeals, G.R. No. 126102, December 4, 2000.
(Police Power; Contracts; Retroactive Application)

Facts:
-In 1976, petitioner Ortigas & Company sold to Emilia Hermoso, a parcel of land
known as Lot 1, Block 21, Psd-66759, with an area of 1,508 square meters, located in
Greenhills Subdivision IV, San Juan, Metro Manila.

-The contract of sale provided certain conditions, to wit: “(1) the lot shall be used
exclusively for residential purposes only, and not more than one single-family
residential building will be constructed thereon; (2) The buyer shall not erect any sign
or billboard on the roof for advertising purposes; (3) Restrictions shall run with the
land and shall be construed as real covenants until December 31, 2025 when they
shall cease and terminate.”

-These and the other conditions were duly annotated on the certificate of title issued
to Emilia.

-In 1981, the MMDA enacted MMC Ordinance No. 81-01, also known as the
Comprehensive Zoning Area for the National Capital Region. The ordinance
reclassified as a commercial area a portion of Ortigas Avenue from Madison to
Roosevelt Streets of Greenhills Subdivision where the lot is located.

-In 1984, private respondent Ismael Mathay III leased the lot from Emilia Hermoso
and J.P. Hermoso Realty Corp. The lease contract did not specify the purposes of the

Page 11 of 323
lease. Thereupon, private respondent constructed a single story commercial building
for Greenhills Autohaus, Inc., a car sales company.

-In 1995, petitioner filed a complaint against Emilia Hermoso with the Regional Trial
Court of Pasig, Branch 261. The complaint sought the demolition of the said
commercial structure for having violated the terms and conditions of the Deed of
Sale.

-Petitioner’s contention: Inasmuch as the restrictions on the use of the lot were duly
annotated on the title it issued to Emilia Hermoso, said restrictions must prevail over
the ordinance, specially since these restrictions were agreed upon before the passage
of MMC Ordinance No. 81-01. Since the MMDA had failed to show that MMC
Ordinance No. 81-01 had retroactive effect, said ordinance should be given prospective
application only.

Issue:
Whether or not MMC Ordinance No. 81-01 nullified the building restrictions annotated
on the certificate of title imposing exclusive residential use on the subject property.

Ruling:
Yes.

In general, we agree that laws are to be construed as having only prospective


operation. Lex prospicit, non respicit. Equally settled, only laws existing at the time of
the execution of a contract are applicable thereto and not later statutes, unless the
latter are specifically intended to have retroactive effect. A later law which enlarges,
abridges, or in any manner changes the intent of the parties to the contract
necessarily impairs the contract itself and cannot be given retroactive effect without
violating the constitutional prohibition against impairment of contracts.

But, the foregoing principles do admit of certain exceptions. One involves police
power. A law enacted in the exercise of police power to regulate or govern
certain activities or transactions could be given retroactive effect and may
reasonably impair vested rights or contracts. Police power legislation is applicable
not only to future contracts, but equally to those already in existence. Non-
impairment of contracts or vested rights clauses will have to yield to the
superior and legitimate exercise by the State of police power to promote the
health, morals, peace, education, good order, safety, and general welfare of the
people. Moreover, statutes in exercise of valid police power must be read into every
contract. Noteworthy, in Sangalang vs. Intermediate Appellate Court, we already upheld
MMC Ordinance No. 81-01 as a legitimate police power measure.

Following our ruling in Ortigas & Co., Ltd. vs. Feati Bank & Trust Co., 94 SCRA 533
(1979), the contractual stipulations annotated on the Torrens Title, on which
Ortigas relies, must yield to the ordinance. When that stretch of Ortigas Avenue
from Roosevelt Street to Madison Street was reclassified as a commercial zone by the
Metropolitan Manila Commission in March 1981, the restrictions in the contract of
sale between Ortigas and Hermoso, limiting all construction on the disputed lot to
single-family residential buildings, were deemed extinguished by the retroactive
operation of the zoning ordinance and could no longer be enforced. While our
legal system upholds the sanctity of contract so that a contract is deemed law between
the contracting parties, nonetheless, stipulations in a contract cannot contravene
"law, morals, good customs, public order, or public policy." Otherwise such
stipulations would be deemed null and void. Respondent court correctly found that
the trial court committed in this case a grave abuse of discretion amounting to want of
or excess of jurisdiction in refusing to treat Ordinance No. 81-01 as applicable to Civil
Case No. 64931. In resolving matters in litigation, judges are not only duty-bound to
ascertain the facts and the applicable laws, they are also bound by their oath of office
to apply the applicable law.

*Republic v. Hon. Tatad, G.R. No. 187677, April 17, 2013.

Page 12 of 323
(Eminent Domain; Conflict in Ownership; Remedial Law: Expropriation)

Facts:
-2001, petitioner Republic of the Philippines, represented by the Department of Public
Works and Highways (DPWH), filed a complaint against several defendants, including
spouses William and Rebecca Genato (private respondents), for the expropriation of
several parcels of land affected by the construction of the EDSA-Quezon Avenue
Flyover.

-Private respondents are the registered owners of a piece of land ("subject property")
having an area of 460 square meters.

-During the pendency of the proceedings, petitioner received a letter from Engr.
Patrick B. Gatan, Project Manager IV of the DPWH-NCR, reporting that the subject
property was "government land and that the transfer certificate of title of the said
claimant respondent x x x is of dubious origin and of fabrication as it encroached or
overlapped on a government property." As a result, petitioner filed an Amended
Complaint.

-Thereafter, petitioner filed a Manifestation and Motion to have the subject property
"declared or considered of uncertain ownership or subject to conflicting claims."

-Eventually, the RTC admitted petitioner’s Amended Complaint and deferred the
release to respondents the amount of eighteen million four hundred thousand pesos
(₱18,400,000) deposited in the bank, equivalent to the current zonal valuation of the
land, and declared the property as the subject of conflicting claims.

-While petitioner was presenting evidence to show that the subject property actually
belonged to the Government, private respondents interposed objections saying that
petitioner was barred from presenting the evidence, as it constituted a collateral
attack on the validity of their Torrens title.

-The RTC then issued an order barring petitioner from presenting evidence as it would
constitute a collateral attack on the validity of the title to the subject.

Issue:
Whether or not petitioner may be barred from presenting evidence to assail the validity
of respondents’ title.

Ruling:
No.

That the court is empowered to entertain the conflicting claims of ownership of


the condemned or sought to be condemned property and adjudge the rightful
owner thereof, in the same expropriation case, is evident from Section 9 of the
Revised Rule 69, which provides:

SEC. 9. Uncertain ownership. Conflicting claims. — If the ownership of the property taken is uncertain, or
there are conflicting claims to any part thereof, the court may order any sum or sums awarded as
compensation for the property to be paid to the clerk of court for the benefit of the persons
adjudged in the same proceeding to be entitled thereto. But the judgment shall require the payment
of the sum or sums awarded to either the defendant or the clerk before the plaintiff can enter upon the
property, or retain it for the public use or purpose if entry has already been made.

In fact, the existence of doubt or obscurity in the title of the person or persons
claiming ownership of the properties to be expropriated would not preclude the
commencement of the action nor prevent the court from assuming jurisdiction
thereof. The Rules merely require, in such eventuality, that the entity exercising the
right of eminent domain should state in the complaint that the true ownership of the
property cannot be ascertained or specified with accuracy. x x x x

Page 13 of 323
Verily, our interpretation of Sec. 9, Rule 67 does not run counter to Section 48 of P.D.
1529. Under Sec. 48, collateral attacks on a Torrens title are prohibited. x x x x

Here, the attempt of petitioner to present evidence cannot be characterized as


an “attack.” It must be emphasized that the objective of the case is to appropriate
private property, and the contest on private respondents' title arose only as an
incident to the issue of whom should be rightly compensated.

Contrary to petitioner's allegations, the Complaint and Amended Complaint cannot


also be considered as a direct attack. The amendment merely limited the coverage of
the expropriation proceedings to the uncontested portion of the subject property. The
RTC's Order declaring the property as subject of conflicting claims is a recognition that
there are varying claimants to the sums to be awarded as just compensation. This
serves as an authority for the court to conduct a limited inquiry on the
property's ownership.

[NOTE:
The case was REMANDED to the RTC to hear the issue of ownership for the purpose
of just compensation.]

*Municipality of Parañaque v. V. M. Realty Corporation, G.R. No. 127820, July


20, 1998.
(Eminent Domain; Remedial Law: Expropriation; Res Judicata)

Facts:
-Pursuant to Sangguniang Bayan Resolution No. 93-95, Series of 1993, the
Municipality of Parañaque filed on September 20, 1993, a Complaint for
expropriation against private respondent V.M. Realty Corporation over two parcels of
land (Lots 2-A-2 and 2-B-1 of Subdivision Plan Psd-17917), with a combined area of
about 10,000 square meters, located at Wakas, San Dionisio, Parañaque, Metro
Manila.

-Private respondent filed its Answer containing affirmative defenses and a


counterclaim, alleging in the main that (a) the complaint failed to state a cause of
action because it was filed pursuant to a resolution and not to an ordinance as
required by RA 7160 (the Local Government Code); and (b) the cause of action, if any,
was barred by a prior judgment or res judicata.

-It turned out that the plaintiff had earlier filed a complaint for expropriation involving
the same parcels of land which was docketed as Civil Case No. 17939. Said case was
ealier dismissed with prejudice. The order of dismissal was not appealed, hence, the
same became final.

-The RTC then dismissed the second complaint, which the CA affirmed upon appeal.

-Contentions of petitioner: (a) A resolution approved by the municipal council for the
purpose of initiating an expropriation case substantially complies with the
requirements of the law; (b) The principle of res judicata is not applicable.

Issues:
(1) Whether or not a resolution duly approved by the municipal council has the same
force and effect of an ordinance insofar as filing a complaint for expropriation is
concerned;
(2) Whether or not the principle of res judicata as a ground for dismissal of case is
applicable under the circumstances.

Ruling:
(1) No.

A local government unit (LGU), like the Municipality of Parañaque, cannot


authorize an expropriation of private property through a mere resolution of its

Page 14 of 323
lawmaking body. The Local Government Code expressly and clearly requires an
ordinance or a local law for the purpose. A resolution that merely expresses the
sentiment or opinion of the Municipal Council will not suffice.

Thus, the following essential requisites must concur before an LGU can exercise the
power of eminent domain:

1. An ordinance is enacted by the local legislative council authorizing the local


chief executive, in behalf of the LGU, to exercise the power of eminent domain
or pursue expropriation proceedings over a particular private property.

2. The power of eminent domain is exercised for public use, purpose or welfare, or for
the benefit of the poor and the landless.

3. There is payment of just compensation, as required under Section 9, Article III of


the Constitution, and other pertinent laws.

4. A valid and definite offer has been previously made to the owner of the property
sought to be expropriated, but said offer was not accepted.

In the case at bar, the local chief executive sought to exercise the power of eminent
domain pursuant to a resolution of the municipal council. Thus, there was no
compliance with the first requisite that the mayor be authorized through an
ordinance.

We are not convinced by petitioner's insistence that the terms "resolution" and
"ordinance" are synonymous. A municipal ordinance is different from a resolution. An
ordinance is a law, but a resolution is merely a declaration of the sentiment or
opinion of a lawmaking body on a specific matter. An ordinance possesses a
general and permanent character, but a resolution is temporary in nature.
Additionally, the two are enacted differently — a third reading is necessary for an
ordinance, but not for a resolution, unless decided otherwise by a majority of all
the Sanggunian members.

Moreover, the power of eminent domain necessarily involves a derogation of a


fundamental or private right of the people. Accordingly, the manifest change in the
legislative language — from "resolution" under BP 337 to "ordinance" under RA 7160
— demands a strict construction.

(2) No.

The principle of res judicata, which finds application in generally all cases and
proceedings, cannot bar the right of the State or its agent to expropriate private
property. The very nature of eminent domain, as an inherent power of the State,
dictates that the right to exercise the power be absolute and unfettered even by
a prior judgment or res judicata. The scope of eminent domain is plenary and, like
police power, can "reach every form of property which the State might need for public
use." All separate interests of individuals in property are held of the government under
this tacit agreement or implied reservation. Thus, the State or its authorized agent
cannot be forever barred from exercising said right by reason alone of previous
non-compliance with any legal requirement.

While the principle of res judicata does not denigrate the right of the State to exercise
eminent domain, it does apply to specific issues decided in a previous case. For
example, a final judgment dismissing an expropriation suit on the ground that there
was no prior offer precludes another suit raising the same issue; it cannot, however,
bar the State or its agent from thereafter complying with this requirement, as
prescribed by law, and subsequently exercising its power of eminent domain over the
same property. By the same token, our ruling that petitioner cannot exercise its
delegated power of eminent domain through a mere resolution will not bar it
from reinstituting similar proceedings, once the said legal requirement and, for

Page 15 of 323
that matter, all others are properly complied with. To rule otherwise will not only
improperly diminish the power of eminent domain, but also clearly defeat social
justice.

*Napocor v. Zabala, G.R. No. 173520, January 30, 2013.


(Eminent Domain; Just Compensation; Remedial Law: Expropriation)

Facts:
-1994, petitioner National Power Corporation (Napocor) filed a complaint for Eminent
Domain against respondents Sps. R. Zabala & L. Baylon.

-After due proceedings, the Commissioners submitted their


Report/Recommendation fixing the just compensation for the use of spouses
Zabala’s property as easement of right of way at ₱150.00 per square meter without
considering the consequential damages.

-Napocor prayed in its Comment to the commissioners’ report, that the report be
recommitted to the commissioners for the modification of the report and the
substantiation of the same with reliable and competent documentary evidence based
on the value of the property at the time of its taking. On their part, spouses Zabala
prayed, in the Comments, for the fixing of the just compensation at ₱250.00 per
square meter.

-Thereafter, the lower court recommitted the report to the Commissioners for further
report on the points raised by the parties.

-After due consideration, the Commissioners submitted their Final Report fixing the
just compensation at ₱500.00 per square meter.

-Subsequently, the RTC ordered Napocor to pay spouses Rodolfo Zabala and Lilia
Baylon the amount of Php150.00 per square meter for the 6,820 square meters
taken from the latter’s property, as the just compensation.

-Napocor appealed.

-Napocor’s contention: The RTC erred in not applying Section 3A of Republic Act
(RA) No. 6395 which limits its liability to easement fee of not more than 10% of the
market value of the property traversed by its transmission lines.

Issue:
Whether or not the RTC erred in not applying Section 3A of Republic Act (RA) No. 6395
which limits Napocor’s liability to easement fee of not more than 10% of the market
value of the property traversed by its transmission lines.

Ruling:
No.

In insisting that the just compensation cannot exceed 10% of the market value of the
affected property, Napocor relies heavily on Section 3A of RA No. 6395, the pertinent
portions of which read:

“Sec. 3A. In acquiring private property or private property rights through expropriation proceedings where
the land or portion thereof will be traversed by the transmission lines, only a right-of-way easement
thereon shall be acquired when the principal purpose for which such land is actually devoted will not be
impaired, and where the land itself or portion thereof will be needed for the projects or works, such land
or portion thereof as necessary shall be acquired.

In determining the just compensation of the property or property sought to be acquired through
expropriation proceedings, the same shall:

(a) With respect to the acquired land or portion thereof, not to exceed the market value declared by the
owner or administrator or anyone having legal interest in the property, or such market value as
determined by the assessor, whichever is lower.

Page 16 of 323
(b) With respect to the acquired right-of-way easement over the land or portion thereof, not to
exceed ten percent (10%) of the market value declared by the owner or administrator or anyone
having legal interest in the property, or such market value as determined by the assessor
whichever is lower. x x x x”

Just compensation has been defined as "the full and fair equivalent of the property
taken from its owner by the expropriator. The measure is not the taker's gain, but the
owner’s loss. The word ‘just’ is used to qualify the meaning of the word ‘compensation’
and to convey thereby the idea that the amount to be tendered for the property to be
taken shall be real, substantial, full and ample." The payment of just compensation for
private property taken for public use is guaranteed no less by our Constitution and is
included in the Bill of Rights. As such, no legislative enactments or executive
issuances can prevent the courts from determining whether the right of the
property owners to just compensation has been violated. It is a judicial function
that cannot be usurped by any other branch or official of the government. Thus,
we have consistently ruled that statutes and executive issuances fixing or
providing for the method of computing just compensation are not binding on
courts and, at best, are treated as mere guidelines in ascertaining the amount
thereof. In National Power Corporation v. Bagui, where the same petitioner also
invoked the provisions of Section 3A of RA No. 6395, we held that:

Moreover, Section 3A-(b) of R.A. No. 6395, as amended, is not binding on the
Court. It has been repeatedly emphasized that the determination of just
compensation in eminent domain cases is a judicial function and that any
valuation for just compensation laid down in the statutes may serve only as a
guiding principle or one of the factors in determining just compensation but it
may not substitute the court’s own judgment as to what amount should be
awarded and how to arrive at such amount.

This ruling was reiterated in Republic v. Lubinao, National Power Corporation v. Tuazon
and National Power Corporation v. Saludares and continues to be the controlling
doctrine. Notably, in all these cases, Napocor likewise argued that it is liable to pay the
property owners for the easement of right-of-way only and not the full market value of
the land traversed by its transmission lines. But we uniformly held in those cases that
since the high-tension electric current passing through the transmission lines will
perpetually deprive the property owners of the normal use of their land, it is only just
and proper to require Napocor to recompense them for the full market value of their
property.

*Guingona v. Carague, G.R. No. 94571, April 22, 1991.


(Priority to Education; National Interest; Appropriations)

Facts:
-In the 1990 National Budget, P86 Billion is appropriated (through automatic
appropriation) for debt service while only P27 Billion is appropriated for the
Department of Education.

-Sec. 5, Art. XIV of the Constitution provides:

“(5) The State shall assign the highest budgetary priority to education and ensure that teaching will
attract and retain its rightful share of the best available talents through adequate remuneration and other
means of job satisfaction and fulfillment.”

-Senators Guingona and Pimentel went to the SC to declare such appropriation


unconstitutional.

Issue:
Whether or not the appropriation of P86 billion for debt service vis-a-vis only 27 billion
for education violates Section 5, Article XIV of the Constitution.

Ruling:

Page 17 of 323
No.

While it is true that under Section 5(5), Article XIV of the Constitution, Congress is
mandated to "assign the highest budgetary priority to education" in order to
"insure that teaching will attract and retain its rightful share of the best available
talents through adequate remuneration and other means of job satisfaction and
fulfillment," it does not thereby follow that the hands of Congress are so
hamstrung as to deprive it the power to respond to the imperatives of the
national interest and for the attainment of other state policies or objectives.

As aptly observed by respondents, since 1985, the budget for education has tripled to
upgrade and improve the facility of the public school system. The compensation of
teachers has been doubled. The amount of P29,740,611,000.008 set aside for the
Department of Education, Culture and Sports under the General Appropriations Act
(R.A. No. 6831), is the highest budgetary allocation among all department
budgets. This is a clear compliance with the aforesaid constitutional mandate
according highest priority to education.

Having faithfully complied therewith, Congress is certainly not without any


power, guided only by its good judgment, to provide an appropriation that can
reasonably service our enormous debt, the greater portion of which was inherited
from the previous administration. It is not only a matter of honor and to protect
the credit standing of the country. More especially, the very survival of our economy
is at stake. Thus, if in the process Congress appropriated an amount for debt service
bigger than the share allocated to education, the Court finds and so holds that said
appropriation cannot be thereby assailed as unconstitutional.

*Bernardo v. NLRC, G.R. No. 122917, July 12, 1999. [PROBABLE BAR
QUESTION!]
(Protection to labor; Labor law: Regularization; Qualified disabled persons)

Facts:
-The 43 petitioners are deaf-mutes who were hired on various periods from 1988 to
1993 by respondent Far East Bank and Trust Co. as Money Sorters and Counters
through a uniformly worded agreement called “Employment Contract for Handicapped
Workers.”

-The said contract provided, among others, that: (1) “there are certain positions in the
bank which may be filled-up by disabled and handicapped persons, particularly deaf-
mutes, and the bank has been approached by some civic-minded citizens and
authorized government agencies regarding the possibility of hiring handicapped
workers for these positions”; (2) “the employee hereby acknowledges that the
provisions of Book Six of the Labor Code of the Philippines as amended,
particularly on regulation of employment and separation pay are not applicable
to him/her”; (3) “the Employment Contract shall be for a period of six (6) months
unless earlier terminated by the bank for any just or reasonable cause.”

-The employments of the said deaf-mute were renewed every six (6) months such that
by the time the present case arose, there were fifty-six (56) deaf-mutes who were
employed by respondent bank under the said employment agreement.

-However, sometime in 1993 and 1994, respondent bank decided not to renew the
contracts anymore and hence terminated petitioners’ employments.

-Petitioners then filed a case with the Labor Arbiter, arguing that they were illegally
dismissed.

-Contentions of the respondent bank: (1) Petitioners were hired only as “special
workers” and should not in any way be considered as part of the regular complement
of the Bank; (2) It never solicited the services of petitioners, whose employment was
merely an “accommodation” in response to the requests of government officials

Page 18 of 323
and civic-minded citizens; (3) Their contracts were renewed several times, not
because of need “but merely for humanitarian reasons”; (4) From the beginning,
there have been no separate items in the respondent bank’s plantilla for sorters
or counters and the tellers themselves already did the sorting and counting
chore as an integral part of their duties; and (5) As of the present, the “special
position” that was created for the petitioners no longer exists in private
respondent bank, after the latter had decided not to renew anymore their special
employment contracts.

-The Labor Arbiter dismissed the case filed by petitioners for lack of merit, which the
NLRC affirmed upon appeal, ratiocinating that, “petitioners were hired as an
accommodation to the recommendation of civic oriented personalities whose
employments were covered by the Employment Contracts with special provisions on
duration of contract as specified under Art. 80. Hence, as correctly held by the Labor
Arbiter a quo, the terms of the contract shall be the law between the parties.”

-Petitioners then went to the SC via petition for certiorari.

Issues:
(1) Whether or not petitioners are regular employees;
(2) Whether or not petitioners were illegally dismissed.

Ruling:
(1) Yes.

At the outset, let it be known that this Court appreciates the nobility of private
respondent's effort to provide employment to physically impaired individuals and to
make them more productive members of society. However, we cannot allow it to elude
the legal consequences of that effort, simply because it now deems their employment
irrelevant. The facts, viewed in light of the Labor Code and the Magna Carta for
Disabled Persons, indubitably show that the petitioners should be deemed regular
employees. As such, they have acquired legal rights that this Court is duty-bound
to protect and uphold, not as a matter of compassion but as a consequence of
law and justice.

According to private respondent, the employment contracts were prepared in


accordance with Article 80 of the Labor code, which provides:

“Art. 80. Employment agreement. — Any employer who employs handicapped workers shall enter
into an employment agreement with them, which agreement shall include:

(a) The names and addresses of the handicapped workers to be employed;

(b) The rate to be paid the handicapped workers which shall be not less than seventy five (75%) per cent
of the applicable legal minimum wage;

(c) The duration of employment period; and

(d) The work to be performed by handicapped workers.

The employment agreement shall be subject to inspection by the Secretary of Labor or


his duly authorized representatives.”

The stipulations in the employment contracts indubitably conform with the aforecited
provision. Succeeding events and the enactment of RA No. 7277 (the Magna Carta
for Disabled Persons), however, justify the application of Article 280 of the Labor
Code.

Respondent bank entered into the aforesaid contract with a total of 56 handicapped
workers and renewed the contracts of 37 of them. In fact, two of them worked from
1988 to 1993. Verily, the renewal of the contracts of the handicapped workers and
the hiring of others lead to the conclusion that their tasks were beneficial and
necessary to the bank. More important, these facts show that they were qualified

Page 19 of 323
to perform the responsibilities of their positions. In other words, their disability
did not render them unqualified or unfit for the tasks assigned to them.

In this light, the Magna Carta for Disabled Persons mandates that a qualified
disabled employee should be given the same terms and conditions of
employment as a qualified able-bodied person. Section 5 of the Magna Carta
provides:

Sec. 5. Equal Opportunity for Employment. — No disabled person shall be denied access to opportunities
for suitable employment. A qualified disabled employee shall be subject to the same terms and
conditions of employment and the same compensation, privileges, benefits, fringe benefits,
incentives or allowances as a qualified able bodied person.

The fact that the employees were qualified disabled persons necessarily removes
the employment contracts from the ambit of Article 80. Since the Magna Carta
accords them the rights of qualified able-bodied persons, they are thus covered
by Article 280 of the Labor Code, which provides:

“Art. 280. Regular and Casual Employment. — The provisions of written agreement to the contrary
notwithstanding and regardless of the oral agreement of the parties, an employment shall be deemed to
be regular where the employee has been engaged to perform activities which are usually necessary
or desirable in the usual business or trade of the employer, except where the employment has been
fixed for a specific project or undertaking the completion or termination of which has been determined at
the time of the engagement of the employee or where the work or services to be performed is seasonal in
nature and the employment is for the duration of the season.

An employment shall be deemed to be casual if it is not covered by the preceding paragraph: Provided,
That, any employee who has rendered at least one year of service, whether such service is continuous or
broken, shall be considered as regular employee with respect to the activity in which he is employed and
his employment shall continue while such activity exists.”

The test of whether an employee is regular was laid down in De Leon v. NLRC, in which
this Court held:

“The primary standard, therefore, of determining regular employment is the


reasonable connection between the particular activity performed by the
employee in relation to the usual trade or business of the employer. The test is
whether the former is usually necessary or desirable in the usual business or
trade of the employer. The connection can be determined by considering the
nature of the work performed and its relation to the scheme of the particular
business or trade in its entirety. Also if the employee has been performing the job
for at least one year, even if the performance is not continuous and merely
intermittent, the law deems repeated and continuing need for its performance as
sufficient evidence of the necessity if not indispensibility of that activity to the
business. Hence, the employment is considered regular, but only with respect to such
activity, and while such activity exists.”

Without a doubt, the task of counting and sorting bills is necessary and desirable
to the business of respondent bank. Petitioners performed these tasks for more
than six (6) months. Thus, petitioners should be deemed regular employees. As
held by the Court, “Articles 280 and 281 of the Labor Code put an end to the
pernicious practice of making permanent casuals of our lowly employees by the simple
expedient of extending to them probationary appointments, ad infinitum.” The
contract signed by petitioners is akin to a probationary employment, during
which the bank determined the employees’ fitness for the job. When the bank
renewed the contract after the lapse of the six-month probationary period, the
employees thereby became regular employees. No employer is allowed to determine
indefinitely the fitness of its employees.

(2) Yes.

As regular employees, petitioners are entitled to security of tenure; that is, their
services may be terminated only for a just or authorized cause. Because
respondent bank failed to show such cause, petitioners are deemed illegally

Page 20 of 323
dismissed and therefore entitled to back wages and reinstatement without loss of
seniority rights and other privileges. Considering the allegation of respondent that the
job of money sorting is no longer available because it has been assigned back to the
tellers to whom it originally belonged, petitioners are hereby awarded separation
pay in lieu of reinstatement.

*Lina v. Hon. Paño, G.R. No. 129093, August 30, 2001.


(Autonomy of local governments)

-In 1995, respondent Tony Calvento was appointed agent by the Philippine Charity
Sweepstakes Office (PCSO) to install Terminal OM 20 for the operation of lotto. He
asked Mayor Calixto Cataquiz, Mayor of San Pedro, Laguna, for a mayor's permit to
open the lotto outlet.

-This was denied by Mayor Cataquiz in a letter dated February 19, 1996. The ground
for said denial was an ordinance passed by the Sangguniang Panlalawigan of Laguna
entitled Kapasiyahan Blg. 508, T. 1995 which prohibits the operation of the lotto in
the province of Laguna.

-As a result of this resolution of denial, respondent Calvento filed a complaint for
declaratory relief with prayer for preliminary injunction and temporary restraining
order with the RTC of San Pedro Laguna, Branch 93.

-After due proceedings, the RTC issued an order enjoining petitioners from
implementing or enforcing resolution or kapasiyahan blg. 508, T. 1995 of the
Sangguniang Panlalawigan ng Laguna prohibiting the operation of the lotto in the
province of Laguna.

-Petitioners’ contention: The assailed resolution is a valid policy declaration of the


Provincial Government of Laguna of its vehement objection to the operation of lotto
and all forms of gambling. It is likewise a valid exercise of the provincial government's
police power under the General Welfare Clause of Republic Act 7160, otherwise known
as the Local Government Code of 1991.

Issue:
Whether or not Kapasiyahan Blg. 508, T. 1995 is a valid exercise of the provincial
government's police power under the General Welfare Clause of Republic Act 7160.

Ruling:
No.

The game of lotto is a game of chance duly authorized by the national


government through an Act of Congress. Republic Act 1169, as amended by Batas
Pambansa Blg. 42, is the law which grants a franchise to the PCSO and allows it to
operate the lotteries. The pertinent provision reads:

“SECTION 1. The Philippine Charity Sweepstakes Office. — The Philippine Charity Sweepstakes Office,
hereinafter designated the Office, shall be the principal government agency for raising and providing for
funds for health programs, medical assistance and services and charities of national character, and as
such shall have the general powers conferred in section thirteen of Act Numbered One thousand four
hundred fifty-nine, as amended, and shall have the authority:

A. To hold and conduct charity sweepstakes races, lotteries, and other similar activities, in
such frequency and manner, as shall be determined, and subject to such rules and regulations as shall
be promulgated by the Board of Directors.”

This statute remains valid today. While lotto is clearly a game of chance, the
national government deems it wise and proper to permit it. Hence, the
Sangguniang Panlalawigan of Laguna, a local government unit, cannot issue a
resolution or an ordinance that would seek to prohibit permits. Stated otherwise,
what the national legislature expressly allows by law, such as lotto, a
provincial board may not disallow by ordinance or resolution.

Page 21 of 323
In our system of government, the power of local government units to legislate and
enact ordinances and resolutions is merely a delegated power coming from Congress.
As held in Tatel vs. Virac, ordinances should not contravene an existing statute
enacted by Congress. The reason for this is obvious, as elucidated in Magtajas v.
Pryce Properties Corp.:

“Municipal governments are only agents of the national government. Local councils
exercise only delegated legislative powers conferred upon them by Congress as the
national lawmaking body. The delegate cannot be superior to the principal or
exercise powers higher than those of the latter. It is a heresy to suggest that the
local government units can undo the acts of Congress, from which they have derived
their power in the first place, and negate by mere ordinance the mandate of the
statute.”

Ours is still a unitary form of government, not a federal state. Being so, any form of
autonomy granted to local governments will necessarily be limited and confined
within the extent allowed by the central authority. Besides, the principle of local
autonomy under the 1987 Constitution simply means “decentralization.” It does not
make local governments sovereign within the state or an “imperium in imperio.”

Bill of Rights

*GSIS v. Montesclaros, G.R. No. 146494, July 14, 2004.


(Aspects of due process: Substantive)

Facts:
-Nicolas Montesclaros (“Nicolas”) married Milagros Orbiso (“Milagros”) on 10 July
1983. Nicolas was a 72- year old widower when he married Milagros who was then 43
years old.

-On 4 January 1985, Nicolas filed with the Government Service Insurance System
("GSIS") an application for retirement benefits effective 18 February 1985 under
Presidential Decree No. 1146 or the Revised Government Service Insurance Act of
1977 (“PD 1146”). In his retirement application, Nicolas designated his wife Milagros
as his sole beneficiary.

-On 31 January 1986, GSIS approved Nicolas’ application for retirement “effective 17
February 1984,” granting a lump sum payment of annuity for the first five years and
a monthly annuity thereafter.

-Nicolas died on 22 April 1992.

-Milagros filed with GSIS a claim for survivorship pension under PD 1146. On 8 June
1992, GSIS denied the claim because under Section 18 of PD 1146, the surviving
spouse has no right to survivorship pension if the surviving spouse contracted the
marriage with the pensioner within three years before the pensioner qualified for
the pension.

-According to GSIS, Nicolas wed Milagros on 10 July 1983, less than one year from
his date of retirement on “17 February 1984.”

-Supposed purpose of the proviso: to prevent “deathbed marriages” contracted for


monetary gain.

-Milagros then filed with the RTC a special civil action for declaratory relief
questioning the validity of Section 18 of PD 1146 disqualifying her from receiving
survivorship pension. After due proceedings, the RTC ruled that Milagros was eligible
for survivorship pension.

Issues:

Page 22 of 323
Whether or not Section 18 of PD 1146 is constitutional;

Ruling:
No.

A. Due Process
Under Section 5 of PD 1146, it is mandatory for the government employee to pay
monthly contributions. Considering the mandatory salary deductions from the
government employee, the government pensions do not constitute mere gratuity
but form part of compensation.

In a pension plan where employee participation is mandatory, the prevailing view is


that employees have contractual or vested rights in the pension where the pension is
part of the terms of employment. x x x x

Thus, where the employee retires and meets the eligibility requirements, he
acquires a vested right to benefits that is protected by the due process clause.
Retirees enjoy a protected property interest whenever they acquire a right to
immediate payment under pre-existing law. Thus, a pensioner acquires a vested
right to benefits that have become due as provided under the terms of the public
employees' pension statute. No law can deprive such person of his pension rights
without due process of law, that is, without notice and opportunity to be heard.

In addition to retirement and disability benefits, PD 1146 also provides for benefits to
survivors of deceased government employees and pensioners. Under PD 1146, the
dependent spouse is one of the beneficiaries of survivorship benefits. A widow’s right
to receive pension following the demise of her husband is also part of the
husband's contractual compensation. x x x x

The proviso is unduly oppressive in outrightly denying a dependent spouse’s


claim for survivorship pension if the dependent spouse contracted marriage to
the pensioner within the three-year prohibited period. There is outright
confiscation of benefits due the surviving spouse without giving the surviving
spouse an opportunity to be heard. The proviso undermines the purpose of PD
1146, which is to assure comprehensive and integrated social security and insurance
benefits to government employees and their dependents in the event of sickness,
disability, death, and retirement of the government employees.

B. Equal Protection Clause


The proviso discriminates against the dependent spouse who contracts marriage to the
pensioner within three years before the pensioner qualified for the pension. Under the
proviso, even if the dependent spouse married the pensioner more than three years
before the pensioner's death, the dependent spouse would still not receive survivorship
pension if the marriage took place within three years before the pensioner qualified for
pension. The object of the prohibition is vague. There is no reasonable connection
between the means employed and the purpose intended. The law itself does not
provide any reason or purpose for such a prohibition. If the purpose of the proviso is
to prevent "deathbed marriages," then we do not see why the proviso reckons the
three-year prohibition from the date the pensioner qualified for pension and not from
the date the pensioner died. The classification does not rest on substantial
distinctions. Worse, the classification lumps all those marriages contracted within
three years before the pensioner qualified for pension as having been contracted
primarily for financial convenience to avail of pension benefits.

*Ynot v. Intermediate Appellate Court, G.R. No. 74457, March 20, 1987.
(Due Process: Lawful means; Trial; Reasonable standard)

Facts:
-The President Marcos had earlier issued an order prohibiting the interprovincial
movement and slaughter of carabaos (purpose: “present conditions demand that the
carabaos and the buffaloes be conserved for the benefit of the small farmers who rely

Page 23 of 323
on them for energy needs. x x x x An epidemic had stricken many of these animals
and the reduction of their number had resulted in an acute decline in agricultural
output, which in turn had caused an incipient famine.”)

-However, observing that the violators still managed to circumvent the prohibition
against inter-provincial movement of carabaos by transporting carabeef instead,
Marcos issued Executive Order No. 626-A, which provided:

SECTION 1. Executive Order No. 626 is hereby amended such that henceforth, no carabao regardless of
age, sex, physical condition or purpose and no carabeef shall be transported from one province to
another. The carabao or carabeef transported in violation of this Executive Order as amended shall be
subject to confiscation and forfeiture by the government, to be distributed to charitable institutions and
other similar institutions as the Chairman of the National Meat Inspection Commission may see fit, in
the case of carabeef, and to deserving farmers through dispersal as the Director of Animal Industry may
see fit, in the case of carabaos. x x x x

-Ynot transported six carabaos in a pump boat from Masbate to Iloilo on January 13,
1984, and said carabaos were confiscated by the police station commander of Barotac
Nuevo, Iloilo, for violation of the above measure.

-The case reached the SC via certiorari.

Issue:
Whether or not Executive Order No. 626-A is constitutional.

Ruling:
No.

A. Lawful means/method
While Executive Order No. 626-A has lawful subject as the original executive order, we
cannot say with equal certainty that it complies with the second requirement, viz., that
there be a lawful method. We note that to strengthen the original measure, Executive
Order No. 626-A imposes an absolute ban not on the slaughter of the carabaos
but on their movement, providing that, “no carabao regardless of age, sex, physical
condition or purpose (sic) and no carabeef shall be transported from one province to
another.” The object of the prohibition escapes us. The reasonable connection
between the means employed and the purpose sought to be achieved by the
questioned measure is missing.

We do not see how the prohibition of the inter-provincial transport of carabaos


can prevent their indiscriminate slaughter, considering that they can be killed
anywhere, with no less difficulty in one province than in another. Obviously,
retaining the carabaos in one province will not prevent their slaughter there, any more
than moving them to another province will make it easier to kill them there. As for the
carabeef, the prohibition is made to apply to it as otherwise, so says executive order, it
could be easily circumvented by simply killing the animal. Perhaps so. However, if the
movement of the live animals for the purpose of preventing their slaughter cannot be
prohibited, it should follow that there is no reason either to prohibit their transfer as,
not to be flippant dead meat.

B. No hearing/trial
The penalty is outright confiscation of the carabao or carabeef being transported, to be
meted out by the executive authorities, usually the police only. In the Toribio Case, the
statute was sustained because the penalty prescribed was fine and imprisonment, to
be imposed by the court after trial and conviction of the accused. Under the
challenged measure, significantly, no such trial is prescribed, and the property
being transported is immediately impounded by the police and declared, by the
measure itself, as forfeited to the government.

In the instant case, the carabaos were arbitrarily confiscated by the police station
commander, were returned to the petitioner only after he had filed a complaint for
recovery and given a supersedeas bond of P12,000.00, which was ordered confiscated

Page 24 of 323
upon his failure to produce the carabaos when ordered by the trial court. The
executive order defined the prohibition, convicted the petitioner and
immediately imposed punishment, which was carried out forthright. The measure
struck at once and pounced upon the petitioner without giving him a chance to be
heard, thus denying him the centuries-old guaranty of elementary fair play.

There was no pressure of time or action calling for the petitioner's peremptory
treatment. The properties involved were not even inimical per se as to require
their instant destruction. There certainly was no reason why the offense prohibited
by the executive order should not have been proved first in a court of justice, with the
accused being accorded all the rights safeguarded to him under the Constitution.
Considering that, as we held in Pesigan v. Angeles, Executive Order No. 626-A is
penal in nature, the violation thereof should have been pronounced not by the
police only but by a court of justice, which alone would have had the authority to
impose the prescribed penalty, and only after trial and conviction of the accused.

C. Invalid delegation of legislative powers/ No standard or reasonable guidelines


It is there authorized that the seized property shall “be distributed to charitable
institutions and other similar institutions as the Chairman of the National Meat
Inspection Commission may see fit, in the case of carabeef, and to deserving farmers
through dispersal as the Director of Animal Industry may see fit, in the case of
carabaos.”

The phrase “may see fit” is an extremely generous and dangerous condition, if
condition it is. It is laden with perilous opportunities for partiality and abuse, and
even corruption. One searches in vain for the usual standard and the reasonable
guidelines, or better still, the limitations that the said officers must observe
when they make their distribution. There is none. Their options are apparently
boundless. Who shall be the fortunate beneficiaries of their generosity and by what
criteria shall they be chosen? Only the officers named can supply the answer, they
and they alone may choose the grantee as they see fit, and in their own exclusive
discretion. Definitely, there is here a “roving commission,” a wide and sweeping
authority that is not “canalized within banks that keep it from overflowing,” in short, a
clearly profligate and therefore invalid delegation of legislative powers.

[NOTE: (Instances when no hearing is valid)


“There are instances when the need for expeditions action will justify omission of these
requisites, as in the summary abatement of a nuisance per se, like a mad dog on
the loose, which may be killed on sight because of the immediate danger it poses to
the safety and lives of the people. Contaminated meat and narcotic drugs are
inherently pernicious and may be summarily destroyed. The passport of a person
sought for a criminal offense may be cancelled without hearing, to compel his return
to the country he has fled. Filthy restaurants may be summarily padlocked in the
interest of the public health and bawdy houses to protect the public morals. In such
instances, previous judicial hearing may be omitted without violation of due
process in view of the nature of the property involved or the urgency of the need to
protect the general welfare from a clear and present danger.”]

*Lim v. Court of Appeals, G.R. No. 111397, August 12, 2002.


(Due process: Notice and hearing)

-Policemen under Lim’s instructions inspected and investigated Bistro’s license as


well as the work permits and health certificates of its staff. This caused the stoppage
of work in Bistro’s night club and restaurant operations.

-Lim also refused to accept Bistro’s application for a business license, as well as the
work permit applications of Bistro’s staff, for the year 1993.

-Bistro then filed with the RTC a petition for mandamus and prohibition, arguing
that Lim’s refusal to issue the business license and work permits violated the doctrine

Page 25 of 323
that municipal corporations cannot prohibit the operation of nightclubs. They may be
regulated, but not prevented from carrying on their business.

-After trial, the RTC ruled in favor of Bistro. However, despite the trial court’s order,
Lim still issued a closure order on Bistro’s operations effective January 23, 1993,
even sending policemen to carry out his closure order.

-Lim’s contention: the power of a mayor to inspect and investigate commercial


establishments and their staff is implicit in the statutory power of the city mayor to
issue, suspend or revoke business permits and licenses. This statutory power is
expressly provided for in Section 11 (l), Article II of the Revised Charter of the City of
Manila and in Section 455, paragraph 3 (iv) of the Local Government Code of 1991.

Issue:
Whether or not Lim’s closure of Bistro was valid.

Ruling:
No.

Lim has no authority to close down Bistro’s business or any business


establishment in Manila without due process of law. Lim cannot take refuge under
the Revised Charter of the City of Manila and the Local Government Code. There is no
provision in these laws expressly or impliedly granting the mayor authority to
close down private commercial establishments without notice and hearing, and
even if there is, such provision would be void. The due process clause of the
Constitution requires that Lim should have given Bistro an opportunity to rebut
the allegations that it violated the conditions of its licenses and permits.

The authority of mayors to issue business licenses and permits is beyond question.
The law expressly provides for such authority. Section 11 (l), Article II of the Revised
Charter of the City of Manila, reads:

“Sec. 11. General duties and powers of the mayor. The general duties and powers of the mayor shall be:
x x x.
(l) To grant and refuse municipal licenses or permits of all classes and to revoke the same for
violation of the conditions upon which they were granted, or if acts prohibited by law or municipal
ordinances are being committed under the protection of such licenses or in the premises in which the
business for which the same have been granted is carried on, or for any other reason of general interest.”

From the language of the two laws, it is clear that the power of the mayor to issue
business licenses and permits necessarily includes the corollary power to suspend,
revoke or even refuse to issue the same. However, the power to suspend or revoke
these licenses and permits is expressly premised on the violation of the
conditions of these permits and licenses. The laws specifically refer to the “violation
of the condition(s)” on which the licenses and permits were issued. Similarly, the
power to refuse to issue such licenses and permits is premised on non-compliance
with the prerequisites for the issuance of such licenses and permits. The mayor
must observe due process in exercising these powers, which means that the mayor
must give the applicant or licensee notice and opportunity to be heard.

True, the mayor has the power to inspect and investigate private commercial
establishments for any violation of the conditions of their licenses and permits.
However, the mayor has no power to order a police raid on these establishments
in the guise of inspecting or investigating these commercial establishments. Lim
acted beyond his authority when he directed policemen to raid the New Bangkok Club
and the Exotic Garden Restaurant.

The regulatory powers granted to municipal corporations must always be exercised in


accordance with law, with utmost observance of the rights of the people to due process
and equal protection of the law. Such power cannot be exercised whimsically,
arbitrarily or despotically. In the instant case, we find that Lim’s exercise of this power

Page 26 of 323
violated Bistro’s property rights that are protected under the due process clause of the
Constitution.

Lim did not charge Bistro with any specific violation of the conditions of its
business license or permits. Still, Lim closed down Bistro’s operations even before
the expiration of its business license on December 31, 1992. Lim also refused to
accept Bistro’s license application for 1993, in effect denying the application without
examining whether it complies with legal prerequisites.

*DOJ v. Hon. Felixberto Olalia Jr, G.R. No. 153675, April 19, 2007.
[IMPORTANT!!!]
(Bail in extradition proceedings; Evidence: Clear and Convincing Evidence;
International Law)

Facts:
-On January 30, 1995, the Republic of the Philippines and the then British Crown
Colony of Hong Kong signed an “Agreement for the Surrender of Accused and
Convicted Persons.” It took effect on June 20, 1997. On July 1, 1997, Hong Kong
reverted back to the People’s Republic of China and became the Hong Kong Special
Administrative Region.

-Private respondent Muñoz was charged before the Hong Kong Court with three (3)
counts of the offense of "accepting an advantage as agent," in violation of Section 9
(1) (a) of the Prevention of Bribery Ordinance, Cap. 201 of Hong Kong. He also faces
seven (7) counts of the offense of conspiracy to defraud, penalized by the common
law of Hong Kong.

-On August 23, 1997 and October 25, 1999, warrants of arrest were issued against
him. If convicted, he faces a jail term of seven (7) to fourteen (14) years for each
charge.

-On September 13, 1999, the DOJ received from the Hong Kong Department of Justice
a request for the provisional arrest of private respondent. The DOJ then forwarded
the request to the National Bureau of Investigation (NBI), which, in turn, filed with the
RTC of Manila an application for the provisional arrest of Muñoz.

-On September 23, 1999, the RTC, Branch 19, Manila issued an Order of Arrest
against private respondent. That same day, the NBI agents arrested and detained
him.

-On October 14, 1999, private respondent filed with the Court of Appeals a petition
for certiorari, prohibition and mandamus with application for preliminary
mandatory injunction and/or writ of habeas corpus questioning the validity of the
Order of Arrest.

-On November 9, 1999, the Court of Appeals rendered its Decision declaring the Order
of Arrest void.

-The DOJ went to the SC via Rule 65, certiorari.

-DOJ’s contention: there is no provision in the Constitution granting bail to a


potential extraditee.

-On December 18, 2000, the SC rendered a Decision granting the petition of the DOJ
and sustaining the validity of the Order of Arrest against private respondent. The
Decision became final and executory on April 10, 2001.

-Meanwhile, as early as November 22, 1999, petitioner Hong Kong Special


Administrative Region filed with the RTC of Manila a petition for the extradition of
private respondent, docketed as Civil Case No. 99-95733, raffled off to Branch 10,

Page 27 of 323
presided by Judge Ricardo Bernardo, Jr. For his part, private respondent filed, in the
same case, a petition for bail which was opposed by the DOJ.

-After hearing, or on October 8, 2001, Judge Bernardo, Jr. issued an Order denying
the petition for bail, holding that there is no Philippine law granting bail in extradition
cases and that private respondent is a high “flight risk.”

-On October 22, 2001, Judge Bernardo, Jr. inhibited himself from further hearing
Civil Case No. 99-95733. It was then raffled off to Branch 8 presided by Judge Olalia.

-On October 30, 2001, private respondent filed a motion for reconsideration of the
Order denying his application for bail. This was granted by respondent judge in an
Order dated December 20, 2001 allowing private respondent to post bail.

-The DOJ went to the SC via Rule 65, certiorari.

-DOJ’s contention: There is nothing in the Constitution or statutory law providing


that a potential extraditee has a right to bail, the right being limited solely to criminal
proceedings.

Issue:
Whether or not a potential extraditee has a right to bail.

Ruling:
Yes.

Jurisprudence on extradition is but in its infancy in this jurisdiction. Nonetheless, this


is not the first time that this Court has an occasion to resolve the question of whether
a prospective extraditee may be granted bail.

In Government of United States of America v. Hon. Guillermo G. Purganan, Presiding


Judge, RTC of Manila, Branch 42, and Mark B. Jimenez, a.k.a. Mario Batacan Crespo,
this Court, speaking through then Associate Justice Artemio V. Panganiban, later
Chief Justice, held that the constitutional provision on bail does not apply to
extradition proceedings. It is “available only in criminal proceedings.”

At first glance, the above ruling applies squarely to private respondent’s case.
However, this Court cannot ignore the following trends in international law: (1)
the growing importance of the individual person in public international law who, in the
20th century, has gradually attained global recognition; (2) the higher value now
being given to human rights in the international sphere; (3) the corresponding
duty of countries to observe these universal human rights in fulfilling their treaty
obligations; and (4) the duty of this Court to balance the rights of the individual under
our fundamental law, on one hand, and the law on extradition, on the other.

The modern trend in public international law is the primacy placed on the worth
of the individual person and the sanctity of human rights. Slowly, the recognition
that the individual person may properly be a subject of international law is now taking
root. x x x x

On a more positive note, also after World War II, both international organizations and
states gave recognition and importance to human rights. Thus, on December 10,
1948, the United Nations General Assembly adopted the Universal Declaration of
Human Rights in which the right to life, liberty and all the other fundamental rights
of every person were proclaimed. While not a treaty, the principles contained in
the said Declaration are now recognized as customarily binding upon the
members of the international community. Thus, in Mejoff v. Director of Prisons, this
Court, in granting bail to a prospective deportee, held that under the Constitution,
the principles set forth in that Declaration are part of the law of the land. In
1966, the UN General Assembly also adopted the International Covenant on Civil
and Political Rights which the Philippines signed and ratified. Fundamental among

Page 28 of 323
the rights enshrined therein are the rights of every person to life, liberty, and due
process.

The Philippine authorities are under obligation to make available to every person
under detention such remedies which safeguard their fundamental right to
liberty. These remedies include the right to be admitted to bail. While this Court
in Purganan limited the exercise of the right to bail to criminal proceedings,
however, in light of the various international treaties giving recognition and
protection to human rights, particularly the right to life and liberty, a re-
examination of this Court’s ruling in Purganan is in order.

First, we note that the exercise of the State’s power to deprive an individual of his
liberty is not necessarily limited to criminal proceedings. Respondents in
administrative proceedings, such as deportation and quarantine, have likewise
been detained.

Second, to limit bail to criminal proceedings would be to close our eyes to our
jurisprudential history. Philippine jurisprudence has not limited the exercise of the
right to bail to criminal proceedings only. This Court has admitted to bail persons who
are not involved in criminal proceedings. In fact, bail has been allowed in this
jurisdiction to persons in detention during the pendency of administrative
proceedings, taking into cognizance the obligation of the Philippines under
international conventions to uphold human rights.

If bail can be granted in deportation cases, we see no justification why it should


not also be allowed in extradition cases. Likewise, considering that the Universal
Declaration of Human Rights applies to deportation cases, there is no reason
why it cannot be invoked in extradition cases. After all, both are administrative
proceedings where the innocence or guilt of the person detained is not in issue.

Clearly, the right of a prospective extraditee to apply for bail in this jurisdiction
must be viewed in the light of the various treaty obligations of the Philippines
concerning respect for the promotion and protection of human rights. Under these
treaties, the presumption lies in favor of human liberty. Thus, the Philippines
should see to it that the right to liberty of every individual is not impaired.

Section 2(a) of Presidential Decree (P.D.) No. 1069 (The Philippine Extradition Law)
defines "extradition" as "the removal of an accused from the Philippines with the object
of placing him at the disposal of foreign authorities to enable the requesting state or
government to hold him in connection with any criminal investigation directed against
him or the execution of a penalty imposed on him under the penal or criminal law of
the requesting state or government."

Extradition has thus been characterized as the right of a foreign power, created by
treaty, to demand the surrender of one accused or convicted of a crime within its
territorial jurisdiction, and the correlative duty of the other state to surrender him to
the demanding state. It is not a criminal proceeding. Even if the potential extraditee
is a criminal, an extradition proceeding is not by its nature criminal, for it is not
punishment for a crime, even though such punishment may follow extradition. It
is sui generis, tracing its existence wholly to treaty obligations between different
nations. It is not a trial to determine the guilt or innocence of the potential
extraditee. Nor is it a full-blown civil action, but one that is merely administrative
in character. Its object is to prevent the escape of a person accused or convicted of a
crime and to secure his return to the state from which he fled, for the purpose of trial
or punishment.

But while extradition is not a criminal proceeding, it is characterized by the


following: (a) it entails a deprivation of liberty on the part of the potential
extraditee and (b) the means employed to attain the purpose of extradition is
also "the machinery of criminal law." This is shown by Section 6 of P.D. No. 1069
(The Philippine Extradition Law) which mandates the "immediate arrest and temporary

Page 29 of 323
detention of the accused" if such "will best serve the interest of justice." We further
note that Section 20 allows the requesting state "in case of urgency" to ask for the
"provisional arrest of the accused, pending receipt of the request for extradition;" and
that release from provisional arrest "shall not prejudice re-arrest and extradition of the
accused if a request for extradition is received subsequently."

Obviously, an extradition proceeding, while ostensibly administrative, bears all


earmarks of a criminal process. A potential extraditee may be subjected to arrest,
to a prolonged restraint of liberty, and forced to transfer to the demanding state
following the proceedings. "Temporary detention" may be a necessary step in the
process of extradition, but the length of time of the detention should be reasonable.

Records show that private respondent was arrested on September 23, 1999, and
remained incarcerated until December 20, 2001, when the trial court ordered his
admission to bail. In other words, he had been detained for over two (2) years
without having been convicted of any crime. By any standard, such an extended
period of detention is a serious deprivation of his fundamental right to liberty. In
fact, it was this prolonged deprivation of liberty which prompted the extradition court
to grant him bail.

An extradition proceeding being sui generis, the standard of proof required in granting
or denying bail can neither be the proof beyond reasonable doubt in criminal cases
nor the standard of proof of preponderance of evidence in civil cases. While
administrative in character, the standard of substantial evidence used in
administrative cases cannot likewise apply given the object of extradition law, which is
to prevent the prospective extraditee from fleeing our jurisdiction. In his Separate
Opinion in Purganan, then Associate Justice, now Chief Justice Reynato S. Puno,
proposed that a new standard which he termed “clear and convincing evidence”
should be used in granting bail in extradition cases. According to him, this
standard should be lower than proof beyond reasonable doubt but higher than
preponderance of evidence. The potential extraditee must prove by “clear and
convincing evidence” that he is not a flight risk and will abide with all the
orders and processes of the extradition court.

In this case, there is no showing that private respondent presented evidence to show
that he is not a flight risk. Consequently, this case should be remanded to the trial
court to determine whether private respondent may be granted bail on the basis of
“clear and convincing evidence.”

*Emin v. De Leon, G.R. No. 139794, February 27, 2002.


(Due Process: Cross-examination)

-Emin was a public school teacher. When the Director of DECS noticed that the
signatures in the certificates of eligibility of certain teachers were forged, the former
ordered an investigation.

-Six (6) teachers, in their sworn statements, identified Emin as the person who helped
them obtain the fake certificates of eligibility.

-Emin was charged for the spurious certificates and was eventually found by the Civil
Service Commission guilty of dishonesty, grave misconduct and conduct prejudicial to
the best interest of the service, and in view thereof, Emin was dismissed from the
service.

-Contention of Emin: he was denied due process because he was not allowed cross-
examination.

Issue: Whether or not Emin was denied due process in view of fact that he was not
allowed cross-examination.

Ruling:

Page 30 of 323
No.

It is well to remember that in administrative proceedings, technical rules of


procedure and evidence are not strictly applied and administrative due process
cannot be fully equated with due process in its strict judicial sense.

Nothing on record shows he asked for cross-examination as most of the


submissions were written. In our view, petitioner cannot argue that he has been
deprived of due process merely because no cross-examination took place. The rule is
well established that due process is satisfied when the parties are afforded fair and
reasonable opportunity to explain their side of the controversy or given
opportunity to move for a reconsideration of the action or ruling complained of.
In the present case, the record clearly shows that petitioner not only filed his Counter-
Affidavit during the preliminary investigation, and later his Motion to Dismiss. He
also filed a Motion for Reconsideration of the October 19, 1993 Order of the
Commission. The essence of due process in administrative proceedings is an
opportunity to explain one’s side or an opportunity to seek reconsideration of the
action or ruling complained of.

*Jesus Garcia v. Hon. Ray Alan Drilon, G.R. No. 179267, June 25, 2013.
[IMPORTANT!!!]
(VAWC; Notice and hearing may be dispensed with without violating due process)

Facts:
-Rosalie and Jesus are married and have 3 children. Rosalie alleged that Jesus is
dominant, controlling, and demands absolute obedience from his wife and children.
He forbade Rosalie to pray, and deliberately isolated her from her friends.

-Things turned for the worse when Jesus took up an affair with a bank manager of
Robinson's Bank, Bacolod City, who is the godmother of one of their sons. Jesus’
infidelity spawned a series of fights that left Rosalie physically and emotionally
wounded.

-In one of their quarrels, petitioner grabbed private respondent on both arms and
shook her with such force that caused bruises and hematoma. At another time,
petitioner hit private respondent forcefully on the lips that caused some bleeding.
Petitioner sometimes turned his ire on their daughter, Jo-Ann, who had seen the text
messages he sent to his paramour and whom he blamed for squealing on him. He beat
Jo-Ann on the chest and slapped her many times.

-Rosalie then filed, for herself and in behalf of her minor children, a verified petition
before the RTC of Bacolod City for the issuance of a Temporary Protection Order
(TPO) against her husband, Jesus (petitioner), pursuant to R.A. 9262. She claimed to
be a victim of physical abuse; emotional, psychological, and economic violence as a
result of marital infidelity on the part of petitioner, with threats of deprivation of
custody of her children and of financial support.

-Finding reasonable ground to believe that an imminent danger of violence against the
private respondent and her children exists or is about to recur, the RTC issued a TPO
on March 24, 2006 effective for thirty (30) days.

-Two days later, or on April 26, 2006, petitioner filed an Opposition to the Urgent Ex-
Parte Motion for Renewal of the TPO seeking the denial of the renewal of the TPO on
the grounds that it did not (1) comply with the three-day notice rule, and (2) contain a
notice of hearing.

-During the pendency of Civil Case No. 06-797, petitioner filed before the Court of
Appeals (CA) a petition for prohibition, with prayer for injunction and temporary
restraining order, challenging (1) the constitutionality of R.A. 9262 for being violative
of the due process and the equal protection clauses, and (2) the validity of the modified
TPO issued in the civil case for being “an unwanted product of an invalid law.”

Page 31 of 323
-The CA eventually dismissed the petition for failure of petitioner to raise the
constitutional issue in his pleadings before the trial court in the civil case, which is
clothed with jurisdiction to resolve the same. Secondly, the challenge to the validity of
R.A. 9262 through a petition for prohibition seeking to annul the protection orders
issued by the trial court constituted a collateral attack on said law.

-Jesus then went to the SC.

-Jesus’ contentions: (1) since R.A. 9262 is intended to prevent and criminalize
spousal and child abuse, which could very well be committed by either the husband or
the wife, gender alone is not enough basis to deprive the husband/father of the
remedies under the law; (2) on the basis of unsubstantiated allegations, and
practically no opportunity to respond, the husband is stripped of family, property,
guns, money, children, job, future employment and reputation, all in a matter of
seconds, without an inkling of what happened.

Issues:
(1) Whether or not R.A. 9262 IS DISCRIMINATORY, UNJUST, AND VIOLATIVE OF THE
EQUAL PROTECTION CLAUSE;

(2) Whether or not R.A. 9262 RUNS COUNTER TO THE DUE PROCESS CLAUSE OF
THE CONSTITUTION;

(3) Whether or not R.A. No. 9262 IS UNCONSTITUTIONAL BECAUSE IT ALLOWS AN


UNDUE DELEGATION OF JUDICIAL POWER TO THE BARANGAY OFFICIALS.

(4) Whether or not the Family Court has the authority to pass upon the
constitutionality of a law.

Ruling:
(1) No.

Equal protection simply requires that all persons or things similarly situated should
be treated alike, both as to rights conferred and responsibilities imposed.

The guaranty of equal protection of the laws is not a guaranty of equality in the
application of the laws upon all citizens of the state. It is not, therefore, a requirement,
in order to avoid the constitutional prohibition against inequality, that every man,
woman and child should be affected alike by a statute. Equality of operation of
statutes does not mean indiscriminate operation on persons merely as such, but on
persons according to the circumstances surrounding them. It guarantees equality, not
identity of rights. The Constitution does not require that things which are different in
fact be treated in law as though they were the same. The equal protection clause
does not forbid discrimination as to things that are different. It does not prohibit
legislation which is limited either in the object to which it is directed or by the territory
within which it is to operate.

A law is not invalid because of simple inequality. The very idea of classification is
that of inequality, so that it goes without saying that the mere fact of inequality in no
manner determines the matter of constitutionality. All that is required of a valid
classification is that it be reasonable, which means that the classification should be
based on substantial distinctions which make for real differences; that it must be
germane to the purpose of the law; that it must not be limited to existing conditions
only; and that it must apply equally to each member of the class. This Court has held
that the standard is satisfied if the classification or distinction is based on a
reasonable foundation or rational basis and is not palpably arbitrary.

I. R.A. 9262 rests on substantial distinctions.

Page 32 of 323
The unequal power relationship between women and men; the fact that women
are more likely than men to be victims of violence; and the widespread gender
bias and prejudice against women all make for real differences justifying the
classification under the law. As Justice McIntyre succinctly states, "the
accommodation of differences ... is the essence of true equality."

Women are the "usual" and "most likely" victims of violence. x x x x On the other
hand, no reliable estimates may be obtained on domestic abuse and violence
against men in the Philippines because incidents thereof are relatively low and,
perhaps, because many men will not even attempt to report the situation.

While there are, indeed, relatively few cases of violence and abuse perpetrated
against men in the Philippines, the same cannot render R.A. 9262 invalid.

II. The classification is germane to the purpose of the law.

The distinction between men and women is germane to the purpose of R.A.
9262, which is to address violence committed against women and children,
spelled out in its Declaration of Policy, as follows:

“SEC. 2. Declaration of Policy. – It is hereby declared that the State values the dignity of women and
children and guarantees full respect for human rights. The State also recognizes the need to protect the
family and its members particularly women and children, from violence and threats to their
personal safety and security.”

III & IV. The classification is not limited to existing conditions only, and apply equally to
all members.

Moreover, the application of R.A. 9262 is not limited to the existing conditions when it
was promulgated, but to future conditions as well, for as long as the safety and
security of women and their children are threatened by violence and abuse.

R.A. 9262 applies equally to all women and children who suffer violence and abuse. x
xxx

There is likewise no merit to the contention that R.A. 9262 singles out the husband or
father as the culprit. As defined above, VAWC may likewise be committed “against
a woman with whom the person has or had a sexual or dating relationship."
Clearly, the use of the gender-neutral word “person” who has or had a sexual or
dating relationship with the woman encompasses even lesbian relationships.

(2) No.

The rules require that petitions for protection order be in writing, signed and verified
by the petitioner thereby undertaking full responsibility, criminal or civil, for every
allegation therein. Since "time is of the essence in cases of VAWC if further
violence is to be prevented," the court is authorized to issue ex parte a TPO after
raffle but before notice and hearing when the life, limb or property of the victim
is in jeopardy and there is reasonable ground to believe that the order is necessary to
protect the victim from the immediate and imminent danger of VAWC or to prevent
such violence, which is about to recur.

There need not be any fear that the judge may have no rational basis to issue an ex
parte order. The victim is required not only to verify the allegations in the
petition, but also to attach her witnesses' affidavits to the petition.

The grant of a TPO ex parte cannot, therefore, be challenged as violative of the right to
due process. Just like a writ of preliminary attachment which is issued without
notice and hearing because the time in which the hearing will take could be
enough to enable the defendant to abscond or dispose of his property, in the
same way, the victim of VAWC may already have suffered harrowing
experiences in the hands of her tormentor, and possibly even death, if notice

Page 33 of 323
and hearing were required before such acts could be prevented. It is a
constitutional commonplace that the ordinary requirements of procedural due process
must yield to the necessities of protecting vital public interests, among which is
protection of women and children from violence and threats to their personal safety
and security.

It should be pointed out that when the TPO is issued ex parte, the court shall
likewise order that notice be immediately given to the respondent directing him
to file an opposition within five (5) days from service. Moreover, the court shall
order that notice, copies of the petition and TPO be served immediately on the
respondent by the court sheriffs. The TPOs are initially effective for thirty (30) days
from service on the respondent.

Where no TPO is issued ex parte, the court will nonetheless order the immediate
issuance and service of the notice upon the respondent requiring him to file an
opposition to the petition within five (5) days from service. The date of the preliminary
conference and hearing on the merits shall likewise be indicated on the notice.

The opposition to the petition which the respondent himself shall verify, must be
accompanied by the affidavits of witnesses and shall show cause why a temporary or
permanent protection order should not be issued.

It is clear from the foregoing rules that the respondent of a petition for protection
order should be apprised of the charges imputed to him and afforded an
opportunity to present his side. Thus, the fear of petitioner of being "stripped of
family, property, guns, money, children, job, future employment and reputation, all in
a matter of seconds, without an inkling of what happened" is a mere product of an
overactive imagination. The essence of due process is to be found in the reasonable
opportunity to be heard and submit any evidence one may have in support of one's
defense. "To be heard" does not only mean verbal arguments in court; one may be
heard also through pleadings. Where opportunity to be heard, either through oral
arguments or pleadings, is accorded, there is no denial of procedural due process.

(3) No.

The pertinent provision reads, as follows:

“SEC. 14. Barangay Protection Orders (BPOs); Who May Issue and How. – Barangay Protection Orders
(BPOs) refer to the protection order issued by the Punong Barangay ordering the perpetrator to desist
from committing acts under Section 5 (a) and (b) of this Act. A Punong Barangay who receives
applications for a BPO shall issue the protection order to the applicant on the date of filing after ex parte
determination of the basis of the application. If the Punong Barangay is unavailable to act on the
application for a BPO, the application shall be acted upon by any available Barangay Kagawad. If the
BPO is issued by a Barangay Kagawad, the order must be accompanied by an attestation by the Barangay
Kagawad that the Punong Barangay was unavailable at the time of the issuance of the BPO. BPOs shall
be effective for fifteen (15) days. Immediately after the issuance of an ex parte BPO, the Punong
Barangay or Barangay Kagawad shall personally serve a copy of the same on the respondent, or direct
any barangay official to effect its personal service.”

As clearly delimited by the aforequoted provision, the BPO issued by the Punong
Barangay or, in his unavailability, by any available Barangay Kagawad, merely orders
the perpetrator to desist from (a) causing physical harm to the woman or her
child; and (2) threatening to cause the woman or her child physical harm. Such
function of the Punong Barangay is, thus, purely executive in nature, in
pursuance of his duty under the Local Government Code to "enforce all laws and
ordinances," and to "maintain public order in the barangay."

We have held that, “the mere fact that an officer is required by law to inquire into
the existence of certain facts and to apply the law thereto in order to determine
what his official conduct shall be and the fact that these acts may affect private
rights do not constitute an exercise of judicial powers.”

Page 34 of 323
In the same manner as the public prosecutor ascertains through a preliminary
inquiry or proceeding "whether there is reasonable ground to believe that an offense
has been committed and the accused is probably guilty thereof," the Punong
Barangay must determine reasonable ground to believe that an imminent danger
of violence against the woman and her children exists or is about to recur that
would necessitate the issuance of a BPO. The preliminary investigation conducted
by the prosecutor is, concededly, an executive, not a judicial, function. The
same holds true with the issuance of a BPO.

(4) Yes.

In spite of its designation as a family court, the RTC of Bacolod City remains
possessed of authority as a court of general original jurisdiction to pass upon all
kinds of cases whether civil, criminal, special proceedings, land registration,
guardianship, naturalization, admiralty or insolvency. It is settled that RTCs
have jurisdiction to resolve the constitutionality of a statute, "this authority
being embraced in the general definition of the judicial power to determine what are
the valid and binding laws by the criterion of their conformity to the fundamental law."
The Constitution vests the power of judicial review or the power to declare the
constitutionality or validity of a law, treaty, international or executive
agreement, presidential decree, order, instruction, ordinance, or regulation not
only in the Supreme Court, but in all RTCs. We said in J.M. Tuason and Co., Inc. v.
CA that, "plainly the Constitution contemplates that the inferior courts should have
jurisdiction in cases involving constitutionality of any treaty or law, for it speaks of
appellate review of final judgments of inferior courts in cases where such
constitutionality happens to be in issue.” Section 5, Article VIII of the 1987
Constitution reads in part as follows:

“SEC. 5. The Supreme Court shall have the following powers:


xxx
2. Review, revise, reverse, modify, or affirm on appeal or certiorari, as the law or the Rules of Court may
provide, final judgments and orders of lower courts in:
a. All cases in which the constitutionality or validity of any treaty, international or executive
agreement, law, presidential decree, proclamation, order, instruction, ordinance, or regulation is in
question. x x x x”

*Shu v. Dee, G.R. No. 182573, April 23, 2014.


(Notice and hearing may be dispensed with without violating due process; NBI)

Facts:
-Shu filed a complaint before the National Bureau of Investigation (NBI) charging Dee,
et al. (as employees of Metrobank) with falsification of two deeds of real estate
mortgage.

-After investigation, the NBI filed a complaint with the City Prosecutor of Makati (city
prosecutor) charging Dee, at al. with the crime of forgery and falsification of public
documents. The NBI supported the complaint with the Questioned Documents
Report No. 746-1098 (questioned documents report) issued by its Questioned
Documents Division. The questioned documents report states that the signatures of
the petitioner which appear on the questioned deeds are not the same as the
standard sample signatures he submitted to the NBI.

-Contention of Dee, et al.: They were denied their right to due process during the
NBI investigation because the NBI never required them and Metrobank to submit the
standard sample signatures of the petitioner for comparison.

Issue:
Whether or not Dee, et al. were denied due process.

Ruling:
No.

Page 35 of 323
The functions of the NBI are merely investigatory and informational in nature. It
has no judicial or quasi-judicial powers and is incapable of granting any relief to any
party. It cannot even determine probable cause. The NBI is an investigative
agency whose findings are merely recommendatory. It undertakes investigation of
crimes upon its own initiative or as public welfare may require in accordance with its
mandate. It also renders assistance when requested in the investigation or detection of
crimes in order to prosecute the persons responsible.

Since the NBI’s findings were merely recommendatory, we find that no denial of the
respondents’ due process right could have taken place; the NBI’s findings were still
subject to the prosecutor’s and the Secretary of Justice’s actions for purposes of
finding the existence of probable cause.

We find it significant that the specimen signatures in the possession of Metrobank


were submitted by the respondents for the consideration of the city prosecutor and
eventually of the Secretary of Justice during the preliminary investigation proceedings.
Thus, these officers had the opportunity to examine these signatures. x x x x

The respondents were not likewise denied their right to due process when the NBI
issued the questioned documents report. We note that this report merely stated that
the signatures appearing on the two deeds and in the petitioner’s submitted sample
signatures were not written by one and the same person. Notably, there was no
categorical finding in the questioned documents report that the respondents
falsified the documents. x x x x The report is inconclusive and does not prevent
the respondents from securing a separate documents examination by
handwriting experts based on their own evidence. On its own, the NBI’s questioned
documents report does not directly point to the respondents’ involvement in the crime
charged. Its significance is that, taken together with the other pieces of evidence
submitted by the parties during the preliminary investigation, these evidence could be
sufficient for purposes of finding probable cause.

*Sanggguniang Panglunsod ng Baguio v. Jadewell Parking Systems Corp., G.R.


No. 160025, April 23, 2014.
(Notice and hearing may be dispensed with without violating due process; Quasi-
legislative)

Facts:
-The officials of the City Government of Baguio and Jadewell Parking Systems
Corporation (Jadewell) executed a Memorandum of Agreement (MOA) on 26 June
2000, whereby the City of Baguio authorized Jadewell to regulate and collect parking
fees for on-street parking in the city, as well as to implement the installation of
modern parking meters.

-Thereafter, the Sangguniang Panlungsod of Baguio City (Sanggunian) revoked the


said MOA through City Resolution No. 037, Series of 2002 (Resolution 37), alleging
substantial breach of the MOA on the part of Jadewell. Then Mayor Alfredo Vergara
vetoed the Resolution.

-The Sanggunian Panlungsod overrode the veto through an unnumbered Resolution


dated 17 April 2002.

-Jadewell denied the breach and commenced an action before the Regional Trial Court
(RTC) of Baguio, questioning the validity of the MOA’s revocation and the
Sanggunian’s capacity to pass a resolution revoking the MOA.

-Jadewell’s contention, among others: It was deprived of due process when the
Sangguniang Panlungsod rescinded the MOA without prior hearing in which
Jadewell could air its side. Neither did the Sanggunian convene an investigatory body
to inquire into Jadewell’s alleged violations nor at least invite Jadewell to a conference
to discuss the alleged violations, if only to give Jadewell the chance to refute any

Page 36 of 323
evidence gathered by it against the latter. As it is, the Sanggunian arrogated upon
itself the role of a prosecutor, judge and executioner in rescinding the MOA, all in
clear violation of Jadewell’s constitutionally embedded right to due process.

Issue:
Whether or not Jadewell was denied due process due to lack of notice and hearing
prior to the rescission of the MOA.

Ruling:
No.

Previous notice and hearing as elements of due process, are constitutionally


required for the protection of life or vested property rights, as well as of liberty,
when its limitation or loss takes place in consequence of a judicial or quasi-
judicial proceeding, generally dependent upon a past act or event which has to be
established or ascertained. It is not essential to the validity of general rules or
regulations promulgated to govern future conduct of a class or persons or
enterprises, unless the law provides otherwise.

In the instant case, the assailed act by the Sanggunian Panlungsod in rescinding
the MOA – be it first or second act of rescission – was clearly in the exercise of
its legislative or administrative functions and was not an exercise of a judicial or
quasi-judicial function. The Sanggunian Panlungsod does not possess any judicial or
quasi-judicial functions. The preamble of the MOA lends support to this view.
Evidently, the foremost reason why the agreement was entered into by the parties was
to provide order, given Baguio City’s parking problems in identified areas, as well as to
generate income.

*Barata v. Abalos, G.R. No. 142888, June 6, 2001.


(Due Process: Right to Appeal; Remedial Law)

Facts:
-Barata, as the head of San Miguel Bukid Homeowners’ Association, Inc. filed an
administrative complaint against Mandaluyong City Mayor Abalos, Jr. for violation
of Section 5(a) of R.A. 6713 (Code of Conduct and Ethical Standards for Public
Officials and Employees) for failing to act promptly on letters and requests sent by the
public, particularly, their repeated requests for the completion of promised housing
project.

-The Office of the Ombudsman rendered a Decision dismissing the administrative


complaint for insufficiency of evidence.

-Barata then filed a Petition for Review on Certiorari with the Court of Appeals
which rendered a Decision dismissing the petition on the ground that the decision
exonerating respondent mayor of administrative charge is not appealable.

-Pursuant to the authority vested in the Office of the Ombudsman to promulgate its
rules of procedure, Section 7, Rule III of Administrative Order No.7 provides:

“SEC. 7. Finality of decision. - Where the respondent is absolved of the charge, and in case of
conviction where the penalty imposed is public censure or reprimand, suspension of not more than one
month, or a fine equivalent to one month salary, the decision shall be final and unappealable. In all
other cases, the decision shall become final after the expiration of ten (10) days from receipt thereof by the
respondent, unless a motion for reconsideration or petition for certiorari shall have been filed by him as
prescribed in Section 27 of RA 6770.”

-Contention of Barata: The Office of the Ombudsman should not restrict the right of
appeal nor limit the power of review of the SC. Whether the decision of the
Ombudsman is for conviction or acquittal of the respondent, it should be reviewed
by the SC.

Issue:

Page 37 of 323
Whether or not the fact that the decision exonerating Mayor Abalos is unappealable, is
tantamount to a denial of due process.

Ruling:
No.

The failure to provide for the right of appeal in certain cases from the decision of
the Ombudsman is not a denial of due process for the right to appeal is not a
natural right nor a part of due process; it is merely a statutory privilege and
may be exercised only in the manner and in accordance with the provisions of
the law. x x x x

In appropriate cases involving oppressive or arbitrary action, the complainant is


not deprived of a legal recourse by certiorari under Rule 65 of the Rules of Court
which apply suppletorily to the Rules of Procedures of the Office of the Ombudsman.

*Estrada v. Ombudsman, G.R. Nos. 212140-41, January 21, 2015.


[IMPORTANT!!!]
(Due process: Counter-affidavits of co-respondents)

Facts:
-The NBI and Atty. Baligod filed a Plunder Complaint with the Ombudsman against
Sen. Jinggoy Estrada (Estrada).

-The following month, another Plunder complaint against Estrada was filed, this time,
by the Field Investigation Office (FIO) of the Ombudsman. Estrada seasonably filed his
counter-affidavits in the two complaints.

-Eighteen of Estrada’s co-respondents in the two complaints likewise filed their


counter-affidavits.

-Subsequently, Estrada filed his “Request to be Furnished with Copies of Counter-


Affidavits of the Other Respondents, Affidavits of New Witnesses and Other
Filings.” The Ombudsman denied the said request.

-Without filing a Motion for Reconsideration of the Ombudsman’s Order denying his
Request, Estrada then went to the SC via a Petition for Certiorari with prayer for the
issuance of a TRO enjoining the Ombudsman and the NBI from conducting further
proceedings.

-During the pendency of Estrada’s petition before the SC, the Ombudsman furnished
him copies (most but not all) of his co-respondents’ counter-affidavits.

Issues:
(1) Whether or not Estrada was denied due process when he was initially denied copies
of his co-respondents’ counter-affidavits;

(2) Whether or not the requirements of due process under the Ang Tibay case are
applicable to preliminary investigation such as Estrada’s case.

Ruling:
(1) No.

First. There is no law or rule which requires the Ombudsman to furnish a


respondent with copies of the counter-affidavits of his co-respondents.

Sen. Estrada claims that the denial of his Request for the counter affidavits of his co-
respondents violates his constitutional right to due process. Sen. Estrada, however,
fails to specify a law or rule which states that it is a compulsory requirement of
due process in a preliminary investigation that the Ombudsman furnish a
respondent with the counter-affidavits of his co-respondents. Neither Section 3(b),

Page 38 of 323
Rule 112 of the Revised Rules of Criminal Procedure nor Section 4(c), Rule II of the
Rules of Procedure of the Office of the Ombudsman supports Sen. Estrada’s claim.
What the Rules of Procedure of the Office of the Ombudsman require is for the
Ombudsman to furnish the respondent with a copy of the complaint and the
supporting affidavits and documents at the time the order to submit the
counter-affidavit is issued to the respondent. x x x x

Although Section 4(c), Rule II of the Rules of Procedure of the Office of the
Ombudsman provides that a respondent "shall have access to the evidence on record,"
this provision should be construed in relation to Section 4(a) and (b) of the same Rule,
as well as to the Rules of Criminal Procedure. First, Section 4(a) states that "the
investigating officer shall require the complainant or supporting witnesses to execute
affidavits to substantiate the complaint." The "supporting witnesses" are the
witnesses of the complainant, and do not refer to the co-respondents.

Second, Section 4(b) states that "the investigating officer shall issue an order attaching
thereto a copy of the affidavits and all other supporting documents, directing the
respondent" to submit his counter-affidavit. The affidavits referred to in Section 4(b)
are the affidavits mentioned in Section 4(a). Clearly, the affidavits to be furnished to
the respondent are the affidavits of the complainant and his supporting
witnesses. The provision in the immediately succeeding Section 4(c) of the same Rule
II that a respondent shall have "access to the evidence on record" does not stand
alone, but should be read in relation to the provisions of Section 4(a and b) of the
same Rule II requiring the investigating officer to furnish the respondent with the
"affidavits and other supporting documents" submitted by "the complainant or
supporting witnesses." Thus, a respondent’s “access to evidence on record” in
Section 4(c), Rule II of the Ombudsman’s Rules of Procedure refers to the
affidavits and supporting documents of “the complainant or supporting
witnesses” in Section 4(a) of the same Rule II.

Third, Section 3(b), Rule 112 of the Revised Rules of Criminal Procedure provides that
“the respondent shall have the right to examine the evidence submitted by the
complainant which he may not have been furnished and to copy them at his expense.”
A respondent’s right to examine refers only to “the evidence submitted by the
complainant.”

Thus, whether under Rule 112 of the Revised Rules of Criminal Procedure or under
Rule II of the Ombudsman’s Rules of Procedure, there is no requirement whatsoever
that the affidavits executed by the co-respondents should be furnished to a
respondent.

(2) No.

Simply put, the Ang Tibay guidelines for administrative cases do not apply to
preliminary investigations in criminal cases. An application of the Ang Tibay
guidelines to preliminary investigations will have absurd and disastrous
consequences. x x x x

The quantum of evidence needed in Ang Tibay, as amplified in GSIS, is greater


than the evidence needed in a preliminary investigation to establish probable
cause, or to establish the existence of a prima facie case that would warrant the
prosecution of a case. Ang Tibay refers to "substantial evidence," while the
establishment of probable cause needs "only more than ‘bare suspicion,’ or ‘less
than evidence which would justify a conviction’." x x x x

To require the application of Ang Tibay, as amplified in GSIS, in preliminary


investigations will change the quantum of evidence required in determining probable
cause from evidence of likelihood or probability of guilt to substantial evidence of guilt.

*Philippine Association of Service Exporters, Inc. v. Drilon, G.R. No. 81958,


June 30, 1988.

Page 39 of 323
(Equal protection of the laws: valid classification)

Facts:
-PASEI is a firm engaged principally in the recruitment of Filipino workers, male and
female, for overseas placement.

-DOLE issued Department Order No. 1, GUIDELINES GOVERNING THE TEMPORARY


SUSPENSION OF DEPLOYMENT OF FILIPINO DOMESTIC AND HOUSEHOLD
WORKERS.

-PASEI then went to the SC via certiorari and prohibition.

-PASEI contentions: (1) D.O. 1 discriminates against females, and it does not apply to
all Filipino workers but only to domestic helpers and females with similar skills; (2) It
is violative of the right to travel; and (3) It constitutes an invalid exercise of legislative
power.

Issues:
(1) Whether or not D.O. 1 violates the equal protection clause;
(2) Whether or not D.O. 1 is violative of the right to travel;
(3) Whether or not D.O. 1 constitutes an invalid exercise of legislative power.

Ruling:
(1) No.

There is no question that Department Order No. 1 applies only to "female


contract workers," but it does not thereby make an undue discrimination
between the sexes. It is well-settled that "equality before the law" under the
Constitution does not import a perfect identity of rights among all men and women. It
admits of classifications, provided that (1) such classifications rest on substantial
distinctions; (2) they are germane to the purposes of the law; (3) they are not confined
to existing conditions; and (4) they apply equally to all members of the same class.

The Court is satisfied that the classification made—the preference for female
workers—rests on substantial distinctions.

As a matter of judicial notice, the Court is well aware of the unhappy plight that
has befallen our female labor force abroad, especially domestic servants, amid
exploitative working conditions marked by, in not a few cases, physical and
personal abuse. The sordid tales of maltreatment suffered by migrant Filipina
workers, even rape and various forms of torture, confirmed by testimonies of returning
workers, are compelling motives for urgent government action. As precisely the
caretaker of Constitutional rights, the Court is called upon to protect victims of
exploitation. In fulfilling that duty, the Court sustains the government's efforts.

The same, however, cannot be said of our male workers. In the first place, there is
no evidence that, except perhaps for isolated instances, our men abroad have
been afflicted with an identical predicament.

(2) No.

The right to travel is subject, among other things, to the requirements of “public
safety,” “as may be provided by law.” Department Order No. 1 is a valid
implementation of the Labor Code, in particular, its basic policy to “afford
protection to labor,” pursuant to the respondent Department of Labor's rule-making
authority vested in it by the Labor Code. The petitioner assumes that it is
unreasonable simply because of its impact on the right to travel, but as we have
stated, the right itself is not absolute. The disputed Order is a valid qualification
thereto.

(3) No.

Page 40 of 323
The Labor Code itself vests the Department of Labor and Employment with rule-
making powers in the enforcement whereof. It is true that police power is the
domain of the legislature, but it does not mean that such an authority may not
be lawfully delegated.

*Himagan v. People, G.R. No. 113811, October 7, 1994.


(Equal protection of the laws: valid classification)

Facts:
-Himagan was a policeman charged with murder and attempted murder before the
RTC. In the course thereof, the trial court issued an Order suspending him until the
termination of the case on the basis of Section 47, R.A. 6975 (Department of
Interior and Local Government Act of 1990), to wit:

Sec. 47. Preventive Suspension Pending Criminal Case. — Upon the filing of a complaint or information
sufficient in form and substance against a member of the PNP for grave felonies where the penalty
imposed by law is six (6) years and one (1) day or more, the court shall immediately suspend the
accused from office until the case is terminated. Such case shall be subject to continuous trial and
shall be terminated within ninety (90) days from arraignment of the accused.

-Himagan filed a motion to lift the order for his suspension, relying on Section 42
of P.D. 807 of the Civil Service Decree, that his suspension should be limited to ninety
(90) days. The RTC denied the same.

-Himagan went to the SC via petition for certiorari and mandamus to set aside the
orders of respondent Judge and to command him to lift petitioner's preventive
suspension.

-Contentions of Himagan: (1) while the first sentence of Sec. 47 provides that the
accused who is charged with grave felonies where the penalty imposed is six years and
one day shall be suspended from office "until the case is terminated", the second
sentence of the same section mandates that the case, which shall be subject to
continuous trial, shall be terminated within 90 days from the arraignment of the
accused; (2) an imposition of preventive suspension of over 90 days is contrary to the
Civil Service Law and would be a violation of his constitutional right to equal
protection of laws.

-Sec. 42 of PD 807 (Civil Service Decree), which limits the maximum period of
suspension to ninety (90) days, provides:

Sec. 42. Lifting of Preventive Suspension Pending Administrative Investigation. — When the
administrative case against the officer or employee under preventive suspension is not finally decided by
the disciplining authority within the period of ninety (90) days after the date of suspension of the
respondent who is not a presidential appointee, the respondent shall be automatically reinstated in
the service; Provided, That when the delay in the disposition of the case is due to the fault, negligence or
petition of the respondent, the period of delay shall not be counted in computing the period of suspension
herein provided.

Issue:
Whether or not Section 47 of R.A. 6975 violates Himagan’s constitutional right to
equal protection of the laws.

Ruling:
No.

First. The language of the first sentence of Sec. 47 of R.A. 6975 is clear, plain
and free from ambiguity. It gives no other meaning than that the suspension from
office of the member of the PNP charged with grave offense where the penalty is
six years and one day or more shall last until the termination of the case. The
suspension cannot be lifted before the termination of the case.

Page 41 of 323
The second sentence of the same Section providing that the trial must be
terminated within ninety (90) days from arraignment does not qualify or limit
the first sentence. The two can stand independently of each other. The first refers
to the period of suspension. The second deals with the time frame within which
the trial should be finished.

Suppose the trial is not terminated within ninety days from arraignment, should the
suspension of accused be lifted? The answer is certainly no. While the law uses the
mandatory word "shall" before the phrase "be terminated within ninety (90)
days", there is nothing in R.A. 6975 that suggests that the preventive
suspension of the accused will be lifted if the trial is not terminated within that
period. Nonetheless, the Judge who fails to decide the case within the period
without justifiable reason may be subject to administrative sanctions and, in
appropriate cases where the facts so warrant, to criminal or civil liability. If the trial is
unreasonably delayed without fault of the accused such that he is deprived of his
right to a speedy trial, he is not without a remedy. He may ask for the dismissal of
the case. Should the court refuse to dismiss the case, the accused can compel its
dismissal by certiorari, prohibition or mandamus, or secure his liberty by habeas
corpus.

Second. Petitioner misapplies Sec. 42 of PD 807. A meticulous reading of the


section clearly shows that it refers to the lifting of preventive suspension in
pending administrative investigation, not in criminal cases, as here. What is
more, Section 42 expressly limits the period of preventive suspension to ninety (90)
days. Sec. 91 of R.A. 6975 which states that "The Civil Service Law and its
implementing rules shall apply to all personnel of the Department" simply means that
the provisions of the Civil Service Law and its implementing rules and
regulations are applicable to members of the Philippine National Police insofar as
the provisions, rules and regulations are not inconsistent with R.A. 6975.
Certainly, Section 42 of the Civil Service Decree which limits the preventive
suspension to ninety (90) days cannot apply to members of the PNP because Sec. 47
of R.A. 6995 provides differently, that is, the suspension where the penalty imposed
by law exceeds six (6) years shall continue until the case is terminated. x x x x

The reason why members of the PNP are treated differently from the other
classes of persons charged criminally or administratively insofar as the application
of the rule on preventive suspension is concerned is that policemen carry weapons
and the badge of the law which can be used to harass or intimidate witnesses
against them. If a suspended policeman criminally charged with a serious offense is
reinstated to his post while his case is pending, his victim and the witnesses
against him are obviously exposed to constant threat and thus easily cowed to
silence by the mere fact that the accused is in uniform and armed. The imposition
of preventive suspension for over 90 days under Section 47 of R.A. 6975 does not
violate the suspended policeman's constitutional right to equal protection of the laws.
x x x x A distinction based on real and reasonable considerations related to a
proper legislative purpose such as that which exists here is neither
unreasonable, capricious nor unfounded.

*International School Alliance of Educators v. Quisumbing, G.R. No. 128845,


June 1, 2000.
(Equal protection of the laws)

Facts:
-International School, Inc. (the School) is a domestic educational institution
established primarily for dependents of foreign diplomatic personnel and other
temporary residents.

-The School hires both foreign and local teachers as members of its faculty, classifying
the same into two: (1) foreign-hires and (2) local-hires.

Page 42 of 323
-The School grants foreign-hires certain benefits not accorded local-hires. These
include housing, transportation, shipping costs, taxes, and home leave travel
allowance. Foreign-hires are also paid a salary rate twenty-five percent (25%) more
than local-hires.

-The School justifies the difference on two “significant economic disadvantages”


foreign-hires have to endure, namely: (a) the “dislocation factor” and (b) limited
tenure.

-Dispute arose and eventually, the DOLE Acting Secretary ruled in favor of the School.

-Petitioner International School Alliance of Educators, a legitimate labor union and


the collective bargaining representative of all faculty members of the School, went to
the SC for relief.

-Petitioner’s contention: the point-of-hire classification employed by the School is


discriminatory to Filipinos and that the grant of higher salaries to foreign-hires
constitutes racial discrimination.

-The School’s contention: there are foreigners who have been hired locally and who
are paid equally as Filipino local hires.

Issue:
Whether or not the lesser salary rate given to local-hires vis-à-vis foreign-hires violates
the equal protection clause.

Ruling:
Yes.

The point-of-hire classification employed by respondent School to justify the


distinction in the salary rates of foreign-hires and local hires is an invalid
classification. There is no reasonable distinction between the services rendered
by foreign-hires and local-hires. The practice of the School of according higher
salaries to foreign-hires contravenes public policy and, certainly, does not deserve the
sympathy of this Court.

This jurisdiction has the long honored legal truism of “equal pay for equal work.”
Persons who work with substantially equal qualifications, skill, effort and
responsibility, under similar conditions, should be paid similar salaries. This rule
applies to the School, its "international character" notwithstanding.

The School contends that petitioner has not adduced evidence that local-hires perform
work equal to that of foreign-hires. The Court finds this argument a little cavalier. If
an employer accords employees the same position and rank, the presumption is
that these employees perform equal work. This presumption is borne by logic and
human experience. If the employer pays one employee less than the rest, it is not
for that employee to explain why he receives less or why the others receive
more. That would be adding insult to injury. The employer has discriminated
against that employee; it is for the employer to explain why the employee is
treated unfairly.

The employer in this case has failed to discharge this burden. There is no evidence
here that foreign-hires perform 25% more efficiently or effectively than the
local-hires. Both groups have similar functions and responsibilities, which they
perform under similar working conditions. x x x x

While we recognize the need of the School to attract foreign-hires, salaries


should not be used as an enticement to the prejudice of local-hires. The local-
hires perform the same services as foreign-hires and they ought to be paid the same
salaries as the latter. For the same reason, the “dislocation factor” and the foreign-
hires’ limited tenure also cannot serve as valid bases for the distinction in salary

Page 43 of 323
rates. The dislocation factor and limited tenure affecting foreign-hires are
adequately compensated by certain benefits accorded them which are not
enjoyed by local-hires, such as housing, transportation, shipping costs, taxes and
home leave travel allowances.

*Biraogo v. The Philippine Truth Commission, G.R. No. 192935, December 7, 2010.
(Equal protection of the laws)

Facts:
-When Senator Benigno Simeon Aquino III (PNoy) catapulted to presidency, he found a
need for a special body to investigate reported cases of graft and corruption allegedly
committed during the previous administration.

-In view thereof, he signed Executive Order No. 1 (E.O. 1) establishing the Philippine
Truth Commission of 2010 (PTC).

-(PTC) is a mere ad hoc body formed under the Office of the President with the primary
task to investigate reports of graft and corruption committed by third-level public
officers and employees, their co-principals, accomplices and accessories during the
previous administration, and thereafter to submit its finding and recommendations to
the President, Congress and the Ombudsman.

-Petitioners, as incumbent members of the House of Representatives, then went to the


SC to have E.O. 1 declared unconstitutional.

Issues:
(1) Whether or not Executive Order No. 1 violates the principle of separation of powers
by usurping the powers of Congress to create and to appropriate funds for public
offices, agencies and commissions;
(2) Whether or not Executive Order No. 1 violates the equal protection clause;
(3) Whether or not Executive Order No. 1 supplants the powers of the Ombudsman
and the DOJ.

Ruling:
(1) No.

The President’s power to conduct investigations to aid him in ensuring the


faithful execution of laws—in this case, fundamental laws on public
accountability and transparency—is inherent in the President’s powers as the
Chief Executive. That the authority of the President to conduct investigations and to
create bodies to execute this power is not explicitly mentioned in the Constitution or in
statutes does not mean that he is bereft of such authority. x x x x

Indeed, the Executive is given much leeway in ensuring that our laws are
faithfully executed. As stated above, the powers of the President are not limited
to those specific powers under the Constitution. One of the recognized powers of
the President granted pursuant to this constitutionally-mandated duty is the
power to create ad hoc committees. This flows from the obvious need to ascertain
facts and determine if laws have been faithfully executed. x x x x

It should be stressed that the purpose of allowing ad hoc investigating bodies to exist
is to allow an inquiry into matters which the President is entitled to know so that he
can be properly advised and guided in the performance of his duties relative to the
execution and enforcement of the laws of the land. x x x x

On the charge that Executive Order No. 1 transgresses the power of Congress to
appropriate funds for the operation of a public office, suffice it to say that there
will be no appropriation but only an allotment or allocations of existing funds
already appropriated. Accordingly, there is no usurpation on the part of the
Executive of the power of Congress to appropriate funds.

Page 44 of 323
(2) Yes.

For a classification to meet the requirements of constitutionality, it must include or


embrace all persons who naturally belong to the class. The classification will be
regarded as invalid if all the members of the class are not similarly treated, both
as to rights conferred and obligations imposed. x x x x

The classification must not be based on existing circumstances only, or so


constituted as to preclude addition to the number included in the class. It must
be of such a nature as to embrace all those who may thereafter be in similar
circumstances and conditions. It must not leave out or “underinclude” those that
should otherwise fall into a certain classification.

Executive Order No. 1 should be struck down as violative of the equal protection
clause. The clear mandate of the envisioned truth commission is to investigate
and find out the truth “concerning the reported cases of graft and corruption
during the previous administration” only. The intent to single out the previous
administration is plain, patent and manifest. Mention of it has been made in at
least three portions of the questioned executive order.

In this regard, it must be borne in mind that the Arroyo administration is but just a
member of a class, that is, a class of past administrations. It is not a class of its
own. Not to include past administrations similarly situated constitutes
arbitrariness which the equal protection clause cannot sanction. Such
discriminating differentiation clearly reverberates to label the commission as a
vehicle for vindictiveness and selective retribution.

Though the OSG enumerates several differences between the Arroyo administration
and other past administrations, these distinctions are not substantial enough to merit
the restriction of the investigation to the “previous administration” only. The reports
of widespread corruption in the Arroyo administration cannot be taken as basis
for distinguishing said administration from earlier administrations which were
also blemished by similar widespread reports of impropriety. They are not inherent
in, and do not inure solely to, the Arroyo administration.

The public needs to be enlightened why Executive Order No. 1 chooses to limit the
scope of the intended investigation to the previous administration only. The OSG
ventures to opine that, “to include other past administrations, at this point, may
unnecessarily overburden the commission and lead it to lose its effectiveness.” The
reason given is specious. It is without doubt irrelevant to the legitimate and noble
objective of the PTC to stamp out or “end corruption and the evil it breeds.”

Given the foregoing physical and legal impossibility, the Court logically recognizes the
unfeasibility of investigating almost a century’s worth of graft cases. However, the fact
remains that Executive Order No. 1 suffers from arbitrary classification. The
PTC, to be true to its mandate of searching for the truth, must not exclude the
other past administrations. The PTC must, at least, have the authority to
investigate all past administrations. While reasonable prioritization is permitted, it
should not be arbitrary lest it be struck down for being unconstitutional.

(3) No.

Contrary to petitioners’ apprehension, the PTC will not supplant the Ombudsman
or the DOJ or erode their respective powers. If at all, the investigative function of
the commission will complement those of the two offices. As pointed out by the
Solicitor General, the recommendation to prosecute is but a consequence of the
overall task of the commission to conduct a fact-finding investigation." The
actual prosecution of suspected offenders, much less adjudication on the merits
of the charges against them, is certainly not a function given to the commission.
The phrase, "when in the course of its investigation," under Section 2(g), highlights
this fact and gives credence to a contrary interpretation from that of the petitioners.

Page 45 of 323
The function of determining probable cause for the filing of the appropriate
complaints before the courts remains to be with the DOJ and the Ombudsman.

At any rate, the Ombudsman’s power to investigate under R.A. No. 6770 is not
exclusive but is shared with other similarly authorized government agencies. x x x x

Finally, nowhere in Executive Order No. 1 can it be inferred that the findings of
the PTC are to be accorded conclusiveness. Much like its predecessors, the Davide
Commission, the Feliciano Commission and the Zenarosa Commission, its findings
would, at best, be recommendatory in nature. And being so, the Ombudsman and
the DOJ have a wider degree of latitude to decide whether or not to reject the
recommendation.

[NOTE: This case was decided by Justices who were mostly appointed by Arroyo. So….
BLEH!]

*Sabio v. Gordon, G.R. No. 174340, October 17, 2006.


(Right to privacy; Inquiries in aid of legislation)

Facts:
-Sen. Gordon wrote PCGG Chairman Camilo L. Sabio inviting him to be one of the
resource persons in the public meeting.

-Chairman Sabio declined the invitation, invoking Section 4(b) of E.O. No. 1.

-Section 4 (b) of E.O. No. 1, as issued by then President Cory Aquino, provides that:

“No member or staff of the Commission shall be required to testify or produce evidence in any
judicial, legislative or administrative proceeding concerning matters within its official cognizance.”

-On the other hand, Article VI, Section 21 of the 1987 Constitution provides:

“The Senate or the House of Representatives or any of its respective committees may conduct
inquiries in aid of legislation in accordance with its duly published rules of procedure. The rights of
persons appearing in or affected by such inquiries shall be respected.”

-The Committee then issued an arrest order against Sabio. The Order bore the
approval of Senate President Villar and the majority of the Committees’ members.

-Sabio was arrested and was brought to the Senate premises where he was detained.

-Chairman Sabio then filed with the SC a petition for habeas corpus against the
Senate Committee.

-Sabio’s contentions: (1) The Senate Committees disregarded Section 4(b) of E.O. No. 1
without any justifiable reason; (2) The inquiries conducted by respondent Senate
Committees are not in aid of legislation; (3) The inquiries were conducted in the
absence of duly published Senate Rules of Procedure Governing Inquiries in Aid of
Legislation; and (4) Respondent Senate Committees are not vested with the power of
contempt.

Issue:
Whether or not the Senate committee has the authority to arrest and detain Sabio
notwithstanding Section 4 (b) of E.O. No. 1 exempting all PCGG members or staff from
testifying in any judicial, legislative or administrative proceeding.

Ruling:
Yes.

Considering these jurisprudential instructions, we find Section 4(b) directly


repugnant with Article VI, Section 21. Section 4(b) exempts the PCGG members and
staff from the Congress' power of inquiry. This cannot be countenanced. Nowhere in

Page 46 of 323
the Constitution is any provision granting such exemption. The Congress’ power
of inquiry, being broad, encompasses everything that concerns the
administration of existing laws as well as proposed or possibly needed statutes. It
even extends "to government agencies created by Congress and officers whose
positions are within the power of Congress to regulate or even abolish." PCGG belongs
to this class.

Certainly, a mere provision of law cannot pose a limitation to the broad power of
Congress, in the absence of any constitutional basis. x x x x

Section 4(b), being in the nature of an immunity, is inconsistent with the


principle of public accountability. It places the PCGG members and staff beyond
the reach of courts, Congress and other administrative bodies. Instead of
encouraging public accountability, the same provision only institutionalizes
irresponsibility and non-accountability.

Furthermore, Section 4(b) is also inconsistent with Article XI, Section 1 of the
Constitution stating that: "Public office is a public trust. Public officers and
employees must at all times be accountable to the people, serve them with utmost
responsibility, integrity, loyalty, and efficiency, act with patriotism and justice, and
lead modest lives."

*Laud v. People, G.R. No. 199032, November 19, 2014. [PROBABLE BAR
QUESTION!!!]
(Searches and Seizures; Crim Pro: Procedural Rules for Search Warrant)

Facts:
-In 2009, the PNP applied with the Regional Trial Court (RTC) of Manila, Branch 50
(Manila-RTC) for a warrant to search three (3) caves located inside the Laud
Compound in Purok 3, Barangay Ma-a, Davao City, where the alleged remains of the
victims summarily executed by the so-called “Davao Death Squad” may be found.

-In support of the application, a certain Ernesto Avasola (Avasola) was presented to
the RTC and there testified that he personally witnessed the killing of six (6) persons
in December 2005, and was, in fact, part of the group that buried the victims.

-The Manila-RTC, presided by Judge Peralta (who was earlier administratively


penalized with fines of ₱15,000.00 and ₱5,000.00 in administrative cases against
him), found probable cause for the issuance of a search warrant, and thus, issued
Search Warrant No. 09-144077 which was later enforced by the elements of the PNP-
CIDG on July 15, 2009. The search of the Laud Compound caves yielded positive
results for the presence of human remains.

-Five days after the search, retired SPO4 Bienvenido Laud (Laud), filed an Urgent
Motion to Quash and to Suppress Illegally Seized Evidence.

-Laud’s contentions: (1) Judge Peralta had no authority to act on the application for a
search warrant since he had been automatically divested of his position as Vice
Executive Judge when several administrative penalties were imposed against him by
the Court; (2) the Manila-RTC had no jurisdiction to issue Search Warrant No. 09-
14407 which was to be enforced in Davao City; (3) human remains are not “personal
property” and, hence, could not be the subject of a search warrant; (4) the search
warrant was issued despite lack of probable cause.

-The RTC-Manila granted Laud’s motion. The People appealed to the CA, then to the
SC via Rule 45.

Issues:
(1) Whether or not the administrative penalties imposed on Judge Peralta invalidated
Search Warrant No. 09-14407;

Page 47 of 323
(2) Whether or not the Manila-RTC had jurisdiction to issue the said warrant despite
non-compliance with the compelling reasons requirement under Section 2, Rule126 of
the Rules of Court;
(3) Whether or not human remains are “personal property” and, hence, could be the
subject of a search warrant.

Ruling:
(1) No.

While the Court does agree that the imposition of said administrative penalties did
operate to divest Judge Peralta’s authority to act as Vice Executive Judge, it
must be qualified that the abstraction of such authority would not, by and of
itself, result in the invalidity of Search Warrant No. 09-14407 considering that
Judge Peralta may be considered to have made the issuance as a de facto officer
whose acts would, nonetheless, remain valid.

Funa v. Agra defines who a de facto officer is and explains that his acts are just as
valid for all purposes as those of a de jure officer, in so far as the public or third
persons who are interested therein are concerned, viz.:

A de facto officer is one who derives his appointment from one having colorable
authority to appoint, if the office is an appointive office, and whose appointment is
valid on its face. He may also be one who is in possession of an office, and is
discharging [his] duties under color of authority, by which is meant authority derived
from an appointment, however irregular or informal, so that the incumbent is not a
mere volunteer. Consequently, the acts of the de facto officer are just as valid for all
purposes as those of a de jure officer, in so far as the public or third persons who are
interested therein are concerned.

The treatment of a de facto officer’s acts is premised on the reality that third
persons cannot always investigate the right of one assuming to hold an
important office and, as such, have a right to assume that officials apparently
qualified and in office are legally such. Public interest demands that acts of persons
holding, under color of title, an office created by a valid statute be, likewise, deemed
valid insofar as the public – as distinguished from the officer in question – is
concerned. Indeed, it is far more cogently acknowledged that the de facto doctrine has
been formulated, not for the protection of the de facto officer principally, but rather for
the protection of the public and individuals who get involved in the official acts of
persons discharging the duties of an office without being lawful officers.

In order for the de facto doctrine to apply, all of the following elements must
concur: (a) there must be a de jure office; (b) there must be color of right or
general acquiescence by the public; and (c) there must be actual physical
possession of the office in good faith.

(2) Yes.

As explicitly mentioned in Section 12, Chapter V of A.M. No. 03-8-02-SC, the rule
on search warrant applications before the Manila and Quezon City RTCs for the
above-mentioned special criminal cases “shall be an exception to Section 2 of Rule
126 of the Rules of Court.”

Perceptibly, the fact that a search warrant is being applied for in connection with
a special criminal case as above-classified already presumes the existence of a
compelling reason; hence, any statement to this effect would be superfluous and
therefore should be dispensed with. By all indications, Section 12, Chapter V of
A.M. No. 03-8-02-SC allows the Manila and Quezon City RTCs to issue warrants
to be served in places outside their territorial jurisdiction for as long as the
parameters under the said section have been complied with, as in this case.
Thus, on these grounds, the Court finds nothing defective in the preliminary issuance

Page 48 of 323
of Search Warrant No. 09-14407. Perforce, the RTC-Manila should not have
overturned it. x x x x

Section 12, Chapter V of A.M.No. 03-8-02-SC states the requirements for the issuance
of search warrants in special criminal cases by the RTCs of Manila and Quezon City.
These special criminal cases pertain to those “involving heinous crimes, illegal
gambling, illegal possession of firearms and ammunitions, as well as violations of
the Comprehensive Dangerous Drugs Act of 2002, the Intellectual Property Code,
the Anti-Money Laundering Act of 2001, the Tariff and Customs Code, as
amended, and other relevant laws that may hereafter be enacted by Congress,
and included herein by the Supreme Court.” Search warrant applications for such
cases may be filed by “the National Bureau of Investigation (NBI), the Philippine
National Police (PNP) and the AntiCrime Task Force (ACTAF),” and “personally
endorsed by the heads of such agencies.”

As the records would show, the search warrant application was filed before the
Manila-RTC by the PNP and was endorsed by its head, PNP Chief Jesus Ame Versosa,
particularly describing the place to be searched and the things to be seized in
connection with the heinous crime of Murder. Finding probable cause therefor, Judge
Peralta, in his capacity as 2nd Vice-Executive Judge, issued Search Warrant No. 09-
14407 which, as the rules state, may be served in places outside the territorial
jurisdiction of the said RTC.

(3) Yes.

Section 3, Rule 126 of the Rules of Court states:

SEC. 3.Personal property to be seized. – A search warrant may be issued for the search and seizure of
personal property:

(a) Subject of the offense;


(b) Stolen or embezzled and other proceeds, or fruits of the offense; or
(c) Used or intended to be used as the means of committing an offense.

“Personal property” in the foregoing context actually refers to the thing’s


mobility, and not to its capacity to be owned or alienated by a particular person.
Article 416 of the Civil Code, which Laud himself cites, states that in general, all
things which can be transported from place to place are deemed to be personal
property. Considering that human remains can generally be transported from
place to place, and considering further that they qualify under the phrase
“subject of the offense” given that they prove the crime’s corpus delicti, it
follows that they may be valid subjects of a search warrant under the above-cited
criminal procedure provision.

*Garaygay v. People, G.R. No. 135503, July 6, 2000.


(Searches and Seizures; Crim Pro: Procedural Rules for Search Warrant)

Facts:
-Presidential Task Force on Intelligence and Counter-Intelligence (PTFIC) applied
before the RTC-Manila for a search warrant against Garaygay whose residence was in
Lapu-lapu.

-The RTC-Manila granted the application. A raid on the house of Garaygay resulted in
the seizure of several items of firearms, explosives, ammunition and other
prohibited paraphernalia.

-An Information for violation of PD 1866 was filed before the RTC of Lapu-Lapu City
against Garaygay who upon being arraigned pleaded not guilty.

-Subsequently, Garaygay filed with the RTC of Lapu-Lapu City a Motion to Quash
Search Warrant and To Exclude Illegally Seized Evidence on the ground that the

Page 49 of 323
search warrant was issued in violation of Supreme Court Circular No. 19, and that it
was a general warrant.

-On the other hand, the prosecution argued that the motion to quash should have
been filed with the RTC of Manila which issued the warrant.

-The RTC of Lapu-Lapu City denied the said motion on the ground that should have
been filed with the RTC of Manila which issued the warrant.

Issue:
Whether or not the RTC of Lapu-Lapu City where the criminal case was filed is clothed
with authority to resolve the Motion to Quash Search Warrant.

Ruling:
Yes.

Where a search warrant is issued by one court and the criminal action based on the
results of the search is afterwards commenced in another court, it is not the rule that
a motion to quash the warrant (or to retrieve things thereunder seized) may be filed
only with the issuing Court. Such a motion may be filed for the first time in either
the issuing Court or that in which the criminal action is pending. However, the
remedy is alternative, not cumulative. The Court first taking cognizance of the
motion does so to the exclusion of the other, and the proceedings thereon are
subject to the Omnibus Motion Rule and the rule against forum-shopping. This is
clearly stated in the third policy guideline which indeed is what properly applies to the
case at bar, to wit:

“3. Where no motion to quash the search warrant was filed in or resolved by the issuing court, the
interested party may move in the court where the criminal case is pending for the suppression as
evidence of the personal property seized under the warrant if the same is offered therein for said
purpose. Since two separate courts with different participations are involved in this situation, a motion
to quash a search warrant and a motion to suppress evidence are alternative and not cumulative
remedies. In order to prevent forum shopping, a motion to quash shall consequently be governed by
the omnibus motion rule, provided, however, that objections not available, existent or known during the
proceedings for the quashal of the warrant may be raised in the hearing of the motion to suppress. The
resolution of the court on the motion to suppress shall likewise be subject to any proper remedy in the
appropriate higher court.”

*People v. Dichoso, G.R. Nos. 101216-18, June 4, 1993.


(Search Warrant: must refer to one specific offense)

Facts:
-Upon application by PNP officers, a search warrant was issued against Dichoso “for
Violation of R.A. 6425 known as the Dangerous Drugs Act of 1972 as amended.”

-The said search warrant authorized the seizure of: “marijuana leaves, shabu and sets
of paraphernalia and bring the same to the undersigned to be dealt with as the law
directs.”

-When police officers conducted the search on Dichoso’s place, they discovered 200
grams more or less of marijuana wrapped in plastic inside a cabinet as well as six (6)
decks of shabu wrapped in an aluminum foil.

-Dichoso’s contention: The search warrant was a general warrant since illegal
possession of shabu, illegal possession of marijuana and illegal possession of
paraphernalia are covered by different articles and sections of the Dangerous
Drugs Act of 1972, and hence, the search warrant is clearly for more than one (1)
specific offense.

Issue:
Whether or not the search warrant herein is a “scatter-shot” warrant.

Ruling:

Page 50 of 323
No.

Dichoso’s contention that the search warrant in question was issued for more than
one (1) offense, hence, in violation of Section 3, Rule 126 of the Rules of Court, is
unpersuasive. He engages in semantic juggling by suggesting that since illegal
possession of shabu, illegal possession of marijuana and illegal possession of
paraphernalia are covered by different articles and sections of the Dangerous Drugs
Act of 1972, the search warrant is clearly for more than one (1) specific offense. In
short, following this theory, there should have been three (3) separate search
warrants, one for illegal possession of shabu, the second for illegal possession of
marijuana and the third for illegal possession of paraphernalia. This argument is
pedantic. The Dangerous Drugs Act of 1972 is a special law that deals
specifically with dangerous drugs which are subsumed into “prohibited” and
“regulated” drugs and defines and penalizes categories of offenses which are
closely related or which belong to the same class or species. Accordingly, one (1)
search warrant may thus be validly issued for the said violations of the
Dangerous Drugs Act.

*Kho v. Judge Makalintal, G.R. No. 94902-0 April 21, 1999.


(Search warrant: Particularity of description)

Facts:
-The search warrant against Kho was for: “Unlicensed firearms of various calibers
and ammunitions for the said firearms.”

-Kho’s contention: the things to be seized were not described and detailed out, i.e. the
firearms listed were not classified as to size or make, etc.

Issue:
Whether or not the subject search warrant is a general warrant.

Ruling:
No.

The Court believes, and so holds, that the said warrants comply with Constitutional
and statutory requirements. The law does not require that the things to be seized
must be described in precise and minute detail as to leave no room for doubt on
the part of the searching authorities. Otherwise, it would be virtually impossible
for the applicants to obtain a warrant as they would not know exactly what kind
of things they are looking for. Since the element of time is very crucial in criminal
cases, the effort and time spent in researching on the details to be embodied in the
warrant would render the purpose of the search nugatory.

In the case under consideration, the NBI agents could not have been in a position
to know before hand the exact caliber or make of the firearms to be seized.
Although the surveillance they conducted did disclose the presence of unlicensed
firearms within the premises to be searched, they could not have known the
particular type of weapons involved before seeing such weapons at close range,
which was of course impossible at the time of the filing of the applications for subject
search warrants.

Verily, the failure to specify detailed descriptions in the warrants did not render
the same general. Retired Justice Ricardo Francisco's book on Criminal Procedure
has this useful insight:

“A description of the property to be seized need not be technically accurate nor


necessarily precise; and its nature will necessarily vary according to whether the
identity of the property, or its character, is the matter of concern. Further, the
description is required to be specific only so far as the circumstances will
ordinarily allow.”

Page 51 of 323
*Uy v. BIR, G.R. No. 129651, October 20, 2000. [IMPORTANT!!!]
(Search warrant; Particularity of description; Severable)

Facts:
-BIR officers applied for search warrants from Branch 28 of the Regional Trial Court
of Cebu against Unifish Packing Corporation and Frank Uy. The RTC granted the
same and issued a search warrant “For: VIOLATION OF SEC. 253” (of the National
Internal Revenue Code), authorizing the seizure of the following:

“1. Multiple sets of Books of Accounts; Ledgers, Journals, Columnar Books, Cash
Register Books, Sales Books or Records; Provisional & Official Receipts;
2. Production Record Books/Inventory Lists, Stock Cards;
3. Unregistered Delivery Receipts;
4. Unregistered Purchase & Sales Invoices;
5. Sales Records, Job Order;
6. Corporate Financial Records; and
7. Bank Statements/Cancelled Checks.”

-Petitioners contend that the said search warrant, being a general warrant, is invalid.

Issues:
(1) Whether or not the herein search warrant is a general warrant;
(2) Whether or not the herein search warrant is severable.

Ruling:
(1) Yes.

We agree that most of the items listed in the warrants fail to meet the test of
particularity, especially since witness Abos had furnished the judge photocopies
of the documents sought to be seized. The issuing judge could have formed a
more specific description of these documents from said photocopies instead of
merely employing a generic description thereof. The use of a generic term or a
general description in a warrant is acceptable only when a more specific
description of the things to be seized is unavailable. The failure to employ the
specificity available will invalidate a general description in a warrant. The use by
the issuing judge of the terms “multiple sets of books of accounts, ledgers, journals,
columnar books, cash register books, sales books or records, provisional & official
receipts,” “production record books/inventory lists, stock cards,” “sales records, job
order,” “corporate financial records,” and "bank statements/cancelled checks" is
therefore unacceptable considering the circumstances of this case. x x x x

The warrant authorized the search for and seizure of records pertaining to all
business transactions of petitioners herein, regardless of whether the
transactions were legal or illegal. The warrants sanctioned the seizure of all
records of the petitioners and the aforementioned corporation, whatever their
nature, thus openly contravening the explicit command of our Bill of Rights -
that the things to be seized be particularly described - as well as tending to
defeat its major object: the elimination of general warrants.

A search warrant may be said to particularly describe the things to be seized when the
description therein is as specific as the circumstances will ordinarily allow; or when
the description expresses a conclusion of fact - not of law - by which the warrant
officer may be guided in making the search and seizure; or when the things described
are limited to those which bear direct relation to the offense for which the warrant is
being issued (Sec. 2, Rule 126, Revised Rules of Court). The herein search warrant
does not conform to any of the foregoing tests. If the articles desired to be seized
have any direct relation to an offense committed, the applicant must necessarily
have some evidence, other than those articles, to prove the said offense; and the
articles subject of search and seizure should come in handy merely to strengthen
such evidence. In this event, the description contained in the herein disputed
warrant should have mentioned, at least, the dates, amounts, persons, and other

Page 52 of 323
pertinent data regarding the receipts of payments, certificates of stocks and
securities, contracts, promissory notes, deeds of sale, messages and
communications, checks, bank deposits and withdrawals, records of foreign
remittances, among others, enumerated in the warrant.

In Asian Surety & Insurance Co., Inc. vs. Herrera, the description of the things to be
seized, i.e., "Fire Registers, Loss, Bordereau, Adjusters' Report, including subrogation
receipts and proof of loss, Loss Registers, Book of Accounts including cash receipts
and disbursements and general ledger, etc." was held to be "an omnibus description"
and, therefore, invalid:

x x x Because of this all embracing description which includes all conceivable


records of petitioner corporation, which if seized x x x, could paralyze its business,
petitioner in several motions filed for early resolution of this case, manifested that the
seizure of TWO carloads of their papers has paralyzed their business to the grave
prejudice of not only the company, its workers, agents, employees but also of its
numerous insured and beneficiaries of bonds issued by it, including the government
itself, and of the general public. And correlating the same to the charges for which the
warrant was issued, We have before Us the infamous general warrants of old. x x x x

As regards the terms "unregistered delivery receipts" and "unregistered


purchase & sales invoices," however, we hold otherwise. The Solicitor General
correctly argues that the serial markings of these documents need not be specified as
it is not possible to do so precisely because they are unregistered. Where, by the
nature of the goods to be seized, their description must be rather general, it is
not required that a technical description be given, as this would mean that no
warrant could issue. Taking into consideration the nature of the articles so
described, it is clear that no other more adequate and detailed description could
have been given, particularly because it is difficult to give a particular
description of the contents thereof. Although it appears that photocopies of these
unregistered documents were among those handed by Abos to the issuing judge, it
would be impractical to require the latter to specify each and every receipt and
invoice, and the contents thereof, to the minutest detail.

(2) Yes.

The general description of most of the documents listed in the warrants does
not render the entire warrant void. Insofar as the warrants authorize the search
and seizure of unregistered delivery receipts and unregistered purchase and
sales invoices, the warrants remain valid. The search warrant is severable, and
those items not particularly described may be cut off without destroying the
whole warrant.

In United States v. Cook, the United States Court of Appeals (Fifth Circuit) made the
following pronouncement:

x x x The leading decision is Aday v. Superior Court. In Aday, a warrant was issued
authorizing the seizure of two particularly described books and myriad other generally
described items. On appeal, the California Supreme Court held that only the books
were particularly described in the warrant and lawfully seized. The court
acknowledged that the warrant was flawed, but rather than suppress everything
seized, the court chose to sever the defective portions of the warrant and suppress
only those items that were not particularly described.

Although the warrant was defective x x x it does not follow that it was invalid as a
whole. Such a conclusion would mean that the seizure of certain articles, even though
proper if viewed separately, must be condemned merely because the warrant was
defective with respect to other articles. The invalid portions of the warrant are
severable from the authorization relating to the named books x x x. The search for
and seizure of these books, if otherwise valid, were not rendered illegal by the defects
concerning other articles.

Page 53 of 323
*Veroy v. Layague, G.R. No. L-95630, June 18, 1992.
(Search Warrant; Consent)

Facts:
-On reports that the residence of spouses Veroy (petitioners) in Davao City was being
used as a safehouse of rebel soldiers, police officers decided to search the same.

-They were able to enter the yard with the help of the caretakers but did not enter the
house since the owners were not present and they did not have a search warrant.
Petitioner Ma. Luisa was contacted by telephone in her Quezon City residence by
Capt. Obrero to ask permission to search the house in Davao City as it was reportedly
being used as a hideout and recruitment center of rebel soldiers.

-Petitioner Ma. Luisa gave her consent for the police officers to search the house for
alleged NPA rebels, on the condition that the search would be conducted in the
presence of Major Ernesto Macasaet, a family friend.

-The following day, police officers and Major Macasaet met at the house of herein
petitioners for the search. The caretakers facilitated their entry into the yard, and
using the key entrusted to the househelp, they were able to gain entrance into the
kitchen. However, a locksmith by the name of George Badiang had to be employed to
open the padlock of the door leading to the children's room.

-Capt. Obrero recovered a .45 caliber handgun with a magazine containing seven (7)
live bullets in a black clutch bag inside an unlocked drawer. Three (3) half-full jute
sacks containing printed materials of RAM-SFP were also found in the children's
room. A search of the children's recreation and study area revealed a big travelling bag
containing assorted polo shirts, men's brief, two (2) pieces polo barong and short
sleeve striped gray polo, sweat shirt, two (2) pairs men's socks, a towel made in U.S.A.,
one blanket, a small black bag, Gandhi brand, containing a book entitled "Islamic
Revolution Future Path of the Nation", a road map of the Philippines, a telescope, a
plastic bag containing assorted medicines and religious pamphlets was found in the
master bedroom.

-Based on the said confiscated items, petitioners were charged with Violation of
Presidential Decree No. 1866 (Illegal Possession of Firearms and Ammunitions in
Furtherance of Rebellion).

Issue:
Whether or not the firearms seized are admissible in evidence.

Ruling:
No.

The reason for searching the house of herein petitioners is that it was reportedly being
used as a hideout and recruitment center for rebel soldiers. While police officers
had permission from Ma. Luisa Veroy to break open the door of their residence,
it was merely for the purpose of ascertaining thereat the presence of the alleged
“rebel” soldiers. The permission did not include any authority to conduct a
room-to-room search once inside the house. The items taken were, therefore,
products of an illegal search, violative of their constitutional rights. x x x x

Permission was indeed granted by Ma. Luisa Veroy to enter the house but only to
ascertain the presence of rebel soldiers. Under the circumstances, it is undeniable
that the police officers had ample time to procure a search warrant but did not.

In a number of cases decided by this Court, (Guazon v. De Villa, supra.; People v.


Aminnudin, G.R. No. L-74869, July 6, 1988 [163 SCRA 402]; Alih v. Castro, G.R. No.
L-69401, June 23, 1987 [151 SCRA 279]), warrantless searches were declared
illegal because the officials conducting the search had every opportunity to

Page 54 of 323
secure a search warrant. The objects seized, being products of illegal searches, were
inadmissible in evidence in the criminal actions subsequently instituted against the
accused-appellants.

*Unilab v. Isip, G.R. No. 163858, June 28, 2005. [IMPORTANT!!!]


(Search warrant; Plain view doctrine)

Facts:
-NBI officers applied for a search warrant for violation of Section 4(a), in relation to
Section 8, of Republic Act (R.A.) No. 8203 against Isip.

-The warrant specifically authorized the officers only to seize “counterfeit Revicon
multivitamins, finished or unfinished, and the documents used in recording,
manufacture and/or importation, distribution and/or sale, or the offering for sale, sale
and/or distribution of the said vitamins.”

-The search warrant was implemented but no fake Revicon multivitamins were found;
instead, there were sealed boxes which, when opened by the NBI agents in the
presence of Isip, contained: (a) 792 bottles of Disudrin 60 ml.; and (b) 30 boxes (100
pieces each) of Inoflox 200 mg.

-Isip filed an “Urgent Motion to Quash the Search Warrant or to Suppress


Evidence,” arguing that the things seized, namely, Disudrin and Inoflox, were not
those described in the search warrant. The RTC granted the motion.

-Aggrieved, Unilab went to the SC via Rule 45.

-Unilab’s contention: the seizure of the items was justified by the plain view doctrine.

Issue:
Whether or not the counterfeit Disudrin and Inoflox are admissible in evidence.

Ruling:
No.

Since the articles allegedly seized during the implementation of the search
warrant – Disudrin and Inoflux products – were not included in the search
warrant, they were, therefore, not lawfully seized by the raiding team; they are
not illegal per se, as it were, like an arms cache, subversive materials or shabu as to
justify their seizure in the course of a lawful search, or being in plain view or some
such. x x x x

Objects, articles or papers not described in the warrant but on plain view of the
executing officer may be seized by him. However, the seizure by the officer of
objects/articles/papers not described in the warrant cannot be presumed as
plain view. The State must adduce evidence, testimonial or documentary, to prove the
confluence of the essential requirements for the doctrine to apply, namely: (a) the
executing law enforcement officer has a prior justification for an initial intrusion
or otherwise properly in a position from which he can view a particular order; (b)
the officer must discover incriminating evidence inadvertently; and (c) it must
be immediately apparent to the police that the items they observe may be
evidence of a crime, contraband, or otherwise subject to seizure.

This doctrine is a recognition of the fact that when executing police officers comes
across immediately incriminating evidence not covered by the warrant, they
should not be required to close their eyes to it, regardless of whether it is evidence
of the crime they are investigating or evidence of some other crime. It would be
needless to require the police to obtain another warrant.

The immediate requirement means that the executing officer can, at the time of
discovery of the object or the facts therein available to him, determine probable cause

Page 55 of 323
of the object’s incriminating evidence. In other words, to be immediate, probable cause
must be the direct result of the officer’s instantaneous sensory perception of the
object. The object is apparent if the executing officer had probable cause to connect
the object to criminal activity. The incriminating nature of the evidence becomes
apparent in the course of the search, without the benefit of any unlawful search or
seizure. It must be apparent at the moment of seizure. x x x x

The NBI manifestation is a glaring admission that it cannot tell without proper
examination that the Disudrin and Inoflox samples allegedly seized from
respondent’s place were counterfeit.

In this case, Disudrin and/or Inoflox were not listed in the search warrant issued by
the court a quo as among the properties to be seized by the NBI agents. x x x x It was
thus incumbent on the NBI agents and the petitioner to prove their claim that
the items were seized based on the plain view doctrine. x x x x The immediately
apparent aspect, after all, is central to the plain view exception relied upon by the
petitioner and the NBI. There is no showing that the NBI and the petitioner even
attempted to adduce such evidence.

It must be stressed that only the NBI agent/agents who enforced the warrant had
personal knowledge whether the sealed boxes and their contents thereof were
incriminating and that they were immediately apparent. There is even no showing
that the NBI agents knew the contents of the sealed boxes before they were
opened.

*People v. Beriarmente, G.R. No. 137612, September 25, 2001.


(Crim Pro: In flagrante de licto arrest; Buy bust; Marked money)

-Beriarmente was caught selling 1,500 grams of marijuana plants and was convicted
by the RTC.

-Beriarmente appealed. His contention: There is reasonable doubt as to his guilt


considering that the prosecution failed to produce the money used during the buy-
bust operation.

Issue:
Whether or not the prosecution’s failure to produce the money used during the buy-
bust operation creates reasonable doubt.

Ruling:
No.

It is well-established that in the prosecution for the sale of illegal drugs, what is
important is the fact that the poseur-buyer received the goods from the accused-
appellant and the same was presented as evidence in court. Neither is there a rule of
law which requires that there must be a simultaneous exchange of the marked money
and the prohibited drug between the poseur-buyer and the pusher.

There is also no rule that requires the police to use only marked money in buy-
bust operations. In fact, this Court has ruled that the failure to use marked money
or to present it in evidence is not material since the sale cannot be essentially
disproved by the absence thereof. The non-presentation of the marked money
does not create a hiatus in the evidence for the prosecution as long as the sale of
the illegal drugs is adequately established and the substance itself is presented
before the court.

*People v. Butial, G.R. No. 192785, February 4, 2015.


(Crim Pro: In flagrante delicto arrest; Buy bust; Marking: Significance)

-Butial allegedly sold shabu to a police officer acting as a poseur buyer. Confiscated
from him after the buy-bust operation were two plastic sachets containing shabu. A

Page 56 of 323
bag confiscated from Butial likewise yielded three more plastic sachets containing
shabu.

-The two plastic sachets confiscated from Butial right after the buy-bust operation,
however, were not marked. PO2 Martirez himself admitted that he did not put any
markings on the two plastic sachets that were handed to him by Borlagdan after the
latter’s purchase of the same from Butial.

-Nonetheless, he was convicted by the RTC.

Issue:
Whether or not the Butial’s conviction was proper.

Ruling:
No.

The initial link in the chain of custody starts with the seizure of the plastic sachets
from appellant and their marking by the apprehending officer. Marking after seizure
is the starting point in the custodial link, thus it is vital that the seized
contraband is immediately marked because succeeding handlers of the
specimens will use the markings as reference. The marking of the evidence
serves to separate the marked evidence from the corpus of all other similar or
related evidence from the time they are seized from the accused until they are
disposed at the end of criminal proceedings, obviating switching, ‘planting,’ or
contamination of evidence. x x x x

Clearly, the absence of markings creates an uncertainty that the two sachets seized
during the buy-bust operation were part of the five sachets submitted to the police
crime laboratory. The prosecution’s evidence failed to establish the marking of
the two sachets of shabu subject of this case, which is the first link in the chain
of custody and which would have shown that the shabu presented in evidence
was the same specimen bought from appellant during the buy-bust operation.
The lack of certainty therefore on a crucial element of the crime i.e., the identity of
the corpus delicti, warrants the reversal of the judgment of conviction.

*People v. Canton, G.R. No. 148825, December 27, 2002.


(Search and seizure; Airport security procedure; Crim Pro: In flagrante delicto arrest)

Facts:
-Susan was at the NAIA being a departing passenger bound for Saigon, Vietnam.

-When she passed through the metal detector booth, a beeping sound was emitted.
Consequently, Mylene Cabunoc (Mylene), a civilian employee of the National Action
Committee on Hijacking and Terrorism (NACHT) and the frisker on duty at that time,
called Susan’s attention.

-Upon frisking SUSAN further, Mylene felt something bulging at her abdominal area.
Mylene inserted her hand under the skirt of SUSAN, pinched the package several
times and noticed that the package contained what felt like rice granules. Susan
claimed it was only money.

-SPO4 De los Reyes instructed Mylene to call Customs Examiner Lorna Jalac and
bring SUSAN to a comfort room for a thorough physical examination. Upon further
frisking in the ladies’ room, Mylene touched something in front of SUSAN’s sex organ.
She directed SUSAN to remove her skirt, girdles and panty. SUSAN obliged. Mylene
and Lorna discovered three packages individually wrapped and sealed in gray colored
packing tape, which SUSAN voluntarily handed to them. The first was taken from
SUSAN’s abdominal area; the second, from in front of her genital area; and the third,
from her right thigh. All packages turned out to be containing shabu.

-The RTC convicted Susan.

Page 57 of 323
-Susan appealed. Her contention: the strip search conducted on her in the ladies’
room was constitutionally infirmed because it was not “incidental to an arrest.” The
arrest could not be said to have been made before the search because at the time of
the strip search, the arresting officers could not have known what was inside the
plastic containers hidden on her body, which were wrapped and sealed with gray
tape. At that point then, they could not have determined whether SUSAN was actually
committing a crime.

-OSG’s contention: it was a search incidental to an arrest.

Issues:
(1) Whether or not the search conducted on Susan was incidental to a lawful arrest;
(2) Whether or not the strip search on Susan in the ladies’ room was justified;
(3) Whether or not the arrest of Susan was valid.

Ruling:
(1) No.

Prior to the strip search in the ladies’ room, the airport security personnel had
no knowledge yet of what were hidden on SUSAN’s body; hence, they did not know
yet whether a crime was being committed. It was only after the strip search upon the
discovery by the police officers of the white crystalline substances inside the packages,
which they believed to be shabu, that Susan was arrested. The search cannot,
therefore, be said to have been done incidental to a lawful arrest. In a search
incidental to a lawful arrest, the law requires that there be first a lawful arrest
before a search can be made; the process cannot be reversed. x x x x

(2) Yes.

In the present case, the search was made pursuant to routine airport security
procedure, which is allowed under Section 9 of Republic Act No. 6235 reading as
follows:

SEC. 9. Every ticket issued to a passenger by the airline or air carrier concerned shall contain among
others the following condition printed thereon: "Holder hereof and his hand-carried luggage(s) are
subject to search for, and seizure of, prohibited materials or substances. Holder refusing to be
searched shall not be allowed to board the aircraft," which shall constitute a part of the contract
between the passenger and the air carrier. x x x x

This constitutes another exception to the proscription against warrantless searches


and seizures. As admitted by SUSAN and shown in Annex "D" of her Brief, the afore-
quoted provision is stated in the "Notice to All Passengers" located at the final security
checkpoint at the departure lounge. From the said provision, it is clear that the
search, unlike in the Terry search, is not limited to weapons. Passengers are also
subject to search for prohibited materials or substances.

(3) Yes.

It must be repeated that R.A. No. 6235 authorizes search for prohibited materials or
substances. To limit the action of the airport security personnel to simply
refusing her entry into the aircraft and sending her home (as suggested by
appellant), and thereby depriving them of "the ability and facility to act accordingly,
including to further search without warrant, in light of such circumstances, would be
to sanction impotence and ineffectivity in law enforcement, to the detriment of
society." x x x x

The packs of shabu seized during the routine frisk at the airport was acquired
legitimately pursuant to airport security procedures and are therefore admissible
in evidence against Susan. Corollarily, her subsequent arrest, although likewise
without warrant, was justified, since it was effected upon the discovery and
recovery of shabu in her person in flagrante delicto.

Page 58 of 323
*People v. Damaso, G.R. No. 93516, August 12, 1992.
(Search: Consent; Waiver)

Facts:
-The rented house of Damaso (a suspected CPP-NPA rebel) was searched by police
officers after Luz Morados, his helper, had allowed the police officers to enter it.

-Inside, the police officers found one (1) M14 Rifle bearing Serial No. 1249935 with
magazine and Fifty-Seven (57) live ammunition.

-Damaso was then charged with the crime of subversion, and was later on convicted
by the RTC.

-Damaso appealed.

-OSG’s contention: Technically speaking, there was no search as the group was
voluntarily shown the articles used in subversion; that besides, a search may be
validly conducted without search warrant with the consent of the person searched in
this case, appellant's allowed them to enter and to look around the appellant's house;
and that since the evidence seized was in plain view of the authorities, the same may
be seized without a warrant.

Issue:
Whether or not the search conducted by the police officers was valid.

Ruling:
No.

The constitutional immunity from unreasonable searches and seizures, being


personal one, cannot be waived by anyone except the person whose rights are
invaded or one who is expressly authorized to do so in his or her behalf. x x x x

The prosecution failed to show if Luz had such an authority. Without this
evidence, the authorities’ intrusion into the appellant’s dwelling cannot be given
any color of legality. While the power to search and seize is necessary to the public
welfare, still it must be exercised and the law enforced without transgressing the
constitutional rights of the citizens, for the enforcement of no statute is of sufficient
importance to justify indifference to the basic principles of government.

*Espano v. Court of Appeals, G.R. No. 120431, April 1, 1998.


(Search: Place of immediate control)

Facts:
-Police officers went to Zamora and Pandacan Streets, Manila to confirm reports of
drug pushing in the area.

-They saw petitioner selling “something” to another person. After the alleged buyer left,
they approached petitioner, identified themselves as policemen, and frisked him. The
search yielded two plastic cellophane tea bags of marijuana.

-When asked if he had more marijuana, he replied that there was more in his house.
The policemen went to his residence where they found ten (10) more cellophane tea
bags of marijuana.

-The RTC convicted him based on the two plastic cellophane tea bags of marijuana
AND the ten more cellophane tea bags of marijuana found in his house.

Issue:
Whether or not the marijuana found inside Espano’s house is admissible in evidence.

Page 59 of 323
Ruling:
No.

An exception to the rule requiring a search warrant is a search incidental to a lawful


arrest for dangerous weapons or anything which may be used as proof of the
commission of an offense. It may extend beyond the person of the one arrested to
include the premises or surroundings under his immediate control. In this case,
the ten cellophane bags of marijuana seized at petitioner's house after his arrest at
Pandacan and Zamora Streets do not fall under the said exceptions.

As regards the brick of marijuana found inside the appellant's house, the trial court
correctly ignored it apparently in view of its inadmissibility. While initially the arrest
as well as the body search was lawful, the warrantless search made inside the
appellant's house became unlawful since the police operatives were not armed
with a search warrant. Such search cannot fall under “search made incidental to a
lawful arrest,” the same being limited to body search and to that point within reach or
control of the person arrested, or that which may furnish him with the means of
committing violence or of escaping. In the case at bar, appellant was admittedly
outside his house when he was arrested. Hence, it can hardly be said that the
inner portion of his house was within his reach or control.

The articles seized from petitioner during his arrest were valid under the doctrine of
search made incidental to a lawful arrest. The warrantless search made in his house,
however, which yielded ten cellophane bags of marijuana became unlawful since the
police officers were not armed with a search warrant at the time. Moreover, it was
beyond the reach and control of petitioner.

*People v. Che Chun Ting, G.R. Nos. 130568-69, March 21, 2000.
(Search: Place of immediate control)

Facts:
-Mabel Cheung Mei Po (Mabel) was a drug courier who later cooperated with police
officers to reveal the source of the drugs, herein accused Che Chun Ting.

-Police agents and Mabel then set up an operation against Che Chun Ting.

-Mabel went just outside Unit 22 (while police officers parked nearby) where Che
Chun Ting handed to Mabel a transparent plastic bag containing 999.43 grams of
shabu.

-The NARCOM agents immediately alighted and arrested the surprised man who was
positively identified by Mabel as Che Chun Ting.

-Police agents searched Unit 22, which was owned by Che Chun Ting’s girlfriend,
Nimfa. During the search, they seized a black bag with several plastic bags containing
5,578.68 grams of shabu in an open cabinet at the second floor.

-The RTC convicted Che Chun Ting.

-Che Chun Ting appealed. His contentions: (1) the RTC erred in convicting him on the
basis of the shabu seized inside Unit 22, which was constitutionally inadmissible as
evidence since it was seized without a search warrant; (2) the RTC erred in assuming
that the entire white crystalline substance seized is positive of methylamphetamine
hydrochloride. The PNP Crime Laboratory should have subjected the entire 999.43
grams and 5,578.66 grams of white crystalline substance taken from him, to
laboratory examination and not merely representative samples thereof in milligrams.

Issue:
(1) Whether or not the search inside Unit 22 was valid;
(2) Whether or not the entire white crystalline substance confiscated from must be
tested for the presence of shabu and not a sample thereof only.

Page 60 of 323
Ruling:
(1) No.

The accused was admittedly outside Unit 22 and in the act of delivering to Mabel
Cheung Mei Po a bag of shabu when he was arrested by the NARCOM operatives.
Moreover, it is borne by the records that Unit 22 was not even his residence but
that of his girlfriend Nimfa Ortiz, and that he was merely a sojourner therein.
Hence, it can hardly be said that the inner portion of the house constituted a
permissible area within his reach or immediate control, to justify a warrantless
search therein.

The lawful arrest being the sole justification for the validity of the warrantless search
under the exception, the same must be limited to and circumscribed by the subject,
time and place of the arrest. As to subject, the warrantless search is sanctioned
only with respect to the person of the suspect, and things that may be seized
from him are limited to "dangerous weapons" or ''anything which may be used as
proof of the commission of the offense." With respect to the time and place of the
warrantless search, it must be contemporaneous with the lawful arrest. Stated
otherwise, to be valid, the search must have been conducted at about the time of
the arrest or immediately thereafter and only at the place where the suspect
was arrested, or the premises or surroundings under his immediate control.

It must be stressed that the purposes of the exception are only to protect the
arresting officer against physical harm from the person being arrested who might
be armed with a concealed weapon, and also to prevent the person arrested from
destroying the evidence within his reach. The exception therefore should not be
strained beyond what is needed in order to serve its purposes.

(2) No.

There is no law or rule of evidence requiring the forensic chemist to test the
entire quantity of seized drugs to determine whether the whole lot is really
prohibited or regulated drugs as suspected. On the contrary, it has always been the
standard procedure in the PNP Crime Laboratory to test only samples of the drugs
submitted for laboratory examination. A sample taken from a package may be
logically presumed to be representative of the whole contents of the package.

At any rate, it was up to the defense to prove by clear and convincing evidence that the
findings of the forensic chemist were erroneous. In the absence of such evidence, the
positive results of the tests conducted by the chemist should be accepted as
conclusive. After all, she has in her favor the presumption that she regularly
performed her official duty, which was to carry out those tests in accordance
with the accepted standard procedure.

*Pita v. Court of Appeals, G.R. No. 80806, October 5, 1989. [IMPORTANT!!!]


(Seizure of pornographic materials)

Facts:
-Pita was a publisher of Pinoy Playboy, a “men’s magazine.”

-PNP officers, as initiated by the Mayor of the City of Manila, Ramon D. Bagatsing
seized and confiscated from dealers, distributors, newsstand owners and peddlers
along Manila sidewalks, magazines, publications and other reading materials believed
to be obscene, pornographic and indecent and later burned the seized materials in
public at the University belt along C.M. Recto Avenue, Manila, in the presence of
Mayor Bagatsing and several officers and members of various student organizations.

-Among the publications seized, and later burned, was “Pinoy Playboy” magazines
published and co-edited by plaintiff Leo Pita.

Page 61 of 323
-Pita then filed a case for injunction with prayer for issuance of the writ of
preliminary injunction against Mayor Bagatsing and PNP, seeking to enjoin and/or
restrain said defendants from confiscating plaintiffs’ magazines or from otherwise
preventing the sale or circulation thereof.

-Pita’s contention: The magazine is a decent, artistic and educational magazine


which is not per se obscene, and that the publication thereof is protected by the
Constitutional guarantees of freedom of speech and of the press.

-The RTC denied the motion for a writ of preliminary injunction, and dismissed the
case for lack of merit.

Issue:
Whether or not the confiscation, seizure and burning of the “men’s magazines” were
valid.

Ruling:
No.

The Court is not convinced that the private Mayor Bagatsing and PNP officers have
shown the required proof to justify a ban and to warrant confiscation of the
magazines. First of all, they were not possessed of a lawful court order: (1) finding
the said materials to be pornographic, and (2) authorizing them to carry out a
search and seizure, by way of a search warrant.

The Court of Appeals has no "quarrel that ... freedom of the press is not without
restraint, as the state has the right to protect society from pornographic literature that
is offensive to public morals." Neither do we. But it brings us back to square one: were
the "literature" so confiscated "pornographic"? That we have laws punishing the
author, publisher and sellers of obscene publications (Sec. 1, Art. 201, Revised Penal
Code, as amended by P.D. No. 960 and P.D. No. 969)," is also fine, but the question,
again, is: Has the petitioner been found guilty under the statute?

The fact that the former respondent Mayor's act was sanctioned by "police
power" is no license to seize property in disregard of due process. x x x x

Significantly, the Decrees themselves lay down procedures for implementation. We


quote:

“Sec. 2. Disposition of the Prohibited Articles. — The disposition of the literature, films, prints,
engravings, sculptures, paintings, or other materials involved in the violation referred to in Section 1
hereof (Art. 201), RPC as amended) shall be governed by the following rules:

(a) Upon conviction of the offender, to be forfeited in favor of the Government to be destroyed.

(b) Where the criminal case against any violator of this decree results in an acquittal, the
obscene/immoral literature, films, prints, engravings, sculptures, paintings or other materials and
articles involved in the violation referred to in Section 1 (referring to Art. 201) hereof shall nevertheless be
forfeited in favor of the government to be destroyed, after forfeiture proceedings conducted by the Chief of
Constabulary.

(c) The person aggrieved by the forfeiture action of the Chief of Constabulary may, within fifteen (15)
days after his receipt of a copy of the decision, appeal the matter to the Secretary of National Defense for
review. The decision of the Secretary of National Defense shall be final and unappealable. (Sec. 2, PD No,
960 as amended by PD No. 969.)

It is basic that searches and seizures may be done only through a judicial warrant;
otherwise, they become unreasonable and subject to challenge. x x x x

The Court is not ruling out warrantless searches, as the Rules of Court (the Rules
then prevailing), provide:

Page 62 of 323
“SEC. 12. Search without warrant of person arrested. — A person charged with an offense may be
searched for dangerous weapons or anything which may be used as proof of the commission of the
offense”

but as the provision itself suggests, the search must have been an incident to a lawful
arrest, and the arrest must be on account of a crime committed. Here, no party has
been charged, nor are such charges being readied against any party, under
Article 201, as amended, of the Revised Penal Code.

We reject outright the argument that "there is no constitutional nor legal provision
which would free the accused of all criminal responsibility because there had been no
warrant," and that "violation of penal law [must] be punished." For starters, there is
no "accused" here to speak of, who ought to be "punished." Second, to say that the
respondent Mayor could have validly ordered the raid (as a result of an anti-smut
campaign) without a lawful search warrant because, in his opinion, "violation of penal
laws" has been committed, is to make the respondent Mayor judge, jury, and
executioner rolled into one.

RULES LAID DOWN IN THIS CASE:

1. The authorities must apply for the issuance of a search warrant from a judge,
if in their opinion, an obscenity rap is in order;

2. The authorities must convince the court that the materials sought to be
seized are “obscene,” and pose a clear and present danger of an evil substantive
enough to warrant State interference and action;

3. The judge must determine whether or not the same are indeed "obscene:" the
question is to be resolved on a case-to-case basis and on His/Her Honor's sound
discretion.

4. If, in the opinion of the court, probable cause exists, it may issue the search
warrant prayed for;

5. The proper suit is then brought in the court under Article 201 of the Revised
Penal Code;

6. Any conviction is subject to appeal. The appellate court may assess whether or not
the properties seized are indeed “obscene.”

*Ramirez v. Court of Appeals, G.R. No. 93833, September 28, 1995.


(Privacy of communications; Wire-tapping)

Facts:
-Ramirez filed a case for damages against Garcia, alleging that Garcia, in a
confrontation in the latter's office, vexed, insulted and humiliated Ramirez in a
"hostile and furious mood" and in a manner offensive to petitioner's dignity and
personality," contrary to morals, good customs and public policy."

-The transcript on which the civil case was based was culled from a tape recording
of the confrontation made by Ramirez wherein it can be heard that Garcia said “wala
kang utak” to Ramirez.

-As a result of Ramirez’s recording of the event and alleging that the said act of
secretly taping the confrontation was illegal, Garcia filed a criminal case before the
RTC for violation of R.A. 4200, entitled, "An Act to prohibit and penalize wire tapping
and other related violations of private communication, and other purposes."

-Ramirez moved to quash the Information against her.

Page 63 of 323
-Ramirez’s contention: the applicable provision of Republic Act 4200 does not apply to
the taping of a private conversation by one of the parties to the conversation.

Issues:
(1) Whether or not R.A. 4200 applies to the taping of a private conversation by one of
the parties to the conversation;
(2) Whether or not the phrase "private communication" in Section 1 of R.A. 4200
includes "private conversations" such as the confrontation that occurred in Garcia’s
office.

Ruling:
(1) Yes.

Section 1 of R.A. 4200 entitled, " An Act to Prohibit and Penalized Wire Tapping and
Other Related Violations of Private Communication and Other Purposes," provides:

Sec. 1. It shall be unlawful for any person, not being authorized by all the parties to any private
communication or spoken word, to tap any wire or cable, or by using any other device or arrangement, to
secretly overhear, intercept, or record such communication or spoken word by using a device commonly
known as a dictaphone or dictagraph or detectaphone or walkie-talkie or tape recorder, or however
otherwise described.

The aforestated provision clearly and unequivocally makes it illegal for any person,
not authorized by all the parties to any private communication to secretly record such
communication by means of a tape recorder. The law makes no distinction as to
whether the party sought to be penalized by the statute ought to be a party
other than or different from those involved in the private communication. The
statute's intent to penalize all persons unauthorized to make such recording is
underscored by the use of the qualifier "any." Consequently, as respondent Court of
Appeals correctly concluded, "even a person privy to a communication who records
his private conversation with another without the knowledge of the latter will
qualify as a violator" under this provision of R.A. 4200. x x x x

The provision clearly seeks to penalize even those privy to the private communications.
Where the law makes no distinctions, one does not distinguish.

(2) Yes.

Petitioner's contention that the phrase "private communication" in Section 1 of R.A.


4200 does not include "private conversations" narrows the ordinary meaning of the
word "communication" to a point of absurdity. The word “communicate” comes
from the Latin word communicare, meaning "to share or to impart." In its
ordinary signification, communication connotes the act of sharing or imparting
signification, communication connotes the act of sharing or imparting, as in a
conversation, or signifies the "process by which meanings or thoughts are shared
between individuals through a common system of symbols (as language signs or
gestures).”

*Waterous Drug Corporation v. NLRC, G.R. No. 113271, October 16, 1997.
(Privacy of communications; Acts of Private persons)

Facts:
-Catolico was hired as a pharmacist by Waterous Drug Corporation (hereafter
WATEROUS) on 15 August 1988.

-An employee of Waterous opened an envelope addressed to Catolico and found


therein a check evidencing an overprice in the purchase of certain medicine.

-After investigation and due notice, Catolico’s employment was terminated on the
ground of dishonesty.

Page 64 of 323
-Catolico’s contention: the check was discovered in violation of the constitutional
provision on the right to privacy and communication; and hence, inadmissible in
evidence.

Issue:
Whether or not the subject check is admissible in evidence.

Ruling:
Yes.

The Bill of Rights does not protect citizens from unreasonable searches and
seizures perpetrated by private individuals.

The constitutional protection against unreasonable searches and seizures refers to the
immunity of one’s person from interference by government and cannot be
extended to acts committed by private individuals so as to bring it within the
ambit of alleged unlawful intrusion by the government.

*Chavez v. Gonzalez, G.R. No. 168338, February 15, 2008. [IMPORTANT


PRINCIPLES HERE!!!]
(Freedom of expression; Freedom of the press; Prior restraint)

Facts:
-On June 8, 2005, respondent DOJ Secretary Raul Gonzalez (Gonzalez) warned
reporters that those who had copies of an audiotape of a mobile phone conversation
allegedly between then President Arroyo and Comelec Chair Abalos, and those
broadcasting or publishing its contents could be held liable under the Anti-
Wiretapping Act. He also stated that persons possessing or airing said tapes were
committing a continuing offense, subject to arrest by anybody who had personal
knowledge if the crime was committed or was being committed in their presence.

-On June 9, 2005, in another press briefing, Secretary Gonzales ordered the National
Bureau of Investigation (NBI) to go after media organizations "found to have caused
the spread, the playing and the printing of the contents of a tape" of an alleged
wiretapped conversation involving the President about fixing votes in the 2004
national elections. Gonzales said that he was going to start with Inq7.net, a joint
venture between the Philippine Daily Inquirer and GMA7 television network, because
by the very nature of the Internet medium, it was able to disseminate the contents of
the tape more widely.

-Subsequently, the NTC released a statement that the continuous airing or broadcast
of the said taped conversations by radio and television stations is a continuing
violation of the Anti-Wiretapping Law and the conditions of the Provisional Authority
and/or Certificate of Authority issued to these radio and television stations. x x x x
NTC further stated that, “the concerned radio and television companies are hereby
warned that their broadcast/airing of such false information and/or willful
misrepresentation shall be just cause for the suspension, revocation and/or
cancellation of the licenses or authorizations issued to the said companies.”

-In view of these, petitioner Chavez filed a petition under Rule 65 of the Rules of Court
against respondents Secretary Gonzales and the NTC, "praying for the issuance of the
writs of certiorari and prohibition, to annul void proceedings, and to prevent the
unlawful, unconstitutional and oppressive exercise of authority by the respondents."

-Contention of Gonzalez and the NTC: the challenged act is valid on the ground that
broadcast media enjoys free speech rights that are lesser in scope to that of print
media.

Issue:
Whether or not the acts of Gonzalez and NTC violate freedom of expression and the
press.

Page 65 of 323
Ruling:
Yes.

[IMPORTANT NOTES FOR THE BAR:

Prior restraint refers to official governmental restrictions on the press or other


forms of expression in advance of actual publication or dissemination. Freedom
from prior restraint is largely freedom from government censorship of publications,
whatever the form of censorship, and regardless of whether it is wielded by the
executive, legislative or judicial branch of the government. Thus, it precludes
governmental acts that required approval of a proposal to publish; licensing or
permits as prerequisites to publication including the payment of license taxes for the
privilege to publish; and even injunctions against publication.

Given that deeply ensconced in our fundamental law is the hostility against all prior
restraints on speech, and any act that restrains speech is presumed invalid, and
"any act that restrains speech is hobbled by the presumption of invalidity and should
be greeted with furrowed brows," it is important to stress not all prior restraints on
speech are invalid. Certain previous restraints may be permitted by the Constitution,
but determined only upon a careful evaluation of the challenged act as against the
appropriate test by which it should be measured against.

Hence, it is not enough to determine whether the challenged act constitutes some form
of restraint on freedom of speech. A distinction has to be made whether the
restraint is (1) a content-neutral regulation, i.e., merely concerned with the
incidents of the speech, or one that merely controls the time, place or manner,
and under well defined standards; or (2) a content-based restraint or censorship,
i.e., the restriction is based on the subject matter of the utterance or speech.
The cast of the restriction determines the test by which the challenged act is assayed
with.

When the speech restraints take the form of a content-neutral regulation, only a
substantial governmental interest is required for its validity. Because regulations
of this type are not designed to suppress any particular message, they are not
subject to the strictest form of judicial scrutiny but an intermediate approach—
somewhere between the mere rationality that is required of any other law and the
compelling interest standard applied to content-based restrictions. The test is called
intermediate because the Court will not merely rubberstamp the validity of a law but
also require that the restrictions be narrowly-tailored to promote an important or
significant governmental interest that is unrelated to the suppression of expression. x
xxx

On the other hand, a governmental action that restricts freedom of speech or of


the press based on content is given the strictest scrutiny in light of its inherent
and invasive impact. Only when the challenged act has overcome the clear and
present danger rule will it pass constitutional muster, with the government having
the burden of overcoming the presumed unconstitutionality. Unless the government
can overthrow this presumption, the content-based restraint will be struck
down.

With respect to content-based restrictions, the government must also show the type of
harm the speech sought to be restrained would bring about— especially the gravity
and the imminence of the threatened harm – otherwise the prior restraint will be
invalid. Prior restraint on speech based on its content cannot be justified by
hypothetical fears, "but only by showing a substantive and imminent evil that has
taken the life of a reality already on ground." As formulated, "the question in every
case is whether the words used are used in such circumstances and are of such a
nature as to create a clear and present danger that they will bring about the
substantive evils that Congress has a right to prevent. It is a question of proximity
and degree." x x x x

Page 66 of 323
Applying the foregoing, it is clear that the challenged acts in the case at bar need to be
subjected to the clear and present danger rule, as they are content-based
restrictions. The acts of respondents focused solely on but one object—a specific
content— fixed as these were on the alleged taped conversations between the
President and a COMELEC official. x x x x

Regardless of the regulatory schemes that broadcast media is subjected to, the Court
has consistently held that the clear and present danger test applies to content-
based restrictions on media, without making a distinction as to traditional print
or broadcast media.]

========

On the basis of the records of the case at bar, respondents who have the burden to
show that these acts do not abridge freedom of speech and of the press failed to hurdle
the clear and present danger test. It appears that the great evil which government
wants to prevent is the airing of a tape recording in alleged violation of the anti-
wiretapping law. The records of the case at bar, however, are confused and confusing,
and respondents’ evidence falls short of satisfying the clear and present danger
test.

Firstly, the various statements of the Press Secretary obfuscate the identity of the
voices in the tape recording. Secondly, the integrity of the taped conversation is also
suspect. The Press Secretary showed to the public two versions, one supposed to be a
"complete" version and the other, an "altered" version. Thirdly, the evidence of the
respondents on the who’s and the how’s of the wiretapping act is ambivalent,
especially considering the tape’s different versions. The identity of the wire-tappers,
the manner of its commission and other related and relevant proofs are some of the
invisibles of this case. Fourthly, given all these unsettled facets of the tape, it is even
arguable whether its airing would violate the anti-wiretapping law.

We rule that not every violation of a law will justify straitjacketing the exercise
of freedom of speech and of the press. Our laws are of different kinds and
doubtless, some of them provide norms of conduct which even if violated have only an
adverse effect on a person’s private comfort but does not endanger national security.
There are laws of great significance but their violation, by itself and without more,
cannot support suppression of free speech and free press. In fine, violation of law is
just a factor, a vital one to be sure, which should be weighed in adjudging
whether to restrain freedom of speech and of the press. The totality of the
injurious effects of the violation to private and public interest must be calibrated in
light of the preferred status accorded by the Constitution and by related international
covenants protecting freedom of speech and of the press. In calling for a careful and
calibrated measurement of the circumference of all these factors to determine
compliance with the clear and present danger test, the Court should not be
misinterpreted as devaluing violations of law. By all means, violations of law should be
vigorously prosecuted by the State for they breed their own evil consequence. But to
repeat, the need to prevent their violation cannot per se trump the exercise of
free speech and free press, a preferred right whose breach can lead to greater
evils. For this failure of the respondents alone to offer proof to satisfy the clear and
present danger test, the Court has no option but to uphold the exercise of free speech
and free press. There is no showing that the feared violation of the anti-
wiretapping law clearly endangers the national security of the State.

This is not all the faultline in the stance of the respondents. We slide to the issue of
whether the mere press statements of the Secretary of Justice and of the NTC in
question constitute a form of content-based prior restraint that has transgressed the
Constitution. In resolving this issue, we hold that it is not decisive that the press
statements made by respondents were not reduced in or followed up with formal
orders or circulars. It is sufficient that the press statements were made by
respondents while in the exercise of their official functions. Undoubtedly,

Page 67 of 323
respondent Gonzalez made his statements as Secretary of Justice, while the NTC
issued its statement as the regulatory body of media. Any act done, such as a speech
uttered, for and on behalf of the government in an official capacity is covered by
the rule on prior restraint. The concept of an “act” does not limit itself to acts
already converted to a formal order or official circular. Otherwise, the non-
formalization of an act into an official order or circular will result in the easy
circumvention of the prohibition on prior restraint. The press statements at bar are
acts that should be struck down as they constitute impermissible forms of prior
restraints on the right to free speech and press.

There is enough evidence of chilling effect of the complained acts on record. The
warnings given to media came from no less the NTC, a regulatory agency that
can cancel the Certificate of Authority of the radio and broadcast media. They
also came from the Secretary of Justice, the alter ego of the Executive, who
wields the awesome power to prosecute those perceived to be violating the laws
of the land. After the warnings, the KBP inexplicably joined the NTC in issuing an
ambivalent Joint Press Statement. After the warnings, petitioner Chavez was left alone
to fight this battle for freedom of speech and of the press. This silence on the sidelines
on the part of some media practitioners is too deafening to be the subject of
misinterpretation.

The constitutional imperative for us to strike down unconstitutional acts should


always be exercised with care and in light of the distinct facts of each case. For there
are no hard and fast rules when it comes to slippery constitutional questions, and the
limits and construct of relative freedoms are never set in stone. Issues revolving on
their construct must be decided on a case to case basis, always based on the peculiar
shapes and shadows of each case. But in cases where the challenged acts are
patent invasions of a constitutionally protected right, we should be swift in
striking them down as nullities per se. A blow too soon struck for freedom is
preferred than a blow too late.

*The Diocese of Bacolod v. Comelec, G.R. No. 205728, January 21, 2015.
[IMPORTANT!]
(Freedom of Expression; Freedom of Religion; Prior restraint; Content-based
regulation)

Facts:
-In 2013, petitioners posted two (2) tarpaulins within a private compound housing
the San Sebastian Cathedral of Bacolod. Each tarpaulin was approximately six feet
(6') by ten feet (10') in size. They were posted on the front walls of the cathedral
within public view.

-The first tarpaulin contains the message "IBASURA RH Law" referring to the
Reproductive Health Law of 2012 or Republic Act No. 10354.

-The second tarpaulin contains the heading "Conscience Vote" and lists candidates as
either "(Anti-RH) Team Buhay" with a check mark, or "(Pro-RH) Team Patay" with an
"X" mark.

-Thereafter, respondent Atty. Majarucon, in her capacity as Election Officer of Bacolod


City, issued a “Notice to Remove Campaign Materials” addressed to petitioner Most
Rev. Bishop Vicente M. Navarra. The election officer ordered the tarpaulin’s removal
within three (3) days from receipt for being oversized. Comelec Resolution No. 9615
provides for the size requirement of two feet (2’) by three feet (3’).

-Petitioners then filed with the SC a petition for certiorari and prohibition with
application for preliminary injunction and temporary restraining order.

-Comelec’s contentions: (1) a petition for certiorari and prohibition under Rule 65 of
the Rules of Court filed before this court is not the proper remedy to question the
notice and letter of respondents; it should have been via appeal to Comelec itself

Page 68 of 323
pursuant to Article IX-C, Section 2(3) of the Constitution, as judicial intervention is
limited to final decisions, orders, rulings and judgments of the Comelec en banc; and
(2) the tarpaulin is an election propaganda subject to regulation by COMELEC
pursuant to its mandate under Article IX-C, Section 4 of the Constitution. Hence,
respondents claim that the issuances ordering its removal for being oversized are valid
and constitutional.

Issues:
(1) Whether or not certiorari and prohibition under Rule 65 is the proper action under
the circumstances;
(2) Whether or not Comelec’s order to remove the tarpaulin is a valid election
regulation;
(3) Whether or not Comelec’s order to remove the tarpaulin violates freedom of religion;
(4) Whether or not the tarpaulin is an election propaganda subject to regulation by
COMELEC under its constitutional mandate.

Ruling:
(1) Yes.

Rule 64 is not the exclusive remedy for all acts of the COMELEC. Rule 65 is
applicable especially to raise objections relating to a grave abuse of discretion
resulting in the ouster of jurisdiction. As a special civil action, there must also be a
showing that there be no plain, speedy, and adequate remedy in the ordinary
course of the law. x x x x

Based on ABS-CBN case, the Supreme Court could review orders and decisions of
COMELEC — in electoral contests — despite not being reviewed by the Comelec en
banc, if:

1) It will prevent the miscarriage of justice;


2) The issue involves a principle of social justice;
3) The issue involves the protection of labor;
4) The decision or resolution sought to be set aside is a nullity; or
5) The need for relief is extremely urgent and certiorari is the only adequate and
speedy remedy available.

The main subject of this case is an alleged constitutional violation: the infringement
on speech and the "chilling effect" caused by respondent COMELEC’s notice and letter.
x x x x We are not confronted here with the question of whether the COMELEC, in its
exercise of jurisdiction, gravely abused it. We are confronted with the question as to
whether the COMELEC had any jurisdiction at all with its acts threatening imminent
criminal action effectively abridging meaningful political speech.

COMELEC’s notice and letter affect preferred speech. Respondents’ acts are
capable of repetition. Under the conditions in which it was issued and in view of the
novelty of this case, it could result in a "chilling effect" that would affect other citizens
who want their voices heard on issues during the elections. Other citizens who wish to
express their views regarding the election and other related issues may choose not to,
for fear of reprisal or sanction by the COMELEC. Direct resort to this court is allowed
to avoid such proscribed conditions. Rule 65 is also the procedural platform for
raising grave abuse of discretion.

Certainly, a breach of the fundamental right of expression by COMELEC is grave


abuse of discretion. Thus, the constitutionality of the notice and letter coming from
COMELEC is within this court’s power to review.

*As to hierarchy of courts (why not file the petition with the RTC first):

The Supreme Court’s role to interpret the Constitution and act in order to protect
constitutional rights when these become exigent should not be emasculated by the

Page 69 of 323
doctrine in respect of the hierarchy of courts. That has never been the purpose of such
doctrine. x x x x

The doctrine of hierarchy of courts is not an iron-clad rule. This court has "full
discretionary power to take cognizance and assume jurisdiction [over] special civil
actions for certiorari . . .filed directly with it for exceptionally compelling reasons or
if warranted by the nature of the issues clearly and specifically raised in the petition."

First, a direct resort to this court is allowed when there are genuine issues of
constitutionality that must be addressed at the most immediate time. A direct
resort to this court includes availing of the remedies of certiorari and prohibition to
assail the constitutionality of actions of both legislative and executive branches of the
government.

A second exception is when the issues involved are of transcendental importance.


In these cases, the imminence and clarity of the threat to fundamental constitutional
rights outweigh the necessity for prudence. The doctrine relating to constitutional
issues of transcendental importance prevents courts from the paralysis of procedural
niceties when clearly faced with the need for substantial protection.

Third, cases of first impression warrant a direct resort to this court. In cases of
first impression, no jurisprudence yet exists that will guide the lower courts on this
matter.

*As to non-exhaustion of administrative remedies (why not file with the Comelec en
banc first):

Prior exhaustion of administrative remedies may be dispensed with and judicial action
may be validly resorted to immediately: (a) when there is a violation of due process; (b)
when the issue involved is purely a legal question; (c) when the administrative
action is patently illegal amounting to lack or excess of jurisdiction; (d) when there is
estoppel on the part of the administrative agency concerned; (e) when there is
irreparable injury; (f) when the respondent is a department secretary whose acts as an
alter ego of the President bear the implied and assumed approval of the latter; (g)
when to require exhaustion of administrative remedies would be unreasonable; (h)
when it would amount to a nullification of a claim; (i) when the subject matter is a
private land in land case proceedings; (j) when the rule does not provide a plain,
speedy and adequate remedy; or (k) when there are circumstances indicating the
urgency of judicial intervention.

First, petitioners allege that the assailed issuances violated their right to freedom of
expression and the principle of separation of church and state. This is a purely legal
question. Second, the circumstances of the present case indicate the urgency of
judicial intervention considering the issue then on the RH Law as well as the
upcoming elections. Thus, to require the exhaustion of administrative remedies in
this case would be unreasonable.

(2) No.

The COMELEC had no legal basis to regulate expressions made by private


citizens. Comelec considered the tarpaulin as a campaign material in their
issuances. The laws regulating the posting of campaign materials only apply to
candidates and political parties, and petitioners are neither of the two.

Respondents cite the Constitution, laws, and jurisprudence to support their position
that they had the power to regulate the tarpaulin. However, all of these provisions
pertain to candidates and political parties. Petitioners are not candidates. Neither do
they belong to any political party. COMELEC does not have the authority to
regulate the enjoyment of the preferred right to freedom of expression exercised
by a non-candidate in this case. x x x x

Page 70 of 323
Section 9 of the Fair Election Act on the posting of campaign materials only mentions
"parties" and "candidates":

“Sec. 9. Posting of Campaign Materials. - The COMELEC may authorize political parties and party-list
groups to erect common poster areas for their candidates in not more than ten (10) public places such as
plazas, markets, barangay centers and the like, wherein candidates can post, display or exhibit election
propaganda: Provided, That the size of the poster areas shall not exceed twelve (12) by sixteen (16) feet or
its equivalent. Independent candidates with no political parties may likewise be authorized to erect
common poster areas in not more than ten (10) public places, the size of which shall not exceed four (4)
by six (6) feet or its equivalent. Candidates may post any lawful propaganda material in private places
with the consent of the owner thereof, and in public places or property which shall be allocated equitably
and impartially among the candidates.”

Similarly, Section 17 of COMELEC Resolution No. 9615, the rules and regulations
implementing the Fair Election Act, provides as follows:

“SECTION 17. Posting of Campaign Materials. - Parties and candidates may post any lawful campaign
material in:

a. Authorized common poster areas in public places subject to the requirements and/or limitations set
forth in the next following section; and

b. Private places provided it has the consent of the owner thereof.”

The tarpaulin was not paid for by any candidate or political party. There was no
allegation that petitioners coordinated with any of the persons named in the tarpaulin
regarding its posting. On the other hand, petitioners posted the tarpaulin as part of
their advocacy against the RH Law. They invoke their constitutional right to
communicate their opinions, views and beliefs about issues and candidates. x x x x

Likewise, even with the clear and present danger test, respondents failed to justify the
regulation. There is no compelling and substantial state interest endangered by
the posting of the tarpaulin as to justify curtailment of the right of freedom of
expression. There is no reason for the state to minimize the right of non-
candidate petitioners to post the tarpaulin in their private property. The size of
the tarpaulin does not affect anyone else’s constitutional rights.

(3) No.

The tarpaulin, on its face, does not convey any religious doctrine of the Catholic
church. That the position of the Catholic church appears to coincide with the
message of the tarpaulin regarding the RH Law does not, by itself, bring the
expression within the ambit of religious speech. On the contrary, the tarpaulin
clearly refers to candidates classified under "Team Patay" and "Team Buhay" according
to their respective votes on the RH Law.

The same may be said of petitioners’ reliance on papal encyclicals to support their
claim that the expression on the tarpaulin is an ecclesiastical matter. With all due
respect to the Catholic faithful, the church doctrines relied upon by petitioners are
not binding upon this court. The position of the Catholic religion in the
Philippines as regards the RH Law does not suffice to qualify the posting by
one of its members of a tarpaulin as religious speech solely on such basis. The
enumeration of candidates on the face of the tarpaulin precludes any doubt as to its
nature as speech with political consequences and not religious speech.

Furthermore, the definition of an “ecclesiastical affair” in Austria v. National Labor


Relations Commission cited by petitioners finds no application in the present case. The
posting of the tarpaulin does not fall within the category of matters that are beyond
the jurisdiction of civil courts as enumerated in the Austria case, such as
“proceedings for excommunication, ordinations of religious ministers,
administration of sacraments and other activities with attached religious
significance.”

(4) No.

Page 71 of 323
While the tarpaulin may influence the success or failure of the named candidates
and political parties, this does not necessarily mean it is election propaganda.
The tarpaulin was not paid for or posted "in return for consideration" by any
candidate, political party, or party-list group. x x x x

The second paragraph of Section 1(4) of COMELEC Resolution No. 9615, or the rules
and regulations implementing Republic Act No. 9006 as an aid to interpret the law
insofar as the facts of this case requires, states:

4. The term "political advertisement" or "election propaganda" refers to any matter broadcasted,
published, printed, displayed or exhibited, in any medium, which contain the name, image, logo, brand,
insignia, color motif, initials, and other symbol or graphic representation that is capable of being
associated with a candidate or party, and is intended to draw the attention of the public or a segment
thereof to promote or oppose, directly or indirectly, the election of the said candidate or candidates to a
public office. In broadcast media, political advertisements may take the form of spots, appearances on TV
shows and radio programs, live or taped announcements, teasers, and other forms of advertising
messages or announcements used by commercial advertisers. Political advertising includes matters,
not falling within the scope of personal opinion, that appear on any Internet website, including,
but not limited to, social networks, blogging sites, and micro-blogging sites, in return for
consideration, or otherwise capable of pecuniary estimation.

It is clear that this paragraph suggests that personal opinions are not included,
while sponsored messages are covered.

*GMA Network, Inc. v. Comelec, G.R. No. 205357, September 2, 2014.


(Freedom of expression; Freedom of the press; Prior restraint)

Facts:
-For the 2013 elections, the Comelec issued Resolution No. 9615 (Resolution),
Section 9 (a) of which limited the broadcast and radio advertisements of candidates
and political parties for national election positions to an “aggregate total” of one
hundred twenty (120) minutes and one hundred eighty (180) minutes, respectively.

-However, during the previous elections of May 14, 2007 and May 10, 2010, the
COMELEC issued Resolutions implementing and interpreting Section 6 of R.A. No.
9006, regarding airtime limitations, to mean that a candidate is entitled to the
aforestated number of minutes “per station.”

--The 2013 Resolution likewise requires that the media entity shall give prior notice
to the COMELEC of appearance or guesting by a candidate on any bona fide newscast,
bona fide news interview, bona fide news documentary. If such prior notice is not
feasible or practicable, the notice shall be sent within twenty-four (24) hours from the
first broadcast or publication.

-GMA and other stations filed a petition challenging the said resolution.

Issues:
(1) Whether or not the Comelec has the power to effect a drastic reduction of the
allowable minutes within which candidates and political parties would be able to
campaign through the air;
(2) Whether or not the resolution violates the constitutional guaranty of freedom of
expression, of speech and of the press;
(3) Whether or not the “prior notice” requirement is constitutional.

Ruling:
(1) No.

COMELEC is duty bound to come up with reasonable basis for changing the
interpretation and implementation of the airtime limits.

There is no question that the COMELEC is the office constitutionally and statutorily
authorized to enforce election laws but it cannot exercise its powers without

Page 72 of 323
limitations - or reasonable basis. It could not simply adopt measures or regulations
just because it feels that it is the right thing to do, in so far as it might be concerned.
It does have discretion, but such discretion is something that must be exercised
within the bounds and intent of the law. The COMELEC is not free to simply
change the rules especially if it has consistently interpreted a legal provision in
a particular manner in the past. If ever it has to change the rules, the same must
be properly explained with sufficient basis. x x x x

While stability in the law, particularly in the business field, is desirable, there is no
demand that the Comelec slavishly follow precedent. However, we think it essential,
for the sake of clarity and intellectual honesty, that if an administrative agency
decides inconsistently with previous action, that it explain thoroughly why a
different result is warranted, or if need be, why the previous standards should no
longer apply or should be overturned. Such explanation is warranted in order to
sufficiently establish a decision as having rational basis. Any inconsistent decision
lacking thorough, ratiocination in support may be struck down as being
arbitrary. And any decision with absolutely nothing to support it is a nullity. x x
xx

In addition, the law, which is the basis of the regulation subject of these petitions,
pertinently provides:

“6.2. (a) Each bona fide candidate or registered political party for a nationally elective office shall be
entitled to not more than one hundred twenty (120) minutes of television advertisement and one hundred
eighty (180) minutes of radio advertisement whether by purchase or donation.

(b) Each bona fide candidate or registered political party for a locally elective office shall be entitled to not
more than sixty (60) minutes of television advertisement and ninety (90) minutes of radio advertisement
whether by purchase or donation; x x x x”

The law, on its face, does not justify a conclusion that the maximum allowable
airtime should be based on the totality of possible broadcast in all television or
radio stations. Senator Cayetano has called our attention to the legislative intent
relative to the airtime allowed - that it should be on a "per station" basis.

(2) Yes.

The guaranty of freedom to speak is useless without the ability to communicate and
disseminate what is said. And where there is a need to reach a large audience, the
need to access the means and media for such dissemination becomes critical. This is
where the press and broadcast media come along. At the same time, the right to speak
and to reach out would not be meaningful if it is just a token ability to be heard by a
few. It must be coupled with substantially reasonable means by which the
communicator and the audience could effectively interact. Section 9 (a) of COMELEC
Resolution No. 9615, with its adoption of the "aggregate-based" airtime limits
unreasonably restricts the guaranteed freedom of speech and of the press. x x x x

The assailed rule on "aggregate-based" airtime limits is unreasonable and


arbitrary as it unduly restricts and constrains the ability of candidates and
political parties to reach out and communicate with the people. Here, the
adverted reason for imposing the "aggregate-based" airtime limits - leveling the
playing field - does not constitute a compelling state interest which would justify
such a substantial restriction on the freedom of candidates and political parties
to communicate their ideas, philosophies, platforms and programs of
government. And, this is specially so in the absence of a clear-cut basis for the
imposition of such a prohibitive measure. In this particular instance, what the
COMELEC has done is analogous to letting a bird fly after one has clipped its wings.

It is also particularly unreasonable and whimsical to adopt the aggregate-based time


limits on broadcast time when we consider that the Philippines is not only composed
of so many islands. There are also a lot of languages and dialects spoken among the
citizens across the country. Accordingly, for a national candidate to really reach out to

Page 73 of 323
as many of the electorates as possible, then it might also be necessary that he conveys
his message through his advertisements in languages and dialects that the people may
more readily understand and relate to. To add all of these airtimes in different dialects
would greatly hamper the ability of such candidate to express himself - a form of
suppression of his political speech.

(3) Yes.

Such a requirement is a reasonable means adopted by the COMELEC to ensure that


parties and candidates are afforded equal opportunities to promote their respective
candidacies. Unlike the restrictive aggregate-based airtime limits, the directive to
give prior notice is not unduly burdensome and unreasonable, much less could it
be characterized as prior restraint since there is no restriction on dissemination
of information before broadcast.

The prior notice requirement is a mechanism designed to inform the COMELEC of the
appearances or guesting of candidates in bona fide news broadcasts. It is for
monitoring purposes only, not censorship. It does not control the subject matter of
news broadcasts in any way. Neither does it prevent media outlets from covering
candidates in news interviews, news events, and news documentaries, nor prevent the
candidates from appearing thereon.

*Adiong v. Comelec, G.R. No. 103956, March 31, 1992.


(Freedom of expression; Prior restraint; Overbreadth doctrine)

Facts:
-The Comelec issued a Resolution prohibiting the posting of decals and stickers in
“mobile” places like cars and other moving vehicles.

-Adiong, a senatorial candidate, went to the SC to challenge the validity thereof.

Issue:
Whether or not the Comelec may prohibit the posting of decals and stickers on
"mobile" places, public or private, and limit their location or publication to the
authorized posting areas that it fixes.

Ruling:
No.

First — the prohibition unduly infringes on the citizen's fundamental right of


free speech enshrined in the Constitution (Sec. 4, Article III). There is no public
interest substantial enough to warrant the kind of restriction involved in this case.

All of the protections expressed in the Bill of Rights are important but we have
accorded to free speech the status of a preferred freedom. x x x x Considering that the
period of legitimate campaign activity is fairly limited and, in the opinion of some, too
short, it becomes obvious that unduly restrictive regulations may prove unfair to
affected parties and the electorate. x x x x

The posting of decals and stickers in mobile places like cars and other moving
vehicles does not endanger any substantial government interest. There is no clear
public interest threatened by such activity so as to justify the curtailment of the
cherished citizen's right of free speech and expression. Under the clear and present
danger rule not only must the danger be patently clear and pressingly present
but the evil sought to be avoided must be so substantive as to justify a clamp
over one's mouth or a writing instrument to be stilled.

Significantly, the freedom of expression curtailed by the questioned prohibition is not


so much that of the candidate or the political party. The regulation strikes at the
freedom of an individual to express his preference and, by displaying it on his
car, to convince others to agree with him. A sticker may be furnished by a

Page 74 of 323
candidate but once the car owner agrees to have it placed on his private vehicle, the
expression becomes a statement by the owner, primarily his own and not of anybody
else.

Second — the questioned prohibition premised on the statute and as couched in


the resolution is void for overbreadth.

A statute is considered void for overbreadth when "it offends the constitutional
principle that a governmental purpose to control or prevent activities
constitutionally subject to state regulations may not be achieved by means
which sweep unnecessarily broadly and thereby invade the area of protected
freedoms."

Even though the governmental purpose be legitimate and substantial, that purpose
cannot be pursued by means that broadly stifle fundamental personal liberties when
the end can be more narrowly achieved. The breadth of legislative abridgment must be
viewed in the light of less drastic means for achieving the same basic purpose.

The resolution prohibits the posting of decals and stickers not more than eight and
one-half (8-1/2) inches in width and fourteen (14) inches in length in any place,
including mobile places whether public or private except in areas designated by the
COMELEC. Verily, the restriction as to where the decals and stickers should be
posted is so broad that it encompasses even the citizen's private property, which
in this case is a privately-owned vehicle. In consequence of this prohibition,
another cardinal rule prescribed by the Constitution would be violated. Section 1,
Article III of the Bill of Rights provides that no person shall be deprived of his property
without due process of law.

Third — the constitutional objective to give a rich candidate and a poor candidate
equal opportunity to inform the electorate as regards their candidacies, mandated by
Article II, Section 26 and Article XIII, Section 1 in relation to Article IX (c) Section 4 of
the Constitution, is not impaired by posting decals and stickers on cars and other
private vehicles. Compared to the paramount interest of the State in guaranteeing
freedom of expression, any financial considerations behind the regulation are of
marginal significance.

It is to be reiterated that the posting of decals and stickers on cars, calesas, tricycles,
pedicabs and other moving vehicles needs the consent of the owner of the vehicle.
Hence, the preference of the citizen becomes crucial in this kind of election
propaganda not the financial resources of the candidate.

Iglesia ni Cristo v. Court of Appeals, G.R. No. 119673, July 26, 1996.
(Freedom of expression; Movie censorship)

Facts:
-Petitioner Iglesia ni Cristo, a duly organized religious organization, has a television
program entitled “Ang Iglesia ni Cristo.”

-Sometime in the months of September, October and November 1992 petitioner


submitted to the respondent Board of Review for Moving Pictures and Television the
VTR tapes of its TV program Series Nos. 116, 119, 121 and 128.

-The Board classified the series as “X” or not for public viewing on the ground that
they "offend and constitute an attack against other religions which is expressly
prohibited by law."

-MTRCB’s contention: the program should only explain petitioner's own faith and
beliefs and avoid attacks on other faiths. It cannot tell or dictate any other religion
that they are right and the rest are wrong.

Issues:

Page 75 of 323
(1) Whether or not the MTRCB has the power to review petitioner’s TV program "Ang
Iglesia ni Cristo”;
(2) Assuming it has the power, whether or not it gravely abused its discretion when it
prohibited the airing of petitioner's religious program, Series Nos. 115, 119 and 121,
for the reason that they constitute an attack against other religions.

Ruling:
(1) Yes.

The right to religious profession and worship has a two-fold aspect, viz., freedom to
believe and freedom to act on one's beliefs. The first is absolute as long as the
belief is confined within the realm of thought. The second is subject to regulation
where the belief is translated into external acts that affect the public welfare.

We thus reject petitioner's postulate that its religious program is per se beyond review
by the respondent Board. Its public broadcast on TV of its religious program brings
it out of the bosom of internal belief. Television is a medium that reaches even the
eyes and ears of children. The Court iterates the rule that the exercise of religious
freedom can be regulated by the State when it will bring about the clear and
present danger of some substantive evil which the State is duty bound to
prevent, i.e., serious detriment to the more overriding interest of public health, public
morals, or public welfare.

(2) Yes.

First. Deeply ensconced in our fundamental law is its hostility against all prior
restraints on speech, including religious speech. Hence, any act that restrains
speech is hobbled by the presumption of invalidity and should be greeted with
furrowed brows. It is the burden of the respondent Board to overthrow this
presumption. If it fails to discharge this burden, its act of censorship will be struck
down. It failed in the case at bar.

Second. The evidence shows that the respondent Board X-rated petitioner’s TV series
for "attacking" either religions, especially the Catholic church. An examination of the
evidence, especially Exhibits "A," "A-1," "B," "C," and "D" will show that the so-called
“attacks” are mere criticisms of some of the deeply held dogmas and tenets of
other religions. The videotapes were not viewed by the respondent court as they were
not presented as evidence. Yet they were considered by the respondent court as
indecent, contrary to law and good customs, hence, can be prohibited from public
viewing under Section 3(c) of PD 1986. This ruling clearly suppresses petitioner's
freedom of speech and interferes with its right to free exercise of religion. x x x x

The respondent Board may disagree with the criticisms of other religions by petitioner
but that gives it no excuse to interdict such criticisms, however unclean they may be.
Under our constitutional scheme, it is not the task of the State to favor any
religion by protecting it against an attack by another religion. Religious dogmas
and beliefs are often at war and to preserve peace among their followers, especially the
fanatics, the establishment clause of freedom of religion prohibits the State from
leaning towards any religion. x x x x

In fine, the MTRCB cannot squelch the speech of petitioner Iglesia ni Cristo
simply because it attacks other religions, even if said religion happens to be the
most numerous church in our country.

Third. The respondents cannot also rely on the ground "attacks against another
religion" in X-rating the religious program of petitioner. Even a sideglance at Section 3
of PD No. 1986 will reveal that it is not among the grounds to justify an order
prohibiting the broadcast of petitioner's television program. The ground "attack
against another religion" was merely added by the respondent Board in its Rules. This
rule is void for it runs smack against the hoary doctrine that administrative rules and
regulations cannot expand the letter and spirit of the law they seek to enforce. x x x x

Page 76 of 323
It is opined that the respondent board can still utilize "attack against any religion" as a
ground allegedly ". . . because section 3 (c) of PD No. 1986 prohibits the showing of
motion pictures, television programs and publicity materials which are contrary to law
and Article 201 (2) (b) (3) of the Revised Penal Code punishes anyone who
exhibits "shows which offend any race or religion." We respectfully disagree for it is
plain that the word “attack” is not synonymous with the word “offend.”
Moreover, Article 201 (2) (b) (3) of the Revised Penal Code should be invoked to justify
the subsequent punishment of a show which offends any religion. It cannot be utilized
to justify prior censorship of speech. It must be emphasized that E.O. 876, the law
prior to PD 1986, included "attack against any religion" as a ground for censorship.
The ground was not, however, carried over by PD 1986. Its deletion is a decree to
disuse it.

Fourth. In X-rating the TV program of the petitioner, the respondents failed to


apply the clear and present danger rule. In American Bible Society v. City of Manila,
this Court held: "The constitutional guaranty of free exercise and enjoyment of
religious profession and worship carries with it the right to disseminate religious
information. Any restraint of such right can be justified like other restraints on
freedom of expression on the ground that there is a clear and present danger of any
substantive evil which the State has the right to prevent." In Victoriano vs. Elizalde
Rope Workers Union, 23 we further ruled that "…it is only where it is unavoidably
necessary to prevent an immediate and grave danger to the security and welfare of the
community that infringement of religious freedom may be justified, and only to the
smallest extent necessary to avoid the danger."

The records show that the decision of the respondent Board, affirmed by the
respondent appellate court, is completely bereft of findings of facts to justify the
conclusion that the subject videotapes constitute impermissible attacks against
another religion. There is no showing whatsoever of the type of harm the tapes
will bring about especially the gravity and imminence of the threatened harm.
Prior restraint on speech, including religious speech, cannot be justified by
hypothetical fears but only by the showing of a substantive and imminent evil which
has taken the life of a reality already on ground.

*ABS-CBN Broadcasting Corporation, G.R. No. 133486, January 28, 2000.


(Exit Poll/Survey; Freedom of Speech; Freedom of the Press; Prior Restraint)

Facts:
-The Comelec allegedly received information from a reliable source that ABS-CBN
(Lopez Group) had prepared a project, with PR groups, to conduct radio-TV coverage
of the elections and to make conduct an exit survey of the vote during the elections
for national officials particularly for President and Vice President.

-An exit poll is a species of electoral survey conducted by qualified individuals or


groups of individuals for the purpose of determining the probable result of an election
by confidentially asking randomly selected voters whom they have voted for,
immediately after they have officially cast their ballots.

-In view of this, the Comelec issued a Resolution approving the issuance of a
restraining order to stop ABS-CBN or any other groups, its agents or representatives
from conducting such exit survey.

-Comelec’s contention: Such project might conflict with the official Comelec count,
as well as the unofficial quick count of the National Movement for Free Elections
(Namfrel). It also had not authorized or deputized ABS-CBN to undertake the exit
survey. Also, exit polls indirectly transgress the sanctity and the secrecy of the
ballot.

Issues:

Page 77 of 323
(1) Whether or not the Comelec acted with grave abuse of discretion amounting to a
lack or excess of jurisdiction when it approved the issuance of the said restraining
order;
(2) Whether or not the conduct of an exit poll violates the secrecy of the ballot.

Ruling:
(1) Yes.

The holding of exit polls and the dissemination of their results through mass
media constitute an essential part of the freedoms of speech and of the press.
Hence, the Comelec cannot ban them totally in the guise of promoting clean, honest,
orderly and credible elections. Quite the contrary, exit polls — properly conducted
and publicized — can be vital tools in eliminating the evils of election-fixing and
fraud. Narrowly tailored countermeasures may be prescribed by the Comelec so as to
minimize or suppress the incidental problems in the conduct of exit polls, without
transgressing in any manner the fundamental rights of our people. x x x x

A limitation on the freedom of expression may be justified only by a danger of


such substantive character that the state has a right to prevent. Unlike in the
"dangerous tendency" doctrine, the danger must not only be clear but also present.
"Present" refers to the time element; the danger must not only be probable but very
likely to be inevitable. The evil sought to be avoided must be so substantive as to
justify a clamp over one's mouth or a restraint of a writing instrument. x x x x

Comelec’s fears are purely speculative and clearly untenable. First, by the very
nature of a survey, the interviewees or participants are selected at random, so that
the results will as much as possible be representative or reflective of the general
sentiment or view of the community or group polled. Second, the survey result is not
meant to replace or be at par with the official Comelec count. It consists merely of
the opinion of the polling group as to who the electorate in general has probably voted
for, based on the limited data gathered from polled individuals. Finally, not at stake
here are the credibility and the integrity of the elections, which are exercises that
are separate and independent from the exit polls. The holding and the reporting of the
results of exit polls cannot undermine those of the elections, since the former is only
part of the latter. If at all, the outcome of one can only be indicative of the other.

The Comelec's concern with the possible noncommunicative effect of exit polls
— disorder and confusion in the voting centers — does not justify a total ban on
them. Undoubtedly, the assailed Comelec Resolution is too broad, since its
application is without qualification as to whether the polling is disruptive or not.
Concededly, the Omnibus Election Code prohibits disruptive behavior around the
voting centers. There is no showing, however, that exit polls or the means to
interview voters cause chaos in voting centers. Neither has any evidence been
presented proving that the presence of exit poll reporters near an election precinct
tends to create disorder or confuse the voters.

(2) No.

The reason behind the principle of ballot secrecy is to avoid vote buying through
voter identification. Thus, voters are prohibited from exhibiting the contents of their
official ballots to other persons, from making copies thereof, or from putting
distinguishing marks thereon so as to be identified. Also proscribed is finding out the
contents of the ballots cast by particular voters or disclosing those of disabled or
illiterate voters who have been assisted. Clearly, what is forbidden is the association
of voters with their respective votes, for the purpose of assuring that the votes
have been cast in accordance with the instructions of a third party. This result
cannot, however, be achieved merely through the voters' verbal and confidential
disclosure to a pollster of whom they have voted for.

[IMPORTANT NOTE:

Page 78 of 323
In the case of SWS v. Comelec, G.R. No. 147571, May 5, 2001, the same principles
were applied against:

“Surveys affecting national candidates shall not be published fifteen (15) days before an election and
surveys affecting local candidates shall not be published seven (7) days before an election.”

It was declared unconstitutional.]

*Chavez v. Comelec, G.R. No. 162777, August 31, 2004. [IMPORTANT!!!]


(Freedom of expression; Election laws; Overbreadth Doctrine)

-On August 18, 2003, he authorized a certain Andrew So to use his name and image
for 96° North, a clothing company. On October 14, 2003 and November 10, 2003,
Chavez also signed Endorsement Agreements with two corporations. Pursuant to
these agreements, three billboards were set up along the Balintawak Interchange of
the North Expressway. One billboard showed petitioner promoting the plastic
products of Konka International Plastics Manufacturing Corporation, and the other
two showed petitioner endorsing the clothes of 96° North. One more billboard was set
up along Roxas Boulevard showing petitioner promoting the game and amusement
parlors of G-Box.

-On December 30, 2003, petitioner filed his certificate of candidacy for the position
of Senator.

-However, on January 6, 2004, the Comelec issued Comelec Resolution No. 6520,
Section 32 of which provided that:

“Section 32. All propaganda materials such as posters, streamers, stickers or paintings on walls and
other materials showing the picture, image, or name of a person, and all advertisements on print,
in radio or on television showing the image or mentioning the name of a person, who subsequent to the
placement or display thereof becomes a candidate for public office shall be immediately removed by said
candidate and radio station, print media or television station within 3 days after the effectivity of
these implementing rules; otherwise, he and said radio station, print media or television station shall be
presumed to have conducted premature campaigning in violation of Section 80 of the Omnibus Election
Code.”

-Chavez then went to the SC via prohibition with prayer for the issuance of a writ of
preliminary injunction.

-Contention of Chavez: The billboards, while they exhibit his name and image, do not
at all announce his candidacy for any public office nor solicit support for such
candidacy from the electorate. They are mere product endorsements and not
election propaganda. Prohibiting, therefore, their exhibition to the public is not
within the scope of the powers of the COMELEC.

Issues:
(1) Whether or not Section 32 of Resolution No. 6520 is a valid exercise of police
power;
(2) Whether or not Section 32 of Resolution No. 6520 is a violation of the non-
impairment clause;
(3) Whether or not Section 32 of Resolution No. 6520 is in the nature of an ex post
facto law;
(4) Whether or not Section 32 of Resolution No. 6520 is invalid because of
overbreadth.

Ruling:
(1) Yes.

A close examination of the assailed provision reveals that its primary objectives are
to prohibit premature campaigning and to level the playing field for candidates
of public office, to equalize the situation between popular or rich candidates, on
one hand, and lesser-known or poorer candidates, on the other, by preventing
the former from enjoying undue advantage in exposure and publicity on account

Page 79 of 323
of their resources and popularity. The latter is a valid reason for the exercise of
police power.

The obvious intention of this provision is to equalize, as far as practicable, the


situations of rich and poor candidates by preventing the former from enjoying the
undue advantage offered by huge campaign "war chests." This objective is of special
importance and urgency in a country which, like ours, is characterized by extreme
disparity in income distribution between the economic elite and the rest of society, and
by the prevalence of poverty, with so many of our population falling below the poverty
line. x x x x

Moreover, petitioner cannot claim that the subject billboards are purely product
endorsements and do not announce nor solicit any support for his candidacy. Under
the Omnibus Election Code, "election campaign" or "partisan political activity" is
defined as an act designed to promote the election or defeat of a particular
candidate or candidates to a public office.

It is true that when petitioner entered into the contracts or agreements to endorse
certain products, he acted as a private individual and had all the right to lend his
name and image to these products. However, when he filed his certificate of
candidacy for Senator, the billboards featuring his name and image assumed
partisan political character because the same indirectly promoted his candidacy.
Therefore, the COMELEC was acting well within its scope of powers when it
required petitioner to discontinue the display of the subject billboards. If the
subject billboards were to be allowed, candidates for public office whose name and
image are used to advertise commercial products would have more opportunity to
make themselves known to the electorate, to the disadvantage of other candidates who
do not have the same chance of lending their faces and names to endorse popular
commercial products as image models.

Similarly, an individual intending to run for public office within the next few months,
could pay private corporations to use him as their image model with the intention of
familiarizing the public with his name and image even before the start of the campaign
period. This, without a doubt, would be a circumvention of the rule against premature
campaigning.

(2) No.

The non-impairment clause of the Constitution must yield to the loftier


purposes targeted by the Government. Equal opportunity to proffer oneself for
public office, without regard to the level of financial resources one may have at his
disposal, is indeed of vital interest to the public. The State has the duty to enact and
implement rules to safeguard this interest. Time and again, this Court has said that
contracts affecting public interest contain an implied reservation of the police
power as a postulate of the existing legal order. This power can be activated at
anytime to change the provisions of the contract, or even abrogate it entirely, for the
promotion or protection of the general welfare. Such an act will not militate
against the impairment clause, which is subject to and limited by the paramount
police power.

(3) No.

Petitioner also claims that Section 32 of Resolution No. 6520 is in the nature of an ex
post facto law. He urges this Court to believe that the assailed provision makes an
individual criminally liable for an election offense for not removing such
advertisement, even if at the time the said advertisement was exhibited, the same
was clearly legal. Hence, it makes a person, whose name or image is featured in any
such advertisement, liable for premature campaigning under the Omnibus Election
Code.

Page 80 of 323
Section 32, although not penal in nature, defines an offense and prescribes a penalty
for said offense. Laws of this nature must operate prospectively, except when they are
favorable to the accused. It should be noted, however, that the offense defined in the
assailed provision is not the putting up of "propaganda materials such as posters,
streamers, stickers or paintings on walls and other materials showing the picture,
image or name of a person, and all advertisements on print, in radio or on television
showing the image or mentioning the name of a person, who subsequent to the
placement or display thereof becomes a candidate for public office." Nor does it
prohibit or consider an offense the entering of contracts for such propaganda
materials by an individual who subsequently becomes a candidate for public office.
One definitely does not commit an offense by entering into a contract with private
parties to use his name and image to endorse certain products prior to his becoming a
candidate for public office. The offense, as expressly prescribed in the assailed
provision, is the non-removal of the described propaganda materials three (3)
days after the effectivity of COMELEC Resolution No. 6520. If the candidate for
public office fails to remove such propaganda materials after the given period, he shall
be liable under Section 80 of the Omnibus Election Code for premature campaigning.
Indeed, nowhere is it indicated in the assailed provision that it shall operate
retroactively. There is, therefore, no ex post facto law in this case.

(4) No.

A statute or regulation is considered void for overbreadth when it offends the


constitutional principle that a governmental purpose to control or prevent activities
constitutionally subject to State regulations may not be achieved by means that sweep
unnecessarily broadly and thereby invade the area of protected freedoms.

The provision in question is limited in its operation both as to time and scope. It
only disallows the continued display of a person's propaganda materials and
advertisements after he has filed a certificate of candidacy and before the start
of the campaign period. Said materials and advertisements must also show his
name and image.

There is no blanket prohibition of the use of propaganda materials and


advertisements. During the campaign period, these may be used subject only to
reasonable limitations necessary and incidental to achieving the purpose of preventing
premature campaigning and promoting equality of opportunities among all candidates.

The provision, therefore, is not invalid on the ground of overbreadth.

*Islamic Da’Wah Council of the Philippines, Inc. v. Executive Secretary, G.R. No.
153888, July 9, 2003.
(Freedom of Religion; Halal)

Facts:
-Petitioner Islamic Da’wah Council of the Philippines, Inc. (IDCP) is a non-
governmental organization that extends voluntary services to the Filipino people,
especially to Muslim communities.

-In 1995, due to demands to certify food products as halal, IDCP formulated internal
rules and procedures based on the Qur’an and the Sunnah for the analysis of food,
inspection thereof and issuance of halal certifications.

-In that same year, petitioner began to issue, for a fee, certifications to qualified
products and food manufacturers. Petitioner even adopted for use on its halal
certificates a distinct sign or logo registered in the Philippine Patent Office under
Patent No. 4-2000-03664.

-In 2001, the Office of the Executive Secretary issued EO 46 creating the Philippine
Halal Certification Scheme and designating the Office of Muslim Affairs (OMA) to

Page 81 of 323
oversee its implementation. Under the EO, OMA has the exclusive authority to
issue halal certificates and perform other related regulatory activities.

-Thereafter, OMA sent letters to food manufacturers asking them to secure the halal
certification only from OMA lest they violate EO 46 and RA 4109. As a result,
petitioner lost revenues after food manufacturers stopped securing certifications from
it.

-IDCP then went to the SC via prohibition.

-OSG’s contention: The freedom of religion is subservient to the police power of the
State. By delegating to OMA the authority to issue halal certifications, the government
seeks to protect and promote the Muslim Filipinos’ right to health, and to instill health
consciousness in them.

Issue:
-Whether or not EO 46 violates freedom of religion.

Ruling:
Yes.

Without doubt, classifying a food product as halal is a religious function because


the standards used are drawn from the Qur'an and Islamic beliefs. By giving OMA
the exclusive power to classify food products as halal, EO 46 encroached on the
religious freedom of Muslim organizations like herein petitioner to interpret for
Filipino Muslims what food products are fit for Muslim consumption. Also, by
arrogating to itself the task of issuing halal certifications, the State has in effect forced
Muslims to accept its own interpretation of the Qur'an and Sunnah on halal food.

Freedom of religion is not subservient to the police power of the State. x x x Only the
prevention of an immediate and grave danger to the security and welfare of the
community can justify the infringement of religious freedom. If the government
fails to show the seriousness and immediacy of the threat, State intrusion is
constitutionally unacceptable. In a society with a democratic framework like ours,
the State must minimize its interference with the affairs of its citizens and instead
allow them to exercise reasonable freedom of personal and religious activity.

In the case at bar, we find no compelling justification for the government to


deprive muslim organizations, like herein petitioner, of their religious right to
classify a product as halal, even on the premise that the health of muslim
Filipinos can be effectively protected by assigning to OMA the exclusive power to
issue halal certifications. The protection and promotion of the muslim Filipinos' right
to health are already provided for in existing laws and ministered to by government
agencies charged with ensuring that food products released in the market are fit for
human consumption, properly labeled and safe. Unlike EO 46, these laws do not
encroach on the religious freedom of muslims. x x x x

Through the laws on food safety and quality, therefore, the State indirectly aids
muslim consumers in differentiating food from non-food products. The NMIC
guarantees that the meat sold in the market has been thoroughly inspected and fit for
consumption. Meanwhile, BFD ensures that food products are properly categorized
and have passed safety and quality standards. Then, through the labeling provisions
enforced by the DTI, muslim consumers are adequately apprised of the products that
contain substances or ingredients that, according to their Islamic beliefs, are not fit for
human intake. These are the non-secular steps put in place by the State to ensure
that the muslim consumers' right to health is protected. The halal certifications issued
by petitioner and similar organizations come forward as the official religious approval
of a food product fit for muslim consumption. x x x x

We do not share respondents' apprehension that the absence of a central


administrative body to regulate halal certifications might give rise to schemers

Page 82 of 323
who, for profit, will issue certifications for products that are not actually halal.
Aside from the fact that muslim consumers can actually verify through the labels
whether a product contains non-food substances, we believe that they are discerning
enough to know who the reliable and competent certifying organizations in their
community are. Before purchasing a product, they can easily avert this perceived evil
by a diligent inquiry on the reliability of the concerned certifying organization.

*Taruc v. Bishop Dela Cruz, G.R. No. 144801, March 10, 2005.
(Freedom of Religion)

Facts:
-Petitioners were lay members of the Philippine Independent Church (PIC) in Socorro,
Surigao del Norte.

-Petitioners, led by Dominador Taruc, clamored for the transfer of Fr. Florano to
another parish but Bishop de la Cruz denied their request. It turned out that the
family of Fr. Florano’s wife belonged to a political party opposed to petitioner Taruc’s,
thus the animosity between the two factions with Fr. Florano being identified with his
wife’s political camp. Bishop de la Cruz, however, found this too flimsy a reason for
transferring Fr. Florano to another parish.

-Meanwhile, hostility among the members of the PIC in Socorro, Surigao del Norte
worsened when petitioner Taruc tried to organize an open mass to be celebrated by a
certain Fr. Renato Z. Ambong during the town fiesta of Socorro. When Taruc informed
Bishop de la Cruz of his plan, the Bishop tried to dissuade him from pushing through
with it because Fr. Ambong was not a member of the clergy of the diocese of
Surigao and his credentials as a parish priest were in doubt.

-However, Taruc and his sympathizers proceeded to hold the open mass with Fr.
Ambong as the celebrant.

-In view thereof, Bishop de la Cruz declared petitioners expelled/excommunicated


from the PIC for disobedience to duly constituted authority in the Church, among
others.

-Petitioners then filed a complaint for damages with preliminary injunction against
Bishop de la Cruz before the RTC of Surigao City.

-Contention of petitioners: Their expulsion was illegal because it was done without
trial, thus violating their right to due process of law.

Issue:
Whether or not the courts have jurisdiction to hear a case involving the
expulsion/excommunication of members of a religious institution.

Ruling:
No.

In our jurisdiction, we hold the Church and the State to be separate and distinct from
each other. "Give to Ceasar what is Ceasar’s and to God what is God’s."

The expulsion/excommunication of members of a religious


institution/organization is a matter best left to the discretion of the officials,
and the laws and canons of said institution/organization. It is not for the courts
to exercise control over church authorities in the performance of their
discretionary and official functions. Rather, it is for the members of religious
institutions/organizations to conform to just church regulations. x x x x

In disputes involving religious institutions or organizations, there is one area


which the Court should not touch: doctrinal and disciplinary differences. Thus,
the amendments of the constitution, restatement of articles of religion and

Page 83 of 323
abandonment of faith or abjuration, having to do with faith, practice, doctrine,
form of worship, ecclesiastical law, custom and rule of a church and having
reference to the power of excluding from the church those allegedly unworthy of
membership, are unquestionably ecclesiastical matters which are outside the
province of the civil courts.

*Estrada v. Escritor, A.M. No. P-02-1651, June 22, 2006. [LONG! LATER!}
(Freedom of Religion; Compelling State Interest Test)

*Manotoc v. Court of Appeals, G.R. No. L-62100, May 30, 1986.


(Right to Travel; Court Order; On Bail)

Facts:
-Petitioner Ricardo L. Manotoc, Jr., was charged with several counts of estafa.

-In all cases, petitioner has been admitted to bail in the total amount of P105,000.00,
with FGU Instance Corporation as surety.

-On March 1, 1982, petitioner filed before each of the trial courts a motion entitled,
“motion for permission to leave the country,” stating as ground therefor his desire
to go to the United States, “relative to his business transactions and opportunities.”

-The RTC denied the said motion.

-Petitioner’s contention: Having been admitted to bail as a matter of right, neither


the courts which granted him bail nor the Securities and Exchange Commission,
could prevent him from exercising his constitutional right to travel.

Issue:
Whether or not a person facing a criminal indictment and provisionally released on
bail has an unrestricted right to travel.

Ruling:
No.

A court has the power to prohibit a person admitted to bail from leaving the
Philippines. This is a necessary consequence of the nature and function of a bail
bond.

Rule 114, Section 1 of the Rules of Court defines bail as the security required and
given for the release of a person who is in the custody of the law, that he will appear
before any court in which his appearance may be required as stipulated in the bail
bond or recognizance.

Its object is to relieve the accused of imprisonment and the state of the burden of
keeping him, pending the trial, and at the same time, to put the accused as much
under the power of the court as if he were in custody of the proper officer, and to
secure the appearance of the accused so as to answer the call of the court and do
what the law may require of him.

The condition imposed upon petitioner to make himself available at all times
whenever the court requires his presence operates as a valid restriction on his
right to travel. Indeed, if the accused were allowed to leave the Philippines
without sufficient reason, he may be placed beyond the reach of the courts.

If the sureties have the right to prevent the principal from leaving the state,
more so then has the court from which the sureties merely derive such right,
and whose jurisdiction over the person of the principal remains unaffected despite the
grant of bail to the latter. In fact, this inherent right of the court is recognized by

Page 84 of 323
petitioner himself, notwithstanding his allegation that he is at total liberty to leave the
country, for he would not have filed the motion for permission to leave the country in
the first place, if it were otherwise. x x x x

As petitioner has failed to satisfy the trial courts and the appellate court of the
urgency of his travel, the duration thereof, as well as the consent of his surety to
the proposed travel, We find no abuse of judicial discretion in their having denied
petitioner's motion for permission to leave the country, in much the same way, albeit
with contrary results, that We found no reversible error to have been committed by the
appellate court in allowing Shepherd to leave the country after it had satisfied itself
that she would comply with the conditions of her bail bond.

The constitutional right to travel being invoked by petitioner is not an absolute


right. Section 5, Article IV of the 1973 Constitution states:

The liberty of abode and of travel shall not be impaired except upon lawful order of the court, or when
necessary in the interest of national security, public safety or public health.

To our mind, the order of the trial court releasing petitioner on bail constitutes
such lawful order as contemplated by the above-quoted constitutional provision.

*Goldenway Merchandising Corporation v. Equitable PCI Bank, G.R. No. 195540, March
13, 2013.
(Non-impairment Clause; Redemption Period; Retroactive Application)

Facts:
-On November 29, 1985, Goldenway Merchandising Corporation (petitioner) executed
a Real Estate Mortgage in favor of Equitable PCI Bank (respondent) over its real
properties situated in Valenzuela City. The mortgage secured the Two Million Pesos
(₱2,000,000.00) loan granted by respondent to petitioner and was duly registered.

-As petitioner failed to settle its loan obligation, respondent extrajudicially foreclosed
the mortgage on December 13, 2000. During the public auction, the mortgaged
properties were sold for ₱3,500,000.00 to respondent. Accordingly, a Certificate of Sale
was issued to respondent on January 26, 2001. On February 16, 2001, the Certificate
of Sale was registered and inscribed on TCT Nos. T-152630, T-151655 and T-214528.

-In the meantime, on June 13, 2000, R.A. No. 8791 otherwise known as "The General
Banking Law of 2000" took effect and amended Act No. 3135.

-Under the new law, an exception is thus made in the case of juridical persons which
are allowed to exercise the right of redemption only “until, but not after, the
registration of the certificate of foreclosure sale” and in no case more than three (3)
months after foreclosure, whichever comes first.

-In a letter dated March 8, 2001, petitioner’s counsel offered to redeem the foreclosed
properties by tendering a check in the amount of ₱3,500,000.00. On March 12, 2001,
petitioner’s counsel met with respondent’s counsel reiterating petitioner’s intention to
exercise the right of redemption.

-However, petitioner was told that such redemption is no longer possible because
the certificate of sale had already been registered. Petitioner also verified with the
Registry of Deeds that title to the foreclosed properties had already been consolidated
in favor of respondent and that new certificates of title were issued in the name of
respondent on March 9, 2001.

-On December 7, 2001, petitioner filed a complaint for specific performance and
damages against the respondent, asserting that it is the one-year period of
redemption under Act No. 3135 which should apply and not the shorter redemption
period (three months) provided in Republic Act (R.A.) No. 8791.

Page 85 of 323
-After due proceedings, the RTC dismissed the petition.

-Contention of petitioner: Since R.A. No. 8791 does not provide for its retroactive
application, courts therefore cannot retroactively apply its provisions to contracts
executed and consummated before its effectivity. Such retroactive application violates
the constitutional proscription against impairment of the obligation of contract.

Issues:
(1) Whether or not R.A. No. 8791 can be applied retroactively;
(2) Whether or not R.A. No. 8791 violates the equal protection clause.

Ruling:
(1) Yes.

Petitioner’s contention that Section 47 of R.A. 8791 violates the constitutional


proscription against impairment of the obligation of contract has no basis.

The purpose of the non-impairment clause of the Constitution is to safeguard the


integrity of contracts against unwarranted interference by the State. As a rule,
contracts should not be tampered with by subsequent laws that would change or
modify the rights and obligations of the parties. Impairment is anything that
diminishes the efficacy of the contract. There is an impairment if a subsequent law
changes the terms of a contract between the parties, imposes new conditions,
dispenses with those agreed upon or withdraws remedies for the enforcement of the
rights of the parties.

Section 47 did not divest juridical persons of the right to redeem their
foreclosed properties but only modified the time for the exercise of such right by
reducing the one-year period originally provided in Act No. 3135. The new
redemption period commences from the date of foreclosure sale, and expires
upon registration of the certificate of sale or three months after foreclosure,
whichever is earlier. There is likewise no retroactive application of the new
redemption period because Section 47 exempts from its operation those properties
foreclosed prior to its effectivity and whose owners shall retain their redemption
rights under Act No. 3135.

(2) No.

Petitioner’s claim that Section 47 infringes the equal protection clause as it


discriminates mortgagors/property owners who are juridical persons is equally bereft
of merit.

The equal protection clause is directed principally against undue favor and individual
or class privilege. It is not intended to prohibit legislation which is limited to the object
to which it is directed or by the territory in which it is to operate. It does not require
absolute equality, but merely that all persons be treated alike under like conditions
both as to privileges conferred and liabilities imposed. Equal protection permits of
reasonable classification. We have ruled that one class may be treated differently
from another where the groupings are based on reasonable and real distinctions. If
classification is germane to the purpose of the law, concerns all members of the class,
and applies equally to present and future conditions, the classification does not violate
the equal protection guarantee.

We agree with the CA that the legislature clearly intended to shorten the period of
redemption for juridical persons whose properties were foreclosed and sold in
accordance with the provisions of Act No. 3135.

The difference in the treatment of juridical persons and natural persons was
based on the nature of the properties foreclosed – whether these are used as
residence, for which the more liberal one-year redemption period is retained, or
used for industrial or commercial purposes, in which case a shorter term is

Page 86 of 323
deemed necessary to reduce the period of uncertainty in the ownership of
property and enable mortgagee-banks to dispose sooner of these acquired assets.
It must be underscored that the General Banking Law of 2000, crafted in the
aftermath of the 1997 Southeast Asian financial crisis, sought to reform the General
Banking Act of 1949 by fashioning a legal framework for maintaining a safe and sound
banking system. In this context, the amendment introduced by Section 47
embodied one of such safe and sound practices aimed at ensuring the solvency
and liquidity of our banks. It cannot therefore be disputed that the said provision
amending the redemption period in Act 3135 was based on a reasonable
classification and germane to the purpose of the law.

*Navallo v. Sandiganbayan, G.R. No. 97214, July 16, 1994.


(Custodial Investigation; Audit Investigation)

Facts:
-On 27 January 1978, the Provincial Auditor of Surigao del Norte made a preliminary
audit examination of cash and other accounts of Ernesto Navallo (petitioner), who
was then Collecting and Disbursing Officer of Numancia National Vocational School.

-Espino found Navallo to be short of P16,483.62. In the course thereof, Navallo


signed the Audit Report indicating his shortage.

-On 11 May 1978, an information charging petitioner with having violated Article 217,
paragraph 4, of the Revised Penal Code, was filed with the RTC of Surigao del Norte.

-Navallo’s contention: His right to counsel was violated during the audit.

Issue:
Whether or not petitioner’s right to counsel was violated during the audit.

Ruling:
No.

Accused-petitioner claims to have been deprived of his constitutional rights under


Section 12, Article III, of the 1987 Constitution. Well-settled is the rule that such
rights are invocable only when the accused is under "custodial investigation," or
is "in custody investigation," which we have since defined as any "questioning
initiated by law enforcement officers after a person has been taken into custody
or otherwise deprived of his freedom of action in any significant way." A person
under a normal audit examination is not under custodial investigation. An audit
examiner himself can hardly be deemed to be the law enforcement officer
contemplated in the above rule.

*Ladiana v. People, G.R. No. 144293, December 4, 2002.


(Custodial Investigation; Preliminary Investigation; Right to Counsel)

Facts:
-Petitioner Ladiana was charged with murder.

-In convicting him of murder, the trial court admitted in evidence his Counter-
Affidatit submitted during the preliminary investigation of the case. In the said
counter-affidavit, Ladiana admitted to having fired the fatal shots that caused the
victim’s death. However, accused Ladiana allegedly did so in self-defense as the victim
was then purportedly attacking accused Ladiana and had, in fact, already inflicted a
stab wound on the arm of accused Ladiana.

-During trial, petitioner repudiated his counter-affidavit, arguing that no counsel was
present when he executed the same, and hence, it is inadmissible in evidence.

Issue:

Page 87 of 323
Whether or not the counter-affidavit executed by the accused in the absence of a
counsel, is admissible in evidence.

Ruling:
Yes.

Undeniably, the resolution of this case hinges mainly on the admissibility of the
Counter-Affidavit submitted by petitioner during the preliminary investigation. He
argues that no counsel was present when the Affidavit was executed. In support of his
argument, he cites the Constitution thus:

"SEC. 12. (1) Any person under investigation for the commission of an offense shall have the right to be
informed of his right to remain silent and to have competent and independent counsel preferably of
his own choice. If the person cannot afford the services of counsel, he must be provided with one. These
rights cannot be waived except in writing and in the presence of counsel.
xxx xxx xxx
(3) Any confession or admission obtained in violation of this or Section 17 hereof shall be inadmissible in
evidence against him."

It is well-settled that the foregoing legal formalities required by the fundamental


law of the land apply only to extra-judicial confessions or admissions obtained
during custodial investigations. Indeed, the rights enumerated in the constitutional
provision "exist only in custodial interrogations, or in-custody interrogation of accused
persons."

Custodial interrogation is the questioning initiated by law enforcement officers after a


person has been taken into custody or otherwise deprived of his freedom of action in
any significant way.

In the present case, petitioner admits that the questioned statements were made
during the preliminary investigation, not during the custodial investigation.
However, he argues that the right to competent and independent counsel also applies
during preliminary investigations.

We disagree. A preliminary investigation is an inquiry or a proceeding to


determine whether there is sufficient ground to engender a well-founded belief
that a crime has been committed, and that the respondent is probably guilty
thereof and should be held for trial.

Evidently, a person undergoing preliminary investigation before the public


prosecutor cannot be considered as being under custodial investigation. In fact,
this Court has unequivocally declared that a defendant on trial or under
preliminary investigation is not under custodial interrogation.

There is no question that even in the absence of counsel, the admissions made by
petitioner in his Counter-Affidavit are not violative of his constitutional rights. It is
clear from the undisputed facts that it was not exacted by the police while he was
under custody or interrogation. Hence, the constitutional rights of a person under
custodial investigation as embodied in Article III, Section 12 of the 1987 Constitution,
are not at issue in this case.

*People v. Andan, G.R. No. 116437, March 3, 1997.


(Extrajudicial Confession; Custodial Investigation; Spontaneous Statements)

Facts:
-Accused-appellant Pablito Andan y Hernandez alias "Bobby" was accused of the crime
of rape with homicide.

-The day after the commission of the rape and homicide, people and media
representatives were already gathered at the police headquarters awaiting the results
of the investigation.

Page 88 of 323
-Mayor Trinidad arrived and proceeded to the investigation room. Upon seeing the
mayor, appellant approached him and whispered a request that they talk privately.

-In the presence of the mayor, the police, representatives of the media and appellant's
own wife and son, appellant confessed his guilt. He disclosed how he killed Marianne
and volunteered to show them the place where he hid her bags.

-After his confession, appellant hugged his wife and son and asked the mayor to help
him. His confession was captured on videotape and covered by the media nationwide.

-At the trial, appellant repudiated his confession, arguing that he was not assisted
by a counsel when he made the same.

Issues:
(1) Whether or not his confession to the Mayor is admissible in evidence;
(2) Whether or not his confession to the media persons is admissible in evidence.

(1) Yes.

Under these circumstances, it cannot be successfully claimed that appellant's


confession before the mayor is inadmissible. It is true that a municipal mayor has
"operational supervision and control" over the local police and may arguably be
deemed a law enforcement officer for purposes of applying Section 12 (1) and (3)
of Article III of the Constitution. However, appellant's confession to the mayor
was not made in response to any interrogation by the latter. In fact, the mayor
did not question appellant at all. No police authority ordered appellant to talk to the
mayor. It was appellant himself who spontaneously, freely and voluntarily
sought the mayor for a private meeting. The mayor did not know that appellant
was going to confess his guilt to him. When appellant talked with the mayor as a
confidant and not as a law enforcement officer, his uncounselled confession to
him did not violate his constitutional rights.

Thus, it has been held that the constitutional procedures on custodial


investigation do not apply to a spontaneous statement, not elicited through
questioning by the authorities, but given in an ordinary manner whereby
appellant orally admitted having committed the crime. What the Constitution bars
is the compulsory disclosure of incriminating facts or confessions. The rights under
Section 12 are guaranteed to preclude the slightest use of coercion by the state as
would lead the accused to admit something false, not to prevent him from freely and
voluntarily telling the truth. Hence, we hold that appellant's confession to the mayor
was correctly admitted by the trial court.

(2) Yes.

Appellant's confessions to the media were likewise properly admitted. The


confessions were made in response to questions by news reporters, not by the
police or any other investigating officer. We have held that statements
spontaneously made by a suspect to news reporters on a televised interview are
deemed voluntary and are admissible in evidence. x x x x

We rule that appellant's verbal confessions to the newsmen are not covered by
Section 12 (1) and (3) of Article III of the Constitution. The Bill of Rights does
not concern itself with the relation between a private individual and another
individual. It governs the relationship between the individual and the State. The
prohibitions therein are primarily addressed to the State and its agents. They
confirm that certain rights of the individual exist without need of any governmental
grant, rights that may not be taken away by government, rights that government has
the duty to protect. Governmental power is not unlimited and the Bill of Rights lays
down these limitations to protect the individual against aggression and unwarranted
interference by any department of government and its agencies.

Page 89 of 323
*Lavides v. Court of Appeals, G.R. No. 129670, February 1, 2000.
(Bail; Arraignment; Motion to Quash; Conditions)

Facts:
-During the criminal proceedings against petitioner Manolet Lavides for child abuse,
the trial court imposed four conditions for the grant of bail to petitioner, one of which
was: “the approval of the bail bonds shall be made only after the arraignment to
enable this Court to immediately acquire jurisdiction over the accused.”

Issue:
Whether or not the condition that “the approval of the bail bonds shall be made only
after the arraignment” is a valid condition.

Ruling:
No.

In requiring that petitioner be first arraigned before he could be granted bail, the trial
court apprehended that if petitioner were released on bail he could, by being absent,
prevent his early arraignment and thereby delay his trial until the complainants got
tired and lost interest in their cases. Hence, to ensure his presence at the
arraignment, approval of petitioner’s bail bonds should be deferred until he could
be arraigned. After that, even if petitioner does not appear, trial can proceed as long
as he is notified of the date of hearing and his failure to appear is unjustified, since
under Art. III, §14(2) of the Constitution, trial in absentia is authorized. This seems to
be the theory of the trial court in its May 16, 1997 order conditioning the grant of
bail to petitioner on his arraignment.

This theory is mistaken. In the first place, as the trial court itself acknowledged, in
cases where it is authorized, bail should be granted before arraignment, otherwise
the accused may be precluded from filing a motion to quash. For if the information
is quashed and the case is dismissed, there would then be no need for the
arraignment of the accused. In the second place, the trial court could ensure the
presence of petitioner at the arraignment precisely by granting bail and ordering
his presence at any stage of the proceedings, such as arraignment. Under Rule
114, §2(b) of the Rules on Criminal Procedure, one of the conditions of bail is that,
"the accused shall appear before the proper court whenever so required by the court or
these Rules," while under Rule 116, §1(b) the presence of the accused at the
arraignment is required.

On the other hand, to condition the grant of bail to an accused on his arraignment
would be to place him in a position where he has to choose between (1) filing a motion
to quash and thus delay his release on bail because until his motion to quash can be
resolved, his arraignment cannot be held, and (2) forgoing the filing of a motion to
quash so that he can be arraigned at once and thereafter be released on bail. These
scenarios certainly undermine the accused's constitutional right not to be put on trial
except upon valid complaint or information sufficient to charge him with a crime and
his right to bail.

It is the condition in the May 16, 1997 order of the trial court that "approval of the bail
bonds shall be made only after arraignment," which the Court of Appeals should
instead have declared void. The condition imposed in the trial court's order of May 16,
1997 that the accused cannot waive his appearance at the trial but that he must be
present at the hearings of the case is valid and is in accordance with Rule 114. For
another condition of bail under Rule 114, §2(c) is that "The failure of the accused to
appear at the trial without justification despite due notice to him or his bondsman
shall be deemed an express waiver of his right to be present on the date specified in
the notice. In such case, trial shall proceed in absentia." x x x x

Undoubtedly, the trial court knew this. Petitioner could delay the proceedings by
absenting himself from the arraignment. But once he is arraigned, trial could proceed

Page 90 of 323
even in his absence. So it thought that to ensure petitioner's presence at the
arraignment, petitioner should be denied bail in the meantime. The fly in the
ointment, however, is that such court strategy violates petitioner's constitutional
rights.

*Yap v. Court of Appeals, G.R. No. 141529, June 6, 2001.


(Right Against Excessive Bail; Estafa)

Facts:
-For misappropriating amounts equivalent to P5,500,000.00, petitioner was
convicted of estafa by the Regional Trial Court of Pasig City1 and was sentenced to
four years and two months of prision correctional, as minimum to eight years of
prision mayor as maximum, "in addition to one (1) year for each additional P10,000.00
in excess of P22,000.00 but in no case shall it exceed twenty (20) years."

-After the records of the case were transmitted to the Court of Appeals, petitioner filed
with the said court a Motion to Fix Bail For the Provisional Liberty of Accused
Appellant Pending Appeal. Asked to comment on this motion, the Solicitor General
opined that petitioner may be allowed to post bail in the amount of P5,500,000.00.

-The CA granted patitioner’s motion and allowed him to post bail in the amount of
P5,500,000.00.

Issue:
Whether or not the Court of Appeals committed grave abuse of discretion in basing the
bail for the provisional liberty of the petitioner on his civil liability.

Ruling:
Yes.

There is no question that in the present case the Court of Appeals exercised its
discretion in favor of allowing bail to petitioner on appeal. Respondent court stated
that it was doing so for "humanitarian reasons", and despite a perceived high risk of
flight, as by petitioner's admission he went out of the country several times during the
pendency of the case, for which reason the court deemed it necessary to peg the
amount of bail at P5,500,000.00.

The prohibition against requiring excessive bail is enshrined in the Constitution.


The obvious rationale, as declared in the leading case of De la Camara vs. Enage, is
that imposing bail in an excessive amount could render meaningless the right to
bail. Thus, in Villaseñor vs. Abano, this Court made the pronouncement that it will not
hesitate to exercise its supervisory powers over lower courts should the latter, after
holding the accused entitled to bail, effectively deny the same by imposing a
prohibitory sum or exacting unreasonable conditions.

At the same time, Section 9, Rule 114 of the Revised Rules of Criminal Procedure
advises courts to consider the following factors in the setting of the amount of bail:

(a) Financial ability of the accused to give bail;


(b) Nature and circumstances of the offense;
(c) Penalty for the offense charged;
(d) Character and reputation of the accused;
(e) Age and health of the accused;
(f) Weight of the evidence against the accused;
(g) Probability of the accused appearing at the trial;
(h) Forfeiture of other bail;
(i) The fact that the accused was a fugitive from justice when arrested; and
(j) Pendency of other cases where the accused is on bail.

Thus, the court has wide latitude in fixing the amount of bail. Where it fears that the
accused may jump bail, it is certainly not precluded from installing devices to

Page 91 of 323
ensure against the same. Options may include increasing the bail bond to an
appropriate level, or requiring the person to report periodically to the court and
to make an accounting of his movements. In the present case, where petitioner was
found to have left the country several times while the case was pending, the Court of
Appeals required the confiscation of his passport and the issuance of a hold-departure
order against him.

Under the circumstances of this case, we find that appropriate conditions have been
imposed in the bail bond to ensure against the risk of flight, particularly, the
combination of the hold-departure order and the requirement that petitioner inform
the court of any change of residence and of his whereabouts. Although an increase in
the amount of bail while the case is on appeal may be meritorious, we find that
the setting of the amount at P5,500,000.00 is unreasonable, excessive, and
constitutes an effective denial of petitioner's right to bail.

The purpose for bail is to guarantee the appearance of the accused at the trial, or
whenever so required by the Court. The amount should be high enough to assure the
presence of the accused when required but no higher than is reasonably calculated to
fulfill this purpose. To fix bail at an amount equivalent to the civil liability of
which petitioner is charged (in this case, P5,500,000.00) is to permit the
impression that the amount paid as bail is an exaction of the civil liability that
accused is charged of; this we cannot allow because bail is not intended as a
punishment, nor as a satisfaction of civil liability which should necessarily await
the judgment of the appellate court. x x x x

The importance attached to conviction is due to the underlying principle that bail
should be granted only where it is uncertain whether the accused is guilty or innocent,
and therefore, where that uncertainty is removed by conviction it would, generally
speaking, be absurd to admit to bail. After a person has been tried and convicted the
presumption of innocence which may be relied upon in prior applications is rebutted,
and the burden is upon the accused to show error in the conviction. From another
point of view it may be properly argued that the probability of ultimate punishment is
so enhanced by the conviction that the accused is much more likely to attempt to
escape if liberated on bail than before conviction. x x x x

Petitioner is seeking bail on appeal. He was in fact declared guilty beyond


reasonable doubt by the RTC, and due to the serious amount of fraud involved,
sentenced to imprisonment for twenty years --the maximum penalty for estafa by false
pretenses or fraudulent acts allowed by the Revised Penal Code. Although it cannot be
controverted that the Court of Appeals, despite the foregoing considerations and the
possibility of flight still wielded its discretion to grant petitioner bail, the setting of
bail in the amount of P5,500,000.00 is unjustified as having no legal nor factual
basis. Guided by the penalty imposed by the lower court and the weight of the
evidence against petitioner, we believe that the amount of P200,000.00 is more
reasonable.

*Feeder International Line v. Court of Appeals, G.R. No. 94262, May 31, 1991.
(Presumption of Innocence; Juridical Person; Corporate Entity)

Facts:
-The Court of Tax Appeals found the vessel M/T "ULU WAI" liable under Section
2530(a) of the Tariff and Customs Code of the Philippines (Presidential Decree No.
1464), as amended.

-On appeal, petitioner argued that it was deprived of property without due process of
law in that its right to be presumed innocent was not recognized and the decision
was not supported by proof beyond reasonable doubt.

Issue:
Whether or not petitioner may invoke its right to be presumed innocent.

Page 92 of 323
Ruling:
No.

Petitioner, which is a corporate entity, has no personality to invoke the right to


be presumed innocent which right is available only to an individual who is an
accused in a criminal case.

*Ilusorio v. Ilusorio, G.R. No. 139789. May 12, 2000.


(Remedial Law: Habeas Corpus; Mental State/Condition; Visitation Rights)

-On July 11, 1942, Erlinda Kalaw and Potenciano Ilusorio contracted matrimony and
lived together for a period of thirty (30) years. In 1972, they separated from bed and
board for undisclosed reasons.

-Potenciano lived at Urdaneta Condominium, Ayala Ave., Makati City when he was in
Manila and at Ilusorio Penthouse, Baguio Country Club when he was in Baguio City.
On the other hand, Erlinda lived in Antipolo City.

-For many years, lawyer Potenciano Ilusorio was Chairman of the Board and President
of Baguio Country Club.

-Out of their marriage, the spouses had six (6) children, namely: Ramon Ilusorio (age
55); Erlinda Ilusorio Bildner (age 52); Maximo (age 50); Sylvia (age 49); Marietta (age
48); and Shereen (age 39).

-On December 30, 1997, upon Potenciano’s arrival from the United States, he stayed
with Erlinda for about five (5) months in Antipolo City. The children, Sylvia and
Erlinda (Lin), alleged that during this time, their mother gave Potenciano an overdose
of 200 mg instead of 100 mg Zoloft, an antidepressant drug prescribed by his doctor in
New York, U.S.A. As a consequence, Potenciano’s health deteriorated.

-On February 25, 1998, Erlinda filed with the Regional Trial Court, Antipolo City a
petition for guardianship over the person and property of Potenciano Ilusorio due to
the latter’s advanced age, frail health, poor eyesight and impaired judgment.

-On May 31, 1998, after attending a corporate meeting in Baguio City, Potenciano
Ilusorio did not return to Antipolo City and instead lived at Cleveland Condominium,
Makati.

-On March 11, 1999, Erlinda filed with the Court of Appeals a petition for habeas
corpus to have the custody of lawyer Potenciano Ilusorio. She alleged that her
children, Erlinda Bildner and Sylvia, refused petitioner’s demands to see and visit her
husband and prohibited Potenciano from returning to Antipolo City.

-At the time of the petition, Potenciano Ilusorio is about 86 years of age possessed of
extensive property valued at millions of pesos.

-After due proceedings, the CA dismissed the petition for habeas corpus for lack of
unlawful restraint or detention of the subject of the petition. However, for
humanitarian consideration, the CA granted visitation rights in favor of Erlinda to
vist Potenciano in his condominium.

Issues:
(1) Whether or not the habeas corpus petition will prosper;
(2) Whether or not the CA’s grant of visitation rights was proper.

Ruling:
No.

A writ of habeas corpus extends to all cases of illegal confinement or detention,


or by which the rightful custody of a person is withheld from the one entitled

Page 93 of 323
thereto. It is available where a person continues to be unlawfully denied of one or
more of his constitutional freedoms, where there is denial of due process, where the
restraints are not merely involuntary but are unnecessary, and where a deprivation of
freedom originally valid has later become arbitrary. It is devised as a speedy and
effectual remedy to relieve persons from unlawful restraint, as the best and only
sufficient defense of personal freedom.

The essential object and purpose of the writ of habeas corpus is to inquire into
all manner of involuntary restraint, and to relieve a person therefrom if such
restraint is illegal. To justify the grant of the petition, the restraint of liberty must
be an illegal and involuntary deprivation of freedom of action. The illegal
restraint of liberty must be actual and effective, not merely nominal or moral.

The evidence shows that there was no actual and effective detention or
deprivation of lawyer Potenciano Ilusorio’s liberty that would justify the
issuance of the writ. The fact that lawyer Potenciano Ilusorio is about 86 years of
age, or under medication does not necessarily render him mentally incapacitated.
Soundness of mind does not hinge on age or medical condition but on the capacity of
the individual to discern his actions.

(2) No.

As to lawyer Potenciano Ilusorio’s mental state, the Court of Appeals observed that he
was of sound and alert mind, having answered all the relevant questions to the
satisfaction of the court.

Being of sound mind, he is thus possessed with the capacity to make choices. In
this case, the crucial choices revolve on his residence and the people he opts to see or
live with. The choices he made may not appeal to some of his family members but
these are choices which exclusively belong to Potenciano. He made it clear before
the Court of Appeals that he was not prevented from leaving his house or seeing
people. With that declaration, and absent any true restraint on his liberty, we have no
reason to reverse the findings of the Court of Appeals.

With his full mental capacity coupled with the right of choice, Potenciano
Ilusorio may not be the subject of visitation rights against his free choice.
Otherwise, we will deprive him of his right to privacy. Needless to say, this will run
against his fundamental constitutional right.

The Court of Appeals exceeded its authority when it awarded visitation rights in a
petition for habeas corpus where Erlinda never even prayed for such right. The ruling
is not consistent with the finding of subject’s sanity.

When the court ordered the grant of visitation rights, it also emphasized that the same
shall be enforced under penalty of contempt in case of violation or refusal to comply.
Such assertion of raw, naked power is unnecessary.

The Court of Appeals missed the fact that the case did not involve the right of a
parent to visit a minor child but the right of a wife to visit a husband. In case
the husband refuses to see his wife for private reasons, he is at liberty to do so
without threat of any penalty attached to the exercise of his right.

No court is empowered as a judicial authority to compel a husband to live with


his wife. Coverture cannot be enforced by compulsion of a writ of habeas corpus
carried out by sheriffs or by any other mesne process. That is a matter beyond judicial
authority and is best left to the man and woman’s free choice.

*Paredes v. Sandiganbayan, G.R. No. 89989, January 28, 1991.


(Habeas Corpus; Crim Pro: Preliminary Investigation; Motion to Quash)

Page 94 of 323
-On October 28, 1986, a criminal complaint charging Attorney Paredes with having
violated Section 3(a) of the Anti-Graft & Corrupt Practices Act (R.A. 3019) because
he allegedly used his office as Provincial Attorney to influence, persuade, and induce
Armando Luison, Land Inspector of the District Land Office in Agusan del Sur, to
favorably indorse Attorney Paredes’ free patent application.

-On February 23, 1987, the Tanodbayan (now Ombudsman) referred the case to Fiscal
Ernesto Brocoy of Butuan City (TBP Case No. 86-03368) for preliminary
investigation.

-Fiscal Brocoy issued summons to Attorney Paredes, Jr. to appear at the preliminary
investigation of the case on August 29, 1987. However, the summons was served on
November 19, 1987 upon the INP Station Commander of San Francisco, instead of
Attorney Paredes. The summons did not reach Attorney Paredes. Nevertheless,
without waiting for proof of service of the summons on the accused, Fiscal Brocoy
proceeded to conduct the preliminary examination of the complainant and his
witnesses.

-On August 29, 1988, the fiscal issued a resolution finding a prima facie case of
violation of Section 3(a) of R.A. 3019 committed by the accused. The Fiscal's resolution
was approved by Tanodbayan Prosecutor Josephine Fernandez.

-Attorney Paredes filed a motion for reconsideration of the Tanodbayan's resolution.


He assailed the validity of the preliminary investigation that was conducted by Fiscal
Brocoy without notice to him. His motion for reconsideration was denied.

-In the local elections on January 18, 1988, Attorney Paredes was elected governor of
Agusan del Sur.

-On August 28,1988, an information was filed against Governor Paredes in the
Sandiganbayan and a warrant for his arrest, fixing bail of P20,000.00 for his
provisional liberty, was issued on August 30, 1989 and served upon him. He refused
to post bail in “protest against the injustice to him as Governor.” Consequently, he
was detained in the municipal jail of San Francisco.

-On September 20, 1989, this petition for habeas corpus was filed by his wife, Mrs.
Eden Paredes, against the Sandiganbayan. She alleged that the warrant for her
husband's arrest was void because the preliminary investigation was void, and, that
the crime charged in the information against him had already prescribed.

Issue:
Whether or not the petition for habeas corpus will prosper.

Ruling:
No.

The settled rule is that the writ of habeas corpus will not issue where the person
alleged to be restrained of his liberty is in custody of an officer under a process
issued by the court which has jurisdiction to do so.

The petitioner alleges that the information against Governor Paredes is invalid because
the preliminary investigation was invalid and the offense charged has already
prescribed. Those circumstances do not constitute valid grounds for the issuance
of a writ of habeas corpus. The absence of a preliminary investigation does not
affect the court's jurisdiction over the case nor impair the validity of the
information or otherwise render it defective.

The remedy of the accused in such a case is to call the attention of the court to the
lack of a preliminary investigation and demand, as a matter of right, that one be
conducted. The court, instead of dismissing the information, should merely
suspend the trial and order the fiscal to conduct a preliminary investigation.

Page 95 of 323
The defense of prescription of offense is a proper ground for a motion to quash
which should be filed before the arraignment of the accused, for whether the crime
may still be prosecuted and penalized should be determined in the criminal case not
in a special proceeding of habeas corpus.

All questions which may arise in the orderly course of a criminal prosecution are to be
determined by the court to whose jurisdiction the defendant has been subjected by the
law, and the fact that a defendant has a good and sufficient defense to a criminal
charge on which he is held will not entitle him to his discharge on habeas
corpus.

[NOTE: Court’s ruling:


“WHEREFORE, finding no merit in the petition, the same is hereby denied. The
accused, Ceferino Paredes, Jr. should file a bail bond of P20,000.00 fixed by the
Sandiganbayan for his provisional liberty. Costs against the petitioner.”]

*Manalo v. Calderon, G.R. No. 178920, October 15, 2007.


(Habeas Corpus; Restrictive Custody; Remedial Law: Mootness)

-Petitioners were formerly police operatives assigned at the Regional Special


Operations Group, PNP Region 4-A, Camp Vicente Lim, Calamba City, Laguna.

-Respondents, meanwhile, were the Chief of the PNP, the Directorate for Investigation
and Detective Management, the Regional Director and Police Sr. Superintendents, at
the time of filing of the petition.

-On May 15, 2007, at around 3:00 a.m., five unidentified malefactors bearing high-
powered firearms suddenly appeared at the Barangay Pinagbayanan Elementary
School in the Municipality of Taysan, Province of Batangas. Earlier, the entire school
grounds were converted into a polling area for the 2007 national and local elections.
The five armed men forcibly entered Polling Precinct 76-A, and poured gasoline over a
ballot box. Then they fired several rounds of ammunitions at the premises, setting it
ablaze.

-In the investigation that ensued, several eyewitnesses identified some of petitioners
as the perpetrators of the school burning. The investigation also yielded that all six
petitioners, who are all members of the PNP Regional Special Operations Group (PNP-
RSOG), failed to timely respond to the incident at the Pinagbayanan Elementary
School.

-Acting on the report, the PNP hierarchy issued a memoranda putting petitioners
under restrictive custody.

-Petitioners then filed s petition for habeas corpus.

-Petitioners’ contentions: (1) Although technically speaking, petitioners as PNP officer


are not detained or imprisoned, their physical movements are, however, limited only
within Camp Vicente Lim, Calamba City, Laguna; they cannot go home to their
respective families and if they would leave Camp Vicente Lim they need to be
escorted; (2) What is only sanctioned is preventive suspension under which they can
enjoy liberty and go home to their families pending administrative investigation.

-During the pendency of the proceedings, the Director of PNP Regional Office 4-A,
recalled, effective immediately, the assailed restrictive custody order.

Issues:
(1) Whether or not the petition for habeas corpus will prosper;
(2) Whether or not the Court should dismiss the petition on the sole ground of
mootness.

Page 96 of 323
Ruling:
(1) No.

The main thrust of the special proceeding of habeas corpus is to inquire into the
legality of one's detention. More specifically, its vital purpose is to obtain immediate
relief from illegal confinement, to liberate those who may be imprisoned without
sufficient cause and to deliver them from unlawful custody.

Only if the Court is satisfied that a person is unlawfully restrained of his liberty will a
petition for habeas corpus be granted and the person detained released from
confinement. If respondents are not detaining nor restraining the applicants or
the person in whose behalf the petition for habeas corpus is filed, the petition
should perforce be dismissed.

Measured by the foregoing yardstick, the petition, on its face, fails to convince us
that petitioners are actually and unlawfully detained and restrained of their
liberty. Sombong v. Court of Appeals, et al. teaches us that for the writ of habeas
corpus to issue, the restraint of liberty must be in the nature of an illegal and
involuntary deprivation of freedom of action. More importantly, the prime
specification of an application for a writ of habeas corpus is an actual and
effective, and not merely nominal or moral, illegal restraint of liberty.

To the mind of the Court, petitioners are not illegally and involuntarily deprived of
their freedom of action.

Firstly, the assailed memoranda decreeing the monitoring of their movements


cannot, by any stretch of the imagination, be considered as a form of
curtailment of their freedom guaranteed under our Constitution.

Perusing the assailed memoranda, it is evident that petitioners are not actually
detained or restrained of their liberties. It is crystal-clear that petitioners are free to
go in and out of Camp Vicente Lim as they please. The only limitation imposed
upon them is that their movements within the premises of the camp shall be
monitored; that they have to be escorted whenever the circumstances warrant
that they leave the camp; and that their estimated time of departure and arrival
shall be entered in a logbook. Even petitioners themselves admit they are not
actually detained or imprisoned.

Secondly, the “restrictive custody” complained of by petitioners is, at best,


nominal restraint which is beyond the ambit of habeas corpus. It is neither actual
nor effective restraint that would call for the grant of the remedy prayed for. It is a
permissible precautionary measure to assure the PNP authorities that the police
officers concerned are always accounted for.

If said custodial procedure were not taken, respondent police superiors themselves
would have been exposed to charges of conspiracy, negligence or laxity in the
enforcement of internal discipline. If petitioners get lost or are able to go abroad or
figure in another untoward incident, respondents would have to explain why they did
not observe the needed precaution, else they would also be administratively liable.

Thirdly, petitioners’ reliance on Moncupa case is misplaced. In said case, petitioner


was ordered released by respondent but his release was saddled with restrictions.
There, petitioner was required to secure prior approval for: (a) any travel outside Metro
Manila; and (b) a change in residence. His freedom of speech was likewise muffled by a
prohibition on granting interviews to local or foreign media. He was likewise ordered to
report regularly to respondent.

In the case at bench, no restrictions in the nature of those imposed in Moncupa exist.
To reiterate, petitioners are merely held to account for their movements inside and
outside the camp’s premises. They are not required to secure prior approval before

Page 97 of 323
they can move out of the camp, only that each of them be accompanied by an
escort and their time of departure and arrival noted.

Fourthly, Republic Act (R.A.) No. 6975 (DILG Act of 1990), as amended by R.A. No.
8551 (PNP Reform and Reorganization Act of 1998), clearly provides that members
of the police force are subject to the administrative disciplinary machinery of
the PNP. Section 41(b) of the said law enumerates the disciplinary actions, including
restrictive custody that may be imposed by duly designated supervisors and
equivalent officers of the PNP as a matter of internal discipline, to wit:

“(b) Internal Discipline. – On dealing with minor offenses involving internal discipline found to have been
committed by any regular member of their respective commands, the duly designated supervisors and
equivalent officers of the PNP shall, after due notice and summary hearing, exercise disciplinary powers
as follows:
xxxx
(4) The Chief of the PNP shall have the power to impose the disciplinary punishment of dismissal from the
service; suspension or forfeiture of salary; or any combination thereof for a period not exceeding one
hundred eighty (180) days: Provided, further, That the chief of the PNP shall have the authority to
place police personnel under restrictive custody during the pendency of a grave administrative
case filed against him or even after the filing of a criminal complaint, grave in nature, against such
police personnel.”

Clearly, placing police officers facing a grave administrative case under


restrictive custody is a disciplinary measure authorized under the PNP law. Thus,
petitioners’ claim that their restrictive custody is an illegal practice “not sanctioned by
any existing provision of our constitution and laws” is not true. It must necessarily
fail.

Lastly, petitioners contend that by placing them under restrictive custody, they are
made to suffer lesser rights than those enjoyed by private citizens. On this score, the
Court’s pronouncement in Canson, et al. v. Hidalgo, et al. is categorical. It was held
there that although the PNP is civilian in character, its members are subject to
the disciplinary authority of the Chief, Philippine National Police, under the
National Police Commission. Courts cannot, by injunction, review, overrule or
otherwise interfere with valid acts of police officials. The police organization must
observe self-discipline and obey a chain of command under civilian officials.

Elsewise stated, police officers are not similarly situated with ordinary civil
service employees. The PNP has its own administrative disciplinary mechanism
different from those of other government employees. x x x x

The ultimate purpose of the writ of habeas corpus is to relieve a person from unlawful
restraint. The writ cannot and will not issue absent a showing that petitioners are
deprived of their liberty. Neither can it relieve petitioners, who are police
officers, from the valid exercise of prescribed discipline over them by the PNP
leadership.

(2) No.

Notwithstanding the mootness of the issues on restrictive custody and monitoring of


movements of petitioners, We opt to resolve them given: (a) the paramount public
interest involved, (b) their susceptibility of recurring yet evading review and (c)
the imperative need to educate the police community on the matter.

The release of petitioners by respondents in a petition for habeas corpus does not
automatically abate a decision on the case. Similarly, a recall of the custody order
challenged by petitioners will not necessarily call for a dismissal on the ground of
mootness alone. Although the general rule is mootness of the issue warrants a
dismissal, there are well-defined exceptions.

In the habeas corpus case of Aquino, Jr. v. Enrile, twenty-six (26) petitioners were
released from custody and one withdrew during the pendency of the petition. The fact

Page 98 of 323
that the petition was rendered moot and academic did not prevent this Court in the
exercise of its symbolic function from promulgating one of the most voluminous
decisions ever.

In Tibo v. The Provincial Commander and Toyoto, et al. v. Ramos, et al., respondents
filed a motion to dismiss the petition for habeas corpus on the ground that petitioners
had been temporarily released and their case had, therefore, become moot and
academic. This Court chose to decide the said cases. The Court sustained petitioners’
plea that their case be considered moot and academic only “if their release would be
permanent.”

*Feria v. Court of Appeals, G.R. No. 122954, February 15, 2000.


(Habeas Corpus; Loss of Judicial Records; Reconstitution of Judicial Records)

Facts:
-Petitioner Norberto Feria y Pacquing has been under detention since May 21, 1981,
up to present by reason of his conviction of the crime of Robbery with Homicide, in
Criminal Case No. 60677, by the Regional Trial Court of Manila, Branch 2, for the
jeepney hold-up and killing of United States Peace Corps Volunteer Margaret Viviene
Carmona.

-Some twelve (12) years later, or on June 9, 1993, petitioner sought to be transferred
from the Manila City Jail to the Bureau of Corrections in Muntinlupa City, but the Jail
Warden of the Manila City Jail informed the Presiding Judge of the RTC-Manila,
Branch 2, that the transfer cannot be effected without the submission of the
requirements, namely, the Commitment Order or Mittimus, Decision, and Information.

-It was then discovered that the entire records of the case, including the copy of the
judgment, were missing. In response to the inquiries made by counsel of petitioner,
both the Office of the City Prosecutor of Manila and the Clerk of Court of Regional
Trial Court of Manila, Branch 2 attested to the fact that the records of Criminal Case
No. 60677 could not be found in their respective offices. Upon further inquiries, the
entire records appear to have been lost or destroyed in the fire which occurred at the
second and third floor of the Manila City Hall on November 3, 1986.

-On October 3, 1994, petitioner filed a Petition for the Issuance of a Writ of Habeas
Corpus with the Supreme Court against the Jail Warden of the Manila City Jail, the
Presiding Judge of Branch 2, Regional Trial Court of Manila, and the City Prosecutor
of Manila, praying for his discharge from confinement on the ground that his
continued detention without any valid judgment is illegal and violative of his
constitutional right to due process.

Petitioners’ contentions: (1) His detention is illegal because there exists no copy of a
valid judgment as required by Sections 1 and 2 of Rule 120 of the Rules of Court; (2)
It is not the fault of the prisoners that the records cannot now be found. If anyone is to
be blamed, it surely cannot be the prisoners, who were not the custodians of those
records.

Issue:
Whether or not the petition for habeas corpus will prosper.

Ruling:
No.

The high prerogative writ of habeas corpus, whose origin is traced to antiquity, was
devised and exists as a speedy and effectual remedy to relieve persons from unlawful
restraint, and as the best and only sufficient defense of personal freedom. It secures
to a prisoner the right to have the cause of his detention examined and
determined by a court of justice, and to have the issue ascertained as to whether he
is held under lawful authority. Consequently, the writ may also be availed of where,
as a consequence of a judicial proceeding, (a) there has been a deprivation of a

Page 99 of 323
constitutional right resulting in the restraint of a person, (b) the court had no
jurisdiction to impose the sentence, or (c) an excessive penalty has been
imposed, as such sentence is void as to such excess. Petitioner's claim is anchored
on the first ground considering, as he claims, that his continued detention,
notwithstanding the lack of a copy of a valid judgment of conviction, is violative of his
constitutional right to due process.

Based on the records and the hearing conducted by the trial court, there is sufficient
evidence on record to establish the fact of conviction of petitioner which serves as the
legal basis for his detention. Petitioner made judicial admissions, both verbal and
written, that he was charged with and convicted of the crime of Robbery with
Homicide, and sentenced to suffer imprisonment “habang buhay.” x x x x

The records also contain a certified true copy of the Monthly Report dated January
1985 of then Judge Rosalio A. De Leon, attesting to the fact that petitioner was
convicted of the crime of Robbery with Homicide on January 11, 1985. Such
Monthly Report constitutes an entry in official records under Section 44 of Rule
130 of the Revised Rules on Evidence, which is prima facie evidence of facts
therein stated. x x x x

As a general rule, the burden of proving illegal restraint by the respondent rests on the
petitioner who attacks such restraint. In other words, where the return is not subject
to exception, that is, where it sets forth process which on its face shows good ground
for the detention of the prisoner, it is incumbent on petitioner to allege and prove new
matter that tends to invalidate the apparent effect of such process. If the detention
of the prisoner is by reason of lawful public authority, the return is considered
prima facie evidence of the validity of the restraint and the petitioner has the
burden of proof to show that the restraint is illegal. Thus, Section 13 of Rule 102
of the Rules of Court provides:

“Sec. 13. When the return evidence, and when only a plea. — If it appears that the prisoner is in
custody under a warrant of commitment in pursuance of law, the return shall be considered prima
facie evidence of the cause of restraint, but if he is restrained of his liberty by any alleged private
authority, the return shall be considered only as a plea of the facts therein set forth, and the party
claiming the custody must prove such facts.”

Public respondents having sufficiently shown good ground for the detention,
petitioner's release from confinement is not warranted under Section 4 of Rule
102 of the Rules of Court which provides that:

“Sec. 4. When writ not allowed or discharge authorized. — If it appears that the person alleged to be
restrained of his liberty is in the custody of an officer under process issued by a court or judge or
by virtue of a judgment or order of a court of record, and that the court or judge had jurisdiction
to issue the process, render the judgment, or make the order, the writ shall not be allowed; or if the
jurisdiction appears after the writ is allowed, the person shall not be discharged by reason of any
informality or defect in the process, judgment, or order. Nor shall anything in this rule be held to
authorize the discharge of a person charged with or convicted of an offense in the Philippines, or of a
person suffering imprisonment under lawful judgment.”

Note further that, in the present case, there is also no showing that petitioner duly
appealed his conviction of the crime of Robbery with Homicide, hence for all intents
and purposes, such judgment has already become final and executory. When a
court has jurisdiction of the offense charged and of the party who is so charged, its
judgment, order, or decree is not subject to collateral attack by habeas corpus. Put
another way, in order that a judgment may be subject to collateral attack by
habeas corpus, it must be void for lack of jurisdiction. Thus, petitioner's invocation
of our ruling in Reyes v. Director of Prisons, supra, is misplaced. In the Reyes case, we
granted the writ and ordered the release of the prisoner on the ground that, “it does
not appear that the prisoner has been sentenced by any tribunal duly established by a
competent authority during the enemy occupation" and not because there were no
copies of the decision and information. Here, a copy of the mittimus is available. And,
indeed, petitioner does not raise any jurisdictional issue.

Page 100 of 323


The proper remedy in this case is for either petitioner or public respondents to
initiate the reconstitution of the judgment of the case under either Act No.
3110, the general law governing reconstitution of judicial records, or under the
inherent power of courts to reconstitute at any time the records of their finished cases
in accordance with Section 5 (h) of Rule 135 of the Rules of Court. Judicial records
are subject to reconstitution without exception, whether they refer to pending
cases or finished cases. There is no sense in limiting reconstitution to pending cases;
finished cases are just as important as pending ones, as evidence of rights and
obligations finally adjudicated.

*Roquero v. The Chancellor of UP-Manila, G.R. No. 181851,March 9, 2010.


(Speedy Disposition of Cases; Administrative Case)

Facts:
-Petitioner Wildredo G. Roquero is an employee of UP-Manila assigned at the
Philippine General Hospital (PGH) Security Division as Special Police Captain. Private
respondent Imelda O. Abutal is a Lady Guard of Ex-Bataan Security Agency who was
applying for a position in the security force assigned at UP-PGH.

-On October 1, 1998, Abutal charged Roquero with Grave Misconduct before the
Administrative Disciplinary Tribunal (ADT) of UP.

-During the proceedings, the prosecution presented its only witness, private
respondent Abutal. After the completion of the cross-examination on the prosecution’s
only witness, the prosecution agreed to submit its Formal Offer of Evidence on or
before 16 July 1999.

-The prosecution, however, failed to submit its formal offer of evidence within the
period agreed upon.

-On 22 October 1999, petitioner filed a motion through counsel praying that
complainant (private respondent herein) be declared to have waived her rights to
formally offer her exhibits since complainant was not able to file her Formal Offer
within the given period of fifteen (15) days from 1 July 1999 or up to 16 July 1999.

-The ADT was not able to act on the said Motion for almost five (5) years.

-On 19 May 2004, due to the unreasonable delay, petitioner filed another motion
asking for the dismissal of the administrative case against him.

-The Motion to Dismiss was anchored on the following reasons: that the prosecution
had not formally offered its evidence; that the ADT had failed to act on the motion filed
on 22 October 1999; that the unfounded charges in the administrative complaint were
filed just to harass him; and that he is entitled to a just and speedy disposition of the
case.

-On 26 May 2004, the prosecution filed its Comment/Opposition to the Motion to
Dismiss. The prosecution alleged that a Formal Offer of Documentary Exhibits had
been filed on 24 January 2004, of which a copy thereof was received by Atty. Lee,
petitioner’s counsel, on 30 January 2004, per registry return receipt. However,
petitioner has not filed his comment to the said Formal Offer.

-Petitioner then filed with the Court of Appeals a Petition for Certiorari under Rule 65,
alleging therein that the ADT committed grave abuse of discretion when it denied the
motion to dismiss the administrative case filed against him.

-The CA denied petitioner’s petition, ratiocinating that, “Section 27 of the Uniform


Rules states that the failure to file a formal offer of evidence amounts to a mere waiver
thereof, and not a dismissal of the action. As such, petitioner cannot claim a vested
right to a dismissal of his case below just because a formal offer was not filed within
the agreed period.”

Page 101 of 323


-Petitioner then went to the SC via a petition for review on certiorari under Rule 45.

Issues:
(1) Whether or not the failure of the ADT to resolve Roquero’s Motion (to declare
complainant Imelda Abutal to have waived her right to submit her Formal Offer of
Exhibit) after almost five years violated the constitutional right of Roquero to a speedy
disposition of cases;

(2) Whether or not the right to speedy disposition of cases applies to administrative
cases.

Ruling:
(1) Yes.

The Court of Appeals faulted petitioner for his failure to present his own evidence
which “he could have done as early as 11 August 1999.” It must be noted, however,
that petitioner’s 22 October 1999 motion to declare complainant to have waived
her right to submit her Formal Offer of Exhibit remained unresolved. This is
reason enough for Roquero to defer presentation of his own evidence.

Indeed, while Section 27 of the Uniform Rules on Administrative Cases in Civil Service
states that the failure to submit the formal offer of evidence within the given period
shall be considered as waiver thereof, the ADT in fact allowed the prosecution to
present its formal offer almost five (5) years later or on 24 January 2004. Starting
on that date, petitioner was presented with the choice to either present his evidence or
to, as he did, file a motion to dismiss owing to the extraordinary length of time that
ADT failed to rule on his motion.

We cannot accept the finding of the Court of Appeals that there was no grave abuse of
discretion on the part of the ADT because "a formal offer of evidence was filed by the
prosecution, a copy of which was received by petitioners’ counsel." The admission by
ADT on 8 June 2004 of the formal offer of exhibits belatedly filed did not cure
the 5-year delay in the resolution of petitioner’s 1999 motion to deem as waived
such formal offer of evidence. Indeed, the delay of almost five (5) years cannot be
justified. x x x x

While it is true that administrative investigations should not be bound by strict


adherence to the technical rules of procedure and evidence applicable to judicial
proceedings, the same however should not violate the constitutional right of
respondents to a speedy disposition of cases.

Section 16, Article III of the 1987 Constitution provides:

Section 16. All person shall have the right to a speedy disposition of their cases before all judicial, quasi-
judicial, or administrative bodies.

The doctrinal rule is that in the determination of whether that right has been violated,
the factors that may be considered and balanced are as follows: (1) the length of
delay; (2) the reasons for the delay; (3) the assertion or failure to assert such
right by the accused; and (4) the prejudice caused by the delay.

Applying the doctrinal ruling vis-a-vis the factual milieu of this case, the violation of
the right to a speedy disposition of the case against petitioner is clear for the following
reasons: (1) the delay of almost five (5) years on the part of ADT in resolving the
motion of petitioner, which resolution petitioner reasonably found necessary before he
could present his defense; (2) the unreasonableness of the delay; and (3) the timely
assertions by petitioner of the right to an early disposition which he did through a
motion to dismiss. Over and above this, the delay was prejudicial to petitioner’s cause
as he was under preventive suspension for ninety (90) days, and during the

Page 102 of 323


interregnum of almost five years, the trial of the accusation against him remained
stagnant at the prosecution stage.

The Constitutional guarantee against unreasonable delay in the disposition of cases


was intended to stem the tide of disenchantment among the people in the
administration of justice by our judicial and quasi-judicial tribunals. The adjudication
of cases must not only be done in an orderly manner that is in accord with the
established rules of procedure but must also be promptly decided to better serve the
ends of justice. Excessive delay in the disposition of cases renders the rights of
the people guaranteed by the Constitution and by various legislations inutile.

(2) Yes.

The constitutional right to a “speedy disposition of cases” is not limited to the


accused in criminal proceedings but extends to all parties in all cases, including
civil and administrative cases, and in all proceedings, including judicial and
quasi-judicial hearings. Hence, under the Constitution, any party to a case may
demand expeditious action by all officials who are tasked with the administration of
justice.

*People v. Dacuycuy, G.R. No. L-45127, May 5, 1989.


(Prohibited Punishments; Undeterminable Period of Imprisonment)

Facts:
-Respondents were criminally charged before the MTC with Violation of Republic Act
No. 4670, otherwise known as The Magna Carta for Public School Teachers.

-While the said case was pending, respondents filed a petition for certiorari and
prohibition with preliminary injunction before the RTC, to restrain the MTC judge
from proceeding with the trial of said criminal case upon the ground that the MTC had
no jurisdiction over the offense charged.

-Respondents likewise argued that Section 32 of said law is unconstitutional as the


duration of the penalty of imprisonment has neither a minimum nor a maximum
duration, and thus is solely left to the discretion of the court as if the latter were the
legislative department of the Government.

-Section 32 of the Republic Act No. 4670 provides:

“Sec. 32. Penal Provision. — A person who shall willfully interfere with, restrain or coerce any teacher in
the exercise of his rights guaranteed by this Act or who shall in any other manner commit any act to
defeat any of the provisions of this Act shall, upon conviction, be punished by a fine of not less than
one hundred pesos nor more than one thousand pesos, or by imprisonment, in the discretion of
the court.”

-After due proceedings, the RTC ruled that the said Section is constitutional,
ratiocinating that since the penalty of imprisonment has been provided for by the
legislature, the court is endowed with the discretion to ascertain the term or
period of imprisonment.

Issues:
(1) Whether or not Section 32 of Republic Act No. 4670 is unconstitutional;
(2) Whether or not the MTC has jurisdiction over the case.

Ruling:
(1) Yes.

Under Section 32, of Republic Act No. 4670, two alternative and distinct
penalties are consequently imposed, to wit: (a) a fine ranging from P100.00 to
P1,000.00; or (b) imprisonment. It is apparent that the law has no prescribed period
or term for the imposable penalty of imprisonment. While a minimum and maximum
amount for the penalty of fine is specified, there is no equivalent provision for

Page 103 of 323


the penalty of imprisonment, although both appear to be qualified by the phrase “in
the discretion of the court.” x x x x

It is not for the courts to fix the term of imprisonment where no points of
reference have been provided by the legislature. What valid delegation
presupposes and sanctions is an exercise of discretion to fix the length of service
of a term of imprisonment which must be encompassed within specific or
designated limits provided by law, the absence of which designated limits well
constitute such exercise as an undue delegation, if not an outright intrusion into
or assumption, of legislative power.

Section 32 of Republic Act No. 4670 provides for an indeterminable period of


imprisonment, with neither a minimum nor a maximum duration having been set by
the legislative authority. The courts are thus given a wide latitude of discretion to
fix the term of imprisonment, without even the benefit of any sufficient
standard, such that the duration thereof may range, in the words of respondent
judge, from one minute to the life span of the accused. Irremissibly, this cannot be
allowed. It vests in the courts a power and a duty essentially legislative in nature
and which, as applied to this case, does violence to the rules on separation of
powers as well as the non-delegability of legislative powers. This time, the
preumption of constitutionality has to yield.

On the foregoing considerations, and by virtue of the separability clause in Section 34


of Republic Act No. 4670, the penalty of imprisonment provided in Section 32 thereof
should be, as it is hereby, declared unconstitutional.

(2) Yes.

With the deletion by invalidation of the provision on imprisonment in Section 32 of


Republic Act No. 4670, as earlier discussed, the imposable penalty for violations of
said law should be limited to a fine of not less than P100.00 and not more than
P1,000.00, the same to serve as the basis in determining which court may properly
exercise jurisdiction thereover.

When the complaint against private respondents was filed in 1975, the pertinent law
then in force was Republic Act No. 296, as amended by Republic Act No. 3828, under
which crimes punishable by a fine of not more than P 3,000.00 fall under the
original jurisdiction of the municipal courts. Consequently, Criminal Case No. 555
against herein private respondents falls within the original jurisdiction of the
Municipal Trial Court of Hindang, Leyte.

*People v. Hon. Villarama, G.R. No. 99287, June 23, 1992. [IMPORTANT!!!]
(Crim Pro: Plea Bargaining; Double Jeopardy)

Facts:
-On August 24, 1990, Jaime Manuel y Ohide was charged with violation of Section 16
(illegal possession of shabu), Republic Act No. 6425, as amended. The penalty
prescribed in the said section is imprisonment ranging from six years and one day to
twelve years and a fine ranging from six thousand to twelve thousand pesos.

-Trial then ensued. On November 21, 1990, the prosecution rested its case.

-On January 9, 1991, counsel for private respondent verbally manifested in open court
that private respondent was willing to change his former plea of "not guilty" to that of
"guilty" to the lesser offense of violation of Section 17, R.A. No. 6425, as amended.
The said section provides a penalty of imprisonment ranging from six months and one
day to four years and a fine ranging from six hundred to four thousand pesos.

-That same day, the respondent Judge issued an order directing private respondent to
secure the consent of the prosecutor to the change of plea, and set the promulgation
of decision on January 30, 1991.

Page 104 of 323


-The prosecutor filed his Opposition to the Request to Plead Guilty to a Lesser Offense
on the grounds that: (1) the prosecution already rested its case; (2) the possibility of
conviction of private respondent of the crime originally charged was high because of
the strong evidence of the prosecution; and (3) the valuable time which the court and
the prosecutor had expended would be put to waste.

-Accused’s contentions: (1) The consent of the fiscal is not required in crimes
involving violation of RA 6425 because there is no offended party to speak of; (2) His
conviction to the lesser offense of violation of Section 17, RA No. 6425 as amended is
no longer open to review otherwise his constitutional right against double jeopardy
will be violated.

-Despite opposition by the fiscal, the judge granted the motion of the accused and
convicted the latter of the lesser offense.

-Aggrieved, the prosecutor then went to the SC.

Issues:
(1) Whether or not the plea bargaining was valid despite opposition by the prosecutor;
(2) Whether or not the the SC’s review of the criminal case against the herein accused
would put him in double jeopardy.

Ruling:
(1) No.

Plea bargaining in criminal cases, is a process whereby the accused and the
prosecution work out a mutually satisfactory disposition of the case subject to court.
It usually involves the defendant's pleading guilty to a lesser offense or to only one or
some of the counts of a multi-count indictment in return for a lighter sentence than
that for the graver charge.

However, the acceptance of an offer to plead guilty to a lesser offense under the
aforequoted rule is not demandable by the accused as a matter of right but is a matter
that is addressed entirely to the sound discretion of the trial court. x x x x

The provision of Section 2, Rule 116 is clear. The consent of both the Fiscal and the
offended party is a condition precedent to a valid plea of guilty to a lesser
offense. The reason for this is obvious. The Fiscal has full control of the prosecution
of criminal actions. Consequently, it is his duty to always prosecute the proper
offense, not any lesser or graver one, when the evidence in his hands can only sustain
the former.

It would not also be correct to state that there is no offended party in crimes under RA
6425 as amended. While the acts constituting the crimes are not wrong in themselves,
they are made so by law because they infringe upon the rights of others. The threat
posed by drugs against human dignity and the integrity of society is malevolent
and incessant. Such pernicious effect is felt not only by the addicts themselves but
also by their families. As a result, society’s survival is endangered because its basic
unit, the family, is the ultimate victim of the drug menace. The state is, therefore,
the offended party in this case. As guardian of the rights of the people, the
government files the criminal action in the name of the People of the Philippines. The
Fiscal who represents the government is duty bound to defend the public interests,
threatened by crime, to the point that it is as though he were the person directly
injured by the offense. Viewed in this light, the consent of the offended party, i.e.
the state, will have to be secured from the Fiscal who acts in behalf of the
government.

(2) No.

Page 105 of 323


The right against double jeopardy given to the accused in Section 2, Rule 116 of
the Rules of Court applies in cases where both the fiscal and the offended party
consent to the private respondent's change of plea. Since this is not the situation
here, the private respondent cannot claim this privilege. Instead, the more pertinent
and applicable provision is that found in Section 7, Rule 117 which states:

“Sec. 7. Former conviction or acquittal; double jeopardy. —


xxxx
However, the conviction of the accused shall not be a bar to another prosecution for an offense which
necessarily includes the offense charged in the former complaint or information under any of the following
instances:

(a) x x x x
(b) x x x x
(c) the plea of guilty to the lesser offense was made without the consent of the Fiscal and of the
offended party; x x x x”

Under this rule, the private respondent could still be prosecuted under the original
charge of violation of Section 16 of RA 6425 as amended because of the lack of
consent of the Fiscal who also represents the offended party, i.e., the state. More
importantly, the trial court's approval of his change of plea was irregular and
improper.

*Paulin v. Hon. Gimenez, G.R. No. 103323, January 21, 1993.


(Double Jeopardy; Dismissal; Acquittal; Consent of the Accused)

Facts:
-A commotion occurred wherein petitioners allegedly pointed a gun at complainant
Mabuyo. In view of this incident, petitioners were charged with Grave Threats by
Station Commander P/Lt. Ariel Palcutos.

-During the initial hearing, upon motion of petitioners, the MTC dismissed the case, it
appearing that the proper charge should have been, “disturbance of public
performance,” punishable under Article 153 of the Revised Penal Code instead of
"grave threats" under Article 282.

-Complainant Mabuyo filed a motion for reconsideration of the said dismissal order,
which the court granted in a resolution dated July 3, 1990.

-Petitioners vigorously sought the setting aside of the July 3, 1990 order, but the MTC
denied the same.

-Feeling aggrieved, petitioners filed a petition for certiorari, prohibition, damages,


with relief for preliminary injunction and the issuance of a temporary restraining order
with the RTC, which was later on denied by the said court.

-Petitioners then sought relief from the SC.

-Contention of petitioners: The decision of the municipal trial court dated June 13,
1990 dismissing the case against them was a judgment of acquittal, and may no
longer be set aside without violating petitioners’ right against double jeopardy.

Issues:
(1) Whether or not the setting aside of the earlier order dismissing the criminal case
violated petitioners’ right against double jeopardy;
(2) Whether or not the dismissal herein is tantamount to an acquittal.

Ruling:
(1) No.

For double jeopardy to attach, the dismissal of the case must be without the
express consent of the accused. Where the dismissal was ordered upon motion or

Page 106 of 323


with the express assent of the accused, he is deemed to have waived his
protection against double jeopardy. In the case at bar, the dismissal was granted
upon motion of petitioners. Double jeopardy thus did not attach. This doctrine of
waiver of double jeopardy was examined and formally introduced in People v. Salico,
where Justice Felicisimo Feria stated:

“When the case is dismissed, with the express consent of the defendant, the dismissal
will not be a bar to another prosecution for the same offense; because his action in
having the case dismissed constitutes a waiver of his constitutional right or
privilege, for the reason that he thereby prevents the court from proceeding to the
trial on the merits and rendering a judgment of conviction against him.” x x x x

(2) No.

Petitioners insist that the June 13, 1990 decision of the MTC is an acquittal since it
was issued after it had allegedly considered the merits of the prosecution's evidence.

In People v. Salico, distinctions between acquittal and dismissal were made, to wit:

Acquittal is always based on the merits, that is, the defendant is acquitted because
the evidence does not show that defendant's guilt is beyond reasonable doubt; but
dismissal does not decide the case on the merits or that the defendant is not
guilty. Dismissals terminate the proceedings, either because the court is not a
court of competent jurisdiction, or the evidence does not show that the offense
was committed within the territorial jurisdiction of the court, or the complaint
or information is not valid or sufficient in form and substance, etc.

The MTC decision dismissing the case is not an acquittal from the charge
considering that no finding was made as to the guilt or innocence of the
petitioners.

Under Section 14, Rule 110 of the 1985 Rules on Criminal Procedure, as amended, it
is stated:

“Sec. 14. Amendments. —

If it appears at any time before judgment that a mistake has been made in charging the proper
offense, the court shall dismiss the original complaint or information upon the filing of a new one
charging the proper offense in accordance with Rule 119, Sec. 11 . . .”

In Section 11 of the same Rule, it is provided:

“When it becomes manifest at any time before judgment, that a mistake has been made in charging the
proper offense, and the accused cannot be convicted of the offense charged, or of any other offense
necessarily included therein, the accused shall not be discharged, if there appears to be good cause to
detain him. In such case, the court shall commit the accused to answer for the proper offense and
dismiss the original case upon the filing of the proper information.”

In the case at bar, the original case was dismissed without the proper
information having been filed, it appearing that the proper charge should have
been, "disturbance of public performance," punishable under Article 153 of the
Revised Penal Code instead of "grave threats," under Article 282 of the same
penal code. x x x x

Petitioners’ motion to dismiss premised on procedural grounds cannot be


considered a demurrer to evidence nor was the dismissal sought by them predicated
on the denial of their right to speedy trial. Hence, the exceptions mentioned find no
application in the instant case, especially so because when the municipal trial court
dismissed the case upon petitioners’ motion, the prosecution still had to present
several witnesses.

*Tupaz v. Hon. Ulep, G.R. No. 127777, October 1, 1999.


(Double Jeopardy; Express Consent; Palpable Mistake)

Page 107 of 323


Facts:
-Petronila C. Tupaz, as a corporate officer of El Oro Engravers Corporation, was
charged before the RTC with nonpayment of deficiency corporate income tax for
the year 1979, amounting to P2,369,085.46.

-On May 20, 1996, Prosecutor Agcaoili filed with the RTC a motion for withdrawal of
the information, which Judge Ulep eventually granted; hence, the case was
dismissed, as prayed for by the prosecution.

-On May 28, 1996, Prosecutor Agcaoili filed with the RTC a motion to reinstate
information, stating that the motion to withdraw information was made through
palpable mistake, and was the result of excusable neglect. He thought that Criminal
Case No. Q-91-17321 was identical to Criminal Case No. Q-90-12896, wherein Tupaz
was charged with nonpayment of deficiency contractor's tax amounting to
P346,879.29.

-Over the vehement objections of accused on the ground of double jeopardy, the RTC
granted the motion and ordered that the information be reinstated.

-The accused then went to the SC via special civil action for certiorari with application
for temporary restraining order.

-Contention of the prosecutor: Assuming arguendo that the case was dismissed
without Tupaz’s consent, there was no valid dismissal of the case since Prosecutor
Agcaoili was under a mistaken assumption that it was a charge of nonpayment of
contractor's tax.

Issue:
Whether or not the reinstatement of the information would place the accused herein in
double jeopardy.

Ruling:
Yes.

The reinstatement of the information would expose her to double jeopardy. An


accused is placed in double jeopardy if he is again tried for an offense for which
he has been convicted, acquitted or in another manner in which the indictment
against him was dismissed without his consent.

In the instant case, there was a valid complaint filed against petitioner to which
she pleaded not guilty. The court dismissed the case at the instance of the
prosecution, without asking for accused-petitioner's consent. This consent
cannot be implied or presumed. Such consent must be expressed as to have no
doubt as to the accused’s conformity.

As petitioner's consent was not expressly given, the dismissal of the case must
be regarded as final and with prejudice to the re-filing of the case. Consequently,
the trial court committed grave abuse of discretion in reinstating the information
against petitioner in violation of her constitutionally protected right against double
jeopardy.

*People v. Alberto, G.R. No. 132374, August 22, 2002.


(Double Jeopardy; Crim Pro: Formal Offer of Exhibits; Due Process)

Facts:
- Accused Lucio Alberto was charged with robbery with homicide.

-Trial ensued. The prosecution presented three witnesses.

Page 108 of 323


-On June 26, 1996, the trial court issued an order dismissing the case for failure of
the prosecution to submit its formal offer of exhibits. The said order was lifted after
the prosecution filed a motion for reconsideration on July 1, 1996. Thereafter, the
prosecution continued to present its evidence. Two more witnesses were presented by
the prosecution.

-On March 26, 1997, the defense orally asked for leave to file demurrer to evidence.
On April 25, 1997, the demurrer was filed but it was denied on May 13, 1997. On
June 25, 1997, the trial court issued an order declaring that the accused should be
deemed to have waived his right to present evidence for the defense, and that the case
be considered submitted for decision.

-On August 21, 1997, the trial court promulgated its judgment convicting Alberto.
Hence, this appeal.

-Accused’s contention (one of the errors assigned): The trial court gravely erred in
reinstating the case after it had dismissed the same for failure of the State to submit
its Formal Offer of Exhibits for almost a year when required by the trial court,
thereby placing him in double jeopardy.

Issue:
Whether or not the accused was placed in double jeopardy.

Ruling:
No.

The three requisites before double jeopardy can be invoked are: (1) the first jeopardy
must have attached prior to the second; (2) the first jeopardy must have been validly
terminated; and (3) the second jeopardy must be for the same offense as that in the
first, or the second offense includes or is necessarily included in the offense charged in
the first information, or is an attempt to commit the same or is a frustration thereof.
As to the first jeopardy, it only arises (1) upon a valid indictment; (2) before a
competent court; (3) after arraignment; (4) when a valid plea has been entered; and (5)
when the defendant was acquitted, convicted, or the case was dismissed.

In our view, it is clear that no double jeopardy has attached in this case. We agree
with the Solicitor General that the dismissal order made by the trial court was not
valid and cannot be used as basis for a claim of double jeopardy. The said right
cannot be grounded on an error of law. As held in People vs. Navarro:

The State is entitled to due process in criminal cases, that is, it must be given
the opportunity to present its evidence in support of the charge. The Court has
always accorded this right to the prosecution, and where the right had been denied,
had promptly annulled the offending court action. We have heretofore held that a
purely capricious dismissal of an information deprives the State of fair
opportunity to prosecute and convict; it denies the prosecution its day in court. For
this reason, it is a dismissal without due process, and, therefore, null and void. Such
dismissal is invalid for lack of a fundamental prerequisite, that is, due process,
and, consequently, will not constitute a proper basis for the claim of double
jeopardy.

We agree with the OSG’s contention that the trial court exceeded its authority when it
dismissed the case without giving the prosecution a right to be heard, hence there was
a violation of due process. Further, the failure of the prosecution to offer its
exhibits is not a ground to dismiss the case. Even without any documentary
exhibits, the prosecution could still prove its case through the testimonies of its
witnesses. Thus, we find that when the trial court reconsidered its order of dismissal,
it merely corrected itself.

*People v. Hon. Tac-an, G.R. No. 148000, February 27, 2003. [PROBABLE BAR
QUESTION!]

Page 109 of 323


(Double Jeopardy; Arbitrary Dismissal; Pre-trial; Crim Pro: Speedy Trial Act)

Facts:
-Mario N. Austria was charged with falsification of public official document.

-In the Information against him, eleven (11) witnesses for the prosecution were
listed, together with their respective addresses where the subpoenas and notices could
be sent.

-During the initial pre-trial, out of the eleven witnesses listed in the Information, only
the first three witnesses were notified by the court of said pre-trial.

-When the case was called for pre-trial, the trial court discovered that none of the
three witnesses who were earlier notified by the court was in attendance. On motion of
the accused and over the vehement objection of the public prosecutor, the trial court
issued an order dismissing the case for failure of said witnesses to appear before it.

-The public prosecutor filed a motion for reconsideration of said order, but the same
was denied.

-The People of the Philippines, through the Office of the Solicitor General, filed a
petition for certiorari with the CA under Rule 65.

-The CA dismissed the petition ratiocinating that: (1) the errors committed by the trial
court were mere errors of judgment which are not correctible by a writ of certiorari;
(2) a reinstatement of the criminal case will place the private respondent in double
jeopardy.

-The People then went to the SC via petition for review on certiorari under Rule 45.

-Contention of the trial court: Under R.A. No. 8493, pre-trial is mandatory and the
presence of the complaining witnesses is likewise required during the trial for the
parties to participate in the plea bargaining and stipulation of facts during said
proceedings. If the dismissal of the case was precipitate, it was the fault of the public
prosecutor and not the trial court.

Issue:
Whether or not, under the circumstances, the reinstatement of the criminal case will
place the accused in double jeopardy.

Ruling:
No.

Under R.A. 8493 (Speedy Trial Act), the absence during pre-trial of any witness for
the prosecution listed in the Information, whether or not said witness is the
offended party or the complaining witness, is not a valid ground for the dismissal of
a criminal case. Although under the law, pre-trial is mandatory in criminal cases,
the presence of the private complainant or the complaining witness is however
not required. Even the presence of the accused is not required unless directed by the
trial court. It is enough that the accused is represented by his counsel.

Indeed, even if none of the witnesses listed in the information for the State
appeared for the pre-trial, the same can and should proceed. After all, the public
prosecutor appeared for the State. The public prosecutor is vested with authority to
consider those matters catalogued in Section 2 of R.A. 8493.

The trial court thus acted without jurisdiction when it dismissed the case merely
because none of the witnesses notified by the trial court appeared for the pre-
trial. The State, like the accused is also entitled to due process in criminal
cases. The order of the trial court dismissing the criminal case deprived the State of
its right to prosecute and prove its case. Said order is, therefore, void for lack of

Page 110 of 323


jurisdiction, and is of no effect. By its ruling, this Court is not abetting or even
glossing over the failure of the three witnesses of the prosecution to appear at the
initial pre-trial of the case. Said witnesses may be cited by the trial court in
contempt of court if their absence was unjustified. Undue delay in the prosecution
of the case should not also be condoned. But the right of the State to prosecute the
case and prove the criminal liability of the private respondent for the crime charged
should not be derailed and stymied by precipitate and capricious dismissal of the case
at the initial pre-trial stage. To do justice to private respondent and injustice to
the State is no justice at all. Justice must be done to all the parties alike. x x x x

When the prosecution is deprived of a fair opportunity to prosecute and prove its case,
its right to due process is thereby violated. The cardinal precept is that where there is
a violation of basic constitutional rights, courts are ousted of their jurisdiction.
Thus, the violation of the State’s right to due process raises a serious jurisdiction
issue, which cannot be glossed over or disregarded at will. Where the denial of the
fundamental right of due process is apparent, a decision rendered in disregard of
that right is void for lack of jurisdiction. x x x x

Thus, apparently, to raise the defense of double jeopardy, three requisites must
be present: (1) a first jeopardy must have attached prior to the second; (2) the
first jeopardy must have been validly terminated; and (3) the second jeopardy
must be for the same offense as that in the first.

Legal jeopardy attaches only (a) upon a valid indictment, (b) before a competent court,
(c) after arraignment, (d) a valid plea having been entered; and (e) the case was
dismissed or otherwise terminated without the express consent of the accused. The
lower court was not competent as it was ousted of its jurisdiction when it violated the
right of the prosecution to due processs.

In effect, the first jeopardy was never terminated, and the remand of the criminal
case for further hearing and/or trial before the lower courts amounts merely to a
continuation of the first jeopardy, and does not expose the accused to a second
jeopardy.

[NOTE: The criminal case was ordered reinstated.]

*Argel v. Hon. Pascua, A.M. No. RTJ-94-1131, August 20, 2001.


(Double Jeopardy; Acquittal; Conviction)

Facts:
-Miguel Argel was acquitted of the crime of murder by Judge Herminia M. Pascua in
her Decision dated 22 July 1993, promulgated on 13 August 1993. The acquittal was
based on the fact that there was no witness who positively identified the accused as
the perpetrator of the crime.

-Subsequently, however, said judge’s attention was called by the lawyer of the private
complainant that there was such an eyewitness who positively identified the accused.
The judge then realized that there was indeed one in the person of Tito Retreta.

-Hon. Pascua clarified that her mistake was brought about by the fact that the
testimony of the eyewitness was not attached to the records at the time she wrote her
decision.

-In view thereof, said judge vacated her previous decision and rendered a new
Decision dated 19 August 1993 finding the accused guilty of murder.

Issues:
(1) Whether or not double jeopardy already set in despite palpable mistake on the part
of the judge;
(2) Whether or not the judge is should be disciplined for gross ignorance of the law.

Page 111 of 323


Ruling:
(1) Yes.

Too elementary is the rule that a decision once final is no longer susceptible to
amendment or alteration except to correct errors which are clerical in nature, to
clarify any ambiguity caused by an omission or mistake in the dispositive portion, or
to rectify a travesty of justice brought about by a mock trial. A final decision is the
law of the case and is immutable and unalterable regardless of any claim of error
or incorrectness.

In criminal cases, a judgment of acquittal is immediately final upon its


promulgation. It cannot be recalled for correction or amendment except in the
cases already mentioned nor withdrawn by another order reconsidering the dismissal
of the case since the inherent power of a court to modify its order or decision does not
extend to a judgment of acquittal in a criminal case.

Complainant herein was already acquitted of murder by respondent in a decision


promulgated on 13 August 1993. Applying the aforestated rule, the decision
became final and immutable on the same day. As a member of the bench who is
always admonished to be conversant with the latest legal and judicial developments,
more so of elementary rules, respondent should have known that she could no
longer “revise” her decision of acquittal without violating not only an
elementary rule of procedure but also the constitutional proscription against
double jeopardy. When the law is so elementary, not to know it constitutes gross
ignorance of the law.

(2) Yes.

We cannot write finis to this case without also commenting on respondent's negligence
in the preparation of her decision. Judges have always been reminded to take down
their own notes of salient portions of hearings and not to rely on the transcripts of
stenographic notes. The pivotal testimony of Tito Retreta would not have been
overlooked and consequently disregarded had respondent prepared her own notes and
read them as she was supposed to.

WHEREFORE, for Gross Ignorance of the Law respondent Judge Herminia M.


Pascua, RTC-Br. 25, Vigan, Ilocos Sur, is FINED P20,000.00, the same to be
deducted from her retirement benefits. Since respondent has already compulsorily
retired as of 18 September 1998, let her retirement benefits be immediately released to
her minus the amount of P20,000.00 herein imposed on her as fine.

*People v. Hon. Relova, G.R. No. L-45129, March 6, 1987.


(Double Jeopardy; Violation of an Ordinance; Violation of a Statute; Same Acts)

Facts:
-Manuel Opulencia was the owner of an ice and cold storage plant.

-On 1 February 1975, members of the Batangas City Police together with personnel of
the Batangas Electric Light System, equipped with a search warran, searched and
examined the premises of the said ice plant and discovered that electric wiring, devices
and contraptions had been installed, without the necessary authority from the city
government, and architecturally concealed inside the walls of the building.

-These electric devices and contraptions were designed purposely to lower or


decrease the readings of electric current consumption in the electric meter of the
said ice and cold storage plant.

-Consequently, on 24 November 1975, an Assistant City Fiscal of Batangas City filed


before the MTC of Batangas City an information against Manuel Opulencia for
“Violation of Ordinance No. 1, Series of 1974, Batangas City.”

Page 112 of 323


-However, said criminal case was eventually dismissed on the ground of prescription,
it appearing that the offense charged was a light felony (which prescribes two
months from the time of discovery thereof), and it appearing further that the
information was filed by the fiscal more than nine months after discovery of the
offense charged.

-Fourteen (14) days later, on 20 April 1976, the Acting City Fiscal of Batangas City
filed before the Court of First Instance of Batangas, Branch 11, another information
against Manuel Opulencia, this time for theft of electric power under Article 308 in
relation to Article 309, paragraph (1), of the Revised Penal Code.

-Before he could be arraigned thereon, Manuel Opulencia filed a Motion to Quash,


dated 5 May 1976, alleging that he had been previously acquitted of the offense
charged in the second information and that the filing thereof was violative of his
constitutional right against double jeopardy.

-The court then dismissed the case on the ground of double jeopardy.

Issue:
Whether or not double jeopardy has already set in under the circumstances.

Ruling:
Yes.

Where the offenses charged are penalized either by different sections of the same
statute or by different statutes, the important inquiry relates to the identity of offenses
charge: the constitutional protection against double jeopardy is available only where
an identity is shown to exist between the earlier and the subsequent offenses charged.
In contrast, where one offense is charged under a municipal ordinance while the
other is penalized by a statute, the critical inquiry is to the identity of the acts
which the accused is said to have committed and which are alleged to have given
rise to the two offenses: the constitutional protection against double jeopardy is
available so long as the acts which constitute or have given rise to the first
offense under a municipal ordinance are the same acts which constitute or have
given rise to the offense charged under a statute.

The question of identity or lack of identity of offenses is addressed by examining the


essential elements of each of the two offenses charged, as such elements are set out in
the respective legislative definitions of the offenses involved. The question of identity
of the acts which are claimed to have generated liability both under a municipal
ordinance and a national statute must be addressed, in the first instance, by
examining the location of such acts in time and space. When the acts of the
accused as set out in the two informations are so related to each other in time and
space as to be reasonably regarded as having taken place on the same occasion
and where those acts have been moved by one and the same, or a continuing,
intent or voluntary design or negligence, such acts may be appropriately
characterized as an integral whole capable of giving rise to penal liability
simultaneously under different legal enactments (a municipal ordinance and a
national statute). x x x x

In the instant case, the relevant acts took place within the same time frame: from
November 1974 to February 1975. During this period, the accused Manuel
Opulencia installed or permitted the installation of electrical wiring and devices in his
ice plant without obtaining the necessary permit or authorization from the municipal
authorities. The accused conceded that he effected or permitted such unauthorized
installation for the very purpose of reducing electric power bill. This corrupt
intent was thus present from the very moment that such unauthorized installation
began. The immediate physical effect of the unauthorized installation was the inward
flow of electric current into Opulencia's ice plant without the corresponding recording
thereof in his electric meter. In other words, the “taking” of electric current was
integral with the unauthorized installation of electric wiring and devices.

Page 113 of 323


While the rule against double jeopardy prohibits prosecution for the same
offense, it seems elementary that an accused should be shielded against being
prosecuted for several offenses made out from a single act.

It remains to point out that the dismissal by the Batangas City Court of the
information for violation of the Batangas City Ordinance upon the ground that such
offense had already prescribed, amounts to an acquittal of the accused of that offense.
Under Article 89 of the Revised Penal Code, “prescription of the crime” is one of
the grounds for “total extinction of criminal liability.” Under the Rules of Court,
an order sustaining a motion to quash based on prescription is a bar to another
prosecution for the same offense.

It is not without reluctance that we deny the people’s petition for certiorari and
mandamus in this case. It is difficult to summon any empathy for a businessman who
would make or enlarge his profit by stealing from the community. Manuel Opulencia
is able to escape criminal punishment because an Assistant City Fiscal by
inadvertence or otherwise chose to file an information for an offense which he
should have known had already prescribed. We are, however, compelled by the
fundamental law to hold the protection of the right against double jeopardy available
even to the private respondent in this case.

*Wright v. Court of Appeals, G.R. No. 113213, August 15, 1994.


(Ex post facto law; Extradition; Treaty)

Facts:
-Petitioner, an Australian citizen, was sought by Australian authorities for indictable
crimes in his country alleged to have been committed by him sometime in 1988 to
1989. At that time of the alleged commission of the said crimes, there was no
extradition treaty yet between the Philippines and Australia. (The Extradition Treaty
was concluded between the Philippines and Australia on September 10, 1990.)

-Extradition proceedings were then filed before the Regional Trial Court of Makati,
which rendered a decision ordering the deportation of petitioner. Said decision was
sustained by the Court of Appeals.

-Hence, petitioner went to the SC via Rule 45, seeking to set aside the order of
deportation.

-Petitioner’s contention: the provision of the Treaty giving retroactive effect to the
extradition treaty amounts to an ex post facto law which violates Section 21 of Article
VI of the Constitution.

Issue:
Whether or not the treaty’s retroactive application violate the Constitutional
prohibition against ex post facto law.

Ruling:
No.

Article 2(4) of the Treaty unequivocally provides that:

“4. Extradition may be granted pursuant to provisions of this Treaty irrespective of when the offense
in relation to which extradition is requested was committed, provided that:

(a) it was an offense in the Requesting State at the time of the acts or omissions constituting the
offense; and
(b) the acts or omissions alleged would, if they had taken place in the Territory of the Requested State at
the time of the making of the request for extradition, have constituted an offense against the laws in force
in that state.”

Page 114 of 323


Thus, the offenses for which petitioner is sought by his government are clearly
extraditable under Article 2 of the Treaty. They were offenses in the Requesting State
at the time they were committed, and, irrespective of the time they were
committed, they fall under the panoply of the Extradition Treaty's provisions,
specifically, Article 2 paragraph 4, quoted above.

Does the Treaty's retroactive application violate the Constitutional prohibition against
ex post facto laws? Early commentators understood ex post facto laws to include all
laws of retrospective application, whether civil or criminal. However, Chief Justice
Salmon P. Chase, citing Blackstone, the Federalist and other early U.S. state
constitutions in Calder vs. Bull concluded that the concept was limited only to
penal and criminal statutes. As conceived under our Constitution, ex post facto laws
are 1) statutes that make an act punishable as a crime when such act was not an
offense when committed; 2) laws which, while not creating new offenses, aggravate the
seriousness of a crime; 3) statutes which prescribes greater punishment for a crime
already committed; or, 4) laws which alter the rules of evidence so as to make it
substantially easier to convict a defendant.

Applying the constitutional principle, the Court has held that the prohibition applies
only to criminal legislation which affects the substantial rights of the accused.
This being so, there is absolutely no merit in petitioner's contention that the ruling of
the lower court sustaining the Treaty's retroactive application with respect to offenses
committed prior to the Treaty’s coming into force and effect, violates the Constitutional
prohibition against ex post facto laws. As the Court of Appeals correctly concluded,
the Treaty is neither a piece of criminal legislation nor a criminal procedural
statute. It merely provides for the extradition of persons wanted for prosecution
of an offense or a crime which offense or crime was already committed or
consummated at the time the treaty was ratified.

*Sesbreño v. Central Board of Assessment Appeals, G.R. No. 106588, March 24, 1997.
(Ex Post Facto Law; Back Taxes)

Facts:
-In 1980, petitioner purchased a house a lot and initially declared the house therein
as a “residential house of strong materials.” The declared property was assessed by
Respondent City Assessor of Cebu City under Tax Declaration No. 02-20454 at a
market value of P60,000.00 and an assessed value of P36,900.00.

-However, during a tax-mapping operation conducted in 1989, the field inspectors of


the Cebu City Assessor discovered that the real property declared and assessed under
Tax Declaration No. 02-20454 was actually a residential building consisting of four (4)
storeys with a fifth storey used as a roof deck. The building had a total floor area of
500.20 square meters. The area for each floor was 100.04 square meters. The building
was found to have been made of Type II-A materials.

-Based on the findings of the field inspectors, Respondent City Assessor of Cebu City
issued Tax Declaration No. GR-06-045-00162 effective in the year 1989, canceling Tax
Declaration No. 02-20454 and assessing the building therein at a net market value of
P499,860.00 and an assessed value of P374,900.00. The 1981-1984 Schedule of
Market Value was applied in the assessment.

-Petitioner’s contentions: (1) the new assessment is excessive and unconscionable; (2)
the imposition of back taxes on his property is unconstitutional for being violative of
Section 22, 37 Article III of the 1987 Constitution (against ex post facto law).

Issue:
Whether or not the retroactive real estate tax assessments made herein constitute an
ex post facto law.

Ruling:
No.

Page 115 of 323


When both Public Respondents CBAA and City Assessor imposed back taxes on
petitioner’s property, they did not violate the rule that laws shall have only
prospective applicability. Respondents were only applying PD 464 which had been in
effect since 1974. Besides, Section 25 of PD 464 is not penal in character; hence,
it may not be considered as an ex post facto law.

*Lacson v. Sandiganbayan, G.R. No. 128096, January 20, 1999.


(Ex Post Facto Law; Jurisdiction; Sandiganbayan)

-Petitioner Panfilo Lacson was among those charged as principal with multiple
murder for the death of eleven (11) persons believed to be members of the Kuratong
Baleleng gang. The killings were alleged to have been committed as a summary
execution or a “rub out.”

-After conducting a reinvestigation, the Ombudsman filed amended the Informations


before the Sandiganbayan, wherein petitioner was charged only as an accessory.

-On March 5-6, 1996, all the accused filed separate motions questioning the
jurisdiction of the Sandiganbayan, asserting that under the amended informations,
the cases fall within the jurisdiction of the Regional Trial Court. They contend that the
said law limited the jurisdiction of the Sandiganbayan to cases where one or more of
the “principal accused” are government officials with Salary Grade (SG) 27 or higher,
or PNP officials with the rank of Chief Superintendent (Brigadier General) or higher.
The highest ranking “principal accused” in the amended informations has the rank of
only a Chief Inspector, and none has the equivalent of at least SG 27.

-While petitioner Lacson has the rank of Chief Superintendent, he is only charged as
an “accessory” and not as a “principal accused.”

-In view of this, the Sandiganbayan ordered the cases transferred to the RTC Quezon
City which has original and exclusive jurisdiction under R.A. 7975, as none of the
“principal accused” has the rank of Chief Superintendent or higher.

-On May 17, 1996, the Office of the Special Prosecutor moved for reconsideration,
insisting that the cases should remain with the Sandiganbayan. While these motions
for reconsideration were still pending resolution, R.A. No. 8249 was approved into law
on February 5, 1997.

-R.A. No. 8249 expanded the jurisdiction of the Sandiganbayan. Specifically, the new
law deleted the word “principal” from the phrase “principal accused” in Section 2
(paragraphs a and c) of R.A. No. 7975. Hence, the jurisdiction of the Sandiganbayan
now covers not only principals but also accomplices and accessories, including
petitioner herein.

Issue:
Whether or not R.A. No. 8249 is an ex post facto law.

Ruling:
No.

Petitioner and intervenors further further argued that the retroactive application of
R.A. 8249 to the Kuratong Baleleng cases constitutes an ex post facto law for they are
deprived of their right to procedural due process as they can no longer avail of the
two-tiered appeal which they had allegedly acquired under R.A. 7975.

Again, this contention is erroneous. There is nothing ex post facto in R.A. 8249. In
Calder v. Bull, an ex post facto law is one —

(a) which makes an act done criminal before the passing of the law and which was
innocent when committed, and punishes such action; or

Page 116 of 323


(b) which aggravates a crime or makes it greater than when it was committed; or

(c) which changes the punishment and inflicts a greater punishment than the law
annexed to the crime when it was committed.

(d) which alters the legal rules of evidence and receives less or different testimony that
the law required at the time of the commission of the offense on order to convict the
defendant.

(e) which, in relation to the offense or its consequences, alters the situation of a person
to his disadvantage.

This Court added two more to the list, namely:

(f) that which assumes to regulate civil rights and remedies only but in effect imposes
a penalty or deprivation of a right which when done was lawful;

(g) deprives a person accussed of crime of some lawful protection to which he has
become entitled, such as the protection of a former conviction or acquittal, or a
proclamation of an amnesty.

Ex post facto law, generally, prohibits retrospectivity of penal laws. R.A. 8249 is
not penal law. It is a substantive law on jurisdiction which is not penal in
character. Penal laws are those acts of the Legislature which prohibit certain
acts and establish penalties for their violations; or those that define crimes,
treat of their nature, and provide dor their punishment. R.A 7975, which amended
P.D. 1606 as regards the Sandiganbayan's jurisdiction, its mode of appeal and other
procedural matters, has been declared by the Court as not a penal law, but clearly a
procedural statute, i.e. one which prescribes rules of procedure by which courts
applying laws of all kinds can properly administer justice. Not being a penal law, the
retroactive application of R.A. 8249 cannot be challenged as unconstitutional.

Petitioner’s and intervenors’ contention that their right to a two-tiered appeal which
they acquired under R.A. 7975 has been diluted by the enactment of R.A. 8249,
is incorrect. The same contention has already been rejected by the court several times
considering that the right to appeal is not a natural right but statutory in nature
that can be regulated by law. The mode of procedure provided for in the statutory
right of appeal is not included in the prohibition against ex post facto laws. R.A.
8249 pertains only to matters of procedure, and being merely an amendatory
statute it does not partake the nature of an ex post facto law. It does not mete out
a penalty and, therefore, does not come within the prohibition. Moreover, the law did
not alter the rules of evidence or the mode of trial. It has been ruled that adjective
statutes may be made applicable to actions pending and unresolved at the time of
their passage.

Citizenship

*Poe-Llamanzares v. Comelec, G.R. No. 221697, March 8, 2016. [IMPORTANT!!!


VERY RECENT!!!]
(Citizenship; Foundling; Comelec’s jurisdiction; Residency Requirement; Material
Misrepresentation)

Facts:
-Mary Grace Natividad S. Poe-Llamanzares (petitioner) was found abandoned as a
newborn infant in the Parish Church of Jaro, Iloilo by a certain Edgardo Militar
(Edgardo) on 3 September 1968. She was then registered as a foundling with the
Office of the Civil Registrar of Iloilo City (OCR-Iloilo).

Page 117 of 323


-In her Foundling Certificate and Certificate of Live Birth, the petitioner was given the
name “Mary Grace Natividad Contreras Militar.”

-When petitioner was five (5) years old, celebrity spouses Ronald Allan Kelley Poe
(a.k.a. Fenando Poe, Jr.) and Jesusa Sonora Poe (a.k.a. Susan Roces) filed a petition
for her adoption with the Municipal Trial Court (MTC) of San Juan City. On 13 May
1974, the trial court granted their petition and ordered that petitioner's name be
changed from "Mary Grace Natividad Contreras Militar" to "Mary Grace Natividad
Sonora Poe."

-Having reached the age of eighteen (18) years in 1986, petitioner registered as a
voter with the local COMELEC Office in San Juan City. On 13 December 1986, she
received her COMELEC Voter's Identification Card for Precinct No. 196 in Greenhills,
San Juan, Metro Manila.

-Initially, the petitioner enrolled and pursued a degree in Development Studies at the
University of the Philippines but she opted to continue her studies abroad and left for
the United States of America (U.S.) in 1988.

-On 27 July 1991, petitioner married Teodoro Misael Daniel V. Llamanzares


(Llamanzares), a citizen of both the Philippines and the U.S., at Sanctuario de San
Jose Parish in San Juan City. Desirous of being with her husband who was then
based in the U.S., the couple flew back to the U.S. two days after the wedding
ceremony or on 29 July 1991.

-While in the U.S., the petitioner gave birth to her eldest child Brian Daniel (Brian) on
16 April 1992. Her two daughters Hanna MacKenzie (Hanna) and Jesusa Anika
(Anika) were both born in the Philippines on 10 July 1998 and 5 June 2004,
respectively.

-On 18 October 2001, petitioner became a naturalized American citizen. She


obtained U.S. Passport No. 017037793 on 19 December 2001.

-On 8 April 2004, the petitioner came back to the Philippines together with Hanna to
support her father's candidacy for President in the May 2004 elections. It was during
this time that she gave birth to her youngest daughter Anika. She returned to the U.S.
with her two daughters on 8 July 2004.

-After a few months, specifically on 13 December 2004, petitioner rushed back to the
Philippines upon learning of her father's deteriorating medical condition. Her father
slipped into a coma and eventually expired. The petitioner stayed in the country until
3 February 2005 to take care of her father's funeral arrangements as well as to assist
in the settlement of his estate.

-In her earnest desire to be with her grieving mother, the petitioner and her husband
decided to move and reside permanently in the Philippines sometime in the first
quarter of 2005.

-In late March 2006, petitioner's husband officially informed the U.S. Postal Service of
the family's change and abandonment of their address in the U.S. The family home
was eventually sold on 27 April 2006. Petitioner's husband resigned from his job in
the U.S. in April 2006, arrived in the country on 4 May 2006 and started working for a
major Philippine company in July 2006.

-On 7 July 2006, petitioner took her Oath of Allegiance to the Republic of the
Philippines pursuant to Republic Act (R.A.) No. 9225 or the Citizenship Retention
and Re-acquisition Act of 2003. Under the same Act, she filed with the Bureau of
Immigration (BI) a sworn petition to reacquire Philippine citizenship together with
petitions for derivative citizenship on behalf of her three minor children on 10 July
2006.

Page 118 of 323


-As can be gathered from its 18 July 2006 Order, the BI acted favorably on
petitioner's petitions and declared that she is deemed to have reacquired her
Philippine citizenship while her children are considered as citizens of the
Philippines. Consequently, the BI issued Identification Certificates (ICs) in petitioner's
name and in the names of her three (3) children.

-Again, petitioner registered as a voter of Barangay Santa Lucia, San Juan City on 31
August 2006. She also secured from the DFA a new Philippine Passport.

-On 6 October 2010, President Benigno S. Aquino III appointed petitioner as


Chairperson of the Movie and Television Review and Classification Board (MTRCB).
Before assuming her post, petitioner executed an "Affidavit of Renunciation of
Allegiance to the United States of America and Renunciation of American
Citizenship" before a notary public in Pasig City on 20 October 2010, in
satisfaction of the legal requisites stated in Section 5 of R.A. No. 9225. The following
day, 21 October 2010 petitioner submitted the said affidavit to the BI and took her
oath of office as Chairperson of the MTRCB. From then on, petitioner stopped using
her American passport.

-On 12 July 2011, the petitioner executed before the Vice Consul of the U.S.
Embassy in Manila an "Oath/Affirmation of Renunciation of Nationality of the
United States." On that day, she accomplished a sworn questionnaire before the U.S.
Vice Consul wherein she stated that she had taken her oath as MTRCB Chairperson
on 21 October 2010 with the intent, among others, of relinquishing her American
citizenship. In the same questionnaire, the petitioner stated that she had resided
outside of the U.S., specifically in the Philippines, from 3 September 1968 to 29
July 1991 and from May 2005 to present.

-On 9 December 2011, the U.S. Vice Consul issued to petitioner a "Certificate of Loss
of Nationality of the United States" effective 21 October 2010.

-On 2 October 2012, the petitioner filed with the COMELEC her Certificate of
Candidacy (COC) for Senator for the 2013 Elections wherein she answered "6
years and 6 months" to the question "Period of residence in the Philippines
before May 13, 2013." Petitioner obtained the highest number of votes and was
proclaimed Senator on 16 May 2013.

-On 19 December 2013, petitioner obtained Philippine Diplomatic Passport No.


DE0004530.

-On 15 October 2015, petitioner filed her COC for the Presidency for the May 2016
Elections. In her COC, the petitioner declared that she is a natural-born citizen and
that her residence in the Philippines up to the day before 9 May 2016 would be
ten (10) years and eleven (11) months counted from 24 May 2005. The petitioner
attached to her COC an "Affidavit Affirming Renunciation of U.S.A. Citizenship"
subscribed and sworn to before a notary public in Quezon City on 14 October 2015.

-Respondents’ contentions: (1) petitioner committed material misrepresentation


when she stated in her COC that she is a natural-born Filipino citizen and that she is
a resident of the Philippines for at least ten (10) years and eleven (11) months up to
the day before the 9 May 2016 Elections. Petitioner was bound by the sworn
declaration she made in her 2012 COC for Senator wherein she indicated that she had
resided in the country for only six (6) years and six (6) months as of May 2013
Elections.; (2) petitioner cannot be considered as a natural-born Filipino on account of
the fact that she was a foundling; (3) Even assuming arguendo that petitioner was a
natural-born Filipino, she is deemed to have lost that status when she became a
naturalized American citizen. According to them, natural-born citizenship must be
continuous from birth; (4) assuming arguendo that petitioner is qualified to regain her
natural-born status under R.A. No. 9225, she still fell short of the ten-year residency
requirement of the Constitution as her residence could only be counted at the earliest
from July 2006, when she reacquired Philippine citizenship under the said Act.

Page 119 of 323


-On 1 December 2015, the COMELEC Second Division promulgated a Resolution
finding that petitioner's COC, filed for the purpose of running for the President of the
Republic of the Philippines in the 9 May 2016 National and Local Elections, contained
material representations which are false. COMELEC Second Division then ordered
petitioner’s COC for presidency cancelled.

-Petitioner then went to the SC via certiorari with urgent prayer for the issuance of an
ex parte temporary restraining order/status quo ante order and/or writ of preliminary
injunction.

Issues:
(1) Whether or not the Comelec, in the COC cancellation case, was acting within its
authority when it decided the qualification (or lack thereof) of Poe;

(2) Whether or not Poe is considered a natural-born despite the fact that she is a
foundling;

(3) Whether or not Poe regained her “natural-born” status when her application for
repatriation was approved;

(4) Whether or not Poe sufficiently proved her COC statement that she has resided in
the Philippines for ten years;

(5) Whether or not she committed a false representation in her 2016 COC, in relation
to her 2013 COC.”

Ruling:
(1) No.

The issue before the COMELEC is whether or not the COC of petitioner should be
denied due course or cancelled “on the exclusive ground” that she made in the
certificate a false material representation. The exclusivity of the ground should hedge
in the discretion of the COMELEC and restrain it from going into the issue of the
qualifications of the candidate for the position, if, as in this case, such issue is yet
undecided or undetermined by the proper authority. The COMELEC cannot by itself,
in the same cancellation case, decide the qualification or lack thereof of the
candidate. “The Supreme Court, sitting en banc, shall be the sole judge of all
contests relating to the election, returns, and qualifications of the President or
Vice-President, and may promulgate its rules for the purpose.” x x x x

The determination of a candidates’ eligibility, e.g., his citizenship or, as in this


case, his domicile, may take a long time to make, extending beyond the beginning
of the term of the office. This is amply demonstrated in the companion case (G.R. No.
120265, Agapito A. Aquino v. COMELEC) where the determination of Aquino's residence
was still pending in the COMELEC even after the elections of May 8, 1995. This is
contrary to the summary character proceedings relating to certificates of
candidacy. That is why the law makes the receipt of certificates of candidacy a
ministerial duty of the COMELEC and its officers. The law is satisfied if candidates
state in their certificates of candidacy that they are eligible for the position that they
seek to fill, leaving the determination of their qualifications to be made after the
election and only in the event they are elected. Only in cases involving charges of
false representations made in certificates of candidacy is the COMELEC given
jurisdiction.

(2) Yes.

At the outset, it must be noted that presumptions regarding paternity is neither


unknown nor unaccepted in Philippine Law. The Family Code of the Philippines has a
whole chapter on Paternity and Filiation. That said, there is more than sufficient
evidence that petitioner has Filipino parents and is therefore a natural-born

Page 120 of 323


Filipino. Parenthetically, the burden of proof was on private respondents to show
that petitioner is not a Filipino citizen. The private respondents should have shown
that both of petitioner's parents were aliens. Her admission that she is a foundling did
not shift the burden to her because such status did not exclude the possibility that
her parents were Filipinos, especially as in this case where there is a high probability,
if not certainty, that her parents are Filipinos.

The fact is that petitioner's blood relationship with a Filipino citizen is


DEMONSTRABLE.

There is a disputable presumption that things have happened according to the


ordinary course of nature and the ordinary habits of life. All of the foregoing evidence,
that a person with typical Filipino features is abandoned in Catholic Church in a
municipality where the population of the Philippines is overwhelmingly Filipinos
such that there would be more than a 99% chance that a child born in the
province would be a Filipino, would indicate more than ample probability if not
statistical certainty, that petitioner's parents are Filipinos. That probability and the
evidence on which it is based are admissible under Rule 128, Section 4 of the Revised
Rules on Evidence. x x x x

Recent legislation is more direct. R.A. No. 8043 entitled "An Act Establishing the Rules
to Govern the Inter-Country Adoption of Filipino Children and For Other Purposes"
(otherwise known as the "Inter-Country Adoption Act of 1995"), R.A. No. 8552, entitled
"An Act Establishing the Rules and Policies on the Adoption of Filipino Children and
For Other Purposes" (otherwise known as the Domestic Adoption Act of 1998) and this
Court's A.M. No. 02-6-02-SC or the "Rule on Adoption," all expressly refer to
"Filipino children" and include foundlings as among Filipino children who may
be adopted. x x x x

Likewise, customary international law dictates that foundlings are entitled to a


nationality and are presumed to be citizens of the country where they are found.

(3) Yes.

The COMELEC also ruled that petitioner's repatriation in July 2006 under the
provisions of R.A. No. 9225 did not result in the reacquisition of natural-born
citizenship. The COMELEC reasoned that since the applicant must perform an act,
what is reacquired is not "natural-born" citizenship but only plain "Philippine
citizenship."

In the seminal case of Bengson Ill v. HRET, repatriation was explained as follows:

Moreover, repatriation results in the recovery of the original nationality. This


means that a naturalized Filipino who lost his citizenship will be restored to his prior
status as a naturalized Filipino citizen. On the other hand, if he was originally a
natural-born citizen before he lost his Philippine citizenship, he will be restored
to his former status as a natural-born Filipino.

R.A. No. 9225 is a repatriation statute and has been described as such in several
cases. They include Sobejana-Condon v. COMELEC where we described it as an
"abbreviated repatriation process that restores one's Filipino citizenship x x x." Also
included is Parreno v. Commission on Audit, which cited Tabasa v. Court of Appeals,
where we said that "the repatriation of the former Filipino will allow him to
recover his natural-born citizenship. Parreno v. Commission on Audit is categorical
that "if petitioner reacquires his Filipino citizenship (under R.A. No. 9225), he
will recover his natural-born citizenship."

The COMELEC construed the phrase "from birth" in the definition of natural citizens
as implying "that natural-born citizenship must begin at birth and remain
uninterrupted and continuous from birth." R.A. No. 9225 was obviously passed in line
with Congress' sole prerogative to determine how citizenship may be lost or

Page 121 of 323


reacquired. Congress saw it fit to decree that natural-born citizenship may be
reacquired even if it had been once lost. It is not for the COMELEC to disagree with
the Congress’ determination.

More importantly, COMELEC's position that natural-born status must be continuous


was already rejected in Bengson III v. HRET where the phrase "from birth" was
clarified to mean at the time of birth: "A person who at the time of his birth, is a
citizen of a particular country, is a natural-born citizen thereof." Neither is
"repatriation" an act to "acquire or perfect" one's citizenship. x x x x

In Bengson III v. HRET, this Court pointed out that there are only two types of citizens
under the 1987 Constitution: natural-born citizen and naturalized, and that there is
no third category for repatriated citizens:

It is apparent from the enumeration of who are citizens under the present Constitution
that there are only two classes of citizens: (1) those who are natural-born and (2) those
who are naturalized in accordance with law. A citizen who is not a naturalized Filipino,
i.e., did not have to undergo the process of naturalization to obtain Philippine
citizenship, necessarily is a natural-born Filipino. Noteworthy is the absence in said
enumeration of a separate category for persons who, after losing Philippine
citizenship, subsequently reacquire it. The reason therefor is clear: as to such
persons, they would either be natural-born or naturalized depending on the reasons
for the loss of their citizenship and the mode prescribed by the applicable law for the
reacquisition thereof. As respondent Cruz was not required by law to go through
naturalization proceedings in order to reacquire his citizenship, he is perforce a
natural-born Filipino. As such, he possessed all the necessary qualifications to be
elected as member of the House of Representatives.

(4) Yes.

Petitioner's claim that she will have been a resident for ten (10) years and eleven (11)
months on the day before the 2016 elections, is true.

In answer to the requested information of "Period of Residence in the Philippines up to


the day before May 09, 2016," she put in "10 years 11 months" which according to her
pleadings in these cases corresponds to a beginning date of 25 May 2005 when she
returned for good from the U.S.

When petitioner immigrated to the U.S. in 1991, she lost her original domicile, which
is the Philippines. There are three requisites to acquire a new domicile: (1)
Residence or bodily presence in a new locality; (2) an intention to remain there;
and (3) an intention to abandon the old domicile. To successfully effect a change of
domicile, one must demonstrate an actual removal or an actual change of domicile; a
bona fide intention of abandoning the former place of residence and establishing a new
one and definite acts which correspond with the purpose. In other words, there must
basically be animus manendi coupled with animus non revertendi. The purpose to
remain in or at the domicile of choice must be for an indefinite period of time; the
change of residence must be voluntary; and the residence at the place chosen for the
new domicile must be actual. x x x x

COMELEC relied on Coquilla v. COMELEC, Japzon v. COMELEC and Caballero v.


COMELEC. During the oral arguments, the private respondents also added Reyes v.
COMELEC. Respondents contend that these cases decree that the stay of an alien
former Filipino cannot be counted until he/she obtains a permanent resident
visa or reacquires Philippine citizenship, a visa-free entry under a balikbayan
stamp being insufficient. Since petitioner was still an American (without any resident
visa) until her reacquisition of citizenship under R.A. No. 9225, her stay from 24 May
2005 to 7 July 2006 cannot be counted.

But as the petitioner pointed out, the facts in these four cases are very different
from her situation. In Coquilla v. COMELEC, the only evidence presented was a

Page 122 of 323


community tax certificate secured by the candidate and his declaration that he would
be running in the elections. Japzon v. COMELEC did not involve a candidate who
wanted to count residence prior to his reacquisition of Philippine citizenship. With the
Court decreeing that residence is distinct from citizenship, the issue there was
whether the candidate’s acts after reacquisition sufficed to establish residence. In
Caballero v. COMELEC, the candidate admitted that his place of work was abroad and
that he only visited during his frequent vacations. In Reyes v. COMELEC, the
candidate was found to be an American citizen who had not even reacquired
Philippine citizenship under R.A. No. 9225 or had renounced her U.S. citizenship. She
was disqualified on the citizenship issue. On residence, the only proof she offered was
a seven-month stint as provincial officer. The COMELEC, quoted with approval by this
Court, said that, "such fact alone is not sufficient to prove her one-year residency."

No case similar to petitioner’s, where the former Filipino's evidence of change in


domicile is extensive and overwhelming, has as yet been decided by the Court.
Petitioner’s evidence of residence is unprecedented. There is no judicial precedent
that comes close to the facts of residence of petitioner. There is no indication in
Coquilla v. COMELEC, and the other cases cited by the respondents that the Court
intended to have its rulings there apply to a situation where the facts are different.
Surely, the issue of residence has been decided particularly on the facts-of-the-case
basis. x x x x

The COMELEC, by its own admission, disregarded the evidence that petitioner
actually and physically returned here on 24 May 2005 not because it was false,
but only because COMELEC took the position that domicile could be established
only from petitioner's repatriation under R.A. No. 9225 in July 2006. However, it
does not take away the fact that in reality, petitioner had returned from the U.S.
and was here to stay permanently, on 24 May 2005. [Pwe! ~ced] When she
claimed to have been a resident for ten (10) years and eleven (11) months, she
could do so in good faith.

(5) No.

To avoid the logical conclusion pointed out by the evidence of residence of petitioner,
the COMELEC ruled that petitioner's claim of residence of ten (10) years and eleven
(11) months by 9 May 2016 in her 2015 COC was false because she put six (6) years
and six (6) months as "period of residence before May 13, 2013" in her 2012 COC for
Senator. Thus, according to the COMELEC, she started being a Philippine resident
only in November 2006. In doing so, the COMELEC automatically assumed as true
the statement in the 2012 COC and the 2015 COC as false.

As explained by petitioner in her verified pleadings, she misunderstood the date


required in the 2013 COC as the period of residence as of the day she submitted
that COC in 2012. She said that she reckoned residency from April-May 2006 which
was the period when the U.S. house was sold and her husband returned to the
Philippines. In that regard, she was advised by her lawyers in 2015 that residence
could be counted from 25 May 2005.

Petitioner's explanation that she misunderstood the query in 2012 (period of residence
before 13 May 2013) as inquiring about residence as of the time she submitted the
COC, is bolstered by the change which the COMELEC itself introduced in the 2015
COC which is now "period of residence in the Philippines up to the day before
May 09, 2016." The COMELEC would not have revised the query if it did not
acknowledge that the first version was vague.

It was grave abuse of discretion for the COMELEC to treat the 2012 COC as a
binding and conclusive admission against petitioner. It could be given in
evidence against her, yes, but it was by no means conclusive. There is precedent
after all where a candidate's mistake as to period of residence made in a COC was
overcome by evidence. x x x x It is the fact of residence, not a statement in a
certificate of candidacy which ought to be decisive in determining whether or

Page 123 of 323


not an individual has satisfied the constitutions residency qualification
requirement.

[NOTE:
There is not much discussion about respondents’ contention that Poe’s residence
could only be counted at the earliest from July 2006, when she reacquired Philippine
citizenship under the said R.A. No. 9225. Note that six Justices dissented here.]

*Republic v. Li Ching Chung, G.R. No. 197450, March 20, 2013.


(Naturalization; Jurisdictional Requirements)

Facts:
-On August 22, 2007, respondent, otherwise known as Bernabe Luna Li or Stephen
Lee Keng, a Chinese national, filed his Declaration of Intention to Become a Citizen of
the Philippines before the OSG.

-On March 12, 2008 or almost seven months after filing his declaration of
intention, respondent filed his Petition for Naturalization before the RTC.

-The OSG filed a motion to dismiss, arguing that the petition for naturalization was
filed less than one (1) year from the time of the declaration of intent before the OSG.
The RTC denied the same, on the ground that such defect was not fatal and not
jurisdictional.

-After due proceedings, the RTC granted the petition for naturalization, which the
Court of Appeals affirmed upon appeal.

- The OSG then went to the SC via Petition for Review on Certiorari under Rule 45.

Issue:
Whether or not respondent should be admitted as a Filipino citizen despite the
undisputed fact that his petition for naturalization was filed less than one (1) year
from the time he filed his declaration of intent before the OSG.

Ruling:
No.

Section 5 of CA No. 473, as amended, expressly states:

“Section 5. Declaration of intention. – One year prior to the filing of his petition for admission to
Philippine citizenship, the applicant for Philippine citizenship shall file with the Office of the
Solicitor General a declaration under oath that it is bona fide his intention to become a citizen of
the Philippines. Such declaration shall set forth name, age, occupation, personal description, place of
birth, last foreign residence and allegiance, the date of arrival, the name of the vessel or aircraft, if any, in
which he came to the Philippines, and the place of residence in the Philippines at the time of making the
declaration. No declaration shall be valid until lawful entry for permanent residence has been established
and a certificate showing the date, place, and manner of his arrival has been issued. The declarant must
also state that he has enrolled his minor children, if any, in any of the public schools or private schools
recognized by the Office of Private Education of the Philippines, where Philippine history, government,
and civics are taught or prescribed as part of the school curriculum, during the entire period of the
residence in the Philippines required of him prior to the hearing of his petition for naturalization as
Philippine citizen. Each declarant must furnish two photographs of himself.” x

The period of one year required therein is the time fixed for the State to make
inquiries as to the qualifications of the applicant. If this period of time is not given
to it, the State will have no sufficient opportunity to investigate the qualifications of
the applicants and gather evidence thereon. An applicant may then impose upon the
courts, as the State would have no opportunity to gather evidence that it may present
to contradict whatever evidence that the applicant may adduce on behalf of his
petition.

The law is explicit that the declaration of intention must be filed one year prior
to the filing of the petition for naturalization. Republic v. Go Bon Lee likewise

Page 124 of 323


decreed that substantial compliance with the requirement is inadequate. In that
case, Go filed his declaration of intention to become a citizen of the Philippines on May
23, 1940. After eleven months, he filed his petition for naturalization on April 18,
1941. In denying his petition, the Court wrote:

The language of the law on the matter being express and explicit, it is beyond the
province of the courts to take into account questions of expediency, good faith and
other similar reasons in the construction of its provisions (De los Santos vs. Mallare,
87 Phil., 289; 48 Off. Gaz., 1787). Were we to accept the view of the lower court on this
matter, there would be no good reason why a petition for naturalization cannot be filed
one week after or simultaneously with the filing of the required declaration of intention
as long as the hearing is delayed to a date after the expiration of the period of one
year. The ruling of the lower court amounts, in our opinion, to a substantial change in
the law, something which courts cannot do, their duty being to apply the law and not
tamper with it. x x x x

The position of the Government is well taken, because no petition for naturalization
may be filed and heard and hence no decree may be issued granting it under the
provisions of Commonwealth Act No. 473, as amended, before the expiration of one
year from and after the date of the filing of a verified declaration of his bona fide
intention to become a citizen of the Philippines. This is mandatory. Failure to
raise in the lower court the question of non-compliance therewith does not preclude
the Government from raising it on appeal.

In naturalization proceedings, the burden of proof is upon the applicant to show


full and complete compliance with the requirements of the law. The opportunity
of a foreigner to become a citizen by naturalization is a mere matter of grace,
favor or privilege extended to him by the State; the applicant does not possess
any natural, inherent, existing or vested right to be admitted to Philippine
citizenship. The only right that a foreigner has, to be given the chance to become a
Filipino citizen, is that which the statute confers upon him; and to acquire such right,
he must strictly comply with all the statutory conditions and requirements. The
absence of one jurisdictional requirement is fatal to the petition as this
necessarily results in the dismissal or severance of the naturalization process.

*Altarejos v. Comelec, G.R. No. 163256, November 10, 2004. [SEVERAL


PRINCIPLES HERE!]
(Repatriation’ Registration; Citizenship Qualification; Retroactivity; Motion for
Reconsideration)

Facts:
-Petitioner Altarejos was a candidate for mayor in the Municipality of San Jacinto,
Masbate in the May 10, 2004 national and local elections.

-On January 15, 2004, private respondents Jose Almiñe Altiche and Vernon Versoza,
registered voters of San Jacinto, Masbate, filed with the COMELEC, a petition to
disqualify and to deny due course or cancel the certificate of candidacy of petitioner
on the ground that he is not a Filipino citizen and that he made a false representation
in his certificate of candidacy that he was not a permanent resident of or immigrant to
a foreign country.

-Private respondents alleged that based on a letter from the Bureau of Immigration
dated June 25, 2001, petitioner was a holder of a permanent U.S. resident visa, an
Alien Certificate of Registration No. E139507 issued on November 3, 1997, and an
Immigration Certificate of Residence No. 320846 issued on November 3, 1997 by the
Bureau of Immigration.

-On January 26, 2004, petitioner filed an Answer stating, among others, that he did
not commit false representation in his application for candidacy as mayor because as
early as December 17, 1997, he was already issued a Certificate of Repatriation by
the Special Committee on Naturalization, after he filed a petition for repatriation

Page 125 of 323


pursuant to Republic Act No. 8171. Thus, petitioner claimed that his Filipino
citizenship was already restored.

-On March 22, 2004, the COMELEC First Division issued a decision disqualifying
petitioner from running as Mayor of San Jacinto, Masbate and his COC was cancelled
and denied due course.

-On March 25, 2004, petitioner filed a motion for reconsideration with the
COMELEC en banc and attached the following documents to prove that he had
completed all the requirements for repatriation which thus entitled him to run for an
elective office, viz:

(1) Oath of Allegiance dated December 17, 1997;

(2) Identification Certificate No. 116543 issued by the Bureau of Immigration on


March 1, 2004;

(3) Certification from the City Civil Registration Office, Makati City, that the
Certificate of Repatriation and Oath of Allegiance of petitioner was received by said
office and registered, with the corresponding fee paid, on February 18, 2004;

(4) A letter dated December 17, 1997 from the Special Committee on Naturalization to
the Bureau on Immigration and Deportation that it was furnishing said office with the
Oath of Allegiance and Certificate of Repatriation of petitioner for the cancellation of
petitioner's registration in said office as an alien, and the issuance to him of the
corresponding Identification Card as Filipino citizen;

(5) A letter dated December 17, 1997 from the Special Committee on Naturalization to
the Local Registrar of San Jacinto, Masbate that it was sending petitioner's Oath of
Allegiance and Certificate of Repatriation for registration in their records and for
petitioner's reacquisition of his former Philippine citizenship.

-The Comelec En Banc eventually denied the MR, ratiocinating that petitioner, in his
Motion for Reconsideration, attempted to introduce to the record new pieces of
evidence, which introduction is not anymore allowed in a Motion for
Reconsideration. It further ruled that, “Assuming that the new evidence of the
respondent are admitted, with more reason should we cancel his certificate of
candidacy for his act of misrepresenting himself as a Filipino citizen when at the time
he filed his certificate of candidacy, he has not yet perfected the process of
repatriation.”

-Petitioner then went to the SC via petition for certiorari.

Issues:
(1) Whether or not the registration of petitioner's repatriation with the proper civil
registry and with the Bureau of Immigration a prerequisite in effecting repatriation;

(2) Whether or not the citizenship qualification of a candidate for an elective office
applies during the filing of the COC;

(3) Whether or not petitioner's repatriation retroacted to the date he filed his
application in 1997;

(4) Whether or not petitioner is qualified to run for a mayoralty position in the
government in the May 10, 2004 elections;

(5) Whether or not the COMELEC en banc gravely abused its discretion in affirming
the Resolution of the COMELEC First Division.

Ruling:
(1) Yes.

Page 126 of 323


The provision of law applicable in this case is Section 2 of Republic Act No. 8171,14
thus:

“SEC. 2. Repatriation shall be effected by taking the necessary oath of allegiance to the Republic of
the Philippines and registration in the proper civil registry and in the Bureau of Immigration. The
Bureau of Immigration shall thereupon cancel the pertinent alien certificate of registration and issue the
certificate of identification as Filipino citizen to the repatriated citizen.”

The law is clear that repatriation is effected "by taking the oath of allegiance to the
Republic of the Philippines and registration in the proper civil registry and in the
Bureau of Immigration." Hence, in addition to taking the Oath of Allegiance to the
Republic of the Philippines, the registration of the Certificate of Repatriation in
the proper civil registry and the Bureau of Immigration is a prerequisite in
effecting the repatriation of a citizen.

In this case, petitioner took his Oath of Allegiance on December 17, 1997, but his
Certificate of Repatriation was registered with the Civil Registry of Makati City only
after six years or on February 18, 2004, and with the Bureau of Immigration on March
1, 2004. Petitioner, therefore, completed all the requirements of repatriation
only after he filed his certificate of candidacy for a mayoralty position, but
before the elections.

(2) No.

In Frivaldo v. Commission on Elections, the Court ruled that the citizenship


qualification must be construed as applying to the time of proclamation of the
elected official and at the start of his term. The Court, through Justice Artemio V.
Panganiban, discussed, thus:

“Under Sec. 39 of the Local Government Code, an elective local official must be:
* a citizen of the Philippines;
* a registered voter in the barangay, municipality, city, or province x x x where he
intends to be elected;
* a resident therein for at least one (1) year immediately preceding the day of the
election;
* able to read and write Filipino or any other local language or dialect."
* In addition, "candidates for the position of governor x x x must be at least twenty-
three (23) years of age on election day."

From the above, it will be noted that the law does not specify any particular date or
time when the candidate must possess citizenship, unlike that for residence
(which must consist of at least one year's residency immediately preceding the day of
election) and age (at least twenty three years of age on election day).

Philippine citizenship is an indispensable requirement for holding an elective public


office, and the purpose of the citizenship qualification is none other than to ensure
that no alien, i.e., no person owing allegiance to another nation, shall govern our
people and our country or a unit of territory thereof. Now, an official begins to
govern or to discharge his functions only upon his proclamation and on the day
the law mandates his term of office to begin.

(3) Yes.

In the case of Frivaldo v. Commission on Elections, the Court ruled that "the
repatriation of Frivaldo RETROACTED to the date of the filing of his
application." In said case, the repatriation of Frivaldo was by virtue of Presidential
Decree No. 725, which took effect on June 5, 1975. The Court therein declared that
Presidential Decree No. 725 was a curative statute, which is retroactive in
nature. The retroactivity of Frivaldo's repatriation to the date of filing of his
application was justified by the Court, thus:

Page 127 of 323


xxxx
…The reason for this is simply that if, as in this case, it was the intent of the
legislative authority that the law should apply to past events—i.e., situations and
transactions existing even before the law came into being—in order to benefit the
greatest number of former Filipinos possible thereby enabling them to enjoy and
exercise the constitutionally guaranteed right of citizenship, and such legislative
intention is to be given the fullest effect and expression, then there is all the more
reason to have the law apply in a retroactive or retrospective manner to situations,
events and transactions subsequent to the passage of such law. That is, the
repatriation granted to Frivaldo x x x can and should be made to take effect as of
date of his application. As earlier mentioned, there is nothing in the law that would
bar this or would show a contrary intention on the part of the legislative authority;
and there is no showing that damage or prejudice to anyone, or anything unjust or
injurious would result from giving retroactivity to his repatriation. Neither has Lee
shown that there will result the impairment of any contractual obligation, disturbance
of any vested right or breach of some constitutional guaranty.
xxxx
Another argument for retroactivity to the date of filing is that it would prevent
prejudice to applicants. If P.D. 725 were not to be given retroactive effect, and the
Special Committee decides not to act, i.e., to delay the processing of applications for
any substantial length of time, then the former Filipinos who may be stateless, as
Frivaldo—having already renounced his American citizenship—was, may be
prejudiced for causes outside their control. This should not be. In case of doubt in
the interpretation or application of laws, it is to be presumed that the law-making
body intended right and justice to prevail.”

Republic Act No. 817118 has impliedly repealed Presidential Decree No. 725. They
cover the same subject matter: Providing for the repatriation of Filipino women who
have lost their Philippine citizenship by marriage to aliens and of natural-born
Filipinos. The Court's ruling in Frivaldo v. Commission on Elections that repatriation
retroacts to the date of filing of one's application for repatriation subsists for the same
reasons quoted above.

(4) Yes.

Considering that petitioner's repatriation retroacted to the date he filed his


application in 1997, he was, therefore, qualified to run for a mayoralty position in the
government in the May 10, 2004 elections.

(5) No.

The Court cannot fault the COMELEC en banc for affirming the decision of the
COMELEC First Division, considering that petitioner failed to prove before the
COMELEC that he had complied with the requirements of repatriation. Petitioner
submitted the necessary documents proving compliance with the requirements
of repatriation only during his motion for reconsideration, when the COMELEC
en banc could no longer consider said evidence.

As the COMELEC en banc correctly stated:

“The Comelec Rules of Procedure provides that insufficiency of evidence to justify the
decision is a ground for a motion for reconsideration (Rule 19, Section 1). The
evidence referred to in the above provision and to be considered in the Motion
for Reconsideration are those which were submitted during the hearing and
attached to the respective Memoranda of the parties which are already part of
the records of the case. In this regard, the evidence of the respondent were not able
to overcome the evidence of the petitioners.”

Page 128 of 323


It is, therefore, incumbent upon candidates for an elective office, who are repatriated
citizens, to be ready with sufficient evidence of their repatriation in case their Filipino
citizenship is questioned to prevent a repetition of this case.

[NOTE: The petition was DISMISSED.]

The Legislative Department

*Aquino III v. Comelec, G.R. No. 189793, April 7, 2010.


(Apportionment of legislative districts)

Facts:
-R.A. 9716 created an additional legislative district for the Province of Camarines
Sur by reconfiguring the existing first and second legislative districts of the province.
As a result of such reconfiguration, the first district ended up with a population of
only 176,383.

-Argument of the petitioners: R.A. 9716 must be declared unconstitutional because


Section 5(3), Article VI of the 1987 Constitution requires a 250,000 minimum
population for the creation of a new legislative district, to wit: “x x x Each legislative
district shall comprise, as far as practicable, contiguous, compact, and adjacent territory.
Each city with a population of at least two hundred fifty thousand, or each province, shall
have at least one representative.”

-Argument of the respondents: The 250,000 minimum population is only a


requirement for the creation of a legislative district in a CITY, but not in a PROVINCE.
Since R.A. 9716 only created an additional legislative district within the province of
Camarines Sur, it should be sustained as a perfectly valid reapportionment law.

Issue:
(1) Whether or not R.A. 9716 is constitutional.

Ruling:
(1) Yes. R.A. 9716 is constitutional.

The provision draws a plain and clear distinction between the entitlement of a city to a
district on one hand, and the entitlement of a province to a district on the other. For
while a province is entitled to at least a representative, with nothing mentioned
about population, a city must first meet a population minimum of 250,000 in
order to be similarly entitled.

The use by the subject provision of a comma to separate the phrase “each city
with a population of at least two hundred fifty thousand” from the phrase “or
each province” point to no other conclusion than that the 250,000 minimum
population is only required for a city, but not for a province. Plainly read, Section
5(3) of the Constitution requires a 250,000 minimum population only for a city to be
entitled to a representative, but not so for a province. x x x x

The Mariano case limited the application of the 250,000 minimum population
requirement for cities only to its initial legislative district. In other words, while
Section 5(3), Article VI of the Constitution requires a city to have a minimum
population of 250,000 to be entitled to a representative, it does not have to
increase its population by another 250,000 to be entitled to an additional
district.

Population is not the only factor but is just one of several other factors in the
composition of the additional district

*Mariano v. Comelec, G.R. No. 118577, March 7, 1995.


(Apportionment of legislative districts)

Page 129 of 323


Facts:
-R.A. No. 7854 converted Makati into a highly-urbanized city and created an
additional legislative district. From having been comprised of just one legislative
district, Makati now has two.

-Argument of petitioners: The addition of another legislative district in Makati is not in


accord with Section 5 (3), Article VI of the Constitution for as of the latest survey (1990
census), the population of Makati stands at only 450,000.

Issue:
Whether or not R.A. No. 7854 is constitutional.

Ruling:
Yes. R.A. No. 7854 is constitutional.

Section 5 (3), Article VI of the Constitution provides, inter alia, that a city with a
population of at least two hundred fifty thousand (250,000) shall have at least one
representative. Even granting that the population of Makati as of the 1990 census
stood at four hundred fifty thousand (450,000), its legislative district may still
be increased since it has met the minimum population requirement of two
hundred fifty thousand (250,000). In fact, Section 3 of the Ordinance appended to
the Constitution provides that a city whose population has increased to more than two
hundred fifty thousand (250,000) shall be entitled to at least one congressional
representative.

*Montejo v. Comelec, G.R. No. 118702, March 16, 1995.


(Apportionment of legislative districts)

Facts:
-Comelec promulgated Resolution No. 2736 where, among others, it transferred the
municipality of Capoocan of the Second District and the municipality of Palompon of
the Fourth District to the Third District of Leyte.

-Petitioner Montejo filed a motion for reconsideration calling the attention of


respondent COMELEC, among others, to the inequitable distribution of inhabitants
and voters between the First and Second Districts. He alleged that the First District
has 178,688 registered voters while the Second District has 156,462 registered voters
or a difference of 22,226 registered voters. To diminish the difference, he proposed
that the municipality of Tolosa with 7,7000 registered voters be transferred from the
First to the Second District.

-Argument of petitioner: Resolution No. 2736 violates the principle of equality of


representation ordained in the Constitution.

Issue:
Whether or not Resolution No. 2736 is constitutional.

Ruling:
No. Resolution No. 2736 is unconstitutional.

First, the Comelec has no authority to transfer municipalities from one


legislative district to another district. It is the Congress that has the legislative
power of redistricting and reapportionment.

Second, the issue involves a problem of reapportionment of legislative districts


and petitioner’s remedy lies with Congress. Section 5(4), Article VI of the
Constitution categorically gives Congress the power to reapportion, thus: "Within
three (3) years following the return of every census, the Congress shall make a
reapportionment of legislative districts based on the standards provided in this section."

Page 130 of 323


*Sema v. Comelec, G.R. No. 177597, July 16, 2008.
(Apportionment of legislative districts)

Facts:
-Congress enacted RA 9054, the Section 19, Article VI of which delegated to the
ARMM Regional Assembly the power to create provinces, cities, municipalities
and barangays.

-Pursuant to such delegation, the ARMM Regional Assembly enacted Muslim


Mindanao Autonomy Act No. 201 (MMA Act 201) creating the Province of Shariff
Kabunsuan composed of the eight municipalities in the first district of Maguindanao.

-(complicated facts but not important to the topic)

Issues:
(1) Whether or not Congress validly delegated to the ARMM Regional Assembly the
power to create legislative districts for the House of Representatives;

(2) Whether or not RA 9054 is constitutional;

(3) Whether or not MMA Act 201 is void.

Ruling:

(1) No. The power to create a province, or a city with a population of 250,000 or
more, requires also the power to create a legislative district.

Under the present Constitution, as well as in past Constitutions, the power to


increase the allowable membership in the House of Representatives, and to
reapportion legislative districts, is vested exclusively in Congress.

Section 5 (1), Article VI of the Constitution vests in Congress the power to increase,
through a law, the allowable membership in the House of Representatives. Section 5
(4) empowers Congress to reapportion legislative districts. The power to reapportion
legislative districts necessarily includes the power to create legislative districts
out of existing ones. Congress exercises these powers through a law that
Congress itself enacts, and not through a law that regional or local legislative
bodies enact.

(2) RA 9054 is unconstitutional. (See above ruling.)

(3) MMA Act 201 is void. (See above ruling.)

*Coquilla v. Comelec, G.R. No. 151914, July 31, 2002.


(Qualifications; Domicile)

Facts:
-Petitioner Coquilla was born on February 17, 1938 of Filipino parents in Oras,
Eastern Samar. He grew up and resided there until 1965, when he joined the United
States Navy. He was subsequently naturalized as a U.S. citizen. From 1970 to 1973,
petitioner thrice visited the Philippines while on leave from the U.S. Navy. Otherwise,
even after his retirement from the U.S. Navy in 1985, he remained in the United
States.

-On October 15, 1998, petitioner came to the Philippines and took out a residence
certificate, although he continued making several trips to the United States, the last
of which took place on July 6, 2000 and lasted until August 5, 2000. Subsequently,
petitioner applied for repatriation under R.A. No. 81715 to the Special Committee on
Naturalization. His application was approved on November 7, 2000, and, on
November 10, 2000, he took his oath as a citizen of the Philippines. Petitioner was

Page 131 of 323


issued Certificate of Repatriation No. 000737 on November 10, 2000 and Bureau of
Immigration Identification Certificate No. 115123 on November 13, 2000.

-On November 21, 2000, petitioner applied for registration as a voter of Butnga, Oras,
Eastern Samar. His application was approved by the Election Registration Board on
January 12, 2001. On February 27, 2001, he filed his certificate of candidacy
stating therein that he had been a resident of Oras, Eastern Samar for "two (2) years."

-His opponent sought the cancellation of petitioner’s certificate of candidacy on the


ground that the latter had made a material misrepresentation in his certificate of
candidacy by stating that he had been a resident of Oras for two years when in truth
he had resided therein for only about six months since November 10, 2000, when he
took his oath as a citizen of the Philippines.

Issue:
Whether or not Coquilla can be considered to have resided in Oras, Eastern Samar for
more than one (1) year.

Ruling:
No. The term "residence" is to be understood not in its common acceptation as
referring to "dwelling" or "habitation," but rather to "domicile" or legal residence,
that is, "the place where a party actually or constructively has his permanent
home, where he, no matter where he may be found at any given time, eventually
intends to return and remain (animus manendi)."

Coquillo lost his domicile of origin in Oras by becoming a U.S. citizen after
enlisting in the U.S. Navy in 1965. From then on and until November 10, 2000,
when he reacquired Philippine citizenship, he was an alien without any right to
reside in the Philippines save as our immigration laws may have allowed him to
stay as a visitor or as a resident alien.

Until his reacquisition of Philippine citizenship on November 10, 2000,


petitioner did not reacquire his legal residence in this country.

[NOTE: This doctrine was not applied to Grace Poe’s case.]

*Atong Paglaum, Inc. v. Comelec, G.R. No. 203766, April 2, 2013. [IMPORTANT!!!]
(Registration; Party-List System)

Facts:
-The Comelec disqualified 52 party-list groups and organizations from participating
in the 13 May 2013 party-list elections, particularly those that did not satisfy these
two criteria: (1) all national, regional, and sectoral groups or organizations must
represent the "marginalized and underrepresented" sectors; AND (2) all nominees
must belong to the "marginalized and underrepresented" sector they represent.

-Aggrieved by the disqualification, said part-list groups via Petitions for Certiorari and
Petitions for Certiorari and Prohibition, alleging grave abuse of discretion amounting
to lack or excess of jurisdiction on the part of the Comelec.

Issues:
(1) Whether or not the Comelec committed grave abuse of discretion amounting to lack
or excess of jurisdiction in disqualifying petitioners from participating in the 13 May
2013 party-list elections;

(2) Whether or not the criteria for participating in the party-list system laid down in
Ang Bagong Bayani and Barangay Association for National Advancement and
Transparency v. Commission on Elections (BANAT) should be applied by the
COMELEC in the coming 13 May 2013 party-list elections.

Ruling:

Page 132 of 323


[NOTE: This case abandoned the ruling in BANAT case.]

(1) No. We cannot, however, fault the COMELEC for following prevailing jurisprudence
in disqualifying petitioners. In following prevailing jurisprudence, the COMELEC could
not have committed grave abuse of discretion. However, for the coming 13 May 2013
party-list elections, we must now impose and mandate the party-list system
actually envisioned and authorized under the 1987 Constitution and R.A. No.
7941.

---
NEW RULE (abandoning BANAT doctrine):

In determining who may participate in the coming 13 May 2013 and subsequent
party-list elections, the COMELEC shall adhere to the following parameters:

1. Three different groups may participate in the party-list system: (1) national
parties or organizations, (2) regional parties or organizations, and (3) sectoral
parties or organizations.

2. National parties or organizations and regional parties or organizations do not


need to organize along sectoral lines and do not need to represent any
"marginalized and underrepresented" sector.

3. Political parties can participate in party-list elections provided they register


under the party-list system and do not field candidates in legislative district
elections. A political party, whether major or not, that fields candidates in
legislative district elections can participate in party-list elections only through its
sectoral wing that can separately register under the party-list system. The sectoral
wing is by itself an independent sectoral party, and is linked to a political party
through a coalition.

4. Sectoral parties or organizations may either be "marginalized and


underrepresented" or lacking in "well-defined political constituencies." It is
enough that their principal advocacy pertains to the special interest and concerns of
their sector. The sectors that are "marginalized and underrepresented" include
labor, peasant, fisherfolk, urban poor, indigenous cultural communities,
handicapped, veterans, and overseas workers. The sectors that lack "well-defined
political constituencies" include professionals, the elderly, women, and the
youth.

5. A majority of the members of sectoral parties or organizations that represent


the "marginalized and underrepresented" must belong to the "marginalized and
underrepresented" sector they represent. Similarly, a majority of the members of
sectoral parties or organizations that lack "well-defined political constituencies"
must belong to the sector they represent. The nominees of sectoral parties or
organizations that represent the "marginalized and underrepresented," or that
represent those who lack "well-defined political constituencies," either must
belong to their respective sectors, or must have a track record of advocacy for
their respective sectors. The nominees of national and regional parties or
organizations must be bona-fide members of such parties or organizations.

6. National, regional, and sectoral parties or organizations shall not be


disqualified if some of their nominees are disqualified, provided that they have
at least one nominee who remains qualified.

(2) No. The experimentations in socio-political engineering have only resulted in


confusion and absurdity in the party-list system. Such experimentations, in clear
contravention of the 1987 Constitution and R.A. No. 7941, must now come to an
end. (See above ruling for the new parameters.)

Page 133 of 323


*Philippine Guardians Brotherhood, Inc. (PGBI) v. Comelec G.R. No. 190529, April 29,
2010.
(Cancellation of registration of a party-list organization)

Facts:
- The COMELEC delisted Philippine Guardians Brotherhood, Inc. (PGBI) from the
roster of registered national, regional or sectoral parties, organizations or coalitions
under the party-list system on the ground that it failed to get 2% of the votes cast
in 2004 and it did not participate in the 2007 elections.

-Section 6(8) of Republic Act No. 7941 (RA 7941), otherwise known as the Party-List
System Act, provides:

“Section 6. Removal and/or Cancellation of Registration. – The COMELEC may motu proprio or upon
verified complaint of any interested party, remove or cancel, after due notice and hearing, the
registration of any national, regional or sectoral party, organization or coalition on any of the
following grounds:
xxxx
(8) It fails to participate in the last two (2) preceding elections or fails to obtain at least two per
centum (2%) of the votes cast under the party-list system in the two (2) preceding elections for the
constituency in which it has registered.”

-Argument of Comelec: Since PGBI failed to get 2% of the votes in 2004 and did not
participate at all in the 2007 elections, it necessarily failed to get at least two per
centum (2%) of the votes cast in the two preceding elections.

Issue:
Whether or not there is legal basis for delisting PGBI.

Ruling:

[NOTE: The Supreme Court partly invalidated the 2% party-list vote requirement
provided in RA 7941 in an earlier case of Barangay Association for Advancement and
National Transparency (BANAT) v. Comelec, G.R. No. 179271, April 21, 2009.]

There is none. PGBI is qualified to be voted upon as a party-list group or


organization in the coming May 2010 elections.

The law is clear – the Comelec may motu proprio or upon verified complaint of any
interested party, remove or cancel, after due notice and hearing, the registration of any
national, regional or sectoral party, organization or coalition if it: (a) fails to
participate in the last two (2) preceding elections; or (b) fails to obtain at least
two per centum (2%) of the votes cast under the party-list system in the two (2)
preceding elections for the constituency in which it has registered.

The word "or" is a disjunctive term signifying disassociation and independence of


one thing from the other things enumerated; it should, as a rule, be construed in
the sense in which it ordinarily implies, as a disjunctive word. Thus, the plain, clear
and unmistakable language of the law provides for two (2) separate reasons for
delisting.

--------
[Note on the invalidity of 2% requirement:

We rule that, in computing the allocation of additional seats, the continued


operation of the two percent (2%) threshold for the distribution of the additional
seats as found in the second clause of Section 11(b) of R.A. No. 7941 is
unconstitutional. This Court finds that the two percent threshold makes it
mathematically impossible to achieve the maximum number of available party list
seats when the number of available party list seats exceeds 50. The continued
operation of the two percent threshold in the distribution of the additional seats

Page 134 of 323


frustrates the attainment of the permissive ceiling that 20% of the members of
the House of Representatives shall consist of party-list representatives.

The disqualification for failure to get 2% party-list votes in two (2) preceding elections
should therefore be understood in light of the Banat ruling that party-list groups or
organizations garnering less than 2% of the party-list votes may yet qualify for a
seat in the allocation of additional seats.

A party-list group or organization which qualified in the second round of seat


allocation cannot now validly be delisted for the reason alone that it garnered
less than 2% in the last two elections. In other words, the application of this
disqualification should henceforth be contingent on the percentage of party-list votes
garnered by the last party-list organization that qualified for a seat in the House of
Representatives, a percentage that is less than the 2% threshold invalidated in Banat.
The disqualification should now necessarily be read to apply to party-list groups
or organizations that did not qualify for a seat in the two preceding elections for
the constituency in which it registered. x x x x

To reiterate, (a) Section 6(8) of RA 7941 provides for two separate grounds for delisting;
these grounds cannot be mixed or combined to support delisting; and (b) the
disqualification for failure to garner 2% party-list votes in two preceding
elections should now be understood, in light of the Banat ruling, to mean failure
to qualify for a party-list seat in two preceding elections for the constituency in
which it has registered.]

*Ang Ladlad LGBT Party v. Comelec, G.R. No. 190582, April 8, 2010.
[IMPORTANT!!!]
(Registration; Party-List System)

Facts:
The Comelec refused to accredit Ang Ladlad as a party-list organization under R.A.
7941, otherwise known as the Party-List System Act, on the ground that the LGBT
sector is neither enumerated in the Constitution and R.A. 7941, nor is it
associated with or related to any of the sectors in the enumeration.

-Ang Ladlad is an organization composed of men and women who identify themselves
as lesbians, gays, bisexuals, or trans-gendered individuals (LGBTs).

-In denying Ang Ladlad's registration, the Comelec's Second Division ruled:
“Until the time comes when Ladlad is able to justify that having mixed sexual
orientations and transgender identities is beneficial to the nation, its application for
accreditation under the party-list system will remain just that. x x x x

Even if society’s understanding, tolerance, and acceptance of LGBT’s is elevated, there


can be no denying that Ladlad constituencies are still males and females, and they
will remain either male or female protected by the same Bill of Rights that
applies to all citizens alike. x x x x

As a society, the Philippines cannot ignore its more than 500 years of Muslim and
Christian upbringing, such that some moral precepts espoused by said religions have
seeped into society and these are not publicly accepted moral norms.”

Issue:
Whether or not there is legal basis for Comelec’s refusal to accredit Ang Ladlad as a
party-list group.

Ruling:
There is none.

Respondent mistakenly opines that our ruling in Ang Bagong Bayani stands for the
proposition that only those sectors specifically enumerated in the law or related to said

Page 135 of 323


sectors (labor, peasant, fisherfolk, urban poor, indigenous cultural communities,
elderly, handicapped, women, youth, veterans, overseas workers, and professionals)
may be registered under the party-list system.

The enumeration of marginalized and under-represented sectors is not exclusive.


The crucial element is not whether a sector is specifically enumerated, but
whether a particular organization complies with the requirements of the
Constitution and R.A. 7941.

Our Constitution provides in Article III, Section 5 that, “no law shall be made
respecting an establishment of religion, or prohibiting the free exercise thereof.” At
bottom, what our non-establishment clause calls for is “government neutrality in
religious matters.” Clearly, governmental reliance on religious justification is
inconsistent with this policy of neutrality. We thus find that it was grave
violation of the non-establishment clause for the Comelec to utilize the Bible
and the Koran to justify the exclusion of Ang Ladlad.

Moral disapproval, without more, is not a sufficient governmental interest to


justify exclusion of homosexuals from participation in the party-list system. The
denial of Ang Ladlad’s registration on purely moral grounds amounts more to a
statement of dislike and disapproval of homosexuals, rather than a tool to further any
substantial public interest.

*ABANG LINGKOD v. Comelec, G R. No. 206952, October 22, 2013.


(Registration; Party-List System)

Facts:
-ABANG LINGKOD is a sectoral organization that represents the interests of peasant
farmers and fisherfolks, and was registered under the party-list system on December
22, 2009. It participated in the May 2010 elections, but failed to obtain the number of
votes needed for a seat in the House of Representatives.

-In a Resolution dated November 7, 2012, cancelled ABANG LINGKOD's registration


as a party-list group on the ground that ABANG LINGKOD failed to establish its
track record in uplifting the cause of the marginalized and underrepresented;
that it merely offered photographs of some alleged activities it conducted after the
May 2010 elections.

Issue:
Whether or not the COMELEC gravely abused its discretion in cancelling ABANG
LINGKOD's registration under the party-list system.

Ruling:
Yes.

The flaw in the COMELEC’s disposition lies in the fact that it insists on requiring
party-list groups to present evidence showing that they have a track record in
representing the marginalized and underrepresented.

Track record is a record of past performance often taken as an indicator of likely


future performance. As a requirement imposed by Ang Bagong Bayani for groups
intending to participate in the party-list elections, track record pertains to the
actual activities undertaken by groups to uplift the cause of the sector/s, which
they represent. x x x x

Under Section 5 of R.A. No. 7941, groups intending to register under the party-
list system are not required to submit evidence of their track record; they are
merely required to attach to their verified petitions their “constitution, by-laws,
platform of government, list of officers, coalition agreement, and other relevant
information as may be required by the COMELEC.”

Page 136 of 323


*Bantay Republic Act v. Comelec, G.R. No. 177271, May 4, 2007.
(Party-List system; Manner of Voting; Disclosure)

Facts:
-The Comelec denied petitioner’s request for the release or disclosure of the names of
the nominees of the fourteen (14) accredited participating party-list groups
mentioned in petitioner’s letter-request.

-Argument of Comelec: The party list elections must not be personality oriented. The
people are to vote for sectoral parties, organizations, or coalitions, not for their
nominees. There is nothing in R.A. 7941 that requires the Comelec to disclose
the names of nominees.

Issue:
Whether or not respondent Comelec committed grave abuse of discretion in refusing to
reveal the names of the nominees of the various party-list groups.

Ruling:
Yes.

No national security or like concerns is involved in the disclosure of the names


of the nominees of the party-list groups in question.

Section 28, Article II of the Constitution reads:

Sec. 28. Subject to reasonable conditions prescribed by law, the State adopts and implements a policy of
full public disclosure of all its transactions involving public interest.

The Comelec's reasoning that a party-list election is not an election of personalities is


valid to a point. It cannot be taken, however, to justify its assailed non-disclosure
stance which comes, as it were, with a weighty presumption of invalidity,
impinging, as it does, on a fundamental right to information.

Respondent Comelec has a constitutional duty to disclose and release the names of
the nominees of the party-list groups named in the herein petitions.

*BANAT v. Comelec, G.R. No. 179271, April 21, 2009. [EDIT!!!]


(Party-List System; Allocation of Seats; Procedure)

Trillanes IV v. Pimenetel Sr., G.R. No. 179817, June 27, 2008.


(Privileges; Freedom from arrest)

Facts:
-Trillanes was charged with coup d’etat together with other junior officers of the AFP
for the so-called “Oakwood incident.” While in detention, Trillanes ran for a seat in
the Senate and won.

-In view thereof, Trillanes filed with the RTC of Makati where his case was pending an
"Omnibus Motion for Leave of Court to be Allowed to Attend Senate Sessions and
Related Requests,” praying that: (1) he be allowed to go to the Senate to attend all
official functions of the Senate, and to attend the regular and plenary sessions of the
Senate, committee hearings, committee meetings, etc.; (2) be allowed to give interviews
and to air his comments, reactions and/or opinions to the press or the media
regarding the important issues affecting the country and the public.

-The RTC denied his motion.

Page 137 of 323


-Argument of Trillanes: The fact that the people, in their sovereign capacity, elected
him to the position of senator provides the proper legal justification to allow him to
work and serve his mandate as a senator.

Issue:
Whether or not election to Congress is a sufficient basis to allow Trillanes liberty for
him to work as a senator.

Ruling:
No.

Election to Congress is not a reasonable classification in criminal law


enforcement as the functions and duties of the office are not substantial
distinctions which lift one from the class of prisoners interrupted in their
freedom and restricted in liberty of movement. x x x x The performance of
legitimate and even essential duties by public officers has never been an excuse
to free a person validly in prison. x x x x Congress continues to function well in the
physical absence of one or a few of its members. x x x x Never has the call of a
particular duty lifted a prisoner into a different classification from those others who
are validly restrained by law.

The Rules state that no person charged with a capital offense, or an offense
punishable by reclusion perpetua or life imprisonment, shall be admitted to bail
when evidence of guilt is strong, regardless of the stage of the criminal action.

That the cited provisions apply equally to rape (like in the case of Jalosjos) and
coup d’etat cases, both being punishable by reclusion perpetua, is beyond cavil.
Within the class of offenses covered by the stated range of imposable penalties, there
is clearly no distinction as to the political complexion of or moral turpitude involved in
the crime charged.

In the present case, it is uncontroverted that petitioner’s application for bail and for
release on recognizance was denied. The determination that the evidence of guilt is
strong, whether ascertained in a hearing of an application for bail or imported from a
trial court’s judgment of conviction, justifies the detention of an accused as a valid
curtailment of his right to provisional liberty.

The presumption of innocence does not carry with it the full enjoyment of civil and
political rights. x x x x The mandate of the people (electing Trillanes to the senate)
yields to the Constitution which the people themselves ordained to govern all
under the rule of law.

[SIDE NOTE: Trillanes and his colleagues were granted amnesty by PNoy in 2010. He
walked free since then.]

*Osmeña v. Pendatun, G.R. No. L-17144, October 28, 1960. [IMPORTANT!!!]


(Privilege of speech and of debate)

Facts:
-During his privilege speech, Congressman Osmeña made serious imputations of
bribery against then President Garcia (“his pardons were for sale, detained persons
charged with grave offenses could bail out if they could afford to pay a large sum,” etc).

-Pendatun and 14 other congressmen issued a Resolution requiring Osmeña to


appear before the Special Committee to substantiate his charges, and if he failed to
do so, he would have to show cause why he should not be punished by the House.

-After giving Congressman Osmeña a chance to defend himself, the committee


submitted its reports finding said Osmeña guilty of serious disorderly behavior; and
acting on such report, the House approved on the same day House Resolution No.

Page 138 of 323


175, declaring him guilty as recommended, and suspending him from office for 15
months.

-Osmeña then went to the Supreme Court for declaratory relief, certiorari and
prohibition with preliminary injunction.”

-Argument of Osmeña: The Resolution violated his constitutional absolute


parliamentary immunity for speeches delivered before the House.

Issues:
(1) Whether or not Osmeña may be validly suspended by the House despite his
parliamentary privilege;
(2) Whether or not delivery of speeches attacking the Chief Executive may constitute
disorderly conduct for which Osmeña may be disciplined.

Ruling:
(1) Yes.

Section 15, Article VI of our Constitution provides that “for any speech or debate” in
Congress, the Senators or Members of the House of Representative "shall not be
questioned in any other place." This section was taken or is a copy of sec. 6, clause
1 of Art. 1 of the Constitution of the United States. In that country, the provision has
always been understood to mean that although exempt from prosecution or civil
actions for their words uttered in Congress, the members of Congress may,
nevertheless, be questioned in Congress itself. Observe that “they shall not be
questioned in any other place” than Congress. x x x x

Parliamentary privilege guarantees the legislator complete freedom of expression


without fear of being made responsible in criminal or civil actions before the
courts or any other forum outside of the Congressional Hall. But it does not
protect him from responsibility before the legislative body itself whenever his
words and conduct are considered by the latter disorderly or unbecoming a
member thereof.

(2) Yes.

However, the House is the judge of what constitutes disorderly behavior, not only
because the Constitution has conferred jurisdiction upon it, but also because the
matter depends mainly on factual circumstances of which the House knows best
but which cannot be depicted in black and white for presentation to, and adjudication
by the Courts. x x x x The House has exclusive power; the courts have no
jurisdiction to interfere.

*Santiago v. Sandiganbayan; G.R. No. 128055, April 18, 2001.


(Preventive Suspension; R.A. 3019; Discipline of Members of the Senate)

Facts:
-Miriam Defensor-Santiago, then Commission of Immigration and Deportation (CID)
Commissioner, was charged with alleged Violation of the Anti-Graft and Corrupt
Practices Act (R.A. 3019).

-In the course of the proceedings, pursuant to R.A. 3019, the Sandiganbayan
preventively suspended her for ninety (90) days. At that time, Santiago was already
a Senator.

-Contention of Santiago: Only the Congress has the power to discipline its own ranks.

Issue:
Whether or not the Sandiganbayan has the authority to preventively suspend a
Senator.

Page 139 of 323


Ruling:
Yes.

The authority of the Sandiganbayan to order the preventive suspension of an


incumbent public official charged with violation of the provisions of Republic Act No.
3019 has both legal and jurisprudential support. Section 13 of the statute provides:

“SECTION 13. Suspension and loss of benefits. — Any incumbent public officer against whom any
criminal prosecution under a valid information under this Act or under Title 7, Book II of the Revised
Penal Code or for any offense involving fraud upon government or public funds or property whether as a
simple or as a complex offense and in whatever stage of execution and mode of participation, is pending
in court, shall be suspended from office. Should he be convicted by final judgment, he shall lose all
retirement or gratuity benefits under any law, but if he is acquitted, he shall be entitled to reinstatement
and to the salaries and benefits which he failed to receive during suspension, unless in the meantime
administrative proceedings have been filed against him.

In the event that such convicted officer, who may have already been separated from the service, has
already received such benefits he shall be liable to restitute the same to the Government.”

It would appear, indeed, to be a ministerial duty of the court to issue an order of


suspension upon determination of the validity of the information filed before it.
Once the information is found to be sufficient in form and substance, the court
is bound to issue an order of suspension as a matter of course, and there seems
to be “no ifs and buts about it.” x x x x

In issuing the preventive suspension of petitioner, the Sandiganbayan merely


adhered to the clear and unequivocal mandate of the law, as well as the
jurisprudence in which the Court has, more than once, upheld Sandiganbayan's
authority to decree the suspension of public officials and employees indicted before it.

Section 13 of Republic Act No. 3019 does not state that the public officer concerned
must be suspended only in the office where he is alleged to have committed the acts
with which he has been charged. Thus, it has been held that the use of the word
“office” would indicate that it applies to any office which the officer charged
may be holding, and not only the particular office under which he stands
accused. x x x x

The order of suspension prescribed by Republic Act No. 3019 is distinct from the
power of Congress to discipline its own ranks under the Constitution which
provides that each —

“x x x house may determine the rules of its proceedings, punish its Members for disorderly behavior, and,
with the concurrence of two-thirds of all its Members, suspend or expel a Member. A penalty of
suspension, when imposed, shall not exceed sixty days.”

The suspension contemplated in the above constitutional provision is a punitive


measure that is imposed upon determination by the Senate or the House of
Representatives, as the case may be, upon an erring member. Thus, in its
resolution in the case of Ceferino Paredes, Jr. vs. Sandiganbayan, et al., the Court
affirmed the order of suspension of Congressman Paredes by the Sandiganbayan,
despite his protestations on the encroachment by the court on the prerogatives of
Congress. The Court ruled:

"x x x Petitioner's invocation of Section 16 (3), Article VI of the Constitution — which


deals with the power of each House of Congress inter alia to 'punish its Members for
disorderly behavior,' and 'suspend or expel a Member' by a vote of two-thirds of all its
Members subject to the qualification that the penalty of suspension, when imposed,
should not exceed sixty days — is unavailing, as it appears to be quite distinct from
the suspension spoken of in Section 13 of RA 3019, which is not a penalty but a
preliminary, preventive measure, prescinding from the fact that the latter is not
being imposed on petitioner for misbehavior as a Member of the House of
Representatives."

Page 140 of 323


The doctrine of separation of powers by itself may not be deemed to have
effectively excluded members of Congress from Republic Act No. 3019 nor from
its sanctions. The maxim simply recognizes each of the three co-equal and
independent, albeit coordinate, branches of the government — the Legislative, the
Executive and the Judiciary — has exclusive prerogatives and cognizance within its
own sphere of influence and effectively prevents one branch from unduly intruding
into the internal affairs of either branch. x x x x

Republic Act No. 3019 does not exclude from its coverage the members of Congress
and that, therefore, the Sandiganbayan did not err in thus decreeing the assailed
preventive suspension order.

*Vilando v. HRET, G.R. Nos. 192147 & 192149, August 23, 2011.
(HRET; Authority to inquire into legality of a judgment)

Facts:
-In the May 14, 2007 elections, Limkaichong filed her certificate of candidacy for the
position of Representative of the First District of Negros Oriental. She won over the
other contender, Olivia Paras.

-On May 25, 2007, she was proclaimed as Representative by the Provincial Board of
Canvassers on the basis of Comelec Resolution No. 80623 issued on May 18, 2007.

-On July 23, 2007, she assumed office as Member of the House of Representatives.

-Eventually, on April 21, 2009 and May 27, 2009, petitioner Renald F. Vilando
(Vilando), as taxpayer; and Jacinto Paras, as registered voter of the congressional
district concerned, filed separate petitions for Quo Warranto against Limkaichong
before the HRET.

-Petitioners asserted that Limkaichong was a Chinese citizen and ineligible for the
office she was elected and proclaimed. They alleged that she was born to a father
(Julio Sy), whose naturalization had not attained finality. Also, they invoked the
jurisdiction of the HRET for a determination of Limkaichong’s citizenship, which
necessarily included an inquiry into the validity of the naturalization certificate of
Julio Sy.

-After due proceedings, the HRET dismissed both petitions.

Issue:
Whether or not Limkaichong should be disqualified based on her alleged Chinese
citizenship.

Ruling:
No.

In this petition, Vilando seeks to disqualify Limkaichong on the ground that she is a
Chinese citizen. To prove his point, he makes reference to the alleged nullity of the
grant of naturalization of Limkaichong’s father which, however, is not allowed as it
would constitute a collateral attack on the citizenship of the father. In our
jurisdiction, an attack on a person’s citizenship may only be done through a
direct action for its nullity. x x x x

The proper proceeding to assail the citizenship of Limkaichong’s father should be in


accordance with Section 18 of Commonwealth Act No. 473. x x x x It may be true that,
as alleged by said respondents, that the proceedings for naturalization were tainted
with certain infirmities, fatal or otherwise, but that is beside the point in this case. The
jurisdiction of the court to inquire into and rule upon such infirmities must be
properly invoked in accordance with the procedure laid down by law. Such procedure
is the cancellation of the naturalization certificate. [Section 1(5), Commonwealth
Act No. 63], in the manner fixed in Section 18 of Commonwealth Act No. 473,

Page 141 of 323


hereinbefore quoted, namely, “upon motion made in the proper proceedings by the
Solicitor General or his representatives, or by the proper provincial fiscal.” In
other words, the initiative must come from these officers, presumably after previous
investigation in each particular case.

Clearly, under law and jurisprudence, it is the State, through its representatives
designated by statute, that may question the illegally or invalidly procured
certificate of naturalization in the appropriate denaturalization proceedings. It is
plainly not a matter that may be raised by private persons in an election case involving
the naturalized citizen’s descendant. x x x x

Time and again, this Court has acknowledged this sole and exclusive jurisdiction of
the HRET. The power granted to HRET by the Constitution is intended to be as
complete and unimpaired as if it had remained originally in the legislature. Such
power is regarded as full, clear and complete and excludes the exercise of any
authority on the part of this Court that would in any wise restrict it or curtail it or
even affect the same.

Such power of the HRET, no matter how complete and exclusive, does not carry
with it the authority to delve into the legality of the judgment of naturalization
in the pursuit of disqualifying Limkaichong. To rule otherwise would operate as a
collateral attack on the citizenship of the father which, as already stated, is not
permissible.

*Manosca v. Court of Appeals, G.R. No. 106440, January 29, 1996.


(Prohibition against appropriations for sectarian benefit)

Facts:
-Petitioners inherited a piece of land located at P. Burgos Street, Calzada, Taguig.
Metro Manila, with an area of about four hundred ninety-two (492) square meters.

-When the parcel was ascertained by the National Historical Institute (NHI) to have
been the birthsite of Felix Y. Manalo, the founder of Iglesia Ni Cristo, it passed
Resolution No. 1, Series of 1986, pursuant to Section 42 of Presidential Decree No.
260, declaring the land to be a national historical landmark.

-Accordingly, on 29 May 1989, the Republic, through the Office of the Solicitor-
General, instituted a complaint for expropriation before the Regional Trial Court of
Pasig for and in behalf of the NHI.

-Petitioners’ contentions: (1) The intended expropriation was not for a public
purpose; (2) The said expropriation would constitute an application of public funds,
directly or indirectly, for the use, benefit, or support of Iglesia ni Cristo, a religious
entity, contrary to the provision of Section 29(2), Article VI, of the 1987 Constitution.

Issue:
Whether or not the expropriation by the State involving the birth site of Felix Manalo is
proper.

Ruling:
Yes.

For condemnation purposes, "public use" is one which confers same benefit or
advantage to the public; it is not confined to actual use by public. It is measured in
terms of right of public to use proposed facilities for which condemnation is sought
and, as long as public has right of use, whether exercised by one or many
members of public, a "public advantage" or "public benefit" accrues sufficient to
constitute a public use. x x x x

Petitioners ask: But “what is the so-called unusual interest that the expropriation of
(Felix Manalo's) birthplace become so vital as to be a public use appropriate for the

Page 142 of 323


exercise of the power of eminent domain" when only members of the Iglesia ni Cristo
would benefit? This attempt to give some religious perspective to the case deserves
little consideration, for what should be significant is the principal objective of, not
the casual consequences that might follow from, the exercise of the power. The
purpose in setting up the marker is essentially to recognize the distinctive
contribution of the late Felix Manalo to the culture of the Philippines, rather
than to commemorate his founding and leadership of the Iglesia ni Cristo.

The practical reality that greater benefit may be derived by members of the
Iglesia ni Cristo than by most others could well be true but such a peculiar
advantage still remains to be merely incidental and secondary in nature. Indeed,
that only a few would actually benefit from the expropriation of property does
not necessarily diminish the essence and character of public use.

*Standard Chartered Bank v. Senate Committee on Banks, G.R. No. 167173, December
27, 2007.
(In aid of legislation; Rights of persons appearing therein)

-The Senate issued a resolution directing the Committee on Banks, Currencies, and
Financial Institutions, to conduct an inquiry, in aid of legislation, into the reported
sale of unregistered and high-risk securities by Standard Chartered Bank (SCB) which
resulted in billions of losses to the investing public.

-Standard Chartered Bank whose actions have reportedly defrauded hundreds of


Filipino investors of billions of pesos through the sale of unregistered securities in the
form of high-risk mutual funds falsely advertised and marketed as safe investment
haven.

-Petitioners, who were corporate officers of SCB, were later served by the Senate with
subpoenae ad testificandum and duces tecum to compel them to attend and testify
at the hearing.

-Petitioners then filed before the SC a Petition for Prohibition (With Prayer for
Issuance of Temporary Restraining Order and/or Injunction), seeking to prohibit the
Senate from compelling petitioners to appear and testify in the inquiry being
conducted.

-Petitioners’ contentions: The issue of the alleged sale in the Philippines by SCB-
Philippines of unregistered foreign securities is the subject of several criminal and
civil actions now pending before various courts. To allow the Senate to investigate
the matter would create the possibility of conflicting judgments, and that the inquiry
into the same justiciable controversy would be an encroachment on the exclusive
domain of judicial jurisdiction that had set in much earlier.

Issue:
Whether or not the conduct of an inquiry in aid of legislation should proceed,
considering that the issue is already subject of several cases pending before various
courts.

Ruling:
Yes.

It is true that in Bengzon case, the Court declared that the issue to be investigated
was one over which jurisdiction had already been acquired by the Sandiganbayan, and
to allow the Senate Blue Ribbon Committee to investigate the matter would create the
possibility of conflicting judgments; and that the inquiry into the same justiciable
controversy would be an encroachment on the exclusive domain of judicial jurisdiction
that had set in much earlier.

To the extent that, in the case at bench, there are a number of cases already pending
in various courts and administrative bodies involving the petitioners, relative to the

Page 143 of 323


alleged sale of unregistered foreign securities, there is a resemblance between this
case and Bengzon. However, the similarity ends there.

Central to the Court’s ruling in Bengzon—that the Senate Blue Ribbon Committee was
without any constitutional mooring to conduct the legislative investigation—was the
Court’s determination that the intended inquiry was not in aid of legislation. The
Court found that the speech of Senator Enrile, which sought such investigation
contained no suggestion of any contemplated legislation; it merely called upon the
Senate to look into possible violations of Section 5, Republic Act No. 3019. Thus, the
Court held that the requested probe failed to comply with a fundamental requirement
of Section 21, Article VI of the Constitution, which states:

The Senate or the House of Representatives or any of its respective committees may conduct inquiries in
aid of legislation in accordance with its duly published rules of procedure. The rights of persons
appearing in or affected by such inquiries shall be respected.

Accordingly, we stopped the Senate Blue Ribbon Committee from proceeding with the
legislative investigation in that case.

Unfortunately for the petitioners, this distinguishing factual milieu in Bengzon


does not obtain in the instant case. P.S. Resolution No. 166 is explicit on the
subject and nature of the inquiry to be (and already being) conducted by the
respondent Committee, as found in the last three Whereas clauses thereof, viz.:

WHEREAS, existing laws including the Securities Regulation Code seem to be inadequate in preventing
the sale of unregistered securities and in effectively enforcing the registration rules intended to protect the
investing public from fraudulent practices;

WHEREAS, the regulatory intervention by the SEC and BSP likewise appears inadequate in preventing
the conduct of proscribed activities in a manner that would protect the investing public;

WHEREAS, there is a need for remedial legislation to address the situation, having in mind the
imposition of proportionate penalties to offending entities and their directors, officers and
representatives among other additional regulatory measures; x x x x

The unmistakable objective of the investigation, as set forth in the said resolution,
exposes the error in petitioners’ allegation that the inquiry, as initiated in a privilege
speech by the very same Senator Enrile, was simply “to denounce the illegal practice
committed by a foreign bank in selling unregistered foreign securities x x x.” This
fallacy is made more glaring when we consider that, at the conclusion of his privilege
speech, Senator Enrile urged the Senate “to immediately conduct an inquiry, in aid of
legislation, so as to prevent the occurrence of a similar fraudulent activity in the
future.”

Indeed, the mere filing of a criminal or an administrative complaint before a


court or a quasi-judicial body should not automatically bar the conduct of
legislative investigation. Otherwise, it would be extremely easy to subvert any
intended inquiry by Congress through the convenient ploy of instituting a
criminal or an administrative complaint. Surely, the exercise of sovereign
legislative authority, of which the power of legislative inquiry is an essential
component, cannot be made subordinate to a criminal or an administrative
investigation.

The Executive Department

*Macalintal v. Comelec, G.R. No. 157013, July 10, 2003. [IMPORTANT!]


(Congress as canvassing board; Absentee Voting; Power of Comelec)

Facts:
-Section 4 of R.A. No. 9189 (The Overseas Absentee Voting Act) provides that all
citizens of the Philippines abroad, who are not otherwise disqualified by law, at least

Page 144 of 323


eighteen (18) years of age on the day of elections, may vote for president, vice-
president, senators and party-list representatives.

-Section 5(d) of R.A. No. 9189 disqualifies from voting an immigrant or permanent
resident who is recognized as such in the host country, UNLESS he/she executes,
upon registration, an affidavit prepared for the purpose by the Commission declaring
that he/she shall resume actual physical permanent residence in the Philippines
not later than three (3) years from approval of his/her registration under the said
law.

-Section 18.5 of R.A. No. 9189 in relation to Section 4 of the same Act empowers the
COMELEC to order the proclamation of the winning candidates (president, vice-
president, senators and party-list representatives).

-Sections 19 and 25 of R.A. No. 9189 created the “Joint Congressional Oversight
Committee” with the power to review, revise, amend and approve the Implementing
Rules and Regulations promulgated by the COMELEC.

-Arguments of Macalintal:
(1) Section 5(d) is unconstitutional because it violates Section 1, Article V of the 1987
Constitution which requires that the voter must be a resident in the Philippines for at
least one year and in the place where he proposes to vote for at least six months
immediately preceding an election. He cites the ruling of the Supreme Court in Caasi
vs. Court of Appeals, wherein it was held that a “green card” holder immigrant to the
United States is deemed to have abandoned his domicile and residence in the
Philippines;

(2) Section 18.5 is unconstitutional, as it affects the canvass of votes and proclamation
of winning candidates for president and vice-president;

(3) Sections 19 and 25 creating the Joint Congressional Oversight Committee are
unconstitutional intrudes into the independence of the COMELEC. Should the rules
promulgated by the COMELEC violate any law, it is the Court that has the power to
review the same via the petition of any interested party, including the legislators.

Issues:
(1) Whether or not Section 5(d) of R.A. No. 9189 violates Section 1, Article V of the
1987 Constitution;

(2) Whether or not Section 18.5 of R.A. No. 9189 is unconstitutional insofar as it
involves the canvass of votes and proclamation of winning candidates for president
and vice-president;

(3) Whether or not the creation of the Joint Congressional Oversight Committee
violates Section 1, Article IX-A of the Constitution mandating the independence of
constitutional commissions.

Ruling:
(1) No.

Contrary to Macalintal’s claim that Section 5(d) circumvents the Constitution,


Congress enacted the law prescribing a system of overseas absentee voting in
compliance with the constitutional mandate. Such mandate expressly requires that
Congress provide a system of absentee voting that necessarily presupposes that the
“qualified citizen of the Philippines abroad” is not physically present in the country.
The provisions of Sections 5(d) and 11 are components of the system of overseas
absentee voting established by R.A. No. 9189. The qualified Filipino abroad who
executed the affidavit is deemed to have retained his domicile in the Philippines. He is
presumed not to have lost his domicile by his physical absence from this country. His
having become an immigrant or permanent resident of his host country does not
necessarily imply an abandonment of his intention to return to his domicile of

Page 145 of 323


origin, the Philippines. Therefore, under the law, he must be given the opportunity
to express that he has not actually abandoned his domicile in the Philippines by
executing the affidavit required by Sections 5(d) and 8(c) of the law. x x x x

Ordinarily, an absentee is not a resident and vice versa; a person cannot be at the
same time, both a resident and an absentee. However, under our election laws and
the countless pronouncements of the Court pertaining to elections, an absentee
remains attached to his residence in the Philippines as residence is considered
synonymous with domicile. x x x x For political purposes the concepts of residence
and domicile are dictated by the peculiar criteria of political laws. As these concepts
have evolved in our election law, what has clearly and unequivocally emerged is the
fact that residence for election purposes is used synonymously with domicile. x x x x

To repeat, the affidavit is required of immigrants and permanent residents abroad


because by their status in their host countries, they are presumed to have
relinquished their intent to return to this country; thus, without the affidavit,
the presumption of abandonment of Philippine domicile shall remain.

(2) Yes.

Section 18.5 of R.A. No. 9189 is repugnant to Section 4, Article VII of the Constitution
only insofar as said Section totally disregarded the authority given to Congress by
the Constitution to proclaim the winning candidates for the positions of
president and vice-president.

Congress could not have allowed the COMELEC to usurp a power that
constitutionally belongs to it or, as aptly stated by petitioner, to encroach “on the
power of Congress to canvass the votes for president and vice-president and the power
to proclaim the winners for the said positions.” x x x x The canvassing of the votes
and the proclamation of the winning candidates for president and vice-president
for the entire nation must remain in the hands of Congress.

[NOTE: Section 18.5 of R.A. No. 9189 was declared UNCONSTITUTIONAL with respect
only to the power given to the Comelec to canvass the votes and proclaim the winning
candidates for President and Vice-President, which is lodged with Congress under
Section 4, Article VII of the Constitution. However, its consitutionality was UPHELD
insofar as the authority given to the COMELEC to proclaim the winning
candidates for the Senators and party-list representatives.]

(3) Yes.

By vesting itself with the powers to approve, review, amend, and revise the IRR
for The Overseas Absentee Voting Act of 2003, Congress went beyond the scope
of its constitutional authority. Congress trampled upon the constitutional
mandate of independence of the COMELEC.

The second sentence of the first paragraph of Section 19 stating that, “the
Implementing Rules and Regulations shall be submitted to the Joint Congressional
Oversight Committee created by virtue of this Act for prior approval," and the second
sentence of the second paragraph of Section 25 stating that, “it shall review, revise,
amend and approve the Implementing Rules and Regulations promulgated by the
Commission," whereby Congress, in both provisions, arrogates unto itself a
function not specifically vested by the Constitution, should be stricken out of
the subject statute for constitutional infirmity. Both provisions brazenly violate the
mandate on the independence of the COMELEC.

*Macalintal v. Presidential Electoral Tribunal, G.R. No. 191618, November 23, 2010.
(The Supreme Court as PET)

Facts:

Page 146 of 323


-Atty. Macalintal filed an undesignated petition questions challenging the
constitution of the Presidential Electoral Tribunal (PET) as an illegal and
unauthorized progeny of Section 4,2 Article VII of the Constitution.

-Arguments of Macalintal:
(1) The creation of a “separate tribunal” complemented by its own budget allocation, a
seal, a set of personnel and confidential employees, violates Section 4, Article VII of the
Constitution;

(2) The constitution of the PET, with the designation of the Members of the Court as
Chairman and Members thereof, contravenes Section 12, Article VIII of the
Constitution, which prohibits the designation of Members of the Supreme Court
and of other courts established by law to any agency performing quasi-judicial or
administrative functions.

Issues:
(1) Whether or not the creation of PET, with its own budget allocation, a seal, a set of
personnel and confidential employees, violates Section 4, Article VII of the
Constitution;

(2) Whether or not the PET performs judicial or quasi-judicial functions.

Ruling:
(1) No.

The PET is not a separate and distinct entity from the Supreme Court, albeit it
has functions peculiar only to the Tribunal. x x x x

The conferment of full authority to the Supreme Court, as a PET, is equivalent to the
full authority conferred upon the electoral tribunals of the Senate and the House of
Representatives, i.e., the Senate Electoral Tribunal (SET) and the House of
Representatives Electoral Tribunal (HRET), which we have affirmed on numerous
occasions. x x x x

The PET, as a constitutional body, is independent of the three departments of


government – Executive, Legislative, and Judiciary – but not separate therefrom.

(2) PET performs judicial functions.

The set up embodied in the Constitution and statutes characterizes the resolution of
electoral contests as essentially an exercise of judicial power. x x x x

With the explicit provision, the present Constitution has allocated to the Supreme
Court, in conjunction with latter’s exercise of judicial power inherent in all courts, the
task of deciding presidential and vice-presidential election contests, with full authority
in the exercise thereof. The power wielded by PET is a derivative of the plenary
judicial power allocated to courts of law, expressly provided in the Constitution. On
the whole, the Constitution draws a thin, but, nevertheless, distinct line between the
PET and the Supreme Court. x x x x

The Constitution which, in Section 17, Article VI, explicitly provides that three
Supreme Court Justices shall sit in the Senate and House Electoral Tribunals,
respectively, effectively exempts the Justices-Members thereof from the prohibition in
Section 12, Article VIII. In the same vein, it is the Constitution itself, in Section 4,
Article VII, which exempts the Members of the Court, constituting the PET, from the
same prohibition.

*Soliven v. Makasiar, G.R. No. 82585, November 14, 1988.


(President’s immunity from suit)

Page 147 of 323


Facts:
-Soliven was charged with libel for an article he had published, and President Aquino
was the complainant therein.

-Judge Makasiar, without personally examining the complainant, found probable


cause and issued a warrant of arrest against Soliven.

-Soliven sought relief from the Supreme Court via certiorari and prohibition.

-Arguments of Makasiar:
(1) The Constitution requires the judge to “personally” examine the complainant and
his witnesses in his determination of probable cause for the issuance of warrants of
arrest;

(2) Considering that the president is immune from suit, consequently, he or she
cannot institute a suit. If criminal proceedings ensue by virtue of the President's filing
of her complaint-affidavit, she may subsequently have to be a witness for the
prosecution, and hence, this would be an indirect way to defeat her privilege of
immunity from suit, as by testifying on the witness stand, she would lose her
supposedly undivided attention to her duties and she would be exposing herself to
possible contempt of court or perjury.

Issues:
(1) Whether or not the judge is required to personally examine the complainant and
his witnesses in his determination of probable cause for the issuance of warrants of
arrest;

(2) Whether or not the president can institute a suit despite his or her presidential
immunity.

Ruling:
(1) No.

In satisfying himself of the existence of probable cause for the issuance of a


warrant of arrest, the judge is not required to personally examine the
complainant and his witnesses.

Following established doctrine and procedure, he shall: (1) personally evaluate the
report and the supporting documents submitted by the fiscal regarding the
existence of probable cause and, on the basis thereof, issue a warrant of arrest; or (2)
if on the basis thereof he finds no probable cause, he may disregard the fiscal's report
and require the submission of supporting affidavits of witnesses to aid him in arriving
at a conclusion as to the existence of probable cause.

Sound policy dictates this procedure, otherwise judges would be unduly laden with
the preliminary examination and investigation of criminal complaints instead of
concentrating on hearing and deciding cases filed before their courts.

(2) Yes.

There is nothing in our laws that would prevent the President from waiving the
privilege.

Moreover, an accused in a criminal case (like Soliven) in which the President is


complainant cannot raise the presidential privilege as a defense to prevent the
case from proceeding against such accused. This privilege of immunity from suit
pertains to the President by virtue of the office and may be invoked only by the
holder of the office.

*Neri v. Senate Committees, G.R. No. 180643, March 25, 2008. [IMPORTANT!!!]

Page 148 of 323


(Executive Privilege)

Facts:
-In 2007, the DOTC entered into a contract with Zhong Xing Telecommunications
Equipment (ZTE) for the supply of equipment and services for the National Broadband
Network (NBN) Project in the amount of approximately P16 Billion Pesos. The Project
was to be financed by the People's Republic of China.

-Several committees were formed by the Senate to investigate, in aid of legislation,


the circumstances leading to the approval of the broadband contract as well as the
legal and economic justification of the NBN project. Neri, as the director general of
the NEDA, was among those summoned to appear and testify before the said
committees.

-On September 26, 2007, petitioner testified before respondent Committees for eleven
(11) hours. He disclosed that then Commission on Elections (COMELEC) Chairman
Benjamin Abalos offered him P200 Million in exchange for his approval of the NBN
Project. He further narrated that he informed President Arroyo about the bribery
attempt and that she instructed him not to accept the bribe.

-When probed further on what they discussed about the NBN Project, petitioner
refused to answer, invoking “executive privilege.” In particular, he refused to
answer the questions on: (a) whether or not President Arroyo followed up the NBN
Project; (b) whether or not she directed him to prioritize it; and (c) whether or
not she directed him to approve.

-During the next scheduled Senate hearings, Neri failed to attend, invoking executive
privilege. The Committees then issued a show cause order, and subsequently,
another order citing him in contempt ordering his arrest and detention at the Office
of the Senate Sergeant-At-Arms until such time that he would appear and give his
testimony.

Issue:
Whether or not Neri, under the circumstances, may legally invoke executive privilege.

Ruling:
Yes.

The claim of executive privilege is highly recognized in cases where the subject of
inquiry relates to a power textually committed by the Constitution to the President,
such as the area of military and foreign relations. Under our Constitution, the
President is the repository of the commander-in-chief, appointing, pardoning, and
diplomatic powers. Consistent with the doctrine of separation of powers, the
information relating to these powers may enjoy greater confidentiality than others.

The elements of presidential communications privilege are:

1) The protected communication must relate to a “quintessential and non-


delegable presidential power.”

2) The communication must be authored or “solicited and received” by a close


advisor of the President or the President himself. The judicial test is that an
advisor must be in “operational proximity” with the President.

3) The presidential communications privilege remains a qualified privilege that


may be overcome by a showing of adequate need, such that the information
sought “likely contains important evidence” and by the unavailability of the
information elsewhere by an appropriate investigating authority.

Using the above elements, we are convinced that, indeed, the communications elicited
by the three (3) questions are covered by the presidential communications privilege.

Page 149 of 323


First, the communications relate to a “quintessential and non-delegable power” of the
President, i.e. the power to enter into an executive agreement with other
countries. This authority of the President to enter into executive agreements without
the concurrence of the Legislature has traditionally been recognized in Philippine
jurisprudence. Second, the communications are “received” by a close advisor of the
President. Under the “operational proximity” test, petitioner can be considered a
close advisor, being a member of President Arroyo's cabinet. And third, there is
no adequate showing of a compelling need that would justify the limitation of
the privilege and of the unavailability of the information elsewhere by an appropriate
investigating authority. x x x x

Respondent Committees further contend that the grant of petitioner's claim of


executive privilege violates the constitutional provisions on the right of the people to
information on matters of public concern. We might have agreed with such
contention if petitioner did not appear before them at all. But petitioner made
himself available to them during the September 26 hearing, where he was
questioned for eleven (11) hours. Not only that, he expressly manifested his
willingness to answer more questions from the Senators, with the exception only
of those covered by his claim of executive privilege.

*Funa v. Agra, G.R. No. 191644, February 19, 2013.


(Prohibitions; Holding of dual/multiple offices)

Facts:
-On March 1, 2010, President Gloria M. Macapagal-Arroyo appointed Agra as the
Acting Secretary of Justice following the resignation of Secretary Agnes VST
Devanadera.

-On March 5, 2010, President Arroyo designated Agra as the Acting Solicitor General
in a concurrent capacity.

-Funa, as a taxpayer, a concerned citizen and a lawyer, commenced a Prohibition suit


to challenge the constitutionality of Agra’s concurrent appointments or designations.

-Contention of Agra: His concurrent designations as the Acting Secretary of Justice


and Acting Solicitor General were only in a temporary capacity, the only effect of
which was to confer additional duties to him. Thus, as the Acting Solicitor General
and Acting Secretary of Justice, Agra was not “holding” both offices in the strict
constitutional sense.

Issue:
Whether or not the designation of Agra as the Acting Secretary of Justice, concurrently
with his position of Acting Solicitor General, violates the constitutional prohibition
against dual or multiple offices for the Members of the Cabinet and their deputies and
assistants.

Ruling:
Yes.

The designation of Agra as Acting Secretary of Justice concurrently with his


position of Acting Solicitor General was unconstitutional and void for being in
violation of the constitutional prohibition under Section 13, Article VII of the
1987 Constitution.

Agra could not validly hold any other office or employment during his tenure as the
Acting Solicitor General, because the Constitution has not otherwise so provided.

It was of no moment that Agra’s designation was in an acting or temporary


capacity. The text of Section 13, supra, plainly indicates that the intent of the
Framers of the Constitution was to impose a stricter prohibition on the
President and the Members of his Cabinet in so far as holding other offices or

Page 150 of 323


employments in the Government or in government-owned or government
controlled-corporations was concerned.

In this regard, to hold an office means to possess or to occupy the office, or to be in


possession and administration of the office, which implies nothing less than the actual
discharge of the functions and duties of the office. Indeed, in the language of Section
13 itself, supra, the Constitution makes no reference to the nature of the
appointment or designation. The prohibition against dual or multiple offices
being held by one official must be construed as to apply to all appointments or
designations, whether permanent or temporary, for it is without question that the
avowed objective of Section 13, supra, is to prevent the concentration of powers in the
Executive Department officials, specifically the President, the Vice-President, the
Members of the Cabinet and their deputies and assistants.

[NOTE: Exception to this prohibition: It does not apply to posts occupied by


Executive officials in an ex-officio capacity—the ex-officio position being actually
and in legal contemplation part of the principal office. Examples: The Secretary of
Transportation and Communications is the ex-officio Chairman of the Boards of
Philippine Ports Authority as well as the Light Trail Authority; the Secretary of
Labor sits in an ex-officio capacity as member of the Board of Directors of the
Philippine Export Processing Zone (PEZA).]

*Almario v. Executive Secretary, G.R. No. 189028, July 16, 2013. [NOTE: The Carlos J.
Caparas controversy]
(Executive Power of the President)

-After several deliberations and screening processes for the conferment of the Order
of National Artists, the NCCA (National Commission for Culture and the Arts) Board of
Commissioners and the CCP (Cultural Center of the Philippines) Board of Trustees
arrived at a final list of four shortlisted individuals: Conde, Santos, Francisco and
Alcuaz.

-The said list of artists was then submitted to the President, and was subsequently
referred by the Office of the President to the Committee on Honors thereof.

-Meanwhile, the Office of the President allegedly received nominations from various
sectors, cultural groups and individuals strongly endorsing four other individuals:
respondents Alvarez, Caparas, Mañosa and Moreno.

-The Committee on Honors of the Office of the President thereafter submitted a


memorandum to then President Gloria Macapagal-Arroyo recommending the
conferment of the Order of National Artists on the four recommendees of the NCCA
and the CCP Boards, as well as on respondents Alvarez, Caparas, Mañosa and
Moreno.

-Acting on this recommendation, Proclamation No. 1823 declaring Conde a National


Artist was issued. Subsequently, Proclamation Nos. 1824 to 1829 were issued
declaring Francisco, Alcuaz and respondents Alvarez, Caparas, Mañosa and
Moreno, as National Artists.

-Convinced that, by law, it is the exclusive province of the NCCA Board of


Commissioners and the CCP Board of Trustees to select those who will be conferred
the Order of National Artists and to set the standard for entry into that select group,
Almario, et al. instituted a petition for prohibition, certiorari and injunction praying
that the Order of National Artists be conferred on Santos and that the conferment of
the Order of National Artists on respondents Alvarez, Caparas, Mañosa and Moreno be
enjoined and declared to have been rendered in grave abuse of discretion.

Issue:
Whether or not the President has the power to name awardees for National Artists
even in the absence of or against the recommendation of the NCCA and the CCP.

Page 151 of 323


Ruling:
No.

The respective powers of the CCP Board of Trustees and of the NCCA Board of
Commissioners with respect to the conferment of the Order of National Artists
are clear. They jointly administer the said award and, upon their
recommendation or advice, the President confers the Order of National Artists.

The President’s discretion in the conferment of the Order of National Artists


should be exercised in accordance with the duty to faithfully execute the
relevant laws. In this connection, the powers granted to the NCCA and the CCP
Boards in connection with the conferment of the Order of National Artists by executive
issuances were institutionalized by two laws, namely, Presidential Decree No. 208
dated June 7, 1973 and Republic Act No. 7356. In particular, Proclamation No. 1144
dated May 15, 1973 constituted the CCP Board as the National Artists Awards
Committee and tasked it to “administer the conferment of the category of National
Artist” upon deserving Filipino artists with the mandate to “draft the rules to guide its
deliberations in the choice of National Artists.”

We have held that an administrative regulation adopted pursuant to law has the force
and effect of law. Thus, the rules, guidelines and policies regarding the Order of
National Artists jointly issued by the CCP Board of Trustees and the NCCA pursuant
to their respective statutory mandates have the force and effect of law. Until set aside,
they are binding upon executive and administrative agencies, including the President
himself/herself as chief executor of laws.

The authority of the Committee on Honors is limited to determining whether the


nominations submitted by a particular awards committee, in this case, the joint
NCCA and CCP Boards, have been tainted by abuse of discretion, and whether the
nominees are in good standing. Should the nominations meet these two criteria, the
Committee on Honors shall make a recommendation to the President for conferment of
the Order of National Artists.

In view of the various stages of deliberation in the selection process and as a


consequence of his/her duty to faithfully enforce the relevant laws, the discretion of
the President in the matter of the Order of National Artists is confined to the
names submitted to him/her by the NCCA and the CCP Boards. This means that
the President could not have considered conferment of the Order of National
Artists on any person not considered and recommended by the NCCA and the
CCP Boards.

Applying this to the instant case, former President Arroyo could not have properly
considered respondents Guidote-Alvarez, Caparas, Mañosa and Moreno, as their
names were not recommended by the NCCA and the CCP Boards. Otherwise, not
only will the stringent selection and meticulous screening process be rendered futile,
the respective mandates of the NCCA and the CCP Board of Trustees under relevant
laws to administer the conferment of Order of National Artists, draft the rules and
regulations to guide its deliberations, formulate and implement policies and plans,
and undertake any and all necessary measures in that regard will also become
meaningless.

*Pimentel v. Ermita, G.R. No. 164978, October 13, 2005.


(President’s appointing power)

Facts:
-The Senate and the House of Representatives (“Congress”) commenced their regular
session on 26 July 2004.

Page 152 of 323


-On 23 August 2004, President Arroyo issued appointments to eight respondents
(Yap, Romulo, Gonzalez, et al.) as acting secretaries of their respective departments
(Departments of Justice, Agriculture, Foreign Affairs, etc.)

-The Commission on Appointments, composed of Senators and Representatives, was


constituted on 25 August 2004.

-On 8 September 2004, nine senators filed a petition for certiorari and prohibition
with the Supreme Court praying to declare unconstitutional the said appointments
issued by President Arroyo.

-Contentions of the senators: (1) President Arroyo should not have appointed
respondents as acting secretaries because “in case of a vacancy in the Office of a
Secretary, it is only an Undersecretary who can be designated as Acting
Secretary;” (2) While Congress is in session, there can be no appointments, whether
regular or acting, to a vacant position of an office needing confirmation by the
Commission on Appointments, without first having obtained the latter’s consent.

-In other words, the senators assert that the President cannot issue appointments
in an acting capacity to department secretaries while Congress is in session
because the law does not give the President such power. In contrast, respondents
insist that the President can issue such appointments because no law prohibits such
appointments.

Issue:
Whether or not the President’s appointment of department secretaries in an acting
capacity while Congress is in session is constitutional.

Ruling:
Yes.

Congress, through a law, cannot impose on the President the obligation to


appoint automatically the undersecretary as her temporary alter ego. An alter
ego, whether temporary or permanent, holds a position of great trust and
confidence. Congress, in the guise of prescribing qualifications to an office, cannot
impose on the President who her alter ego should be. x x x x

The office of a department secretary may become vacant while Congress is in


session. Since a department secretary is the alter ego of the President, the acting
appointee to the office must necessarily have the President’s confidence. Thus, by the
very nature of the office of a department secretary, the President must appoint
in an acting capacity a person of her choice even while Congress is in session.
That person may or may not be the permanent appointee, but practical reasons may
make it expedient that the acting appointee will also be the permanent appointee.

The essence of an appointment in an acting capacity is its temporary nature. It is


a stop-gap measure intended to fill an office for a limited time until the appointment of
a permanent occupant to the office. In case of vacancy in an office occupied by an alter
ego of the President, such as the office of a department secretary, the President must
necessarily appoint an alter ego of her choice as acting secretary before the permanent
appointee of her choice could assume office.

Finally, petitioners claim that the issuance of appointments in an acting capacity is


susceptible to abuse. Petitioners fail to consider that acting appointments cannot
exceed one year as expressly provided in Section 17(3), Chapter 5, Title I, Book
III of EO 292. The law has incorporated this safeguard to prevent abuses, like the use
of acting appointments as a way to circumvent confirmation by the Commission on
Appointments.

In distinguishing ad interim appointments from appointments in an acting capacity, a


noted textbook writer on constitutional law has observed:

Page 153 of 323


Ad-interim appointments must be distinguished from appointments in an acting
capacity. Both of them are effective upon acceptance. But ad-interim appointments
are extended only during a recess of Congress, whereas acting appointments
may be extended any time there is a vacancy. Moreover, ad-interim appointments
are submitted to the Commission on Appointments for confirmation or rejection;
acting appointments are not submitted to the Commission on Appointments.
Acting appointments are a way of temporarily filling important offices but, if abused,
they can also be a way of circumventing the need for confirmation by the Commission
on Appointments.

*De Rama v. Court of Appeals, G.R. No. 131136, February 28, 2001.
(Appointing power)

Facts:
-Upon his assumption to the position of Mayor of Pagbilao, Quezon, petitioner
Conrado L. de Raffia wrote a letter to the Civil Service Commission (or CSC), seeking
the recall of the appointments of 14 municipal employees on the ground that the
appointments of the said employees were “midnight” appointments of the former
mayor, Ma. Evelyn S. Abeja, done in violation of Article VII, Section 15 of the 1987
Constitution.

-The CSC later denied petitioner's request for the recall of the appointments of the
fourteen employees for lack of merit and for petitioner's failure to present evidence
that would warrant the revocation or recall of the said appointments.

-Petitioner then filed a “supplement” to the appeal and motion for reconsideration
where, for the very first time, he alleged that the appointments were obtained through
fraud, considering that: (1) the rules on screening of applicants based on adopted
criteria were not followed; (2) there was no proper posting of notice of vacancy; and (3)
the merit and fitness requirements set by the civil service rules were not observed.

-Nevertheless, the CSC overruled petitioner's assertions, holding that no new evidence
had been presented to warrant a reversal of its earlier resolution.

-Petitioner then filed a petition for review before the Court of Appeals, which in turn,
denied the same for lack of merit.

-Petitioner’s contentions: (1) the appointments must be recalled because they were
“midnight appointments,” (2) the CSC erred in its conclusions because it had
ignored the allegations and documents he presented in the “supplement” to his earlier
consolidated appeal and motion for reconsideration.

Issues:
(1) Whether or not “midnight appointments” made by an outgoing mayor may be
validly recalled on that ground;

(2) Whether or not the “supplemental” pleading constituted “new evidence” that should
have convinced the CSC to reverse its earlier ruling.

Ruling:
(1) No.

The constitutional prohibition on so-called “midnight appointments,”


specifically those made within two (2) months immediately prior to the next
presidential elections, applies only to the President or Acting President.

(2) No.

Supplemental pleadings must be with reasonable notice, and it is discretionary


upon the court or tribunal to allow the same or not. Thus, the CSC was under no

Page 154 of 323


obligation to admit the supplemental pleading, or even to consider the
averments therein.

Moreover, a supplemental pleading must state transactions, occurrences or


events which took place since the time the pleading sought to be supplemented
was filed. In the instant case, petitioner alleged fraud and irregularities that
supposedly occurred contemporaneous to the execution of the appointments.
They should have been raised at the very first opportunity. They are not new events
which petitioner could not have originally included as grounds for the recall of the
appointments.

When the petitioner brought the matter of recalling the appointments of the fourteen
(14) private respondents before the CSC, the only reason he cited to justify his action
was that these were “midnight appointments” that are forbidden under Article VII,
Section 15 of the Constitution. Petitioner certainly did not raise the issue of fraud
on the part of the outgoing mayor who made the appointments. Neither did he
allege that the said appointments were tainted by irregularities or anomalies that
breached laws and regulations governing appointments. It is rather too late for
petitioner to raise these issues for the first time on appeal. It is well-settled that
issues or questions of fact cannot be raised for the first time on appeal.
Consequently, the CSC and the Court of Appeals did not err in refusing to give
credence to the supplemental pleading.

[IMPORTANT NOTE:
After this case, the CSC issued CSC Resolution No. 010988 dated June 4, 2001,
which provides that, “All appointments x x x x issued AFTER the elections,
regardless of their dates of effectivity and/or date of receipt by the Commission,
including its Regional or Field Offices, of said appointments or the Report of
Personnel Actions (ROPA) as the case may be, shall be disapproved x x x x.”

The validity of said resolution has been upheld by the Supreme Court. See the case
of: Nazareno v. City of Dumaguete, G.R. No. 181559, October 2, 2009 under the
topic: Law on Local Government.

*Domingo v. Rayala, G.R. No. 155831, February 18, 2008. [IMPORTANT!]


(President’s power of removal)

Facts:
-Domingo, then Stenographic Reporter III at the NLRC, filed a complaint for sexual
harassment before the Secretary of the DOLE against Rayala, the Chairman of the
NLRC.

-Upon receipt of the complaint, the DOLE Secretary referred the complaint to the
Office of the President, Rayala being a presidential appointee.

-DOLE’s Committee on Decorum and Investigation later found Rayala guilty of the
offense charged and recommended the imposition of the minimum penalty provided
under AO 250, which is suspension for six (6) months and one (1) day.

-The following day, the DOLE Secretary submitted a copy of the Committee Report and
Recommendation to the OP. Acting on the said recommendation, the OP issued AO
119, dismissing Rayala from service for disgraceful conduct.

-Contention of Rayala: The OP has no authority to dismiss him considering that


under the pertinent Civil Service Rules, disgraceful and immoral conduct is
punishable by suspension for a period of six (6) months and one (1) day to one (1)
year, and not by dismissal.

-Contention of Domingo: The President has the prerogative to determine the


proper penalty to be imposed on an erring Presidential appointee. The President was

Page 155 of 323


well within his power when he fittingly used that prerogative in deciding to dismiss the
respondent from the service.

Issue:
Whether or not the President may validly dismiss Rayala as Chairman of the NLRC for
committing acts of sexual harassment.

Ruling:
No.

Under AO 250, the penalty for the first offense is suspension for six (6) months
and one (1) day to one (1) year, while the penalty for the second offense is
dismissal.

When the President found that Rayala was indeed guilty of disgraceful and immoral
conduct, the Chief Executive did not have unfettered discretion to impose a
penalty other than the penalty provided by law for such offense. As cited above,
the imposable penalty for the first offense of either the administrative offense of sexual
harassment or for disgraceful and immoral conduct is suspension of six (6) months
and one (1) day to one (1) year. Accordingly, it was error for the Office of the
President to impose upon Rayala the penalty of dismissal from the service, a
penalty which can only be imposed upon commission of a second offense.

*DENR v. DENR Region 12 Employees, G.R. No. 149724, August 19, 2003.
(The alter ego principle)

Facts:
-The Regional Executive Director of the DENR for Region 12 issued a Memorandum
directing the immediate transfer of the DENR XII Regional Offices from Cotabato City
to Koronadal, South Cotabato.

-This transfer was pursuant to the DENR Administrative Order No. 99-14, issued by
then DENR Cerilles mandating that the supervision of the Provinces of South Cotabato
and Sarangani shall be transferred from Region 11 to Region 12.

-Respondents, employees of the DENR Region XII, challenged the subject


Memorandum, contending that (1) only the President and not the DENR Secretary
has the authority to reorganize the DENR offices; (2) the transfer of the offices is
impractical and untimely.

Issues:
(1) Whether the DENR Secretary has the authority to reorganize the DENR;
(2) Whether DAO-99-14 and the Memorandum implementing the same were valid.

Ruling:
(1) Yes.

Applying the doctrine of qualified political agency, the power of the President to
reorganize the National Government may validly be delegated to his cabinet
members exercising control over a particular executive department. Thus, the
DENR Secretary can validly reorganize the DENR by ordering the transfer of the
DENR XII Regional Offices from Cotabato City to Koronadal, South Cotabato. The
exercise of this authority by the DENR Secretary, as an alter ego, is presumed to
be the acts of the President for the latter had not expressly repudiated the same.

(2) Yes. (See above.)

Additionally:
It may be true that the transfer of the offices may not be timely considering that: (1)
there are no buildings yet to house the regional offices in Koronadal, (2) the transfer
falls on the month of Ramadan, (3) the children of the affected employees are already

Page 156 of 323


enrolled in schools in Cotabato City, (4) the Regional Development Council was not
consulted, and (5) the Sangguniang Panglungsond, through a resolution, requested
the DENR Secretary to reconsider the orders. However, these issues are addressed to
the wisdom of the transfer rather than to its legality. It is basic in our form of
government that the judiciary cannot inquire into the wisdom or expediency of
the acts of the executive or the legislative department, for each department is
supreme and independent of the others, and each is devoid of authority not only to
encroach upon the powers or field of action assigned to any of the other department,
but also to inquire into or pass upon the advisability or wisdom of the acts performed,
measures taken or decisions made by the other departments.

*Dadole v. Commission on Audit, G.R. No. 125350, December 3, 2002.


(President’s general supervision over local government units)

Facts:
-In 1986, the RTC and MTC judges of Mandaue City started receiving monthly
allowances of P1,260 each through the yearly appropriation ordinance enacted by the
Sangguniang Panlungsod of the said city. In 1991, Mandaue City increased the
amount to P1,500 for each judge.

-In 1994, the Department of Budget and Management (DBM) issued the disputed Local
Budget Circular No. 55 (LBC 55), providing that additional allowances in the form of
honorarium at rates must not exceed P1,000.00 for cities.

-Acting on the DBM directive, the Mandaue City Auditor issued notices of
disallowance to petitioners who are RTC judges, in excess of the amount authorized
by LBC 55. Beginning October 1994, the additional monthly allowances of the
petitioner judges were reduced to P1,000 each.

-The RTC judges filed a protest.

-Contention of DBM: The constitutional and statutory authority of a city government


to provide allowances to judges stationed therein is not absolute. Congress may set
limitations on the exercise of autonomy. It is for the President, through the
DBM, to check whether these legislative limitations are being followed by the
local government units.

Issue:
Whether or not LBC 55 is valid.

Ruling:
No. It is void.

The President can only interfere in the affairs and activities of a local
government unit if he or she finds that the latter has acted contrary to law. This
is the scope of the President's supervisory powers over local government units. Hence,
the President or any of his or her alter egos cannot interfere in local affairs as
long as the concerned local government unit acts within the parameters of the
law and the Constitution. Any directive therefore by the President or any of his or
her alter egos seeking to alter the wisdom of a law-conforming judgment on local
affairs of a local government unit is a patent nullity because it violates the principle
of local autonomy and separation of powers of the executive and legislative
departments in governing municipal corporations.

Section 458, par. (a)(1)(xi), of RA 7160, the law that supposedly serves as the
legal basis of LBC 55, allows the grant of additional allowances to judges “when
the finances of the city government allow.” The said provision does not
authorize setting a definite maximum limit to the additional allowances granted
to judges. Thus, we need not belabor the point that the finances of a city
government may allow the grant of additional allowances higher than P1,000 if
the revenues of the said city government exceed its annual expenditures. Thus,

Page 157 of 323


to illustrate, a city government with locally generated annual revenues of P40 million
and expenditures of P35 million can afford to grant additional allowances of more than
P1,000 each to, say, ten judges inasmuch as the finances of the city can afford it.

Setting a uniform amount for the grant of additional allowances is an inappropriate


way of enforcing the criterion found in Section 458, par. (a)(1)(xi), of RA 7160. The
DBM over-stepped its power of supervision over local government units by
imposing a prohibition that did not correspond with the law it sought to
implement. In other words, the prohibitory nature of the circular had no legal basis.

*Pimentel v. Aguirre, G.R. No. 132988, July 19, 2000.


(President’s general supervision over local government units)

Facts:
-In 1997, in view of the economic difficulties brought about by peso depreciation,
then President Ramos issued A.O. 372, Section 4 of which requires local government
units to, among others, withhold 5% of their internal revenue allotment (IRA).

-Contention of the OSG: AO 372 was validly issued as the objective was to alleviate
the “economic difficulties brought about by the peso devaluation” and constituted
merely an exercise of the President's power of supervision over LGUs. The 5%
withholding is merely temporary in nature.

Issue:
Whether or not Section 4 of A.O. 372 is a valid exercise of the President's power of
general supervision over local governments.

Ruling:
No.

A basic feature of local fiscal autonomy is the automatic release of the shares of
LGUs in the national internal revenue. This is mandated by no less than the
Constitution. The Local Government Code specifies further that the release shall be
made directly to the LGU concerned within five (5) days after every quarter of
the year and “shall not be subject to any lien or holdback that may be imposed
by the national government for whatever purpose.” As a rule, the term “shall” is a
word of command that must be given a compulsory meaning. The provision is,
therefore, imperative.

Such withholding clearly contravenes the Constitution and the law. Although
temporary, it is equivalent to a holdback, which means "something held back or
withheld, often temporarily.” Hence, the “temporary” nature of the retention by
the national government does not matter. Any retention is prohibited.

Concededly, the President was well-intentioned in issuing his Order to withhold the
LGUs’ IRA, but the rule of law requires that even the best intentions must be carried
out within the parameters of the Constitution and the law. Verily, laudable purposes
must be carried out by legal methods.

*Ampatuan v. Puno, G.R. No. 190259, June 7, 2011. [IMPORTANT!!!]


(President’s general supervision over local government units; Calling Out Power)

Facts:
-The day after the gruesome Ampatuan massacre, then President Arroyo issued
Proclamation 1946, placing “the Provinces of Maguindanao and Sultan Kudarat and
the City of Cotabato under a state of emergency.”

-She directed the AFP and the PNP “to undertake such measures as may be allowed by
the Constitution and by law to prevent and suppress all incidents of lawless
violence” in the named places.

Page 158 of 323


-Three days later, President Arroyo also issued A.O. 273 delegating supervision of the
ARMM from the Office of the President to the DILG.

-The Ampatuans went to the SC via Rule 65 for petition for prohibition.

-Ampatuans’ contentions: (1) the proclamation and the orders empowered the DILG
Secretary to take over ARMM’s operations and seize the regional government’s powers,
in violation of the principle of local autonomy; (2) The deployment of troops and
the taking over of the ARMM constitutes an invalid exercise of the President’s
emergency powers; (3) the President had no factual basis for declaring a state of
emergency, especially in the Province of Sultan Kudarat and the City of Cotabato,
where no critical violent incidents occurred.

Issues:
(1) Whether or not Proclamation 1946 and AOs 273 and 273-A violate the principle of
local autonomy;
(2) Whether or not President Arroyo invalidly exercised emergency powers when she
called out the AFP and the PNP to prevent and suppress all incidents of lawless
violence in Maguindanao, Sultan Kudarat, and Cotabato City;
(3) Whether or not the President had factual bases for her actions.

Ruling:
(1) No.

The DILG Secretary did not take over control of the powers of the ARMM. After
law enforcement agents took respondent Governor of ARMM into custody for alleged
complicity in the Maguindanao massacre, the ARMM Vice-Governor, Ansaruddin
Adiong, assumed the vacated post pursuant to the rule on succession. In turn,
Acting Governor Adiong named the then Speaker of the ARMM Regional Assembly,
petitioner Sahali-Generale, as Acting ARMM Vice-Governor. In short, the DILG
Secretary did not take over the administration or operations of the ARMM.

(2) No.

Petitioners contend that the President unlawfully exercised emergency powers when
she ordered the deployment of AFP and PNP personnel in the places mentioned in the
proclamation. But such deployment is not by itself an exercise of emergency
powers as understood under Section 23 (2), Article VI of the Constitution, which
provides:

“SECTION 23. x x x (2) In times of war or other national emergency, the Congress may, by law,
authorize the President, for a limited period and subject to such restrictions as it may prescribe, to
exercise powers necessary and proper to carry out a declared national policy. Unless sooner withdrawn by
resolution of the Congress, such powers shall cease upon the next adjournment thereof.”

The President did not proclaim a national emergency, only a state of


emergency in the three places mentioned. And she did not act pursuant to any law
enacted by Congress that authorized her to exercise extraordinary powers. The calling
out of the armed forces to prevent or suppress lawless violence in such places is
a power that the Constitution directly vests in the President. She did not need a
congressional authority to exercise the same.

(3) Yes.

The President’s call on the armed forces to prevent or suppress lawless violence
springs from the power vested in her under Section 18, Article VII of the
Constitution, which provides.

“SECTION 18. The President shall be the Commander-in-Chief of all armed forces of the Philippines and
whenever it becomes necessary, he may call out such armed forces to prevent or suppress lawless
violence, invasion or rebellion. x x x”

Page 159 of 323


While it is true that the Court may inquire into the factual bases for the
President’s exercise of the above power, it would generally defer to her judgment
on the matter. As the Court acknowledged in Integrated Bar of the Philippines v. Hon.
Zamora, it is clearly to the President that the Constitution entrusts the
determination of the need for calling out the armed forces to prevent and
suppress lawless violence. Unless it is shown that such determination was attended
by grave abuse of discretion, the Court will accord respect to the President’s judgment.

If the petitioner fails, by way of proof, to support the assertion that the President acted
without factual basis, then this Court cannot undertake an independent
investigation beyond the pleadings. The factual necessity of calling out the
armed forces is not easily quantifiable and cannot be objectively established
since matters considered for satisfying the same is a combination of several
factors which are not always accessible to the courts. Besides the absence of
textual standards that the court may use to judge necessity, information necessary to
arrive at such judgment might also prove unmanageable for the courts. Certain
pertinent information might be difficult to verify, or wholly unavailable to the
courts. In many instances, the evidence upon which the President might decide that
there is a need to call out the armed forces may be of a nature not constituting
technical proof.

On the other hand, the President, as Commander-in-Chief has a vast intelligence


network to gather information, some of which may be classified as highly
confidential or affecting the security of the state. In the exercise of the power to
call, on-the-spot decisions may be imperatively necessary in emergency situations to
avert great loss of human lives and mass destruction of property. Indeed, the decision
to call out the military to prevent or suppress lawless violence must be done swiftly
and decisively if it were to have any effect at all. x x x x

Since petitioners are not able to demonstrate that the proclamation of state of
emergency in the subject places and the calling out of the armed forces to prevent or
suppress lawless violence there have clearly no factual bases, the Court must respect
the President’s actions.

*Province of Negros Occidental v. COA, G.R. No. 182574, September 28, 2010.
(President’s general supervision over local government units)

Facts:
-In 1994, the Sangguniang Panlalawigan of Negros Occidental passed Resolution No.
720-A allocating ₱4,000,000.00 of its retained earnings for the hospitalization and
health care insurance benefits of its 1,949 officials and employees of the province.

-In 1997, after a post-audit investigation, the Provincial Auditor issued Notice of
Suspension No. 97-001-1015 suspending the premium payment because of lack of
approval from the Office of the President (OP) as provided under A.O. 103.

-Sections 1 and 2 of A.O. 103 state:

“SECTION 1. All agencies of the National Government including government-owned and/or -


controlled corporations and government financial institutions, and local government units, are
hereby authorized to grant productivity incentive benefit in the maximum amount of TWO THOUSAND
PESOS (₱2,000.00) each to their permanent and full-time temporary and casual employees, including
contractual personnel with employment in the nature of a regular employee, who have rendered at least
one (1) year of service in the Government as of December 31, 1993.

SECTION 2. All heads of government offices/agencies, including government owned and/or


controlled corporations, as well as their respective governing boards are hereby enjoined and
prohibited from authorizing/granting Productivity Incentive Benefits or any and all forms of
allowances/benefits without prior approval and authorization via Administrative Order by the
Office of the President. Henceforth, anyone found violating any of the mandates in this Order, including
all officials/agency found to have taken part thereof, shall be accordingly and severely dealt with in
accordance with the applicable provisions of existing administrative and penal laws.”

Page 160 of 323


-The Provincial Auditor explained that the premium payment for health care benefits
violated R.A. 6758, otherwise known as the Salary Standardization Law. The COA
later affirmed the said disallowance and ordered the refund thereof.

-COA’s contention: Under A.O. 103, no government entity, including a local


government unit, is exempt from securing prior approval from the President granting
additional benefits to its personnel. This is in conformity with the policy of
standardization of compensation laid down in RA 6758.

-The Sangguniang Panlalawigan of Negros Occidental went to the SC via certiorari.

Issue:
Whether or not the COA committed grave abuse of discretion in affirming the
disallowance of ₱3,760,000.00 for premium paid for the hospitalization and health
care insurance benefits granted by the Province of Negros Occidental to its 1,949
officials and employees.

Ruling:
Yes.

The prohibition applies only to “government offices/agencies, including


government-owned and/or controlled corporations, as well as their respective
governing boards.” Nowhere is it indicated in Section 2 that the prohibition also
applies to LGUs. The requirement then of prior approval from the President
under A.O. 103 is applicable only to departments, bureaus, offices and
government-owned and controlled corporations under the Executive branch. In
other words, A.O. 103 must be observed by government offices under the President’s
control as mandated by Section 17, Article VII of the Constitution which states:

“Section 17. The President shall have control of all executive departments, bureaus and offices. He
shall ensure that the laws be faithfully executed.”

Being an LGU, petitioner is merely under the President’s general supervision


pursuant to Section 4, Article X of the Constitution. Since LGUs are subject only
to the power of general supervision of the President, the President’s authority is
limited to seeing to it that rules are followed and laws are faithfully executed.
The President may only point out that rules have not been followed but the President
cannot lay down the rules, neither does he have the discretion to modify or replace the
rules. Thus, the grant of additional compensation like hospitalization and health
care insurance benefits in the present case does not need the approval of the
President to be valid.

*Gudani v. Senga, G.R. No. 170165, August 15, 2006.


(The President as the Commander-in-Chief)

Facts:
-Gudani and Balutan are high-ranking officers of the AFP. At the time of the subject
incidents, both Gudani and Balutan were assigned to the PMA in Baguio City, the
former as the PMA Assistant Superintendent, and the latter as the Assistant
Commandant of Cadets.

-On 22 September 2005, Sen. Biazon invited several senior officers of the AFP to
appear at a public hearing before the Senate Committee on National Defense and
Security (Senate Committee) scheduled on 28 September 2005, regarding allegations
of massive cheating and the surfacing of copies of an audio excerpt purportedly of a
phone conversation between then President Arroyo and an official of the Comelec
widely reputed as then COMELEC Commissioner Virgilio Garcillano. (“Hello Garci”
controversy)

Page 161 of 323


-Gen. Gudani, Col. Balutan, and AFP Chief of Staff Lieutenant General Generoso
Senga (Gen. Senga) were among the several AFP officers who received a letter
invitation from Sen. Biazon.

-On the evening of 27 September 2005, at around 10:10 p.m., a message was
transmitted to the PMA Superintendent from the office of Gen. Senga, stating as
follows:

PER INSTRUCTION OF HER EXCELLENCY PGMA, NO AFP PERSONNEL SHALL


APPEAR BEFORE ANY CONGRESSIONAL OR SENATE HEARING WITHOUT HER
APPROVAL. INFORM BGEN FRANCISCO F GUDANI AFP AND LTC ALEXANDER
BALUTAN PA (GSC) ACCORDINGLY.

-The following day, Gen. Senga sent another letter to Sen. Biazon, this time informing
the senator that “no approval has been granted by the President to any AFP officer
to appear” before the hearing scheduled on that day. Nonetheless, both Gen. Gudani
and Col. Balutan were present as the hearing started, and they both testified as to
the conduct of the 2004 elections.

-On the very day of the hearing, 28 September 2005, President Arroyo issued E.O. 464
enjoining officials of the executive department including the military establishment
from appearing in any legislative inquiry without her approval. [NOTE: The
constitutionality of the said E.O was upheld by the SC.]

-An investigation ensued; during which Gen. Gudani was compulsorily retired from
military service, having reached the age of 56.

-After conducting investigation on Gudani and Balutan, the Office of the Provost
Marshal General (OPMG) recommended that petitioners be charged with Violation of
Article of War 65, on willfully disobeying a superior officer, in relation to Article of
War 97, on conduct prejudicial to the good order and military discipline.
Subsequently, petitioners were directed to appear in person before Col. Roa at the Pre-
Trial Investigation of the Charges for violation of Articles 6516 and 9717 of
Commonwealth Act No. 408, and to submit their counter-affidavits and affidavits of
witnesses at the Office of the Judge Advocate General.

-Petitioners then went to the SC via certiorari and prohibition, praying that: (1) the
order of President Arroyo coursed through Gen. Senga preventing petitioners from
testifying before Congress without her prior approval be declared unconstitutional;
(2) the charges stated in the charge sheets against petitioners be quashed; and (3)
Gen. Senga, Col. Galarpe, Col. Roa, and their successors-in-interest be permanently
enjoined from proceeding against petitioners, as a consequence of their having testified
before the Senate on 28 September 2005.

-Contentions of petitioners: (1) the directive from President Arroyo requiring her prior
approval before any AFP personnel appear before Congress is a “gag order,” which
violates the principle of separation of powers in government as it interferes with the
investigation of the Senate Committee conducted in aid of legislation; (2) there is no
law prohibiting them from testifying before the Senate, and in fact, they were
appearing in obeisance to the authority of Congress to conduct inquiries in aid of
legislation; (3) the Filipino people have every right to hear their testimonies; (4)
considering that Article 2, Title I of Commonwealth Act No. 408 defines the persons
subject to military law as “all officers and soldiers in the active service of the AFP,”
Gen. Gudani was no longer subject to military jurisdiction on account of his
compulsory retirement.

Issues:
(1) Whether or not the President may prevent a member of the armed forces from
testifying before a legislative inquiry;
(2) Whether or not the violation of the order of Gen. Senga, which emanated from the
President, could lead to any investigation for court-martial of petitioners;

Page 162 of 323


(3) Whether or not Gen. Gudani is still subject to military jurisdiction on account of
his compulsory retirement.

Ruling:
(1) Yes.

The President has constitutional authority to do so, by virtue of her power as


commander-in-chief, and that as a consequence a military officer who defies
such injunction is liable under military justice. x x x x The President’s prerogatives
as commander-in-chief are not hampered by the same limitations as in executive
privilege. x x x x A contrary rule unduly diminishes the prerogatives of the President
as commander-in-chief.

The ability of the President to require a military official to secure prior consent
before appearing before Congress pertains to a wholly different and independent
species of presidential authority—the commander-in-chief powers of the
President. By tradition and jurisprudence, the commander-in-chief powers of the
President are not encumbered by the same degree of restriction as that which
may attach to executive privilege or executive control. x x x x

The commander-in-chief provision in the Constitution is denominated as Section 18,


Article VII, which begins with the simple declaration that “the President shall be the
Commander-in-Chief of all armed forces of the Philippines x x x.” The commander-in-
chief clause vests on the President, as commander-in-chief, absolute authority
over the persons and actions of the members of the armed forces. Such authority
includes the ability of the President to restrict the travel, movement and speech
of military officers, activities which may otherwise be sanctioned under civilian
law. x x x x

It cannot be gainsaid that certain liberties of persons in the military service,


including the freedom of speech, may be circumscribed by rules of military
discipline. Thus, to a certain degree, individual rights may be curtailed, because the
effectiveness of the military in fulfilling its duties under the law depends to a large
extent on the maintenance of discipline within its ranks. Hence, lawful orders must
be followed without question and rules must be faithfully complied with,
irrespective of a soldier’s personal views on the matter. x x x x

Critical to military discipline is obeisance to the military chain of command.


Willful disobedience of a superior officer is punishable by court-martial under Article
65 of the Articles of War. "An individual soldier is not free to ignore the lawful orders
or duties assigned by his immediate superiors. For there would be an end of all
discipline if the seaman and marines on board a ship of war [or soldiers deployed in
the field], on a distant service, were permitted to act upon their own opinion of their
rights [or their opinion of the President’s intent], and to throw off the authority of the
commander whenever they supposed it to be unlawfully exercised." x x x x

The necessity of upholding the ability to restrain speech becomes even more
imperative if the soldier desires to speak freely on political matters. The
Constitution requires that "the armed forces shall be insulated from partisan politics,"
and that “no member of the military shall engage directly or indirectly in any partisan
political activity, except to vote.” Certainly, no constitutional provision or military
indoctrination will eliminate a soldier’s ability to form a personal political opinion, yet
it is vital that such opinions be kept out of the public eye.

(2) Yes.

As a general principle that AFP personnel of whatever rank are liable under military
law for violating a direct order of an officer superior in rank. (See above ruling.)

(3) Yes.

Page 163 of 323


In Abadilla v. Ramos, the SC declared that an officer whose name was dropped from
the roll of officers cannot be considered to be outside the jurisdiction of military
authorities when military justice proceedings were initiated against him before
the termination of his service. Once jurisdiction has been acquired over the
officer, it continues until his case is terminated. Thus, the Court held:

The military authorities had jurisdiction over the person of Colonel Abadilla at the
time of the alleged offenses. This jurisdiction having been vested in the military
authorities, it is retained up to the end of the proceedings against Colonel Abadilla.
Well-settled is the rule that jurisdiction once acquired is not lost upon the
instance of the parties but continues until the case is terminated.

*David v. Arroyo, G.R. No. 171396, May 3, 2006.


(The President as the Commander-in-Chief; Calling out power)

Facts:
-On February 24, 2006, as the nation celebrated the 20th Anniversary of the Edsa
People Power I, President Arroyo issued PP 1017 declaring a state of national
emergency, thus:

NOW, THEREFORE, I, Gloria Macapagal-Arroyo, President of the Republic of the Philippines and
Commander-in-Chief of the Armed Forces of the Philippines, by virtue of the powers vested upon me by
Section 18, Article 7 of the Philippine Constitution which states that: “The President. . . whenever it
becomes necessary, . . . may call out (the) armed forces to prevent or suppress. . .rebellion. . .,” and
in my capacity as their Commander-in-Chief, do hereby command the Armed Forces of the Philippines, to
maintain law and order throughout the Philippines, prevent or suppress all forms of lawless violence
as well as any act of insurrection or rebellion and to enforce obedience to all the laws and to all
decrees, orders and regulations promulgated by me personally or upon my direction; and as
provided in Section 17, Article 12 of the Constitution do hereby declare a State of National
Emergency.”

-On the same day, the President issued G. O. No. 5 implementing PP 1017, thus:

“NOW, THEREFORE, I GLORIA MACAPAGAL-ARROYO, by virtue of the powers vested in me under the
Constitution as President of the Republic of the Philippines, and Commander-in-Chief of the Republic of
the Philippines, and pursuant to Proclamation No. 1017 dated February 24, 2006, do hereby call upon
the Armed Forces of the Philippines (AFP) and the Philippine National Police (PNP), to prevent and
suppress acts of terrorism and lawless violence in the country;

I hereby direct the Chief of Staff of the AFP and the Chief of the PNP, as well as the officers and men of the
AFP and PNP, to immediately carry out the necessary and appropriate actions and measures to
suppress and prevent acts of terrorism and lawless violence.”

-During the dispersal of the rallyists along EDSA, police arrested (without warrant)
petitioner Randolf S. David, a professor at the University of the Philippines and
newspaper columnist. Also arrested was his companion, Ronald Llamas, president of
party-list Akbayan.

-At around 12:20 in the early morning of February 25, 2006, operatives of the
Criminal Investigation and Detection Group (CIDG) of the PNP, on the basis of PP 1017
and G.O. No. 5, raided the Daily Tribune offices in Manila. The raiding team
confiscated news stories by reporters, documents, pictures, and mock-ups of the
Saturday issue. Policemen from Camp Crame in Quezon City were stationed inside the
editorial and business offices of the newspaper; while policemen from the Manila
Police District were stationed outside the building.

- Section 17, Article XII reads:

Sec. 17. In times of national emergency, when the public interest so requires, the State may, during the
emergency and under reasonable terms prescribed by it, temporarily take over or direct the operation of
any privately-owned public utility or business affected with public interest.

-Also on February 25, 2006, the police arrested Congressman Crispin Beltran,
representing the Anakpawis Party and Chairman of Kilusang Mayo Uno (KMU), while
leaving his farmhouse in Bulacan.

Page 164 of 323


-Retired Major General Ramon Montaño, former head of the Philippine Constabulary,
was arrested while with his wife and golfmates at the Orchard Golf and Country Club
in Dasmariñas, Cavite.

-Petitioners filed this petition in view of these events.

-On March 3, 2006, exactly one week after the declaration of a state of national
emergency, Arroyo lifted PP 1017.

-Contentions of petitioners: (1) it is a subterfuge to avoid the constitutional


requirements for the imposition of martial law; (2) the CIDG’s act of raiding the Daily
Tribune offices is a clear case of “censorship” or “prior restraint;” (3) President Arroyo
gravely abused her discretion in calling out the armed forces without clear and
verifiable factual basis of the possibility of lawless violence and a showing that there is
necessity to do so;” (4) PP 1017 is unconstitutional for being violative of the freedom of
expression, including its cognate rights such as freedom of the press and the right to
access to information on matters of public concern, all guaranteed under Article III,
Section 4 of the 1987 Constitution.

Issues:
(1) Whether or not PP 1017 and GO5 are unconstitutional; (Related: Can this Court
adjudge as unconstitutional PP 1017 and G.O. No 5 on the basis of these illegal acts?
In general, does the illegal implementation of a law render it unconstitutional?)
(2) Whether or not the President can validly take over any privately-owned public
utility or business affected with public interest during national emergency;
(3) Whether or not the Supreme Court can review the factual bases of PP 1017 and
GO5;
(4) Whether or not President Arroyo can enforce obedience to all decrees and laws
through the military;
(5) Whether or not President Arroyo can issue decrees pursuant to PP 1017.

Ruling:
(1) No. They are valid.

The only criterion for the exercise of the calling-out power is that “whenever it
becomes necessary,” the President may call the armed forces “to prevent or suppress
lawless violence, invasion or rebellion.” x x x x

Under the calling-out power, the President may summon the armed forces to aid
him in suppressing lawless violence, invasion and rebellion. This involves ordinary
police action. But every act that goes beyond the President’s calling-out power is
considered illegal or ultra vires. x x x x

(As to the abuse of the same:)

Settled is the rule that courts are not at liberty to declare statutes invalid
although they may be abused and misabused and may afford an opportunity for
abuse in the manner of application. The validity of a statute or ordinance is to
be determined from its general purpose and its efficiency to accomplish the end
desired, not from its effects in a particular case.

PP 1017 is merely an invocation of the President’s calling-out power. Its general


purpose is to command the AFP to suppress all forms of lawless violence, invasion or
rebellion. It had accomplished the end desired which prompted President Arroyo to
issue PP 1021. But there is nothing in PP 1017 allowing the police, expressly or
impliedly, to conduct illegal arrest, search or violate the citizens’ constitutional
rights.

[NOTE:

Page 165 of 323


The operative portion of PP 1017 may be divided into three important provisions, thus:

First provision: (Calling-out Power)


“by virtue of the power vested upon me by Section 18, Article VII … do hereby
command the Armed Forces of the Philippines, to maintain law and order throughout
the Philippines, prevent or suppress all forms of lawless violence as well any act of
insurrection or rebellion”

Second provision: (“Take Care” Power)


“and to enforce obedience to all the laws and to all decrees, orders and regulations
promulgated by me personally or upon my direction;”

Third provision: (Power to Take Over)


“as provided in Section 17, Article XII of the Constitution do hereby declare a State of
National Emergency.”

(2) No.

Section 17, Article XII must be understood as an aspect of the emergency powers
clause. The taking over of private business affected with public interest is just
another facet of the emergency powers generally reposed upon Congress. Thus,
when Section 17 states that the “the State may, during the emergency and under
reasonable terms prescribed by it, temporarily take over or direct the operation
of any privately owned public utility or business affected with public interest,” it
refers to Congress, not the President. Now, whether or not the President may
exercise such power is dependent on whether Congress may delegate it to him
pursuant to a law prescribing the reasonable terms thereof.

While the President alone can declare a state of national emergency, however,
without legislation, he has no power to take over privately-owned public utility
or business affected with public interest.

(3) Yes.

However, judicial inquiry can go no further than to satisfy the Court not that the
President’s decision is correct, but that the President did not act arbitrarily.
Thus, the standard laid down is not correctness, but arbitrariness.

It is incumbent upon the petitioner/s to show that the President’s decision is


totally bereft of factual basis and that if he fails, by way of proof, to support his
assertion, then the Court cannot undertake an independent investigation beyond
the pleadings.

Petitioners failed to show that President Arroyo’s exercise of the calling-out power, by
issuing PP 1017, is totally bereft of factual basis. A reading of the Solicitor General’s
Consolidated Comment and Memorandum shows a detailed narration of the events
leading to the issuance of PP 1017, with supporting reports forming part of the
records. Mentioned are the escape of the Magdalo Group, their audacious threat of
the Magdalo D-Day, the defections in the military, particularly in the Philippine
Marines, and the reproving statements from the communist leaders. There was
also the Minutes of the Intelligence Report and Security Group of the Philippine Army
showing the growing alliance between the NPA and the military. Petitioners
presented nothing to refute such events. Thus, absent any contrary allegations, the
Court is convinced that the President was justified in issuing PP 1017 calling for
military aid.

(4) No.

Arroyo cannot call the military to enforce or implement certain laws, such as
customs laws, laws governing family and property relations, laws on obligations

Page 166 of 323


and contracts and the like. She can only order the military, under PP 1017, to
enforce laws pertinent to its duty to suppress lawless violence.

(5) No.

PP 1017 states in part: “to enforce obedience to all the laws and decrees x x x
promulgated by me personally or upon my direction.”

This Court rules that the assailed PP 1017 is unconstitutional insofar as it grants
President Arroyo the authority to promulgate “decrees.” Legislative power is
peculiarly within the province of the Legislature. Section 1, Article VI categorically
states that, “the legislative power shall be vested in the Congress of the Philippines
which shall consist of a Senate and a House of Representatives.” To be sure, neither
Martial Law nor a state of rebellion nor a state of emergency can justify
President Arroyo’s exercise of legislative power by issuing decrees.

[NOTE:
The warrantless arrests and searches were all declared illegal and
unconstitutional.

*Sanlakas v. Reyes, G.R. No. 159085, February 3, 2004.


(The President as the Commander-in-Chief; Calling out power)

Facts:
-In the wee hours of July 27, 2003, some 300 junior officers and enlisted men of the
AFP stormed into the Oakwood Premiere apartments in Makati City in the wee hours
of July 27, 2003. The soldiers demanded, among other things, the resignation of the
President, the Secretary of Defense and the Chief of the PNP.

-In the wake of the Oakwood occupation, the President issued later in the day
Proclamation No. 427 and General Order No. 4, both declaring "a state of rebellion"
and calling out the Armed Forces to suppress the rebellion.

-By the evening of July 27, 2003, the Oakwood occupation had ended. After hours-
long negotiations, the soldiers agreed to return to barracks. The President, however,
did not immediately lift the declaration of a state of rebellion and did so only on
August 1, 2003, through Proclamation No. 435.

-In the interim, several petitions were filed before this Court challenging the validity of
Proclamation No. 427 and General Order No. 4.

-Contentions of petitioners: (1) because of the cessation of the Oakwood occupation,


there exists no sufficient factual basis for the proclamation by the President of a state
of rebellion for an indefinite period; (2) the declaration of a state of rebellion is a
“superfluity,” and is actually an exercise of emergency powers; (3) the declaration of a
state of rebellion opens the door to the unconstitutional implementation of warrantless
arrests for the crime of rebellion.

Issue:
Whether or not the calling out power the Constitution requires the President to make a
declaration of a state of rebellion.

Ruling:
No.

Section 18, Article VII provides:

Sec. 18. The President shall be the Commander-in-Chief of all armed forces of the Philippines and
whenever it becomes necessary, he may call out such armed forces to prevent or suppress lawless
violence, invasion or rebellion. In case of invasion or rebellion, when the public safety requires it, he may,
for a period not exceeding sixty days, suspend the privilege of the writ of habeas corpus or place the
Philippines or any part thereof under martial law. Within forty-eight hours from the proclamation of

Page 167 of 323


martial law or the suspension of the writ of habeas corpus, the President shall submit a report in person
or in writing to the Congress. The Congress, voting jointly, by a vote of at least a majority of all its
Members in regular or special session, may revoke such proclamation or suspension, which revocation
shall not be set aside by the President. Upon the initiative of the President, the Congress may, in the
same manner, extend such proclamation or suspension for a period to be determined by the Congress, if
the invasion or rebellion shall persist and public safety requires it.

The Congress, if not in session, shall, within twenty-four hours following such proclamation or
suspension, convene in accordance with its rules without need of a call.

The Supreme Court may review, in an appropriate proceeding filed by any citizen, the sufficiency of the
factual basis for the proclamation of martial law or the suspension of the privilege of the writ of habeas
corpus or the extension thereof, and must promulgate its decision thereon within thirty days from its
filing.

A state of martial law does not suspend the operation of the Constitution, nor supplant the functioning of
the civil courts or legislative assemblies, nor authorize the conferment of the jurisdiction on military
courts and agencies over civilians where civil courts are able to function, nor automatically suspend the
privilege of the writ.

The suspension of the privilege of the writ shall apply only to persons judicially charged for rebellion or
offenses inherent in or directly connected with invasion.

During the suspension of the privilege of the writ, any person thus arrested or detained shall be judicially
charged within three days, otherwise he shall be released.

The above provision grants the President, as Commander-in-Chief, a “sequence” of


“graduated powers.” From the most to the least benign, these are: the calling out
power, the power to suspend the privilege of the writ of habeas corpus, and the
power to declare martial law. In the exercise of the latter two powers, the
Constitution requires the concurrence of two conditions, namely, an actual
invasion or rebellion, and that public safety requires the exercise of such power.
However, as we observed in Integrated Bar of the Philippines v. Zamora, these
conditions are not required in the exercise of the calling out power. The only
criterion is that, “whenever it becomes necessary,” the President may call the
armed forces “to prevent or suppress lawless violence, invasion or rebellion.”

In calling out the armed forces, a declaration of a state of rebellion is an utter


superfluity. At most, it only gives notice to the nation that such a state exists
and that the armed forces may be called to prevent or suppress it. Perhaps the
declaration may wreak emotional effects upon the perceived enemies of the State, even
on the entire nation. But this Court's mandate is to probe only into the legal
consequences of the declaration. This Court finds that such a declaration is devoid
of any legal significance. For all legal intents, the declaration is deemed not written.

The mere declaration of a state of rebellion cannot diminish or violate


constitutionally protected rights. x x x x In quelling or suppressing the rebellion,
the authorities may only resort to warrantless arrests of persons suspected of
rebellion, as provided under Section 5, Rule 113 of the Rules of Court, if the
circumstances so warrant. x x x x A person may be subjected to a warrantless arrest
for the crime of rebellion whether or not the President has declared a state of rebellion,
so long as the requisites for a valid warrantless arrest are present. x x x x

The President, in declaring a state of rebellion and in calling out the armed
forces, was merely exercising a wedding of her Chief Executive and Commander-
in-Chief powers. These are purely executive powers, vested on the President by
Sections 1 and 18, Article VII, as opposed to the delegated legislative powers
contemplated by Section 23 (2), Article VI.

*Kulayan v. Tan, G.R. No. 187298, July 03, 2012.


(The President as the Commander-in-Chief; Calling out power)

Facts:
-On 15 January 2009, three members from the International Committee of the Red
Cross (ICRC) were kidnapped in the vicinity of the Provincial Capitol in Patikul, Sulu

Page 168 of 323


by three armed men who were later confirmed to be members of the Abu Sayyaf
Group (ASG).

-On 31 March 2009, Governor Tan issued Proclamation No. 1, Series of 2009
(Proclamation 1-09), declaring a state of emergency in the province of Sulu. It cited
the kidnapping incident as a ground for the said declaration, describing it as a
terrorist act pursuant to the Human Security Act (R.A. 9372). It also invoked Section
465 of the Local Government Code of 1991 (R.A. 7160), which bestows on the
Provincial Governor the power to carry out emergency measures during man-made
and natural disasters and calamities, and to call upon the appropriate national law
enforcement agencies to suppress disorder and lawless violence.

-In the same Proclamation, respondent Tan called upon the PNP and the CEF to
set up checkpoints and chokepoints, conduct general search and seizures
including arrests, and other actions necessary to ensure public safety.

Issues:
(1) Whether or not a provincial governor is vested with calling-out powers;
(2) Whether or not Section 465, in relation to Section 16, of the Local Government
Code authorizes the respondent governor to declare a state of emergency, and exercise
the powers enumerated under Proclamation 1-09, specifically the conduct of general
searches and seizures.

Ruling:
(1) No.

Only the President is vested with calling-out powers, as the commander-in-chief


of the Republic. There is one repository of executive powers, and that is the
President of the Republic. This means that when Section 1, Article VII of the
Constitution speaks of executive power, it is granted to the President and no one
else.

Corollarily, it is only the President, as Executive, who is authorized to exercise


emergency powers as provided under Section 23, Article VI, of the Constitution, as
well as what became known as the calling-out powers under Section 7, Article VII
thereof. x x x x

Respondent provincial governor is not endowed with the power to call upon the
armed forces at his own bidding. In issuing the assailed proclamation, Governor
Tan exceeded his authority when he declared a state of emergency and called
upon the Armed Forces, the police, and his own Civilian Emergency Force. The
calling-out powers contemplated under the Constitution is exclusive to the President.
An exercise by another official, even if he is the local chief executive, is ultra vires,
and may not be justified by the invocation of Section 465 of the Local
Government Code. x x x x

(2) No.

Section 465 of the Local Government Code cannot be invoked to justify the powers
enumerated under Proclamation 1-09. There is nothing in the Local Government Code
which justifies the acts sanctioned under the said Proclamation. Not even Section 465
of the said Code.

*Fortun v. Arroyo, G.R. No. 190293, March 20, 2012.


(The President as the Commander-in-Chief; Martial Law)

Facts:
-In response to the Ampatuan Massacre, President Arroyo issued Presidential
Proclamation 1946, declaring a state of emergency in Maguindanao, Sultan
Kudarat, and Cotabato City to prevent and suppress similar lawless violence in
Central Mindanao.

Page 169 of 323


-Believing that she needed greater authority to put order in Maguindanao and secure
it from large groups of persons that have taken up arms against the constituted
authorities in the province, President Arroyo issued Presidential Proclamation 1959
declaring martial law and suspending the privilege of the writ of habeas corpus in
that province except for identified areas of the MILF.

-After eight days, and before the Congress could review the validity of the President’s
action, Arroyo issued Presidential Proclamation 1963, lifting martial law and restoring
the privilege of the writ of habeas corpus in Maguindanao.

-Petitioners went to the SC to challenge the constitutionality of President Arroyo’s


Proclamation 1959 affecting Maguindanao.

Issue:
Whether or not, given the circumstances, the SC should review the constitutionality of
President Arroyo’s Proclamation 1959 affecting Maguindanao.

Ruling:
No.

Although the Constitution reserves to the Supreme Court the power to review the
sufficiency of the factual basis of the proclamation or suspension in a proper suit, it is
implicit that the Court must allow Congress to exercise its own review powers,
which is automatic rather than initiated. Only when Congress defaults in its
express duty to defend the Constitution through such review should the
Supreme Court step in as its final rampart. The constitutional validity of the
President’s proclamation of martial law or suspension of the writ of habeas
corpus is first a political question in the hands of Congress before it becomes a
justiciable one in the hands of the Court.

Here, President Arroyo withdrew Proclamation 1959 before the joint houses of
Congress, which had in fact convened, could act on the same. Consequently, the
petitions in these cases have become moot and the Court has nothing to review.
The lifting of martial law and restoration of the privilege of the writ of habeas
corpus in Maguindanao was a supervening event that obliterated any justiciable
controversy. x x x x

The problem in this case is that the President aborted the proclamation of martial law
and the suspension of the privilege of the writ of habeas corpus in Maguindanao in
just eight days. In a real sense, the proclamation and the suspension never took off.
The Congress itself adjourned without touching the matter, it having become
moot and academic.

Of course, the Court has in exceptional cases passed upon issues that ordinarily
would have been regarded as moot. But the present cases do not present sufficient
basis for the exercise of the power of judicial review. The proclamation of martial
law and the suspension of the privilege of the writ of habeas corpus in this case,
unlike similar Presidential acts in the late 60s and early 70s, appear more like saber-
rattling than an actual deployment and arbitrary use of political power.

*In Re: Petition for Habeas Corpus of Wilfredo Sumulong, G.R. No. 122338, December 29,
1995.
(President’s pardoning power)

-Torres was convicted for two counts of estafa by the RTC some time before 1979.
These convictions were affirmed by the Court of Appeals. The maximum sentence
would expire on November 2, 2000.

-On April 18, 1979, a conditional pardon was granted to Torres by the President of
the Philippines on condition that Torres would not again violate any of the penal

Page 170 of 323


laws of the Philippines. Petitioner accepted the conditional pardon and was
consequently released from confinement.

-On May 21, 1986, the Board of Pardons and Parole resolved to recommend to the
President the cancellation of the conditional pardon granted to Torres because
Torres had been charged with 20 counts of estafa before the RTC of Quezon City,
and convicted of sedition by, same court.

-On September 8, 1986, the President cancelled the conditional pardon of Torres. An
Order of Arrest and Recommitment was issued against petitioner. Torres was
accordingly arrested and confined in Muntinlupa to serve the unexpired portion of his
sentence.

-His wife and daughters filed a petition for habeas corpus.

-Contentions of Torres: (1) the exercise of the President's prerogative to determine the
occurrence of a breach of a condition of a pardon in violation of pardonee's right to
due process and the constitutional presumption of innocence, constitutes a grave
abuse of discretion amounting to lack or excess of jurisdiction; (2) he was acquitted in
two of the three criminal cases filed against him subsequent to his conditional pardon,
and that the third case remains pending for thirteen (13) years in apparent violation
of his right to a speedy trial.

Issue:
Whether or not the writ of habeas corpus must be granted.

Ruling:
No.

A conditional pardon is in the nature of a contract between the sovereign power or the
Chief Executive and the convicted criminal to the effect that the former will release the
latter subject to the condition that if he does not comply with the terms of the pardon,
he will be recommitted to prison to serve the unexpired portion of the sentence or an
additional one. By the pardonee's consent to the terms stipulated in this
contract, the pardonee has thereby placed himself under the supervision of the
Chief Executive or his delegate who is duty-bound to see to it that the pardonee
complies with the terms and conditions of the pardon. Under Section 64 (i) of the
Revised Administrative Code, the Chief Executive is authorized to order “the arrest and
re-incarceration of any such person who, in his judgment, shall fail to comply with the
condition, or conditions of his pardon, parole, or suspension of sentence.”

It is now a well-entrenched rule in this jurisdiction that this exercise of


presidential judgment is beyond judicial scrutiny. The determination of the
violation of the conditional pardon rests exclusively in the sound judgment of
the Chief Executive, and the pardonee, having consented to place his liberty on
conditional pardon upon the judgment of the power that has granted it, cannot
invoke the aid of the courts, however erroneous the findings may be upon which
his recommitment was ordered.

It matters not that in the case of Torres, he has allegedly been acquitted in two of the
three criminal cases filed against him subsequent to his conditional pardon, and that
the third case remains pending for thirteen (13) years in apparent violation of his right
to a speedy trial.

Habeas corpus lies only where the restraint of a person's liberty has been
judicially adjudged as illegal or unlawful. In the instant petition, the incarceration of
Torres remains legal considering that, were it not for the grant of conditional pardon
which had been revoked because of a breach thereof, the determination of which is
beyond judicial scrutiny, he would have served his final sentence for his first
conviction until November 2, 2000.

Page 171 of 323


Courts have no authority to interfere with the grant by the President of a pardon
to a convicted criminal. It has been our fortified ruling that a final judicial
pronouncement as to the guilt of a pardonee is not a requirement for the President to
determine whether or not there has been a breach of the terms of a conditional
pardon. There is likewise nil a basis for the courts to effectuate the
reinstatement of a conditional pardon revoked by the President in the exercise
of powers undisputedly solely and absolutely lodged in his office.

*Vera v. People, G.R. No. L-18184, January 31, 1963.


(President’s pardoning power)

-Petitioners were charged with the complex crime of kidnapping with murder of
Amadeo Lozanes, alias Azarcon (who was believed to have aided the enemy during the
Japanese occupation.

-Upon petitioners’ motion, invoking the benefits of Amnesty Proclamation of the


President, series of 1946, the case was referred to the Eighth Guerrilla Amnesty
Commission, which actually tried it.

-During the hearing, none of the petitioner-defendants admitted having committed


the crime charged. In fact, Gaudencio Vera, the only defendant who took the witness
stand, instead of admitting the killing of the deceased Lozanes categorically denied it.

-Hence, the Commission held that it could not take cognizance of the case on the
ground of such non-admission of guilt. Consequently, it ordered that the case be
remanded to the court of origin for trial.

-Petitioners sought remedy from CA and then SC.

Issue:
Whether or not it is necessary to admit guilt in order to entitle a person to the benefits
of amnesty.

Ruling:
Yes.

Amnesty presupposes the commission of a crime, and when an accused


maintains that he has not committed a crime, he cannot have any use for
amnesty. Where an amnesty proclamation imposes certain conditions, as in this
case, it is incumbent upon the accused to prove the existence of such
conditions. The invocation of amnesty is in the nature of a plea of confession and
avoidance, which means that the pleader admits the allegations against him but
disclaims liability therefor on account of intervening facts which, if proved, would
being the crime charged within the scope of the amnesty proclamation.

*Bayan v. Zamora, G.R. No. 138570, October 10, 2000. [EDIT!!!]


(President’s Diplomatic Power; Treaties)

Facts:
-On March 14, 1947, the Philippines and the United States of America forged a
Military Bases Agreement which formalized, among others, the use of installations in
the Philippine territory by United States military personnel. To further strengthen their
defense and security relationship, the Philippines and the United States entered into a
Mutual Defense Treaty on August 30, 1951. Under the treaty, the parties agreed to
respond to any external armed attack on their territory, armed forces, public vessels,
and aircraft.

-In view of the impending expiration of the RP-US Military Bases Agreement in 1991,
the Philippines and the United States negotiated for a possible extension of the
military bases agreement. On September 16, 1991, the Philippine Senate rejected the

Page 172 of 323


proposed RP-US Treaty of Friendship, Cooperation and Security which, in effect,
would have extended the presence of US military bases in the Philippines.

-With the expiration of the RP-US Military Bases Agreement, the periodic military
exercises conducted between the two countries were held in abeyance.
Notwithstanding, the defense and security relationship between the Philippines and
the United States of America continued pursuant to the Mutual Defense Treaty.

-xxxxx

Contention of petitioners: the phrase “recognized as a treaty,” embodied in section 25,


Article XVIII, means that the VFA should have the advice and consent of the United
States Senate pursuant to its own constitutional process, and that it should not be
considered merely an executive agreement by the United States.

Contention of respondents: the letter of United States Ambassador Hubbard stating


that the VFA is binding on the United States Government is conclusive.

Issues:
(1) Whether or not the VFA is governed by the provisions of Section 21, Article VII or of
Section 25, Article XVIII of the Constitution;
(2) Whether or not the VFA constitute an abdication of Philippine sovereignty (as to
whether Philippine courts are deprived of their jurisdiction to hear and try offenses
committed by US military personnel);
(3) Whether or not the VFA violates the equal protection clause under Section 1,
Article III of the Constitution;
(4) Whether or not the VFA violates the Prohibition against nuclear weapons under
Article II, Section 8;
(5) Whether or not the VFA violates Section 28 (4), Article VI of the Constitution in
granting the exemption from taxes and duties for the equipment, materials supplies
and other properties imported into or acquired in the Philippines by, or on behalf, of
the US Armed Forces;

Ruling:

whether under Section 21, Article VII or Section 25, Article XVIII, the fundamental law
is crystalline that the concurrence of the Senate is mandatory to comply with the strict
constitutional requirements.

Section 21, Article VII of the Constitution reads:

“No treaty or international agreement shall be valid and effective unless concurred in
by at least two-thirds of all the Members of the Senate.”

Section 25, Article XVIII, provides:

“After the expiration in 1991 of the Agreement between the Republic of the Philippines
and the United States of America concerning Military Bases, foreign military bases,
troops, or facilities shall not be allowed in the Philippines except under a treaty duly
concurred in by the senate and, when the Congress so requires, ratified by a majority
of the votes cast by the people in a national referendum held for that purpose, and
recognized as a treaty by the other contracting State.”

=======

whether under Section 21, Article VII or Section 25, Article XVIII, the fundamental law
is crystalline that the concurrence of the Senate is mandatory to comply with the strict
constitutional requirements.

=======

Page 173 of 323


Undoubtedly, Section 25, Article XVIII, which specifically deals with treaties involving
foreign military bases, troops, or facilities, should apply in the instant case. To a
certain extent and in a limited sense, however, the provisions of section 21, Article VII
will find applicability with regard to the issue and for the sole purpose of determining
the number of votes required to obtain the valid concurrence of the Senate.

It is a finely-imbedded principle in statutory construction that a special provision or


law prevails over a general one. Lex specialis derogat generali. Thus, where there is in
the same statute a particular enactment and also a general one which, in its most
comprehensive sense, would include what is embraced in the former, the particular
enactment must be operative, and the general enactment must be taken to affect only
such cases within its general language which are not within the provision of the
particular enactment.

=======

Section 25, Article XVIII disallows foreign military bases, troops, or facilities in the
country, unless the following conditions are sufficiently met, viz: (a) it must be under a
treaty; (b) the treaty must be duly concurred in by the Senate and, when so required
by congress, ratified by a majority of the votes cast by the people in a national
referendum; and (c) recognized as a treaty by the other contracting state.

There is no dispute as to the presence of the first two requisites in the case of the VFA.
The concurrence handed by the Senate through Resolution No. 18 is in accordance
with the provisions of the Constitution, whether under the general requirement in
Section 21, Article VII, or the specific mandate mentioned in Section 25, Article XVIII,
the provision in the latter article requiring ratification by a majority of the votes cast in
a national referendum being unnecessary since Congress has not required it.

=======

As to the matter of voting, Section 21, Article VII particularly requires that a treaty or
international agreement, to be valid and effective, must be concurred in by at least
two-thirds of all the members of the Senate. On the other hand, Section 25, Article
XVIII simply provides that the treaty be “duly concurred in by the Senate.” x x x x

As noted, the “concurrence requirement” under Section 25, Article XVIII must be
construed in relation to the provisions of Section 21, Article VII. In a more particular
language, the concurrence of the Senate contemplated under Section 25, Article XVIII
means that at least two-thirds of all the members of the Senate favorably vote to
concur with the treaty-the VFA in the instant case.

=======

This Court is of the firm view that the phrase “recognized as a treaty” means that the
other contracting party accepts or acknowledges the agreement as a treaty. To require
the other contracting state, the United States of America in this case, to submit the
VFA to the United States Senate for concurrence pursuant to its Constitution, is to
accord strict meaning to the phrase.

Well-entrenched is the principle that the words used in the Constitution are to be
given their ordinary meaning except where technical terms are employed, in which
case the significance thus attached to them prevails. Its language should be
understood in the sense they have in common use.

Moreover, it is inconsequential whether the United States treats the VFA only as an
executive agreement because, under international law, an executive agreement is as
binding as a treaty. To be sure, as long as the VFA possesses the elements of an
agreement under international law, the said agreement is to be taken equally as a
treaty.

Page 174 of 323


In international law, there is no difference between treaties and executive agreements
in their binding effect upon states concerned, as long as the negotiating functionaries
have remained within their powers.38 International law continues to make no
distinction between treaties and executive agreements: they are equally binding
obligations upon nations.

=======

As regards the power to enter into treaties or international agreements, the


Constitution vests the same in the President, subject only to the concurrence of at
least two-thirds vote of all the members of the Senate. In this light, the negotiation of
the VFA and the subsequent ratification of the agreement are exclusive acts which
pertain solely to the President, in the lawful exercise of his vast executive and
diplomatic powers granted him no less than by the fundamental law itself. Into the
field of negotiation the Senate cannot intrude, and Congress itself is powerless to
invade it. Consequently, the acts or judgment calls of the President involving the VFA-
specifically the acts of ratification and entering into a treaty and those necessary or
incidental to the exercise of such principal acts - squarely fall within the sphere of his
constitutional powers and thus, may not be validly struck down, much less calibrated
by this Court, in the absence of clear showing of grave abuse of power or discretion.

=======

It is the Court’s considered view that the President, in ratifying the VFA and in
submitting the same to the Senate for concurrence, acted within the confines and
limits of the powers vested in him by the Constitution. It is of no moment that the
President, in the exercise of his wide latitude of discretion and in the honest belief that
the VFA falls within the ambit of Section 21, Article VII of the Constitution, referred
the VFA to the Senate for concurrence under the aforementioned provision. Certainly,
no abuse of discretion, much less a grave, patent and whimsical abuse of judgment,
may be imputed to the President in his act of ratifying the VFA and referring the same
to the Senate for the purpose of complying with the concurrence requirement
embodied in the fundamental law. In doing so, the President merely performed a
constitutional task and exercised a prerogative that chiefly pertains to the functions of
his office. Even if he erred in submitting the VFA to the Senate for concurrence under
the provisions of Section 21 of Article VII, instead of Section 25 of Article XVIII of the
Constitution, still, the President may not be faulted or scarred, much less be adjudged
guilty of committing an abuse of discretion in some patent, gross, and capricious
manner.

The Judicial Department

*Resolution dated February 14, 2012. [IMPORTANT!!!]


(Deliberative process privilege)

Facts:
-Title of the Resolution:
In Re: Production of Court Records and Documents and the Attendance of Court
officials and employees as witnesses under the subpoenas of February 10, 2012 and
the various letters for the Impeachment Prosecution Panel dated January 19 and 25,
2012.

-For SC’s resolution were letters from the Impeachment Prosecution Panel (against
then CJ Corona), asking for the examination of records, and the issuance of certified
true copies of the rollos and the Agenda and Minutes of the Deliberations involving
certain cases still under litigation and pending with the Supreme Court, as well as
two cases that were already closed and terminated, for purposes of Articles 3 and 7 of
the Impeachment Complaint.

Page 175 of 323


-The Clerk of Court of the Supreme Court likewise received from the panel a
Subpoena Ad Testificandum et Duces Tecum, commanding her to appear before it with
the original and certified true copies of the subject documents, and to likewise appear
in the afternoon at 2:00 of the same day and every day thereafter, to produce subject
documents and to testify.

Issue:
Whether or not the Impeachment Panel has the authority to compel the Supreme
Court to produce subject documents and to compel its employees to testify and bring
the subject documents.

Resolution:
No.

The right to information, by its very nature and by the Constitution’s own terms, is not
absolute.

Section 11, Rule 136 of the Rules of Court, which states:

Section. 11. Certified copies.—The clerk shall prepare, for any person demanding the same, a copy
certified under the seal of the court of any paper, record, order, judgment, or entry in his office, proper to
be certified, for the fees prescribed by these rules.

Notably, the rule grants access to court records to any person, subject to payment of
fees and compliance with rules; it is not necessary that the request be made by a party
to the case. This grant, however, is not as open nor as broad as its plain terms
appear to project, as it is subject to the limitations the laws and the Court’s own
rules provide.

Certain informations contained in the records of cases before the Supreme Court
are considered confidential and are exempt from disclosure. To reiterate, the need
arises from the dictates of the integrity of the Court’s decision-making function which
may be affected by the disclosure of information.

The following are privileged documents or communications, and are not subject to
disclosure:

(1) Court actions such as the result of the raffle of cases and the actions taken by the
Court on each case included in the agenda of the Court’s session on acts done
material to pending cases, except where a party litigant requests information on the
result of the raffle of the case, pursuant to Rule 7, Section 3 of the IRSC;

(2) Court deliberations or the deliberations of the Members in court sessions on cases
and matters pending before the Court;

(3) Court records which are “predecisional” and “deliberative” in nature, in particular,
documents and other communications which are part of or related to the deliberative
process, i.e., notes, drafts, research papers, internal discussions, internal memoranda,
records of internal deliberations, and similar papers.

(4) Confidential Information secured by justices, judges, court officials and employees
in the course of their official functions, mentioned in (2) and (3) above, are privileged
even after their term of office.

(5) Records of cases that are still pending for decision are privileged materials that
cannot be disclosed, except only for pleadings, orders and resolutions that have been
made available by the court to the general public.

(6) The principle of comity or inter-departmental courtesy demands that the highest
officials of each department be exempt from the compulsory processes of the other
departments.

Page 176 of 323


(7) These privileges belong to the Supreme Court as an institution, not to any justice
or judge in his or her individual capacity. Since the Court is higher than the individual
justices or judges, no sitting or retired justice or judge, not even the Chief Justice,
may claim exception without the consent of the Court.

Court deliberations are traditionally recognized as privileged communication.


Section 2, Rule 10 of the IRSC provides:

Section 2. Confidentiality of court sessions. – Court sessions are executive in character, with only the
Members of the Court present. Court deliberations are confidential and shall not be disclosed to outside
parties, except as may be provided herein or as authorized by the Court.

The rules on confidentiality will enable the Members of the Court to “freely
discuss the issues without fear of criticism for holding unpopular positions” or
fear of humiliation for one’s comments. The privilege against disclosure of these
kinds of information/communication is known as deliberative process privilege,
involving as it does the deliberative process of reaching a decision.

The privilege is not exclusive to the Judiciary. A Senator may invoke legislative
privilege when he or she is questioned outside the Senate about information gathered
during an executive session of the Senate’s legislative inquiry in aid of legislation. In
the same manner, a justice of the court or a judge may invoke judicial privilege in the
Senate sitting as an Impeachment Court, for proceedings in the performance of his or
her own judicial functions. What applies to magistrates applies with equal force to
court officials and employees who are privy to these deliberations. They may
likewise claim exemption when asked about this privileged information.

While Section 2, Rule 10 of the IRSC cited above speaks only of the confidentiality of
court deliberations, it is understood that the rule extends to documents and other
communications which are part of or are related to the deliberative process. The
deliberative process privilege protects from disclosure documents reflecting advisory
opinions, recommendations and deliberations that are component parts of the process
for formulating governmental decisions and policies. Obviously, the privilege may also
be claimed by other court officials and employees when asked to act on these
documents and other communications.

Additionally, two other grounds may be cited for denying access to court records, as
well as preventing members of the bench, from being subjected to compulsory process:
(1) the disqualification by reason of privileged communication; and (2) the
pendency of an action or matter.

Under the law, the Members of the Court may not be compelled to testify in the
impeachment proceedings against the Chief Justice or other Members of the
Court about information they acquired in the performance of their official
function of adjudication, such as information on how deliberations were conducted
or the material inputs that the justices used in decision-making, because the end-
result would be the disclosure of confidential information that could subject them to
criminal prosecution. Such act violates judicial privilege (or the equivalent of executive
privilege) as it pertains to the exercise of the constitutional mandate of adjudication.

Jurisprudence implies that justices and judges may not be subject to any
compulsory process in relation to the performance of their adjudicatory
functions. x x x x Inter-departmental courtesy demands that the highest levels of
each department be exempt from the compulsory processes of the other
departments on matters related to the functions and duties of their office.

To state the rule differently, Justices of the Court cannot be compelled to testify
on matters relating to the internal deliberations and actions of the Court, in the
exercise of their adjudicatory functions and duties. This is to be differentiated from a

Page 177 of 323


situation where the testimony is on a matter which is external to their adjudicatory
functions and duties.

For example, where the ground cited in an impeachment complaint is bribery, a


Justice may be called as a witness in the impeachment of another Justice, as
bribery is a matter external to or is not connected with the adjudicatory
functions and duties of a magistrate. A Justice, however, may not be called to testify
on the arguments the accused Justice presented in the internal debates as these
constitute details of the deliberative process.

Entries in official records may be presented without the necessity of presenting


in court the officer or person who made the entries. Entries in public or official
books or records may be proved by the production of the books or records themselves
or by a copy certified by the legal keeper thereof. These records, however, may be
presented and marked in evidence only where they are not excluded by reasons
of privilege and the other similar reasons.

* Resolution dated August 26, 2014.


(Re: Request for Copies of the Statement of Assets, Liabilities and Net Worth (SALNs)
of the Justices of the Supreme Court and Court of Tax Appeals)

Facts:
-In a letter-request by the Commissioner of Internal Revenue (CIR), Atty. Kim Henares
requested certified copies of the Statements of Assets, Liabilities, and Net Worth
(SALNs) of all incumbent Justices of the Supreme Court and the Court of Tax Appeals
(CTA), for the years 2003 to 2012.

-According to Henares, her request was “in relation to the ‘Ma’am Arlene
Controversy’ in the judiciary” and for “tax investigation purposes pursuant to Section
S(B) of the National Internal Revenue Code of 1997.”

-Henares manifested that she had earlier received a letter that alleging that largesse
from ‘Ma'am Arlene’ benefited the higher echelons of the judiciary, including the
members of the SC and the CTA. The CIR also cited, as basis, the column of Jarius
Bondoc entitled, “Just call her Ma’am Arlene, the Judiciary’s Napoles,” in the
Philippine Star, to justify Henraes’ assertion that she is simply performing her duty to
ensure that the proper taxes are being paid.

Resolution:
The request for SALNs was DENIED.

Requests for SALNs must be made under circumstances that must not endanger,
diminish or destroy the independence and objectivity of the members of the
Judiciary in the performance of their judicial functions or expose them to
revenge for adverse decisions, kidnapping, extortion, blackmail or other
untoward incidents. x x x x

Moreover, the Tax Code provides prescriptive limits on her power to assess and
collect taxes. Under Section 203 of the Tax Code, a 3-year limit is imposed on
the assessment of internal revenue taxes. Section 222 of the Tax Code, on the other
hand, extends the prescriptive period to assess deficiency taxes to 10 years in cases
when there is a false or fraudulent return with intent to evade tax, or non-filing of a
tax return.

Examined in this light, it becomes apparent this time that the CIR has exceeded her
authority to investigate and examine the tax liabilities of the members of the
Court and the CTA by requesting certified copies of their SALNs. It is incredulous
that the CIR seems to imply that all the Justices of the Court and the CTA are being
investigated for fraud. To begin with, she fails to identify those she wants to be
investigated. Neither has she given notices to those she wants to be investigated for
having committed fraud. Stated differently, there is no prima facie showing or well-

Page 178 of 323


founded suspicion that fraud has been committed to justify the application of the 10-
year prescriptive period. Verily, fraud is never presumed. One who alleges fraud
must, at the very least, allege the acts and omissions that constitute fraud, as the
prescriptive periods imposed by the Tax Code are precisely intended to give taxpayers
peace of mind. x x x x

Even the so-called “Ma’am Arlene controversy” cannot be utilized as a blanket


authority to investigate the alleged tax deficiencies of the members of the Court
or the CTA. For one thing, the alleged controversy only involves specific members of
the regional trial courts for their alleged complicity with certain individuals in
influencing the outcome of the election of officers of the Philippine Judges Association
(PJA). Undeniably, the controversy does not involve any member of the Court or
the CTA. At the very least, departmental courtesy demands that the CIR first
allow the Court to conclude its formal investigation of the matter.

Curiously, while it is the claim of the CIR that the subject requests for the SALNs are
upon her interest to “ensure tax compliance by members of the judiciary,” she has
shown no interest in obtaining copies of the SALNs of the members of the
Judiciary in the Sandiganbayan or the first-level courts. It appears that it is her
dogged determination to focus the investigation of her office on just the
members of the Court and the CTA, tribunals exercising jurisdiction over cases
involving her office. Thus, there appears to be a basis to agree with the position of
the CT A that the request of the CIR smacks of a “fishing expedition.”

The inescapable conclusion is that the CIR has questionable motives for seeking
copies of the SALNs of the members of the Court and the CTA. To grant the
subject requests under these circumstances would authorize a veritable witch-hunt
which the Court cannot, and will not, countenance. To accede to the CIR's requests
will undermine the bedrock of judicial independence, which this Court has zealously
guarded. x x x x [NOTE: This letter request was made after the DAP decision by the
SC came out. PNoy was unhappy with the invalidation of DAP.]

In invoking one's constitutional right to information - whether in the spirit of public


accountability, transparency or some other cause -the need to preserve the
integrity and independence of the Judiciary must be weighed. It must be invoked,
and can only be upheld, if under the circumstances it would not result in
endangering, diminishing or destroying the independence and security of the
members of the Judiciary in the performance of their judicial functions, or expose
them to revenge for adverse decisions.

The reason for this is obvious: Judicial integrity and security is an essential
element in the maintenance of a free and democratic society. The independence
of the Judiciary should be protected not because the Court seeks to “create an
exception for themselves,” but because there is a need to ensure that the Court will be
able to perform its role in the system of checks and balances. While the Judiciary is
well-known to be the weakest branch of government, the bases of its limited power
should never be further clipped and undermined, lest we all become witnesses to the
rise of an undemocratic regime.

*De Castro v. JBC, G.R. No. 191002, March 17, 2010. [IMPORTANT!!!]
(Appointment to the Judiciary)

Facts:
-Chief Justice Puno was set to compulsorily retire on May 17, 2010, or seven days
after the presidential election.

-Under Section 4(1), in relation to Section 9, Article VIII, that “vacancy shall be filled
within ninety days from the occurrence thereof” from a “list of at least three
nominees prepared by the Judicial and Bar Council for every vacancy.”

Page 179 of 323


-The JBC already began the process for the filling of the position of Chief Justice
Puno in accordance with its rules.

-Petitioners went to the Supreme Court via Prohibition to prevent the JBC from
recommending appointees in the Judiciary.

Issue:
Whether or not the incumbent President has the authority to appoint CJ’s successor,
considering that Section 15, Article VII of the Constitution prohibits the President or
Acting President from making appointments within two months immediately before the
next presidential elections and up to the end of his term, in relation to Section 4 (1),
Article VIII of the Constitution, which provides that any vacancy in the Supreme Court
shall be filled within 90 days from the occurrence thereof.

Ruling:
Yes.

The prohibition under Section 15, Article VII does not apply to appointments to
fill a vacancy in the Supreme Court or to all other appointments to the
Judiciary. x x x x

As can be seen, Article VII is devoted to the Executive Department, and, among
others, it lists the powers vested by the Constitution in the President. The
presidential power of appointment is dealt with in Sections 14, 15 and 16 of the
Article.

Article VIII is dedicated to the Judicial Department and defines the duties and
qualifications of Members of the Supreme Court, among others. Section 4(1) and
Section 9 of this Article are the provisions specifically providing for the
appointment of Supreme Court Justices. In particular, Section 9 states that the
appointment of Supreme Court Justices can only be made by the President upon the
submission of a list of at least three nominees by the JBC; Section 4(1) of the Article
mandates the President to fill the vacancy within 90 days from the occurrence of
the vacancy.

Had the framers intended to extend the prohibition contained in Section 15,
Article VII to the appointment of Members of the Supreme Court, they could
have explicitly done so. They could not have ignored the meticulous ordering of
the provisions. They would have easily and surely written the prohibition made
explicit in Section 15, Article VII as being equally applicable to the appointment
of Members of the Supreme Court in Article VIII itself, most likely in Section 4
(1), Article VIII. That such specification was not done only reveals that the
prohibition against the President or Acting President making appointments within two
months before the next presidential elections and up to the end of the President’s or
Acting President’s term does not refer to the Members of the Supreme Court. x x x
x

Moreover, the usage in Section 4(1), Article VIII of the word ‘shall’ - an
imperative, operating to impose a duty that may be enforced - should not be
disregarded. Thereby, Sections 4(1) imposes on the President the imperative duty
to make an appointment of a Member of the Supreme Court within 90 days from
the occurrence of the vacancy. The failure by the President to do so will be a clear
disobedience to the Constitution. x x x x

Given the background and rationale for the prohibition in Section 15, Article VII, we
have no doubt that the Constitutional Commission confined the prohibition to
appointments made in the Executive Department. The framers did not need to
extend the prohibition to appointments in the Judiciary, because their
establishment of the JBC and their subjecting the nomination and screening of
candidates for judicial positions to the unhurried and deliberate prior process of

Page 180 of 323


the JBC ensured that there would no longer be midnight appointments to the
Judiciary.

Also, the intervention of the JBC eliminates the danger that appointments to the
Judiciary can be made for the purpose of buying votes in a coming presidential
election, or of satisfying partisan considerations. The experience from the time of
the establishment of the JBC shows that even candidates for judicial positions at any
level backed by people influential with the President could not always be assured of
being recommended for the consideration of the President, because they first had to
undergo the vetting of the JBC and pass muster there. Indeed, the creation of the
JBC was precisely intended to de-politicize the Judiciary by doing away with the
intervention of the Commission on Appointments.

*Chavez v. JBC, G.R. No. 202242, April 16, 2013.


(The Judicial and Bar Council)

Facts:
-Pursuant to Section 8, Article VIII of the 1987 Constitution, the Congress designated
one (1) representative to sit in the JBC to act as one of the ex-officio members.
Considering that the Congress is entitled to only one (1) representative, each House
sent a representative to the JBC, not together, but alternately or by rotation.

-However, in 1994, the seven-member composition of the JBC was substantially


altered. An eighth member was added to the JBC as the two (2) representatives from
Congress began sitting simultaneously in the JBC, with each having one-half (1/2) of
a vote.

-In 2001, the JBC En Banc decided to allow the representatives from the Senate and
the House of Representatives one full vote each. It has been the situation since
then.

-Upon CJ Corona’s impeachment, petitioner Chavez was nominated as Corona’s


potential successor. He then went to the Supreme Court to challenge the addition of
the eight member of the JBC.

-Contentions of the respondents: (1) allowing only one representative from Congress in
the JBC would lead to absurdity considering its bicameral nature; (2) that the failure
of the Framers to make the proper adjustment when there was a shift from
unilateralism to bicameralism was a plain oversight; and (3) that two representatives
from Congress would not subvert the intention of the Framers to insulate the JBC
from political partisanship.

Issues:
(1) Whether or not Section 8, Article VIII of the Constitution allows more than one (1)
member of Congress to sit in the JBC;
(2) Whether or not the practice of having two (2) representatives from each House of
Congress with one (1) vote each is sanctioned by the Constitution.

Ruling:
(1) No.

In opting to use the singular letter “a” to describe “representative of Congress,” the
Filipino people through the Framers intended that Congress be entitled to only one
(1) seat in the JBC. Had the intention been otherwise, the Constitution could have, in
no uncertain terms, so provided, as can be read in its other provisions. x x x x

In the creation of the JBC, the Framers arrived at a unique system by adding to the
four (4) regular members, three (3) representatives from the major branches of
government - the Chief Justice as ex-officio Chairman (representing the Judicial
Department), the Secretary of Justice (representing the Executive Department),
and a representative of the Congress (representing the Legislative Department).

Page 181 of 323


The total is seven (7), not eight. In so providing, the Framers simply gave
recognition to the Legislature, not because it was in the interest of a certain
constituency, but in reverence to it as a major branch of government.

Respondents cannot just lean on plain oversight to justify a conclusion favorable to


them. It is very clear that the Framers were not keen on adjusting the provision on
congressional representation in the JBC because it was not in the exercise of its
primary function – to legislate. JBC was created to support the executive power to
appoint, and Congress, as one whole body, was merely assigned a contributory non-
legislative function. x x x x

The aforesaid provision is clear and unambiguous and does not need any further
interpretation. Perhaps, it is apt to mention that the oft-repeated doctrine that
“construction and interpretation come only after it has been demonstrated that
application is impossible or inadequate without them.” Further, to allow Congress to
have two representatives in the Council, with one vote each, is to negate the
principle of equality among the three branches of government which is
enshrined in the Constitution.

(2) No. (See above ruling.)

*Jardeleza v. Sereno, G.R. No. 213181, August 19, 2014.


(Powers and Functions of the JBC)

Facts:
-Due to the vacated post of Justice Abad for compulsory retirement, the JBC
announced the opening for application or recommendation for the said vacated
position.

-Jardeleza, then the Solicitor General, was nominated by Dean Concepcion of the UP
Law. On May 29, 2014, Jardeleza was interviewed by the JBC.

-During the meetings held on June 5 and 16, 2014, CJ Sereno manifested that she
would be invoking Section 2, Rule 10 of JBC-0094 against Jardeleza. (opposition
based on doubtful integrity)

-Section 2, Rule 10 of JBC-009 provides:

SEC. 2. Votes required when integrity of a qualified applicant is challenged. - In every case where the
integrity of an applicant who is not otherwise disqualified for nomination is raised or challenged,
the affirmative vote of all the Members of the Council must be obtained for the favorable
consideration of his nomination.

-This so-called “unanimity rule” was invoked by CJ Sereno during the JBC meeting
held on June 5, 2014, where she expressed her position that Jardeleza did not possess
the integrity required to be a member of the Court. In the same meeting, the Chief
Justice shared with the other JBC members the details of Jardeleza’s chosen manner
of framing the government’s position in an international dispute case and how this
could have been detrimental to the national interest.

-On June 17, 2014, Jardeleza was directed to “make himself available” before the JBC
on June 30, 2014, during which he would be informed of the objections to his
integrity.

-In view of the challenge against his integrity, Jardeleza filed a letter-petition with the
SC praying that the Court, in the exercise of its constitutional power of supervision
over the JBC, issue an order: (1) directing the JBC to give him at least five (5)
working days written notice of any hearing of the JBC to which he would be
summoned; and the said notice to contain the sworn specifications of the charges
against him by his oppositors, the sworn statements of supporting witnesses, if any,
and copies of documents in support of the charges; and notice and sworn statements
shall be made part of the public record of the JBC; (2) allowing him to cross-examine

Page 182 of 323


his oppositors and supporting witnesses, if any, and the cross-examination to be
conducted in public, under the same conditions that attend the public interviews held
for all applicants; and (3) directing the JBC to reset the hearing scheduled on June
30, 2014 to another date.

-During the June 30, 2014 meeting, Jardeleza’s integrity was questioned regarding: (1)
his actuations in the handling of the international dispute case; (2) a supposed
extra-marital affair in the past; and (3) alleged acts of insider trading. A JBC
member manifested that he wanted to hear for himself Jardeleza’s explanation on the
matter. Jardeleza, however, refused as he maintained that he would not be lulled
into waiving his rights. Jardeleza requested the JBC to defer its meeting considering
that the Court en banc would meet the next day to act on his pending letter-petition.

-Apparently denying Jardeleza’s request for deferment of the proceedings, the JBC
continued its deliberations and proceeded to vote for the nominees to be included in
the shortlist. Thereafter, the JBC released the subject shortlist of four (4) nominees,
excluding Jardeleza.

-Jardelza then filed with the SC petition for certiorari and mandamus with prayer for
the issuance of a Temporary Restraining Order (TRO), seeking to compel the JBC to
include him in the list of nominees, on the grounds that the JBC and CJ Sereno acted
in grave abuse of discretion amounting to lack or excess of jurisdiction in excluding
him, despite having garnered a sufficient number of votes to qualify for the
position.

-Contention of Jardeleza: His exclusion from the shortlist is illegal due to the
deprivation of his constitutional right to due process.

Issues:
(1) Whether or not the right to due process is available and demandable as a matter of
right during JBC proceedings in cases where an objection or opposition to an
application is raised;
(2) Whether or not petitioner Jardeleza may be included in the shortlist of nominees
submitted to the President.

Ruling:
(1) Yes.

JBC proceedings are unique and special in nature. While the facets of criminal and
administrative due process are not strictly applicable to JBC proceedings, their
peculiarity is insufficient to justify the conclusion that due process is not
demandable.

In JBC proceedings, an aspiring judge or justice justifies his qualifications for the
office when he presents proof of his scholastic records, work experience and laudable
citations. His goal is to establish that he is qualified for the office applied for. The JBC
then takes every possible step to verify an applicant’s track record for the purpose of
determining whether or not he is qualified for nomination. It ascertains the factors
which entitle an applicant to become a part of the roster from which the President
appoints. x x x x

The fact that a proceeding is sui generis and is impressed with discretion,
however, does not automatically denigrate an applicant’s entitlement to due
process. Notwithstanding being “a class of its own,” the right to be heard and to
explain one’s self is availing. The Court subscribes to the view that in cases where
an objection to an applicant’s qualifications is raised, the observance of due process
neither negates nor renders illusory the fulfillment of the duty of JBC to recommend. x
x x x The JBC is not expected to strictly apply the rules of evidence in its
assessment of an objection against an applicant. Just the same, to hear the side of
the person challenged complies with the dictates of fairness for the only test that an
exercise of discretion must surmount is that of soundness. x x x x

Page 183 of 323


The provisions of JBC-010, per se, provide that: any complaint or opposition
against a candidate may be filed with the Secretary within ten (10) days thereof;
the complaint or opposition shall be in writing, under oath and in ten (10) legible
copies; the Secretary of the Council shall furnish the candidate a copy of the
complaint or opposition against him; the candidate shall have five (5) days from
receipt thereof within which to file his comment to the complaint or opposition,
if he so desires; and the candidate can be made to explain the complaint or
opposition against him. x x x x

It would all be too well to remember that the allegations of his extra-marital affair and
acts of insider trading sprung up only during the June 30, 2014 meeting. While the
said issues became the object of the JBC discussion on June 16, 2014, Jardeleza was
not given the idea that he should prepare to affirm or deny his past behavior.
These circumstances preclude the very idea of due process in which the right to
explain oneself is given, not to ensnare by surprise, but to provide the person a
reasonable opportunity and sufficient time to intelligently muster his response.
Otherwise, the occasion becomes an idle and futile exercise.

(2) Yes.

This consequence arose from the violation by the JBC of its own rules of procedure
and the basic tenets of due process.

The Court refuses to turn a blind eye on the palpable defects in its implementation
and the ensuing treatment that Jardeleza received before the Council. True, Jardeleza
has no vested right to a nomination, but this does not prescind from the fact
that the JBC failed to observe the minimum requirements of due process.

Having been able to secure four (4) out of six (6) votes, the only conclusion left to
propound is that a majority of the members of the JBC, nonetheless, found
Jardeleza to be qualified for the position of Associate Justice and this grants him
a rightful spot in the shortlist submitted to the President.

=========

[NOTE: Jardeleza was eventually chosen and appointed by PNoy.]

[ANOTHER NOTE: Another matter that convinced the Court that the challenge against
Jardeleza’s integrity was misplaced was the fact that the primary ground for
opposition against him was the manner he handled the international dispute as the
Solicitor General:

“The approach taken by Jardeleza in that case was opposed to that preferred by the
legal team. However, a lawyer has complete discretion on what legal strategy to
employ in a case entrusted to him provided that he lives up to his duty to serve
his client with competence and diligence, and that he exert his best efforts to
protect the interests of his client within the bounds of the law.

Stripped of a clear showing of gross neglect, iniquity, or immoral purpose, a strategy


of a legal mind remains a legal tactic acceptable to some and deplorable to
others. It has no direct bearing on his moral choices. x x x x

CJ Sereno’s efforts in the determination of Jardeleza’s professional background, while


commendable, have not produced a patent demonstration of a connection between the
act complained of and his integrity as a person. Nonetheless, the Court cannot
consider her invocation of Section 2, Rule 10 of JBC-009 as conformably within the
contemplation of the rule. To fall under Section 2, Rule 10 of JBC-009, there must be
a showing that the act complained of is, at the least, linked to the moral
character of the person and not to his judgment as a professional.”

Page 184 of 323


*Fortich v. Corona, G.R. No. 131457, August 19, 1999.
(The Supreme Court; Divisions and En Banc)

Facts:
-The SC Second Division promulgated a decision. The OSG filed a motion for
reconsideration, which was denied by the SC Second Division in a two-two vote.

-Article VIII, Section 4(3) provides:

Cases or matters heard by a division shall be decided or resolved with the concurrence of a
majority of the Members who actually took part in the deliberations on the issues in the case and
voted thereon, and in no case without the concurrence of at least three of such Members. When the
required number is not obtained, the case shall be decided en banc: Provided, that no doctrine or
principle of law laid down by the Court in a decision rendered en banc or in division may be modified or
reversed except by the Court sitting en banc.

-Argument of the OSG: Considering that the MR was resolved by a vote of two-two, the
required number to carry a decision, i.e., three, was not met. Hence, the case must be
referred to the SC en banc.

Issue:
Whether or not the case must be referred to the SC en banc.

Ruling:
No.

The intention of the framers was to draw a distinction between cases, on the one
hand, and matters, on the other hand, such that cases are “decided” while
matters, which include motions, are “resolved.” Otherwise put, the word
“decided” must refer to “cases;” while the word “resolved” must refer to
“matters,” applying the rule of reddendo singula singulis.

Only cases are referred to the Court en banc for decision whenever the required
number of votes is not obtained. Conversely, the rule does not apply where, as in
this case, the required three votes is not obtained in the resolution of a motion
for reconsideration. Hence, the second sentence of the aforequoted provision
speaks only of “case” and not “matter.”

If there is a tie in the voting, there is no decision. The only way to dispose of the
case then is to refer it to the Court en banc. On the other hand, if a case has
already been decided by the division and the losing party files a motion for
reconsideration, the failure of the division to resolve the motion because of a tie
in the voting does not leave the case undecided. There is still the decision which
must stand in view of the failure of the members of the division to muster the
necessary vote for its reconsideration. Quite plainly, if the voting results in a tie,
the motion for reconsideration is lost. The assailed decision is not reconsidered
and must therefore be deemed affirmed.

*Primicias v. Ocampo, G.R. No. L-6120, June 30, 1953.


(The SC’s rule-making power)

Facts:
-Primicias was charged before the RTC with two offenses, namely, (1) a violation of
Commonwealth Act No. 606, on the basis that he knowingly chartered a vessel of
Philippine registry to an alien without the approval of the President of the Philippines
and (2) a violation of section 129 of the Revised Administrative Code, on the basis that
he failed to submit to the Collector of Customs the manifests and certain
authenticated documents for the vessel “Antarctic” and failed to obtain the necessary
clearance from the Bureau of Customs prior to the departure of said vessel for a
foreign port.

Page 185 of 323


-Before the trial of said criminal cases, petitioner filed a motion praying that
assessors be appointed to assist the court in considering the questions of fact
involved in said cases as authorized by section 49 of Republic Act No. 409, otherwise
known as Revised Charter of the City of Manila, which provides that “the aid of
assessors in the trial of any civil or criminal action in the MTC, or the RTC,
within the City, may be invoked in the manner provided in the Code of Civil
Procedure.” This motion was opposed by the City Fiscal who appeared for the People
of the Philippines.

-The RTC denied the motion, holding in effect that with the promulgation of the Rules
of Court by the Supreme Court, all rules concerning pleading, practice and procedure
in all courts of the Philippines previously existing were not only superseded but
expressly repealed.

-Contention of the RTC: Considering that that the Supreme Court, having been vested
with the rule-making power, expressly omitted the portions of the Code of Civil
Procedure regarding assessors in said Rules of Court, the aid of assessors is no longer
demandable as a matter of right.

-Primicias then went to the SC via petition for prohibition.

Issue:
Whether or not the right of the petitioner to a trial with the aid of assessors is a
substantive right or a mere procedural right that can be dispensed with by the
Supreme Court through its rule-making power.

Ruling:
A trial with the aid of assessors is a substantive right.

Rules of procedure should be distinguished from substantive law. A substantive law


creates, defines or regulates rights concerning life, liberty or property, or the powers of
agencies or instrumentalities for the administration of public affairs, whereas rules of
procedure are provisions prescribing the method by which substantive rights may be
enforced in courts of justice.

The trial with the aid of assessors as granted by Section 2477 of the old Charter
of Manila is part of substantive law and as such is not embraced by the rule-
making power of the Supreme Court. x x x x

A trial with the aid of assessors is another security given by the law to the
litigants, and as such, it is a substantial right of which they cannot be deprived
without vitiating all the proceedings. x x x x

Being substantive in nature, it is not difficult to see why the provisions concerning
trial by assessors embodied in the Code of Civil Procedure have not been incorporated
by the Supreme Court in the present Rules of Court. To have done so, it would have
been a travesty of its rule-making power which, by direct mandate of the
Constitution, is limited to matters referring to pleading, practice and procedure. The
application that the respondents draw from the failure to incorporate these provisions
in the present Rules of Court to the effect that the intention was to eliminate them or
repeal them all together cannot, therefore, stand.

*Canlas v. Napico Homeowners Association, G.R. No. 182795, June 5, 2008.


(The SC’s rule-making power: Writ of Amparo)

Facts:
-Petitioners are settlers in a certain parcel of land situated in Pasig City. Their
dwellings/houses have either been demolished as of the time of filing of the petition,
or are about to be demolished pursuant to a court judgment.

Page 186 of 323


-The petition prayed that certain government officials be summoned to answer their
participation in the issuances of fraudulent and spurious titles covering the lands
where their houses are erected.

Issue:
Whether or not the said petition for writ of amparo will prosper.

Ruling:
No.

The Rule on the Writ of Amparo provides:

“Section 1. Petition. – The petition for a writ of amparo is a remedy available to any person whose right to
life, liberty and security is violated or threatened with violation by an unlawful act or omission of a
public official or employee, or of a private individual or entity.
The writ shall cover extralegal killings and enforced disappearances or threats thereof.”

The threatened demolition of a dwelling by virtue of a final judgment of the court,


which in this case was affirmed with finality by this Court in G.R. Nos. 177448,
180768, 177701, 177038, is not included among the enumeration of rights as stated
in the above-quoted Section 1 for which the remedy of a writ of amparo is made
available. Their claim to their dwelling, assuming they still have any despite the
final and executory judgment adverse to them, does not constitute right to life,
liberty and security. There is, therefore, no legal basis for the issuance of the writ of
amparo.

Besides, the factual and legal basis for petitioners’ claim to the land in question
is not alleged in the petition at all. The Court can only surmise that these rights
and interest had already been threshed out and settled in the four cases cited above.
No writ of amparo may be issued unless there is a clear allegation of the supposed
factual and legal basis of the right sought to be protected.

Under Section 6 of the same rules, the court shall issue the writ upon the filing of the
petition, only if on its face, the court ought to issue said writ.

“Section 6. Issuance of the Writ. – Upon the filing of the petition, the court, justice or judge shall
immediately order the issuance of the writ if on its face it ought to issue. The clerk of court shall issue the
writ under the seal of the court; or in case of urgent necessity, the justice or the judge may issue the writ
under his or her own hand, and may deputize any officer or person to serve it.

The writ shall also set the date and time for summary hearing of the petition which shall not be later than
seven (7) days from the date of its issuance.”

Considering that there is no legal basis for its issuance, as in this case, the writ
will not be issued and the petition will be dismissed outright.

This new remedy of writ of amparo which is made available by this Court is intended
for the protection of the highest possible rights of any person, which is his or her right
to life, liberty and security. The Court will not spare any time or effort on its part in
order to give priority to petitions of this nature. However, the Court will also not waste
its precious time and effort on matters not covered by the writ.

*Tapuz v. Del Rosario, G.R. No. 182484, June 17, 2008.


(The SC’s rule-making power: Writ of Amparo; Writ of Habeas Data)

Facts:
-Respondents filed with the MCTC a complaint for forcible entry and damages
against petitioners.

-The MCTC, after due proceedings, rendered a decision in respondents’ favor.


Petitioners appealed to the RTC.

Page 187 of 323


-On appeal, the RTC judge granted respondents’ motion for the issuance of a writ of
preliminary mandatory injunction, with the issuance conditioned on respondents’
posting of a bond. The petitioners moved to reconsider the issuance of the writ; the
respondents, on the other hand, filed a motion for demolition.

-The respondent Judge subsequently issued via a Special Order a writ of demolition to
be implemented fifteen (15) days after the Sheriff’s written notice to the petitioners to
voluntarily demolish their house/s to allow the private respondents to effectively take
actual possession of the land.

-The petitioners thereafter filed with the Court of Appeals a Petition for Review (under
Rule 42 of the 1997 Rules of Civil Procedure) of the Permanent Mandatory Injunction
and Order of Demolition of the RTC. Meanwhile, respondent Sheriff Nelson R. dela
Cruz issued the Notice to Vacate and for Demolition.

-Petitioners then went to the SC. The petition contained and prayed for three
remedies, namely: a petition for certiorari under Rule 65 of the Revised Rules of Court;
the issuance of a writ of habeas data under the Rule on the Writ of Habeas Data; and
finally, the issuance of the writ of amparo under the Rule on the Writ of Amparo.

-To support the petition and the remedies prayed for, the petitioners present factual
positions diametrically opposed to the MCTC’s findings and legal reasons. Most
importantly, the petitioners maintain their claims of prior possession of the disputed
land and of intrusion into this land by the private respondents.

-To support their petition for the writ of habeas data, petitioners alleged that:

“…Similarly, a petition for a WRIT OF HABEAS DATA is prayed for so that the PNP
may release the report on the burning of the homes of the petitioners and the
acts of violence employed against them by the private respondents, furnishing the
Court and the petitioners with copy of the same;

…Petitioners apply for a WRIT OF HABEAS DATA commanding the Philippine National
Police [PNP] to produce the police report pertaining to the burning of the houses
of the petitioners in the land in dispute and likewise the investigation report if an
investigation was conducted by the PNP.”

Issue:
Whether or not the Writ of Amparo and Writ of Habeas Data must issue under the
circumstances.

Ruling:
No.

On the whole, what is clear from these statements - both sworn and unsworn - is
the overriding involvement of property issues as the petition traces its roots to
questions of physical possession of the property disputed by the private parties.
If at all, issues relating to the right to life or to liberty can hardly be discerned except
to the extent that the occurrence of past violence has been alleged. The right to
security, on the other hand, is alleged only to the extent of the threats and
harassments implied from the presence of "armed men bare to the waist" and the
alleged pointing and firing of weapons. Notably, none of the supporting affidavits
compellingly show that the threat to the rights to life, liberty and security of the
petitioners is imminent or is continuing. x x x x

Under these legal and factual situations, we are far from satisfied with the prima
facie existence of the ultimate facts that would justify the issuance of a writ of
amparo. Rather than acts of terrorism that pose a continuing threat to the persons of
the petitioners, the violent incidents alleged appear to us to be purely property-
related and focused on the disputed land. Thus, if the petitioners wish to seek
redress and hold the alleged perpetrators criminally accountable, the remedy may lie

Page 188 of 323


more in the realm of ordinary criminal prosecution rather than on the use of the
extraordinary remedy of the writ of amparo. x x x x

The writ of amparo was originally conceived as a response to the extraordinary


rise in the number of killings and enforced disappearances, and to the perceived
lack of available and effective remedies to address these extraordinary concerns. It is
intended to address violations of or threats to the rights to life, liberty or security, as
an extraordinary and independent remedy beyond those available under the prevailing
Rules, or as a remedy supplemental to these Rules. What it is not, is a writ to
protect concerns that are purely property or commercial. Neither is it a writ that
we shall issue on amorphous and uncertain grounds. x x x x

To be sure, when recourses in the ordinary course of law fail because of deficient legal
representation or the use of improper remedial measures, neither the writ of certiorari
nor that of amparo - extraordinary though they may be - will suffice to serve as a
curative substitute. The writ of amparo, particularly, should not issue when
applied for as a substitute for the appeal or certiorari process, or when it will
inordinately interfere with these processes - the situation obtaining in the
present case.

As to the writ of habeas data, petitioners’ allegations obviously lack what the Rule on
Writ of Habeas Data requires as a minimum, thus rendering the petition fatally
deficient. Specifically, we see no concrete allegations of unjustified or unlawful
violation of the right to privacy related to the right to life, liberty or security.
The petition likewise has not alleged, much less demonstrated, any need for
information under the control of police authorities other than those it has already set
forth as integral annexes. The necessity or justification for the issuance of the writ,
based on the insufficiency of previous efforts made to secure information, has not also
been shown. In sum, the prayer for the issuance of a writ of habeas data is
nothing more than the “fishing expedition” that this Court - in the course of
drafting the Rule on habeas data - had in mind in defining what the purpose of a
writ of habeas data is not. In these lights, the outright denial of the petition for the
issuance of the writ of habeas data is fully in order.

*Yusa v. Segui, G.R. No. 193652, August 5, 2014.


(The SC’s rule-making power: Writ of Amparo)

Facts:
-Christina had an amorous relationship with Marcelino and eventually became
pregnant with the latter’s child without the benefit of marriage. After getting pregnant,
Christina misled Marcelino into believing that she had an abortion when in fact she
proceeded to complete the term of her pregnancy. During this time, she intended to
have the child adopted through Sun and Moon Home for Children (Sun and Moon) to
avoid placing her family in a potentially embarrassing situation for having a second
illegitimate son.

-Christina subsequently gave birth to Baby Julian. Sun and Moon shouldered all the
hospital and medical expenses. Thereafter, Christina voluntarily surrendered Baby
Julian by way of a Deed of Voluntary Commitment to the DSWD.

-After a few months, Marcelino suffered a heart attack and died without knowing
about the birth of his son. Thereafter, during the wake, Christina disclosed to
Marcelino’s family that she and the deceased had a son that she gave up for adoption
due to financial distress and initial embarrassment. After the emotional revelation,
Marcelino’s family vowed to help her recover and raise the baby.

-In the same month of Marcelino’s death, the DSWD issued a certificate declaring
Baby Julian as “Legally Available for Adoption.” A local matching conference was
held eventually, Baby Julian was “matched” with the spouses Medina of the
Kaisahang Bahay Foundation. Supervised trial custody then commenced.

Page 189 of 323


-After six months, Christina who had changed her mind about the adoption, wrote a
letter to the DSWD asking for the suspension of Baby Julian’s adoption proceedings.
She also said she wanted her family back together.

-The DSWD then sent a Memorandum stating that the certificate declaring Baby
Julian legally available for adoption had attained finality six months ago, or three
months after Christina signed the Deed of Voluntary Commitment which terminated
her parental authority and effectively made Baby Julian a ward of the State.

-Therafter, Marcelino’s brother sent a letter to DSWD stating that a DNA testing was
scheduled at the DNA Analysis Laboratory at the University of the Philippines.
DSWD responded that it would not allow Baby Julian to undergo DNA testing.

-Christina then filed a petition for the issuance of a writ of amparo before the RTC of
Quezon City seeking to obtain custody of Baby Julian from DSWD, which the RTC
eventually dismissed without prejudice to the filing of the appropriate action in court.

-Christina then went to the SC via Rule 45.

-Christina’s contention: The life, liberty and security of Baby Julian is being violated or
threatened by the respondent DSWD officers’ enforcement of an illegal Deed of
Voluntary Commitment between her and Sun and Moon. She had been “blackmailed”
through the said Deed by the DSWD officers and Sun and Moon’s representatives into
surrendering her child thereby causing the “forced separation” of the said infant from
his mother. She likewise reiterated that DSWD officers acted beyond the scope of their
authority when they deprived her of Baby Julian’s custody.

Issue:
Whether or not a petition for a writ of amparo is the proper recourse for obtaining
parental authority and custody of a minor child.

Ruling:
No.

In the landmark case of Secretary of National Defense, et al. v. Manalo, et al., this Court
held:

The Amparo Rule was intended to address the intractable problem of “extralegal
killings” and “enforced disappearances,” its coverage, in its present form, is
confined to these two instances or to threats thereof. “Extralegal killings” are
“killings committed without due process of law, i.e., without legal safeguards or
judicial proceedings.” On the other hand, “enforced disappearances” are “attended by
the following characteristics: an arrest, detention or abduction of a person by a
government official or organized groups or private individuals acting with the direct or
indirect acquiescence of the government; the refusal of the State to disclose the fate or
whereabouts of the person concerned or a refusal to acknowledge the deprivation of
liberty which places such persons outside the protection of law. x x x x

This pronouncement on the coverage of the writ was further cemented in the latter
case of Lozada, Jr. v. Macapagal-Arroyo where this Court explicitly declared that as it
stands, the writ of amparo is confined only to cases of extrajudicial killings and
enforced disappearances, or to threats thereof. As to what constitutes "enforced
disappearance," the Court in Navia v. Pardico enumerated the elements constituting
"enforced disappearances" as the term is statutorily defined in Section 3(g) of R.A.
No. 985134 to wit:

(a) that there be an arrest, detention, abduction or any form of deprivation of liberty;
(b) that it be carried out by, or with the authorization, support or acquiescence of, the
State ora political organization;

Page 190 of 323


(c) that it be followed by the State or political organization’s refusal to acknowledge or
give information on the fate or whereabouts of the person subject of the amparo
petition; and,
(d) that the intention for such refusal is to remove subject person from the protection
of the law for a prolonged period of time.

In this case, Christina alleged that the respondent DSWD officers caused her
“enforced separation” from Baby Julian and that their action amounted to an
“enforced disappearance” within the context of the Amparo rule. Contrary to her
position, however, the respondent DSWD officers never concealed Baby Julian’s
whereabouts. In fact, Christina obtained a copy of the DSWD's Memorandum
explicitly stating that Baby Julian was in the custody of the Medina Spouses when
she filed her petition before the RTC. Besides, she even admitted in her petition for
review on certiorari that the respondent DSWD officers presented Baby Julian before
the RTC during the hearing held in the afternoon of August 5, 2010. There is
therefore, no “enforced disappearance” as used in the context of the Amparo
rule as the third and fourth elements are missing.

Christina's directly accusing the respondents of forcibly separating her from her child
and placing the latter up for adoption, supposedly without complying with the
necessary legal requisites to qualify the child for adoption, clearly indicates that she is
not searching for a lost child but asserting her parental authority over the child
and contesting custody over him. Since it is extant from the pleadings filed that
what is involved is the issue of child custody and the exercise of parental rights over a
child, who, for all intents and purposes, has been legally considered a ward of the
State, the Amparo rule cannot be properly applied.

To reiterate, the privilege of the writ of amparo is a remedy available to victims of


extra-judicial killings and enforced disappearances or threats of a similar nature,
regardless of whether the perpetrator of the unlawful act or omission is a public
official or employee or a private individual. It is envisioned basically to protect and
guarantee the right to life, liberty and security of persons, free from fears and threats
that vitiate the quality of life.

*Lee v. Ilagan, G.R. No. 203254, October 08, 2014.


(The SC’s rule-making power: Writ of Habeas Data)

Facts:
-Joy and Neri were former common law partners. One day, Joy got hold of Neri’s
digital camera and allegedly found a sex video of Neri with another woman.

-Neri denied the video and demanded Joy to return the camera, but to no avail. During
the confrontation, Neri allegedly slammed Joy’s head against a wall inside his office
and walked away. Subsequently, Joy utilized the said video as evidence in filing
various complaints against Neri, namely: (a) a criminal complaint for violation of the
“Anti-Violence Against Women and Their Children Act of 2004, before the Office of
the City Prosecutor; and (b) an administrative complaint for grave misconduct before
the Napolcom.

-Neri then filed a Petition for Issuance of the Writ of Habeas Data, alleging that
Joy’s acts of reproducing the subject video and threatening to distribute the same to
the upper echelons of the NAPOLCOM and uploading it to the internet violated not
only his right to life, liberty, security, and privacy but also that of the other woman,
and thus, the issuance of a writ of habeas data in his favor is warranted.

-Finding the petition prima facie meritorious, the RTC issued a Writ of Habeas Data,
directing Joy to appear before the court a quo, and to produce Neri’s digital camera, as
well as the negative and/or original of the subject video and copies thereof, and to file
a verified written return within five (5) working days from date of receipt thereof.

Page 191 of 323


-In her Verified Return, Joy admitted that she indeed kept the memory card of the
digital camera and reproduced the aforesaid video but averred that she only did so to
utilize the same as evidence in the cases she filed against Neri. She also admitted that
her relationship with Neri started sometime in 2003 and ended under disturbing
circumstances in 2011, and that she only happened to discover the subject video
when Neri left his camera in her condominium.

-Afrer due proceedings, the RTC granted the privilege of the writ of habeas data in
Neri’s favor, and accordingly, ordered the implementing officer to turn-over copies of
the subject video to him, and enjoined Joy from further reproducing the same.

-Joy then went to the SC via Rule 45.

Issue:
Whether or not the RTC, under the circumstances, correctly granted the privilege of
the writ of habeas data in favor of Neri.

Ruling:
No.

A.M. No. 08-1-16-SC, or the Rule on the Writ of Habeas Data (Habeas Data Rule), was
conceived as a response, given the lack of effective and available remedies, to address
the extraordinary rise in the number of killings and enforced disappearances. It was
conceptualized as a judicial remedy enforcing the right to privacy, most especially the
right to informational privacy of individuals, which is defined as “the right to control
the collection, maintenance, use, and dissemination of data about oneself.”

As defined in Section 1 of the Habeas Data Rule, the writ of habeas data now stands
as “a remedy available to any person whose right to privacy in life, liberty or security is
violated or threatened by an unlawful act or omission of a public official or employee,
or of a private individual or entity engaged in the gathering, collecting or storing of
data or information regarding the person, family, home, and correspondence of the
aggrieved party.” Thus, in order to support a petition for the issuance of such
writ, Section 6 of the Habeas Data Rule essentially requires that the petition
sufficiently alleges, among others, “the manner the right to privacy is violated or
threatened and how it affects the right to life, liberty or security of the aggrieved
party.” In other words, the petition must adequately show that there exists a
nexus between the right to privacy on the one hand, and the right to life, liberty
or security on the other. Corollarily, the allegations in the petition must be
supported by substantial evidence showing an actual or threatened violation of the
right to privacy in life, liberty or security of the victim. In this relation, it bears
pointing out that the writ of habeas data will not issue to protect purely property or
commercial concerns nor when the grounds invoked in support of the petitions
therefor are vague and doubtful.

In this case, the Court finds that Neri was not able to sufficiently allege that his right
to privacy in life, liberty or security was or would be violated through the supposed
reproduction and threatened dissemination of the subject sex video. While Neri
purports a privacy interest in the suppression of this video – which he fears would
somehow find its way to Quiapo or be uploaded in the internet for public consumption
– he failed to explain the connection between such interest and any violation of
his right to life, liberty or security. Indeed, courts cannot speculate or contrive
versions of possible transgressions. As the rules and existing jurisprudence on the
matter evoke, alleging and eventually proving the nexus between one’s privacy right to
the cogent rights to life, liberty or security are crucial in habeas data cases, so much
so that a failure on either account certainly renders a habeas data petition
dismissible, as in this case.

In fact, even discounting the insufficiency of the allegations, the petition would
equally be dismissible due to the inadequacy of the evidence presented. As the
records show, all that Neri submitted in support of his petition was his self-serving

Page 192 of 323


testimony which hardly meets the substantial evidence requirement as
prescribed by the Habeas Data Rule. This is because nothing therein would indicate
that Joy actually proceeded to commit any overt act towards the end of violating Neri’s
right to privacy in life, liberty or security. Nor would anything on record even lead a
reasonable mind to conclude that Joy was going to use the subject video in order to
achieve unlawful ends – say for instance, to spread it to the public so as to ruin Neri’s
reputation. Contrastingly, Joy even made it clear in her testimony that the only
reason why she reproduced the subject video was to legitimately utilize the same as
evidence in the criminal and administrative cases that she filed against Neri. Hence,
due to the insufficiency of the allegations as well as the glaring absence of substantial
evidence, the Court finds it proper to reverse the RTC Decision and dismiss the habeas
data petition.

*Gamboa v. Chan, G.R. No. 193636, July 24, 2012.


(The SC’s rule-making power: Writ of Habeas Data)

Facts:
-Gamboa was the Mayor of Dingras, Ilocos Norte. Chan, meanwhile, was the Officer-
in-Charge of the Provincial Investigation and Detective Management Branch of the
Ilocos Norte Police Provincial Office.

-In 2009, former President Gloria Arroyo issued Administrative Order No. 275 (A.O.
275), “Creating an Independent Commission to Address the Alleged Existence of
Private Armies in the Country.” The body, which was later on referred to as the
Zeñarosa Commission, was formed to investigate the existence of private army
groups (PAGs) in the country with a view to eliminating them before the 10 May 2010
elections and dismantling them permanently in the future. Upon the conclusion of its
investigation, the Zeñarosa Commission released and submitted to the Office of the
President a confidential report entitled “A Journey Towards H.O.P.E.: The Independent
Commission Against Private Armies’ Report to the President” (the Report).

-Gamboa alleged that the Philippine National Police in Ilocos Norte (PNP–Ilocos Norte)
conducted a series of surveillance operations against her and her aides, and
classified her as someone who keeps a PAG. Purportedly without the benefit of data
verification, PNP–Ilocos Norte forwarded the information gathered on her to the
Zeñarosa Commission, thereby causing her inclusion in the Report’s
enumeration of individuals maintaining PAGs.

-On 6 and 7 July 2010, ABS-CBN broadcasted on its evening news program the
portion of the Report naming Gamboa as one of the politicians alleged to be
maintaining a PAG. Gamboa averred that her association with a PAG also appeared on
print media. Thus, she was publicly tagged as someone who maintains a PAG on
the basis of the unverified information that the PNP-Ilocos Norte gathered and
forwarded to the Zeñarosa Commission. As a result, she claimed that her malicious or
reckless inclusion in the enumeration of personalities maintaining a PAG as published
in the Report also made her, as well as her supporters and other people identified with
her, susceptible to harassment and police surveillance operations.

-Contending that her right to privacy was violated and her reputation maligned and
destroyed, Gamboa filed a Petition for the issuance of a writ of habeas data against
respondents in their capacities as officials of the PNP-Ilocos Norte. In her Petition, she
prayed for the following reliefs: (a) destruction of the unverified reports from the
PNP-Ilocos Norte database; (b) withdrawal of all information forwarded to higher PNP
officials; (c) rectification of the damage done to her honor; (d) ordering respondents to
refrain from forwarding unverified reports against her; and (e) restraining
respondents from making baseless reports.

-After due hearing, the RTC dismissed the Petition on the ground that Gamboa failed
to prove through substantial evidence that the subject information originated from
Chan and colleagues, and that they forwarded this database to the Zeñarosa
Commission without the benefit of prior verification.

Page 193 of 323


Issue:
Whether or not, under the circumstances, the privilege of the writ of habeas data must
be granted.

Ruling:
No.

The forwarding of information by the PNP to the Zeñarosa Commission was not
an unlawful act that violated or threatened Gamboa’s right to privacy in life,
liberty or security.

The fact that the PNP released information to the Zeñarosa Commission without prior
communication to Gamboa and without affording her the opportunity to refute the
same cannot be interpreted as a violation or threat to her right to privacy since that
act is an inherent and crucial component of intelligence-gathering and investigation.
Additionally, Gamboa herself admitted that the PNP had a validation system, which
was used to update information on individuals associated with PAGs and to ensure
that the data mirrored the situation on the field. Thus, safeguards were put in place to
make sure that the information collected maintained its integrity and accuracy.

The writ of habeas data is an independent and summary remedy designed to


protect the image, privacy, honor, information, and freedom of information of an
individual, and to provide a forum to enforce one’s right to the truth and to
informational privacy. It seeks to protect a person’s right to control information
regarding oneself, particularly in instances in which such information is being
collected through unlawful means in order to achieve unlawful ends. It must be
emphasized that in order for the privilege of the writ to be granted, there must exist a
nexus between the right to privacy on the one hand, and the right to life, liberty or
security on the other. x x x x

The Constitution explicitly mandates the dismantling of private armies and


other armed groups not recognized by the duly constituted authority. It also
provides for the establishment of one police force that is national in scope and civilian
in character, and is controlled and administered by a national police commission.

Taking into account these constitutional fiats, it is clear that the issuance of A.O.
275 articulates a legitimate state aim, which is to investigate the existence of
PAGs with the ultimate objective of dismantling them permanently.

To enable the Zeñarosa Commission to achieve its goals, A.O. 275 clothed it with the
powers of an investigative body, including the power to summon witnesses, administer
oaths, take testimony or evidence relevant to the investigation and use compulsory
processes to produce documents, books, and records. A.O. 275 likewise authorized the
Zeñarosa Commission to deputize the Armed Forces of the Philippines, the National
Bureau of Investigation, the Department of Justice, the PNP, and any other law
enforcement agency to assist the commission in the performance of its functions. x x x
x

The PNP was rationally expected to forward and share intelligence regarding
PAGs with the body specifically created for the purpose of investigating the
existence of these notorious groups. Moreover, the Zeñarosa Commission was
explicitly authorized to deputize the police force in the fulfillment of the former’s
mandate, and thus had the power to request assistance from the latter.

However, to accord the right to privacy with the kind of protection established in
existing law and jurisprudence, this Court nonetheless deems it necessary to caution
these investigating entities that information-sharing must observe strict
confidentiality. Intelligence gathered must be released exclusively to the authorities
empowered to receive the relevant information. After all, inherent to the right to
privacy is the freedom from “unwarranted exploitation of one’s person or from

Page 194 of 323


intrusion into one’s private activities in such a way as to cause humiliation to a
person’s ordinary sensibilities.” x x x x

It is clear from the foregoing discussion that the state interest of dismantling PAGs
far outweighs the alleged intrusion on the private life of Gamboa, especially
when the collection and forwarding by the PNP of information against her was
pursuant to a lawful mandate. Therefore, the privilege of the writ of habeas data
must be denied.

*Vivares v. St. Theresa’s College, G.R. No. 202666, September 29, 2014.
[PROBABLE BAR QUESTION!!]
(The SC’s rule-making power: Writ of Habeas Data)

Facts:
-Julia and Julianne were graduating high school students at St. Theresa's College
(STC). Sometime in January 2012, while changing into their swimsuits for a beach
party they were about to attend, Julia and Julienne, along with several others, took
digital pictures of themselves clad only in their undergarments. These pictures were
then uploaded by their friend Angela on her Facebook profile.

-Back at the school, Escudero, a computer teacher at STC’s high school department,
learned from her students that some seniors at STC posted pictures online, depicting
themselves from the waist up, dressed only in brassieres. Escudero then asked her
students if they knew who the girls in the photos are. In turn, they readily identified
Julia, Julienne, Angela and Chloe, among others.

-Using STC’s computers, Escudero’s students logged in to their respective personal


Facebook accounts and showed her photos of the identified students, which include:
(a) Julia and Julienne drinking hard liquor and smoking cigarettes inside a bar; and
(b) Julia and Julienne along the streets of Cebu wearing articles of clothing that show
virtually the entirety of their black brassieres. What is more, Escudero’s students
claimed that there were times when access to or the availability of the identified
students’ photos was not confined to the girls’ Facebook friends, but were, in fact,
viewable by any Facebook user.

-Escudero then referred the matter to the STC’s Discipline-in-Charge, for appropriate
action. A meeting was called between STC heads and all the students involved.
Eventually, STC found the identified students to have deported themselves in a
manner proscribed by the school’s Student Handbook and barred them from joining
the commencement exercises scheduled on March 30, 2012.

-A week before graduation, Angela’s mother, Tan, filed a Petition for Injunction and
Damages before the RTC of Cebu City against STC, et al. Tan prayed that defendants
therein be enjoined from implementing the sanction that precluded Angela from
joining the commencement exercises.

-The RTC issued a temporary restraining order (TRO) allowing the students to attend
the graduation ceremony, to which STC filed a motion for reconsideration. Despite
the issuance of the TRO, STC, nevertheless, barred the sanctioned students from
participating in the graduation rites, arguing that, on the date of the commencement
exercises, its adverted motion for reconsideration on the issuance of the TRO remained
unresolved.

-Thereafter, petitioners filed before the RTC a Petition for the Issuance of a Writ of
Habeas Data.

-Contentions of the parents: (1) The privacy setting of their children’s Facebook
accounts was set at "Friends Only." They, thus, have a reasonable expectation of
privacy which must be respected; (2) STC heads ought to have known that the girls,
whose privacy has been invaded, are the victims in this case, and not the offenders;
(3) The photos accessed belong to the girls and, thus, cannot be used and

Page 195 of 323


reproduced without their consent. The act of reproducing the photos constitutes an
invasion of their children’s privacy and hence, a writ of habeas data must be
issued.

-Another contention of the minors’ parents: Their children’s disclosure was only
limited since their profiles were not open to public viewing. Therefore, according to
them, people who are not their Facebook friends, including STC heads, are barred
from accessing said post without the minors’ knowledge and consent. As
petitioners’ children testified, it was Angela who uploaded the subject photos which
were only viewable by the five of them, although who these five are do not appear on
the records.

-After due proceedings, the RTC dismissed the petition. Petitioners went to the SC via
Rule 45.

Issues:
(1) Whether or not there was indeed an actual or threatened violation of the right to
privacy in the life, liberty, or security of the minors involved;
(2) Whether or not, under the circumstances, a writ of habeas data must be issued.

Ruling:
(1) No.

Facebook extends its users an avenue to make the availability of their Facebook
activities reflect their choice as to “when and to what extent to disclose facts about
[themselves] – and to put others in the position of receiving such confidences.” Ideally,
the selected setting will be based on one’s desire to interact with others, coupled with
the opposing need to withhold certain information as well as to regulate the spreading
of his or her personal information. Needless to say, as the privacy setting becomes
more limiting, fewer Facebook users can view that user’s particular post. x x x x

Before one can have an expectation of privacy in his or her online social network
(OSN) activity, it is first necessary that said user, in this case the children of
petitioners, manifest the intention to keep certain posts private, through the
employment of measures to prevent access thereto or to limit its visibility. And
this intention can materialize in cyberspace through the utilization of the OSN’s
privacy tools. In other words, utilization of these privacy tools is the manifestation, in
cyber world, of the user’s invocation of his or her right to informational privacy.

Did the minors limit the disclosure of the photos such that the images were kept
within their zones of privacy? This determination is necessary in resolving the issue of
whether the minors carved out a zone of privacy when the photos were uploaded to
Facebook so that the images will be protected against unauthorized access and
disclosure.

Escudero stated in her affidavit that, “my students showed me some pictures of girls
clad in brassieres. These students of mine informed me that these are senior high
school students of STC, who are their friends in Facebook.” Without any evidence to
corroborate their statement that the images were visible only to the five of them, and
without their challenging Escudero’s claim that the other students were able to view
the photos, their statements are, at best, self-serving, thus deserving scant
consideration.

This only goes to show that no special means to be able to view the allegedly
private posts were ever resorted to by Escudero’s students, and that it is
reasonable to assume, therefore, that the photos were, in reality, viewable either
by (1) their Facebook friends, or (2) by the public at large.

The students cannot invoke the protection attached to the right to


informational privacy. The ensuing pronouncement in US v. Gines-Perez is most
instructive:

Page 196 of 323


A person who places a photograph on the Internet precisely intends to forsake
and renounce all privacy rights to such imagery, particularly under
circumstances such as here, where the Defendant did not employ protective
measures or devices that would have controlled access to the Web page or the
photograph itself.

It is well to emphasize at this point that setting a post’s or profile detail’s privacy to
“Friends” is no assurance that it can no longer be viewed by another user who is not
Facebook friends with the source of the content. The user’s own Facebook friend can
share said content or tag his or her own Facebook friend thereto, regardless of
whether the user tagged by the latter is Facebook friends or not with the former. Also,
when the post is shared or when a person is tagged, the respective Facebook friends of
the person who shared the post or who was tagged can view the post, the privacy
setting of which was set at “Friends.”

Even assuming that the photos in issue are visible only to the sanctioned students’
Facebook friends, STC heads can hardly be taken to task for the perceived privacy
invasion since it was the minors’ Facebook friends who showed the pictures to
Escudero. They did not resort to any unlawful means of gathering the
information as it was voluntarily given to them by persons who had legitimate
access to the said posts. Clearly, the fault, if any, lies with the friends of the minors.
Curiously enough, however, neither the minors nor their parents imputed any
violation of privacy against the students who showed the images to Escudero.

(2) No.

There was no actual or threatened violation of the right to privacy in the life, liberty, or
security of the minors involved. (See above ruling.)

[NOTE: The writ of habeas data is not only confined to cases of extralegal killings
and enforced disappearances. The Writ of Habeas Data was not enacted solely for
the purpose of complementing the Writ of Amparo in cases of extralegal killings
and enforced disappearances.]

[NOTE: Respondents’ contention that the habeas data writ may not issue against STC,
it not being an entity engaged in the gathering, collecting or storing of data or
information regarding the person, family, home and correspondence of the aggrieved
party, while valid to a point, is, nonetheless, erroneous.

To be sure, nothing in the Rule would suggest that the habeas data protection shall be
available only against abuses of a person or entity engaged in the business of
gathering, storing, and collecting of data.

The provision, when taken in its proper context, as a whole, irresistibly conveys the
idea that habeas data is a protection against unlawful acts or omissions of public
officials and of private individuals or entities engaged in gathering, collecting, or
storing data about the aggrieved party and his or her correspondences, or about his or
her family. Such individual or entity need not be in the business of collecting or
storing data.

To “engage” in something is different from undertaking a business endeavor. To


“engage” means “to do or take part in something.” It does not necessarily mean
that the activity must be done in pursuit of a business. What matters is that the
person or entity must be gathering, collecting or storing said data or information
about the aggrieved party or his or her family. Whether such undertaking carries
the element of regularity, as when one pursues a business, and is in the nature of a
personal endeavor, for any other reason or even for no reason at all, is immaterial and
such will not prevent the writ from getting to said person or entity.]

Page 197 of 323


*MMDA v. Concerned Residents of Manila Bay, G.R. Nos. 171947-48, December 18,
2008.
(The SC’s rule-making power: Procedure for Environmental Cases)

Facts:
-In 1999, Concerned Residents of Manila Bay (CRMB) filed a complaint before the RTC
in Imus, Cavite against several government agencies, including DENR, MMDA, DepEd,
DA, DOH, DPWH, DBM and DILG, for the cleanup, rehabilitation, and protection of
the Manila Bay. The complaint alleged that the water quality of the Manila Bay had
fallen way below the allowable standards set by law, specifically P.D.1152 or the
Philippine Environment Code, and prayed that MMDA, et al. be ordered to clean the
Manila Bay and submit to the RTC a concerted concrete plan of action for the purpose.

-CRMB anchored their cause of action on Secs. 17 and 20 of the Environment Code, to
wit:

Section 17. Upgrading of Water Quality.––Where the quality of water has deteriorated to a degree where
its state will adversely affect its best usage, the government agencies concerned shall take such measures
as may be necessary to upgrade the quality of such water to meet the prescribed water quality standards.
Section 20. Clean-up Operations.––It shall be the responsibility of the polluter to contain, remove and
clean-up water pollution incidents at his own expense. In case of his failure to do so, the government
agencies concerned shall undertake containment, removal and clean-up operations and expenses
incurred in said operations shall be charged against the persons and/or entities responsible for such
pollution.

-The RTC rendered a decision in favor of CRMB, directing DENR et al. within six (6)
months from receipt of the decision, to act and perform their respective duties
(specified for every agency and relating directly to their functions) by devising a
consolidated, coordinated and concerted scheme of action for the rehabilitation and
restoration of the bay.

-Contention of DENR, et al: (1) The pertinent provisions of the Environment Code
relate only to the cleaning of specific pollution incidents and do not cover cleaning in
general; (2) The duty to take measures and maintain adequate solid waste and liquid
disposal systems necessarily involves policy evaluation and the exercise of judgment
on the part of the agency concerned; hence, cannot be compelled by mandamus.

Issues:
(1) Whether or not petitioners can be compelled by mandamus to clean up and
rehabilitate the Manila Bay;
(2) Whether or not Sections 17 and 20 of PD 1152 under the headings, “Upgrading of
Water Quality and Clean-up Operations,” encompass a cleanup in general or they are
limited only to the cleanup of specific pollution incidents.

Ruling:
(1) Yes.

Generally, the writ of mandamus lies to require the execution of a ministerial


duty. A ministerial duty is one that “requires neither the exercise of official discretion
nor judgment.”

Petitioners’ obligation to perform their duties as defined by law, on one hand, and how
they are to carry out such duties, on the other, are two different concepts. While the
implementation of the MMDA’s mandated tasks may entail a decision-making
process, the enforcement of the law or the very act of doing what the law exacts
to be done is ministerial in nature and may be compelled by mandamus. x x x x

A perusal of other petitioners’ respective charters or like enabling statutes and


pertinent laws would yield this conclusion: these government agencies are enjoined, as
a matter of statutory obligation, to perform certain functions relating directly or
indirectly to the cleanup, rehabilitation, protection, and preservation of the Manila
Bay. They are precluded from choosing not to perform these duties.

Page 198 of 323


(2) Sections 17 and 20 of PD 1152 include cleaning in general.

Sec. 17 does not in any way state that the government agencies concerned ought to confine themselves to
the containment, removal, and cleaning operations when a specific pollution incident occurs. On the
contrary, Sec. 17 requires them to act even in the absence of a specific pollution incident, as long as
water quality “has deteriorated to a degree where its state will adversely affect its best usage.”

A perusal of Sec. 20 of the Environment Code, as couched, indicates that it is properly


applicable to a specific situation in which the pollution is caused by polluters who fail
to clean up the mess they left behind. In such instance, the concerned government
agencies shall undertake the cleanup work for the polluters’ account.

PD 1152 aims to introduce a comprehensive program of environmental protection and


management. This is better served by making Secs. 17 & 20 of general application
rather than limiting them to specific pollution incidents.

[NOTE: (WRIT OF CONTINUING MANDAMUS)

The government agencies did not file any motion for reconsideration and the Decision
became final in January 2009.

In the absence of specific completion periods, the Committee recommended that time
frames be set for the agencies to perform their assigned tasks. This may be viewed as
an encroachment over the powers and functions of the Executive Branch headed by
the President of the Philippines.

The issuance of subsequent resolutions by the Court is simply an exercise of judicial


power under Art. VIII of the Constitution, because the execution of the Decision is but
an integral part of the adjudicative function of the Court. None of the agencies ever
questioned the power of the Court to implement the December 18, 2008 Decision nor
has any of them raised the alleged encroachment by the Court over executive
functions.

It is clear that the final judgment includes not only what appears upon its face
to have been so adjudged but also those matters "actually and necessarily
included therein or necessary thereto." Certainly, any activity that is needed to fully
implement a final judgment is necessarily encompassed by said judgment.

With the final and executory judgment in MMDA, the writ of continuing mandamus
issued in MMDA means that until petitioner-agencies have shown full compliance
with the Court’s orders, the Court exercises continuing jurisdiction over them
until full execution of the judgment.]

*Echegaray v. Secretary of Justice, G.R. No. 132601, January 19, 1999.


(The SC’s rule-making power vis-à-vis power of Congress)

Facts:
-Echegaray filed his Very Urgent Motion for Issuance of TRO, invoking several
grounds, to wit: (a) that his execution has been set on January 4, the first working day
of 1999; (b) that members of Congress had either sought for his executive clemency
and/or review or repeal of the law authorizing capital punishment; (b.1) that
Senator Aquilino Pimentel's resolution asking that clemency be granted to the
petitioner and that capital punishment be reviewed has been concurred by thirteen
(13) other senators; (b.2) Senate President Marcelo Fernan and Senator Miriam S.
Defensor have publicly declared they would seek a review of the death penalty law;
(b.3) Senator Paul Roco has also sought the repeal of capital punishment, and (b.4)
Congressman Salacrib Baterina, Jr., and thirty five (35) other congressmen are
demanding review of the same law.

-The SC issued an order temporarily restraining the execution of Echegaray “until


June 15, 1999, coeval with the constitutional duration of the present regular session

Page 199 of 323


of Congress, unless it sooner becomes certain that no repeal or modification of the law
is going to be made.”

-The Secretary of Justice filed an Urgent Motion for Reconsideration, arguing that:
(1) The Decision in this case having become final and executory, its execution enters
the exclusive ambit of authority of the executive authority. The issuance of the TRO
may be construed as trenching on that sphere of executive authority; (2) The issuance
of the temporary restraining order creates dangerous precedent as there will never be
an end to litigation because there is always a possibility that Congress may repeal a
law; (3) By granting the TRO, the SC has in effect granted reprieve which is an
executive function.

Issue:
Whether or not the SC has the authority to temporarily restrain the execution of a
final judgment, specifically, the execution of lethal injection on Echegaray.

Ruling:
Yes.

The Court is not changing even a comma of its final Decision.

The rule on finality of judgment cannot divest this Court of its jurisdiction to execute
and enforce the same judgment.

The power to control the execution of its decision is an essential aspect of


jurisdiction. It cannot be the subject of substantial subtraction for our
Constitution vests the entirety of judicial power in one Supreme Court and in
such lower courts as may be established by law. To be sure, the important part of a
litigation, whether civil or criminal, is the process of execution of decisions where
supervening events may change the circumstance of the parties and compel courts to
intervene and adjust the rights of the litigants to prevent unfairness. It is because of
these unforeseen, supervening contingencies that courts have been conceded the
inherent and necessary power of control of its processes and orders to make them
conformable to law and justice. x x x x

The provision stating that, “the President may grant reprieves, commutations,
and pardons, and remit fines and forfeitures after conviction by final judgment”
cannot be interpreted as denying the power of courts to control the enforcement
of their decisions after their finality. In truth, an accused who has been convicted
by final judgment still possesses collateral rights and these rights can be claimed in
the appropriate courts. For instance, a death convict who become insane after his final
conviction cannot be executed while in a state of insanity. x x x x The suspension of
such a death sentence is undisputably an exercise of judicial power. It is not a
usurpation of the presidential power of reprieve though its effects is the same —
the temporary suspension of the execution of the death convict.

In the same vein, it cannot be denied that Congress can at any time amend R.A. No.
7659 by reducing the penalty of death to life imprisonment. The effect of such an
amendment is like that of commutation of sentence. But by no stretch of the
imagination can the exercise by Congress of its plenary power to amend laws be
considered as a violation of the power of the President to commute final sentences of
conviction. The powers of the Executive, the Legislative and the Judiciary to save
the life of a death convict do not exclude each other for the simple reason that
there is no higher right than the right to life.

Likewise, the SC hardly had five (5) hours to resolve petitioner's motion as he was due
to be executed at 3 p.m. the same day the special session was held. The extreme
caution taken by the Court was compelled, among others, by the fear that any error of
the Court in not stopping the execution of the petitioner will preclude any further relief
for all rights stop at the graveyard.

Page 200 of 323


*Prudential Bank v. Castro, A.M. No. 2756, March 15, 1988.
(Consultations/Decisions of the SC)

Facts:
-Atty. Benjamin Grecia was disbarred by the SC.

-Grecia filed an MR, arguing that the decision is void considering that the same was
violative of the 1987 Constitution due to lack of certification by the Chief Justice
that the conclusions of the Court were reached in consultation before the case was
assigned to a member for the writing of the opinion of the Court.

Issue:
Whether or not the decision is void due to lack of the aforementioned certification.

Ruling:
No.

The certification requirement refers to decisions in judicial, not administrative


cases. From the very beginning, resolutions/decisions of the Court in administrative
cases have not been accompanied by any formal certification. In fact, such a
certification would be a superfluity in administrative cases, which by their very
nature, have to be deliberated upon considering the collegiate composition of
this Court.

But even if such a certification were required, it is beyond doubt that the conclusions
of the Court in its decision were arrived at after consultation and deliberation. The
signatures of the members who actually took part in the deliberations and voted attest
to that. Besides, being a per curiam decision, or an opinion of the Court as a whole,
there is no ponente although any member of the Court may be assigned to write the
draft. In such cases, a formal certification is obviously not required.

*Solid Homes, Inc. v. Laserna, G.R. No. 166051, April 8, 2008.


(Decisions of the SC: Memorandum Decisions)

Facts:
-The dispute stemmed from the contract to sell between buyers and developer of
subdivision lots. The buyers filed a complaint against Solid Homes before the Housing
and Land Use Regulatory Board (HLURB).

-Solid Homes was unsatisfied with the Decision of the HLURB Board of
Commissioners, and hence, it appealed the same before the Office of the President.

-After evaluating the established facts and pieces of evidence on record, the Office of
the President rendered a one-page Decision affirming in toto the Decision of the
HLURB Board of Commissioners. In rendering its Decision, the Office of the President
merely adopted by reference the findings of fact and conclusions of law
contained in the Decision of the HLURB Board of Commissioners.

-Solid Homes then went to the CA via Rule 43; then to the SC via Rule 45.

-Contention of Solid Homes: the Decision of the Office of the President, as affirmed
by the Court of Appeals, which merely adopted by reference the Decision of the
HLURB Board of Commissioners, without a recitation of the facts and law on which it
was based, runs afoul of the mandate of Section 14, Article VIII of the 1987 Philippine
Constitution which provides that: “No decision shall be rendered by any court without
expressing therein clearly and distinctly the facts and law on which it is based.” The
Office of the President, being a government agency, should have adhered to this
principle.

Issue:

Page 201 of 323


Whether or not the decision by the Office of the President violates Section 14, Article
VIII of the Constitution.

Ruling:
No.

The constitutional mandate that, “no decision shall be rendered by any court without
expressing therein clearly and distinctly the facts and the law on which it is based,”
does not preclude the validity of “memorandum decisions,” which adopt by
reference the findings of fact and conclusions of law contained in the decisions
of inferior tribunals. In fact, in Yao v. Court of Appeals, this Court has sanctioned the
use of “memorandum decisions,” a species of succinctly written decisions by appellate
courts in accordance with the provisions of Section 40, B.P. Blg. 129, as amended, on
the grounds of expediency, practicality, convenience and docket status of our
courts. This Court likewise declared that “memorandum decisions” comply with the
constitutional mandate. x x x x

In Francisco v. Permskul, this Court similarly held that the following memorandum
decision of the Regional Trial Court (RTC) of Makati City did not transgress the
requirements of Section 14, Article VIII of the 1997 Philippine Constitution:

“MEMORANDUM DECISION

After a careful perusal, evaluation and study of the records of this case, this Court
hereby adopts by reference the findings of fact and conclusions of law contained in the
decision of the Metropolitan Trial Court of Makati, Metro Manila, Branch 63 and finds
that there is no cogent reason to disturb the same.

"WHEREFORE, judgment appealed from is hereby affirmed in toto.”

Hence, incorporation by reference is allowed if only to avoid the cumbersome


reproduction of the decision of the lower courts, or portions thereof, in the
decision of the higher court.

However, also in Permskul, this Court laid down the conditions for the validity of
memorandum decisions, to wit:

“The memorandum decision, to be valid, cannot incorporate the findings of fact and
the conclusions of law of the lower court only by remote reference, which is to say that
the challenged decision is not easily and immediately available to the person reading
the memorandum decision. For the incorporation by reference to be allowed, it must
provide for direct access to the facts and the law being adopted, which must be
contained in a statement attached to the said decision. In other words, the
memorandum decision authorized under Section 40 of B.P. Blg. 129 should actually
embody the findings of fact and conclusions of law of the lower court in an annex
attached to and made an indispensable part of the decision.

It is expected that this requirement will allay the suspicion that no study was made of
the decision of the lower court and that its decision was merely affirmed without a
proper examination of the facts and the law on which it is based. The proximity at
least of the annexed statement should suggest that such an examination has been
undertaken. It is, of course, also understood that the decision being adopted should,
to begin with, comply with Article VIII, Section 14 as no amount of incorporation or
adoption will rectify its violation.

The Court finds necessary to emphasize that the memorandum decision should be
sparingly used lest it become an addictive excuse for judicial sloth. It is an additional
condition for the validity that this kind of decision may be resorted to only in cases
where the facts are in the main accepted by both parties and easily determinable by
the judge and there are no doctrinal complications involved that will require an
extended discussion of the laws involved. The memorandum decision may be

Page 202 of 323


employed in simple litigations only, such as ordinary collection cases, where the
appeal is obviously groundless and deserves no more than the time needed to dismiss
it.

xxxx

Henceforth, all memorandum decisions shall comply with the requirements herein set
forth both as to the form prescribed and the occasions when they may be rendered.
Any deviation will summon the strict enforcement of Article VIII, Section 14 of the
Constitution and strike down the flawed judgment as a lawless disobedience.”

*Dizon v. Judge Lopez, A.M. No. RTJ-96-1338, September 5, 1997.


(Decisions of the SC: Sin Perjuicio Judgment)

Facts:
-On April 22, 1993, judgment was rendered, convicting Dizon of falsification of private
document. The promulgation of the judgment consisted of reading the dispositive
portion of the decision sentencing him to imprisonment, without serving a copy of
the decision on him.

-The accused and his counsel were told to return in a few days for their copy of the
decision, but although petitioner and his father by turns went to the court to obtain a
copy of the decision they were not able to do so. Dizon then filed a partial motion for
reconsideration on May 5, 1993, expressly reserving his light to submit a more
elaborate one upon receipt of the decision.

-The hearing of the motion for reconsideration was scheduled on May 12, 1993, but
the case was not called as complainant’s counsel was told that the decision had not
yet been finished.

-On November 29, 1994, complainant filed an “Omnibus Motion to Annul


Promulgation of Sentence and to Dismiss” the case. On December 16, 1994, the
date set for hearing the motion, complainant was served a copy of the decision, dated
April 22, 1993.

-Judge Lopez’s contention: On April 22, 1993, when the judgment was promulgated
with the reading of the dispositive portion, her decision was already prepared,
although to prevent leakage in the process of preparing it, she withheld its dispositive
portion until the day of its promulgation. Respondent judge states that after the
dispositive portion had been read to complainant, respondent gave it to Ma. Cleotilde
Paulo (Social Worker II, presently OIC of Branch 109) for typing and incorporation into
the text of the decision. The delay in furnishing complainant with a copy of the
decision was unintentional.

Issue:
Whether or not Judge Lopez violated Art. VIII, §15(1) of the Constitution which
provides that all cases or matters filed after the effectivity of this Constitution must be
decided or resolved within twenty-four months from date of submission for the
Supreme Court, and, unless reduced by the Supreme Court, twelve months for all
lower collegiate courts and three months for all other lower courts.

Ruling:
Yes.

Although respondent judge promulgated her decision within three months of the
submission of the case for decision, the fact is that only the dispositive portion was
read at such promulgation.

Rule 120 of the Rules on Criminal Procedure provides:

Page 203 of 323


§1. Judgment defined. — The term judgment as used in this Rule means the adjudication by the
court that the accused is guilty or is not guilty of the offense charged, and the imposition of the proper
penalty and civil liability provided for by law on the accused.

§2. Form and contents of judgment. — The judgment must be written in the official language,
personally and directly prepared by the judge and signed by him and shall contain clearly and distinctly a
statement of the facts proved or admitted by the accused and the law upon which the judgment is based.

§. Promulgation of judgment. — The judgment is promulgated by reading the same in the presence
of the accused and any judge of the court in which it was rendered. However, if the conviction is for a
light offense, the judgment may be pronounced in the presence of his counsel or representative. When the
judge is absent or outside of the province or city, the judgment may be promulgated by the clerk of court.

It is clear that merely reading the dispositive portion of the decision to the
accused is not sufficient. It is the judgment that must be read to him, stating
the facts and the law on which such judgment is based. Since this was done only
on December 16, 1994 when a copy of the complete decision was served on
complainant, it is obvious that the respondent failed to render her decision
within three months as required by Art. VIII, §15 of the Constitution.

What respondent did in this case was to render what is known as a “sin
perjuicio” judgment, which is a judgment without a statement of the facts in
support of its conclusion to be later supplemented by the final judgment.

*Sison-Barias v. Judge Rubia,


(SC’s power to discipline judges)

Facts:
-Judge Rubia agreed to have a dinner meeting with a party (Barias) in a case pending
before his sala. He allegedly asked inappropriate questions, and thereafter, Judge
Rubia showed manifest partiy in favor of Barias’ opponent.

-After 8 months, Barias complainant filed a complaint with the Office of the Court
Administrator charging Judge Rubia with conduct unbecoming of a judge, partiality,
gross ignorance of the law or procedure, incompetence, and gross misconduct.

Issue:
Whether or not the eight-month delay in filing of administrative complaint is a defense
in an administrative case.

Ruling:
No.

Prescription does not apply to administrative cases. The eight-month delay in


the filing of the administrative complaint, hence, is of no consequence.

To dismiss the commission of the offense based on this eight-month period is to ignore
the distinct and tangible possibility that the offense was actually committed. The
commission of the offense is not contingent on the period of revelation or
disclosure. To dismiss the complaint on this ground is tantamount to attaching
a period of prescription to the offense, which does not apply in administrative
charges. x x x x

Delay in filing an administrative complaint should not be construed as basis to


question its veracity or credibility. There are considerations that a litigant must
think about before filing an administrative case against judges and court personnel.
This is more so for lawyers where the possibility of appearing before the judge where
an administrative complaint has been filed is high.

Here, respondent Judge Rubia presided over three cases that involved complainant
and her late husband’s estate. He wielded an unmistakable amount of control over the
proceedings.

Page 204 of 323


Filing an administrative case against respondents is a time-consuming ordeal,
and it would require additional time and resources that litigants would rather
not expend in the interest of preserving their rights in the suit. Complainant
might have decided to tread with caution so as not to incur the ire of respondent
Judge Rubia for fear of the reprisal that could take place after the filing of an
administrative complaint.

Judges and court personnel wield extraordinary control over court proceedings of
cases filed. Thus, litigants are always cautious in filing administrative cases against
judges and court personnel.

In any case, administrative offenses, including those committed by members of the


bench and bar, are not subject to a fixed period within which they must be reported. x
xxx

The interval between the time when the offense was committed and the time when the
offense was officially reported cannot serve as a basis to doubt the veracity of
complainant’s allegations. This court’s mandate to discipline members of the judiciary
and its personnel is implemented by pertinent rules and statutes. Judges are
disciplined based on whether their actions violated the New Code of Judicial Conduct.
No rule for administrative discipline mandates a period within which a complaint must
be filed after the commission or discovery of the offense. The gravity of an
administrative offense cannot be diminished by a delay in the filing of a complaint.

XI. Constitutional Commissions

*CSC v. DBM, G.R. No. 158791, July 22, 2005.


(CSC; Fiscal Autonomy; Automatic Release; Revenue Shortfall)

Facts:
-The amount of ₱215,270,000.00 was appropriated for the Central Office of the Civil
Service Commission (petitioner) by the General Appropriations Act (GAA) of 2002,
while the total allocations for the same Office, if all sources of funds are considered,
amount to ₱285,660,790.44.

-The CSC complains, however, that the total fund releases by the Department of
Budget and Management (respondent) to its Central Office during the fiscal year 2002
was only ₱279,853,398.14, thereby leaving an unreleased balance of ₱5,807,392.30.

-This balance was intentionally withheld by respondent on the basis of its “no report,
no release” policy whereby allocations for agencies are withheld pending their
submission of the documents mentioned in Sections 3.8 to 3.10 and Section 7.0 of
National Budget Circular No. 478 on Guidelines on the Release of the FY 2002 Funds,
such as Quarterly Report of Actual Income, Quarterly Financial Report of Operations,
Monthly Statement of Charges to Accounts Payable, etc.

-The CSC then went to the SC via a petition for mandamus to compel the DBM to
release the balance of its budget for fiscal year 2002.

-DBM’s contention: The delay in releasing the balance of petitioner’s budget was not
on account of any failure on petitioner’s part to submit the required reports; rather, it
was due to a shortfall in revenues.

Issues:
(1) Whether or not the “no report, no release” policy may not be validly enforced
against the CSC and other Constitutional Commissions.
(2) Whether or not the alleged shortfall in revenues justifies the withholding of funds
from the CSC.

Ruling:

Page 205 of 323


(1) No.

That the “no report, no release” policy may not be validly enforced against
offices vested with fiscal autonomy is not disputed. Indeed, such policy cannot be
enforced against offices possessing fiscal autonomy without violating Article IX (A),
Section 5 of the Constitution which provides:

Sec. 5. The Commission shall enjoy fiscal autonomy. Their approved appropriations shall be
automatically and regularly released.

In Province of Batangas v. Romulo, this Court, in construing the phrase "automatic


release" in Section 6, Article X of the Constitution reading:

Section 6. Local government units shall have a just share, as determined by law, in the national taxes
which shall be automatically released to them,

held:

Webster’s Third New International Dictionary defines "automatic" as "involuntary


either wholly or to a major extent so that any activity of the will is largely negligible; of
a reflex nature; without volition; mechanical; like or suggestive of an automaton."
Further, the word "automatically" is defined as "in an automatic manner: without
thought or conscious intention." Being "automatic," thus, connotes something
mechanical, spontaneous and perfunctory. As such the LGUs are not required to
perform any act to receive the "just share" accruing to them from the national coffers.
x x x"

By parity of construction, “automatic release” of approved annual appropriations


to petitioner, a constitutional commission which is vested with fiscal autonomy,
should thus be construed to mean that no condition to fund releases to it may
be imposed. This conclusion is consistent with the above-cited June 3, 1993
Resolution of this Court which effectively prohibited the enforcement of a "no report,
no release" policy against the Judiciary which has also been granted fiscal autonomy
by the Constitution.

(2) No.

In the first place, the alleged shortfall is totally unsubstantiated. In the second place,
even assuming that there was indeed such a shortfall, that does not justify non-
compliance with the mandate of above-quoted Article IX (A), Section 5 of the
Constitution.

Asturias Sugar Central, Inc. v. Commissioner of Customs teaches that “an


interpretation should, if possible, be avoided under which a statute or provision
being construed is defeated, or as otherwise expressed, nullified, destroyed,
emasculated, repealed, explained away, or rendered insignificant, meaningless,
inoperative, or nugatory."

If respondent’s theory were adopted, then the constitutional mandate to


automatically and regularly release approved appropriations would be suspended
every year, or even every month that there is a shortfall in revenues, thereby
emasculating to a significant degree, if not rendering insignificant altogether,
such mandate.

Furthermore, the Constitution grants the enjoyment of fiscal autonomy only to the
Judiciary, the Constitutional Commissions of which petitioner is one, and the
Ombudsman. To hold that petitioner may be subjected to withholding or
reduction of funds in the event of a revenue shortfall would, to that extent, place
petitioner and the other entities vested with fiscal autonomy on equal footing
with all others which are not granted the same autonomy, thereby reducing to
naught the distinction established by the Constitution.

Page 206 of 323


The agencies which the Constitution has vested with fiscal autonomy should
thus be given priority in the release of their approved appropriations over all
other agencies not similarly vested when there is a revenue shortfall.

*CHREA v. CHR, G.R. No. 155336, November 25, 2004.


(CHR; Fiscal Autonomy; Reclassification of Personnel Positions; Salary Standardization
Law)

Facts:
-On 19 October 1998, CHR issued Resolution No. A98-0555 providing for the
upgrading or raising of salary grades of certain positions in the Commission.

-To support the implementation of such scheme, the CHR, in the same resolution,
authorized the augmentation of a commensurate amount generated from savings
under Personnel Services.

-The CHR forwarded said staffing modification and upgrading scheme to the DBM with
a request for its approval, but the then DBM secretary Benjamin Diokno denied the
request.

-In light of the DBM's disapproval of the proposed personnel modification scheme, the
CSC-National Capital Region Office, through a memorandum, recommended to the
CSC-Central Office that the subject appointments be rejected owing to the DBM's
disapproval of the plantilla reclassification.

-Meanwhile, the officers of petitioner CHREA, in representation of the rank and file
employees of the CHR, requested the CSC-Central Office to affirm the recommendation
of the CSC-Regional Office.

-However, the CSC-Central Office denied CHREA's request and reversed the
recommendation of the CSC-Regional Office that the upgrading scheme be censured.

-CHREA then went to the CA via Rule 43, then to the SC via Rule 45.

-CHR’s contention: The CHR has the authority to cause the upgrading,
reclassification, plantilla creation, and collapsing scheme without the approval of the
DBM because CHR enjoys fiscal autonomy.

Issues:
(1) Whether or not the CHR can validly implement an upgrading, reclassification,
creation, and collapsing of plantilla positions in the Commission without the prior
approval of the Department of Budget and Management;

(2) Whether or not the CHR enjoys fiscal autonomy.

Ruling:
(1) No.

The approval of the DBM must first be sought prior to implementation of any
reclassification or upgrading of positions in government. This is consonant to the
mandate of the DBM under the Revised Administrative Code of 1987, Section 3,
Chapter 1, Title XVII, to wit:

“SEC. 3. Powers and Functions. – The Department of Budget and Management shall assist the President
in the preparation of a national resources and expenditures budget, preparation, execution and control of
the National Budget, preparation and maintenance of accounting systems essential to the budgetary
process, achievement of more economy and efficiency in the management of government operations,
administration of compensation and position classification systems, assessment of organizational
effectiveness and review and evaluation of legislative proposals having budgetary or organizational
implications.”

Page 207 of 323


Irrefragably, it is within the turf of the DBM Secretary to disallow the upgrading,
reclassification, and creation of additional plantilla positions in the CHR based
on its finding that such scheme lacks legal justification. x x x x

Rep. Act No. 6758, An Act Prescribing a Revised Compensation and Position
Classification System in the Government and For Other Purposes, or the Salary
Standardization Law, dated 01 July 1989, which provides in Sections 2 and 4 thereof
that it is the DBM that shall establish and administer a unified Compensation
and Position Classification System. Thus:

SEC. 2. Statement of Policy. -- It is hereby declared the policy of the State to provide equal pay for
substantially equal work and to base differences in pay upon substantive differences in duties and
responsibilities, and qualification requirements of the positions. In determining rates of pay, due regard
shall be given to, among others, prevailing rates in the private sector for comparable work. For this
purpose, the Department of Budget and Management (DBM) is hereby directed to establish and
administer a unified Compensation and Position Classification System, hereinafter referred to as the
System as provided for in Presidential Decree No. 985, as amended, that shall be applied for all
government entities, as mandated by the Constitution. (Emphasis supplied.)

SEC. 4. Coverage. – The Compensation and Position Classification System herein provided shall
apply to all positions, appointive or elective, on full or part-time basis, now existing or hereafter
created in the government, including government-owned or controlled corporations and government
financial institutions.

The term “government” refers to the Executive, the Legislative and the Judicial Branches and the
Constitutional Commissions and shall include all, but shall not be limited to, departments,
bureaus, offices, boards, commissions, courts, tribunals, councils, authorities, administrations,
centers, institutes, state colleges and universities, local government units, and the armed forces.
The term "government-owned or controlled corporations and financial institutions" shall include all
corporations and financial institutions owned or controlled by the National Government, whether such
corporations and financial institutions perform governmental or proprietary functions. (Emphasis
supplied.)

The disputation of the Court of Appeals that the CHR is exempt from the long
arm of the Salary Standardization Law is flawed considering that the coverage
thereof, as defined above, encompasses the entire gamut of government offices,
without qualification. x x x x

This power to “administer” is not purely ministerial in character as erroneously


held by the Court of Appeals. The word to administer means to control or regulate in
behalf of others; to direct or superintend the execution, application or conduct of; and
to manage or conduct public affairs, as to administer the government of the state.

(2) No.

Article IX of the Constitution states in no uncertain terms that only the CSC, the
Commission on Elections, and the Commission on Audit shall be tagged as
Constitutional Commissions with the appurtenant right to fiscal autonomy. Thus:

“Sec. 1. The Constitutional Commissions, which shall be independent, are the Civil Service Commission,
the Commission on Elections, and the Commission on Audit.

Sec. 5. The Commission shall enjoy fiscal autonomy. Their approved annual appropriations shall be
automatically and regularly released.”

Along the same vein, the Administrative Code, in Chapter 5, Sections 24 and 26 of
Book II on Distribution of Powers of Government, the constitutional commissions shall
include only the Civil Service Commission, the Commission on Elections, and the
Commission on Audit, which are granted independence and fiscal autonomy. In
contrast, Chapter 5, Section 29 thereof, is silent on the grant of similar powers to
the other bodies including the CHR. Thus:

“SEC. 24. Constitutional Commissions. – The Constitutional Commissions, which shall be independent,
are the Civil Service Commission, the Commission on Elections, and the Commission on Audit.

SEC. 26. Fiscal Autonomy. – The Constitutional Commissions shall enjoy fiscal autonomy. The approved
annual appropriations shall be automatically and regularly released.

Page 208 of 323


SEC. 29. Other Bodies. – There shall be in accordance with the Constitution, an Office of the
Ombudsman, a Commission on Human Rights, and independent central monetary authority, and a
national police commission. Likewise, as provided in the Constitution, Congress may establish an
independent economic and planning agency.”

From the 1987 Constitution and the Administrative Code, it is abundantly clear
that the CHR is not among the class of Constitutional Commissions. As expressed
in the oft-repeated maxim expressio unius est exclusio alterius, the express mention
of one person, thing, act or consequence excludes all others. Stated otherwise,
expressium facit cessare tacitum – what is expressed puts an end to what is implied.

Nor is there any legal basis to support the contention that the CHR enjoys fiscal
autonomy. In essence, fiscal autonomy entails freedom from outside control and
limitations, other than those provided by law. It is the freedom to allocate and
utilize funds granted by law, in accordance with law, and pursuant to the wisdom and
dispatch its needs may require from time to time. In Blaquera v. Alcala and Bengzon v.
Drilon, it is understood that it is only the Judiciary, the Civil Service Commission,
the Commission on Audit, the Commission on Elections, and the Office of the
Ombudsman, which enjoy fiscal autonomy.

Neither does the fact that the CHR was admitted as a member by the Constitutional
Fiscal Autonomy Group (CFAG) ipso facto clothed it with fiscal autonomy. Fiscal
autonomy is a constitutional grant, not a tag obtainable by membership.

All told, the CHR, although admittedly a constitutional creation is, nonetheless,
not included in the genus of offices accorded fiscal autonomy by constitutional
or legislative fiat.

Even assuming en arguendo that the CHR enjoys fiscal autonomy, we share the
stance of the DBM that the grant of fiscal autonomy notwithstanding, all
government offices must, all the same, kowtow to the Salary Standardization
Law. We are of the same mind with the DBM on its standpoint, thus-

Being a member of the fiscal autonomy group does not vest the agency with the
authority to reclassify, upgrade, and create positions without approval of the
DBM. While the members of the Group are authorized to formulate and implement the
organizational structures of their respective offices and determine the compensation of
their personnel, such authority is not absolute and must be exercised within the
parameters of the Unified Position Classification and Compensation System
established under RA 6758 more popularly known as the Compensation
Standardization Law. x x x x

The Salary Standardization Law has gained impetus in addressing one of the basic
causes of discontent of many civil servants.27 For this purpose, Congress has
delegated to the DBM the power to administer the Salary Standardization Law
and to ensure that the spirit behind it is observed. This power is part of the system
of checks and balances or system of restraints in our government. The DBM's exercise
of such authority is not in itself an arrogation inasmuch as it is pursuant to the
paramount law of the land, the Salary Standardization Law and the Administrative
Code. x x x x

The courts will not interfere in matters which are addressed to the sound
discretion of government agencies entrusted with the regulation of activities
coming under the special technical knowledge and training of such agencies.

To be sure, considering his expertise on matters affecting the nation's coffers, the
Secretary of the DBM, as the President's alter ego, knows from where he speaks
inasmuch as he has the front seat view of the adverse effects of an unwarranted
upgrading or creation of positions in the CHR in particular and in the entire
government in general.

Page 209 of 323


*Funa v. CSC, G.R. No. 191672, November 25, 2014. [IMPORTANT!!!]
(Ex officio capacity; Holding of multiple offices; Independence; De facto officers)

Facts:
-Through Executive Order No. 864 (EO 864), then President Arroyo designated Hon.
Francisco T. Duque III, who was then the Chairman of the CSC, as a member of the
Board of Directors or Trustees in an ex officio capacity of the (a) Government Service
Insurance System (GSIS); (b) Philippine Health Insurance Corporation (PHILHEALTH),
(c) the Employees Compensation Commission (ECC), and (d) the Home Development
Mutual Fund (HDMF).

-Petitioners then went to the SC via special civil action for certiorari and prohibition
asserting that that EO 864 and Section 14, Chapter 3, Title I-A, Book V of EO 292
violate the independence of the CSC.

-Respondents’ contentions: (1) Duque is just one member of the CSC, or of the Boards
of the GSIS, PHILHEALTH, ECC and HDMF, such that matters resolved by these
bodies may be resolved with or without Duque’s participation; (2) Section 2, Article IX-
A of the 1987 Constitution does not cover positions held without additional
compensation in ex officio capacities; (3) While the charters of these GOCCs do not
provide that CSC Chairman shall be a member of their respective governing Boards,
there is likewise no prohibition mentioned under said charters.

Issues:
(1) Whether or not the designation of Duque as member of the Board of Directors or
Trustees of the GSIS, PHILHEALTH, ECC and HDMF, in an ex officio capacity, is
unconstitutional;
(2) Whether or not Duque can be considered a de facto officer during his tenure as a
Director or Trustee of the GSIS, PHILHEALTH, ECC and HDMF.

Ruling:
(1) Yes.

Section 1, Article IX-A of the 1987 Constitution expressly describes all the
Constitutional Commissions as “independent.” Although their respective functions
are essentially executive in nature, they are not under the control of the President
of the Philippines in the discharge of such functions. Each of the Constitutional
Commissions conducts its own proceedings under the applicable laws and its own
rules and in the exercise of its own discretion. Its decisions, orders and rulings are
subject only to review on certiorari by the Court as provided by Section 7, Article IX-A
of the 1987 Constitution. To safeguard the independence of these Commissions, the
1987 Constitution, among others, imposes under Section 2, Article IX-A of the
Constitution certain inhibitions and disqualifications upon the Chairmen and
members to strengthen their integrity, to wit:

(a) Holding any other office or employment during their tenure;

(b) Engaging in the practice of any profession;

(c) Engaging in the active management or control of any business which in any way
may be affected by the functions of his office; and

(d) Being financially interested, directly or indirectly, in any contract with, or in any
franchise or privilege granted by the Government, any of its subdivisions, agencies or
instrumentalities, including government-owned or – controlled corporations or their
subsidiaries. x x x x

When the CSC Chairman sits as a member of the governing Boards of the GSIS,
PHILHEALTH, ECC and HDMF, he may exercise these powers and functions,
which are not anymore derived from his position as CSC Chairman, such as
imposing intereston unpaid or unremitted contributions, issuing guidelines for

Page 210 of 323


the accreditation of health care providers, or approving restructuring proposals
in the payment of unpaid loan amortizations. The Court also notes that Duque’s
designation as member of the governing Boards of the GSIS, PHILHEALTH, ECC
and HDMF entitles him to receive per diem, a form of additional compensation
that is disallowed by the concept of an ex officio position by virtue of its clear
contravention of the proscription set by Section 2, Article IX-A of the 1987
Constitution. This situation goes against the principle behind an ex officio position,
and must, therefore, be held unconstitutional.

Apart from violating the prohibition against holding multiple offices, Duque’s
designation as member of the governing Boards of the GSIS, PHILHEALTH, ECC
and HDMF impairs the independence of the CSC. Under Section 17,42 Article VII
of the Constitution, the President exercises control over all government offices
in the Executive Branch. An office that is legally not under the control of the
President is not part of the Executive Branch. x x x x

As provided in their respective charters, PHILHEALTH and ECC have the status of a
government corporation and are deemed attached to the Department of Health
and the Department of Labor, respectively. On the other hand, the GSIS and
HDMF fall under the Office of the President. The corporate powers of the GSIS,
PHILHEALTH, ECC and HDMF are exercised through their governing Boards,
members of which are all appointed by the President of the Philippines. Undoubtedly,
the GSIS, PHILHEALTH, ECC and HDMF and the members of their respective
governing Boards are under the control of the President. As such, the CSC
Chairman cannot be a member of a government entity that is under the control
of the President without impairing the independence vested in the CSC by the
1987 Constitution.

(2) Yes.

A de facto officer is one who derives his appointment from one having colorable
authority to appoint, if the office is an appointive office, and whose appointment
is valid on its face. He may also be one who is in possession of an office, and is
discharging its duties under color of authority, by which is meant authority derived
from an appointment, however irregular or informal, so that the incumbent is not a
mere volunteer. Consequently, the acts of the de facto officer are just as valid for
all purposes as those of a de jure officer, in so far as the public or third persons
who are interested therein are concerned.

In order to be clear, therefore, the Court holds that all official actions of Duque as a
Director or Trustee of the GSIS, PHILHEAL TH, ECC and HDMF, were presumed
valid, binding and effective as if he was the officer legally appointed and
qualified for the office. This clarification is necessary in order to protect the sanctity
and integrity of the dealings by the public with persons whose ostensible authority
emanates from the State. Duque’s official actions covered by this clarification extend
but are not limited to the issuance of Board resolutions and memoranda approving
appointments to positions in the concerned GOCCs, promulgation of policies and
guidelines on compensation and employee benefits, and adoption of programs to carry
out the corporate powers of the GSIS, PHILHEAL TH, ECC and HDMF.

*Legaspi v. Comelec, G.R. No. 216572, September 1, 2015.


( )

Accountability of Public Officers

*Francisco v. House of Representatives, G.R. No. 160261, November 10, 2003.


(Impeachment; Judicial Review; One-Year Bar)

Facts:

Page 211 of 323


-On June 2, 2003, former President Joseph E. Estrada filed an impeachment
complaint (first impeachment complaint) against Chief Justice Hilario G. Davide Jr.
and seven Associate Justices of this Court for "culpable violation of the Constitution,
betrayal of the public trust and other high crimes." The complaint was endorsed by
Representatives Rolex T. Suplico, Ronaldo B. Zamora and Didagen Piang Dilangalen,
and was referred to the House Committee on Justice on August 5, 2003.

-The House Committee on Justice ruled on October 13, 2003 that the first
impeachment complaint was "sufficient in form," but voted to dismiss the same on
October 22, 2003 for being insufficient in substance.

-To date, the Committee Report to this effect has not yet been sent to the House
in plenary in accordance with the said Section 3(2) of Article XI of the
Constitution.

-Four months and three weeks since the filing on June 2, 2003 of the first complaint
or on October 23, 2003, a day after the House Committee on Justice voted to dismiss
it, the second impeachment complaint was filed with the Secretary General of the
House by Representatives Gilberto C. Teodoro, Jr. (First District, Tarlac) and Felix
William B. Fuentebella (Third District, Camarines Sur) against Chief Justice Hilario G.
Davide, Jr.

-This second impeachment complaint was accompanied by a "Resolution of


Endorsement/Impeachment" signed by at least one-third (1/3) of all the Members of
the House of Representatives.

-Pertinent to the impeachment proceedings, Sections 16 and 17 of Rule V of the House


Impeachment Rules provide:

“Section 16. – Impeachment Proceedings Deemed Initiated. – In cases where a Member of the House files a
verified complaint of impeachment or a citizen files a verified complaint that is endorsed by a Member of
the House through a resolution of endorsement against an impeachable officer, impeachment
proceedings against such official are deemed initiated on the day the Committee on Justice finds
that the verified complaint and/or resolution against such official, as the case may be, is sufficient
in substance, or on the date the House votes to overturn or affirm the finding of the said
Committee that the verified complaint and/or resolution, as the case may be, is not sufficient in
substance.

In cases where a verified complaint or a resolution of impeachment is filed or endorsed, as the case may
be, by at least one-third (1/3) of the Members of the House, impeachment proceedings are deemed
initiated at the time of the filing of such verified complaint or resolution of impeachment with the
Secretary General.

Section 17. Bar Against Initiation Of Impeachment Proceedings. – Within a period of one (1) year from
the date impeachment proceedings are deemed initiated as provided in Section 16 hereof, no
impeachment proceedings, as such, can be initiated against the same official.”

-Petitioners then went to the SC via petition for Certiorari/Prohibition to stop a


purported unconstitutional impeachment.

-Respondents’ contentions: (1) impeachment is a political action which cannot assume


a judicial character. Hence, any question, issue or incident arising at any stage of the
impeachment proceeding is beyond the reach of judicial review; (2) the one-year bar
prohibiting the initiation of impeachment proceedings against the same officials could
not have been violated as the impeachment complaint against Chief Justice Davide
and seven Associate Justices had not been “initiated” as the House of
Representatives, acting as the collective body, has yet to act on it.

Issues:
(1) Whether or not questions relative to impeachment are beyond the scope of judicial
review;
(2) Whether or not the one-year bar prohibiting the initiation of impeachment
proceedings against the same officials applies under the circumstances;

Page 212 of 323


(3) Whether or not the provisions of Sections 16 and 17 of Rule V of the House
Impeachment Rules are unconstitutional.

Ruling:
(1) No.

The major difference between the judicial power of the Philippine Supreme Court and
that of the U.S. Supreme Court is that while the power of judicial review is only
impliedly granted to the U.S. Supreme Court and is discretionary in nature, that
granted to the Philippine Supreme Court and lower courts, as expressly provided
for in the Constitution, is not just a power but also a duty, and it was given an
expanded definition to include the power to correct any grave abuse of
discretion on the part of any government branch or instrumentality.

There are also glaring distinctions between the U.S. Constitution and the Philippine
Constitution with respect to the power of the House of Representatives over
impeachment proceedings. While the U.S. Constitution bestows sole power of
impeachment to the House of Representatives without limitation, our Constitution,
though vesting in the House of Representatives the exclusive power to initiate
impeachment cases, provides for several limitations to the exercise of such power
as embodied in Section 3(2), (3), (4) and (5), Article XI thereof. These limitations
include the manner of filing, required vote to impeach, and the one-syear bar on
the impeachment of one and the same official.

The Constitution did not intend to leave the matter of impeachment to the sole
discretion of Congress. Instead, it provided for certain well-defined limits, or in the
language of Baker v. Carr, "judicially discoverable standards" for determining the
validity of the exercise of such discretion, through the power of judicial review.

Finally, there exists no constitutional basis for the contention that the exercise of
judicial review over impeachment proceedings would upset the system of checks and
balances. Verily, the Constitution is to be interpreted as a whole and "one section is
not to be allowed to defeat another." Both are integral components of the calibrated
system of independence and interdependence that insures that no branch of
government act beyond the powers assigned to it by the Constitution.

(2) Yes.

From the records of the Constitutional Commission, to the amicus curiae briefs of two
former Constitutional Commissioners, it is without a doubt that the term “to
initiate” refers to the filing of the impeachment complaint coupled with
Congress’ taking initial action of said complaint.

During the oral arguments before this Court, Father Bernas clarified that the word
“initiate,” appearing in the constitutional provision on impeachment, viz:

Section 3 (1) The House of Representatives shall have the exclusive power to initiate all cases of
impeachment.

xxx

(5) No impeachment proceedings shall be initiated against the same official more than once within a
period of one year”

refers to two objects, “impeachment case” and “impeachment proceeding.”

Father Bernas explains that in these two provisions, the common verb is "to initiate."
The object in the first sentence is "impeachment case." The object in the second
sentence is "impeachment proceeding." Following the principle of reddendo singuala
sinuilis, the term "cases" must be distinguished from the term "proceedings." An
impeachment case is the legal controversy that must be decided by the Senate. Above-
quoted first provision provides that the House, by a vote of one-third of all its

Page 213 of 323


members, can bring a case to the Senate. It is in that sense that the House has
"exclusive power" to initiate all cases of impeachment. No other body can do it.
However, before a decision is made to initiate a case in the Senate, a "proceeding"
must be followed to arrive at a conclusion. A proceeding must be "initiated." To
initiate, which comes from the Latin word initium, means to begin. On the other hand,
proceeding is a progressive noun. It has a beginning, a middle, and an end. It takes
place not in the Senate but in the House and consists of several steps: (1) there is the
filing of a verified complaint either by a Member of the House of Representatives or by
a private citizen endorsed by a Member of the House of the Representatives; (2) there
is the processing of this complaint by the proper Committee which may either reject
the complaint or uphold it; (3) whether the resolution of the Committee rejects or
upholds the complaint, the resolution must be forwarded to the House for further
processing; and (4) there is the processing of the same complaint by the House of
Representatives which either affirms a favorable resolution of the Committee or
overrides a contrary resolution by a vote of one-third of all the members. If at least
one third of all the Members upholds the complaint, Articles of Impeachment are
prepared and transmitted to the Senate. It is at this point that the House
"initiates an impeachment case." It is at this point that an impeachable public
official is successfully impeached. That is, he or she is successfully charged with an
impeachment "case" before the Senate as impeachment court.

Father Bernas further explains: The "impeachment proceeding" is not initiated


when the complaint is transmitted to the Senate for trial because that is the end
of the House proceeding and the beginning of another proceeding, namely the
trial. Neither is the "impeachment proceeding" initiated when the House deliberates on
the resolution passed on to it by the Committee, because something prior to that has
already been done. The action of the House is already a further step in the proceeding,
not its initiation or beginning. Rather, the proceeding is initiated or begins, when
a verified complaint is filed and referred to the Committee on Justice for action.
This is the initiating step which triggers the series of steps that follow. x x x x

Having concluded that the initiation takes place by the act of filing of the
impeachment complaint and referral to the House Committee on Justice, the
initial action taken thereon, the meaning of Section 3 (5) of Article XI becomes
clear. Once an impeachment complaint has been initiated in the foregoing
manner, another may not be filed against the same official within a one year
period following Article XI, Section 3(5) of the Constitution.

In fine, considering that the first impeachment complaint was filed by former President
Estrada against Chief Justice Hilario G. Davide, Jr., along with seven associate
justices of this Court, on June 2, 2003 and referred to the House Committee on
Justice on August 5, 2003, the second impeachment complaint filed by
Representatives Gilberto C. Teodoro, Jr. and Felix William Fuentebella against the
Chief Justice on October 23, 2003 violates the constitutional prohibition against the
initiation of impeachment proceedings against the same impeachable officer within a
one-year period.

(3) Yes.

The provisions of Sections 16 and 17 of Rule V of the House Impeachment Rules


which state that impeachment proceedings are deemed initiated: (1) if there is a
finding by the House Committee on Justice that the verified complaint and/or
resolution is sufficient in substance, or (2) once the House itself affirms or overturns
the finding of the Committee on Justice that the verified complaint and/or resolution
is not sufficient in substance or (3) by the filing or endorsement before the Secretary-
General of the House of Representatives of a verified complaint or a resolution of
impeachment by at least 1/3 of the members of the House thus clearly contravene
Section 3 (5) of Article XI as they give the term “initiate” a meaning different
from “filing.”

Page 214 of 323


*Villaseñor v. Sandiganbayan, G.R. No. 180700, March 4, 2008. [PROBABLE BAR
QUESTION!!!]
(Preventive suspension; Criminal liability; Administrative liability)

Facts:
-On August 18, 2001, the Quezon City Manor Hotel went ablaze resulting in the death
of seventy-four (74) people and injuries to scores of others. Investigation into the
tragedy revealed that the hotel was a veritable fire trap.

-Petitioners, together with other officials of the City Engineering Office of Quezon City,
are presently facing criminal charges before the 5th Division of the Sandiganbayan
for: (1) multiple homicide through reckless imprudence and (2) violation of Section 3(e)
of R.A. No. 3019.

-Petitioners were also charged administratively with gross negligence, gross


misconduct and conduct prejudicial to the interest of the service in connection with
the Manor Hotel inferno.

-In two separate Orders dated August 29, 20015 and September 7, 20016 in the
administrative case, the Sandiganbayan preventively suspended petitioners
Villaseñor and Mesa for a period of six (6) months, effective upon receipt of the
suspension order.

-Meanwhile, on September 20, 2006, during the pendency of the criminal case,
respondent special prosecutor Louella Mae Oco-Pesquera filed a motion for suspension
pendente lite of petitioners.

-Petitioners opposed the motion, contending that they had already been suspended for
six (6) months relative to the administrative case, based on the same facts and
circumstances. They posited that any preventive suspension that may be warranted in
the criminal case was already absorbed by the preventive suspension in the
administrative case because both the criminal and administrative cases were anchored
on the same set of facts.

-The Sandiganbayan then issued a resolution preventively suspending petitioners for a


period of ninety (90) days from receipt thereof.

-Petitioners then went to the SC via petition for certiorari under Rule 65.

Issues:
(1) Whether or not preventive suspension under Section 13 of R.A. No. 3019 is
mandatory;
(2) Whether or not the Sandiganbayan committed grave abuse of discretion when it
ordered the preventive suspension of petitioners despite the fact that they had already
been previously suspended administratively based on the same facts and
circumstances;
(3) Whether or not Section 13 of R.A. No. 3019 on preventive suspension is a penal
provision.

Ruling:
(1) Yes.

It is well-settled that preventive suspension under Section 13 of R.A. No. 3019 is


mandatory. It is evident from the very wording of the law:

“Suspension and loss of benefits. – Any incumbent public officer against whom any criminal prosecution
under a valid information under this Act or under Title 7, Book II of the Revised Penal Code or for any
offense involving fraud upon the government or public funds or property, whether as a simple or as a
complex offense and in whatever stage of the execution and mode of participation, is pending in court,
shall be suspended from office. x x x x”

Page 215 of 323


A whole slew of cases reinforce this provision of law. In Luciano v. Provincial Governor,
the Court pronounced that suspension of a public officer under Section 13 of R.A. No.
3019 is mandatory. This was reiterated in Luciano v. Mariano, People v. Albano,
Gonzaga v. Sandiganbayan and Bunye v. Escareal. In the last mentioned case, the
Court said:

“It is now settled that Sec. 13 of Republic Act No. 3019 makes it mandatory for
the Sandiganbayan to suspend any public official against whom a valid information
charging violation of that law, Book II, Title 7 of the Revised Penal Code, or any offense
involving fraud upon government or public funds or property is filed. The court trying
a case has neither discretion nor duty to determine whether preventive
suspension is required to prevent the accused from using his office to intimidate
witnesses or frustrate his prosecution or continuing committing malfeasance in
office. The presumption is that unless the accused is suspended he may frustrate his
prosecution or commit further acts of malfeasance or do both, in the same way that
upon a finding that there is probable cause to believe that a crime has been committed
and that the accused is probably guilty thereof, the law requires the judge to issue a
warrant for the arrest of the accused. The law does not require the court to determine
whether the accused is likely to escape or evade the jurisdiction of the court.”

(2) No.

Significantly, there are three kinds of remedies that are available against a public
officer for impropriety in the performance of his powers and the discharge of his
duties: (1) civil, (2) criminal, and (3) administrative. These remedies may be
invoked separately, alternately, simultaneously or successively. Sometimes, the
same offense may be the subject of all three kinds of remedies.

Defeat of any of the three remedies will not necessarily preclude resort to other
remedies or affect decisions reached thereunder, as different degrees of evidence are
required in these several actions. In criminal cases, proof beyond reasonable doubt is
needed whereas a mere preponderance of evidence will suffice in civil cases. In
administrative proceedings, only substantial evidence is required.

It is clear, then, that criminal and administrative cases are distinct from each
other. The settled rule is that criminal and civil cases are altogether different from
administrative matters, such that the first two will not inevitably govern or affect the
third and vice versa. Verily, administrative cases may proceed independently of
criminal proceedings.

Socrates v. Sandiganbayan, citing the Court’s pronouncements in Luciano v. Provincial


Governor, recounted:

“The Court then hastened to clarify that such a view may not be taken as an
encroachment upon the power of suspension given other officials, reiterating in the
process that a line should be drawn between administrative proceedings and
criminal actions in court, that one is apart from the other.”

Based on the foregoing, criminal actions will not preclude administrative


proceedings, and vice-versa, insofar as the application of the law on preventive
suspension is concerned.

(3) No.

It is petitioners’ contention that as a penal statute, the provision on preventive


suspension should be strictly construed against the State and liberally in their favor.

We cannot agree. Section 13 of R.A. No. 3019 on preventive suspension is not a


penal provision. It is procedural in nature. Hence, the strict construction rule
finds no application. The Court expounded on this point in Buenaseda v. Flavier:

Page 216 of 323


“Penal statutes are strictly construed while procedural statutes are liberally
construed (Crawford, Statutory Construction, Interpretation of Laws, pp. 460-461;
Lacson v. Romero, 92 Phil. 456 [1953]). The test in determining if a statute is penal is
whether a penalty is imposed for the punishment of a wrong to the public or for the
redress of an injury to an individual (59 Corpuz Juris, Sec. 658; Crawford, Statutory
Construction, pp. 496-497). A Code prescribing the procedure in criminal cases is
not a penal statute and is to be interpreted liberally (People v. Adler, 140 N.Y. 331;
35 N.E. 644).”

As We have already established, preventive suspension is not, in actual fact, a


penalty at all. It is a procedural rule.

*Contes v. Ombudsman, G.R. Nos. 187896-97, June 10, 2013. [GOOD FOR
REMEDIAL LAW!]
(Ombudsman procedure)

-In his Complaint-Affidavit, petitioner charged respondents with violation of Section


3(c) of Republic Act No. 3019 or the Anti-Graft and Corrupt Practices Act and
Misconduct. Respondents were sued in their capacity as Provincial Engineer,
Barangay Captain of Barangay Soncolan and Grader Operator of the Province of
Aklan.

-In its Consolidated Evaluation Report, the Office of the Ombudsman (Visayas)
recommended the dismissal of the cases due to the fact that two (2) other cases
involving the same parties and issues had already been filed by petitioner.

-Petitioner moved for the reconsideration of the Consolidated Evaluation Report, but
the same was denied.

-Petitioner takes the appeal directly to the Supreme Court via a petition for
review on certiorari under Rule 45, pursuant to Section 27 of the Ombudsman Act,
assailing the denial of his motion for reconsideration by the Office of the Ombudsman
(Visayas).

Issue:
Whether or not the petition will prosper under the circumstances.

Ruling:
No.

Petitioner, in filing this petition for review, committed a procedural misstep which
warrants an outright dismissal.

Petitioner misconstrued Section 27 of Republic Act No. 6770 or the Ombudsman Act
of 1989 and disregarded prevailing jurisprudence. Section 27 provides, in part, that:

In all administrative disciplinary cases, orders, directives, or decisions of the Office of the Ombudsman
may be appealed to the Supreme Court by filing a petition for certiorari within ten (10) days from receipt
of the written notice of the order, directive or decision or denial of the motion for reconsideration in
accordance with Rule 45 of the Rules of Court.

This provision, insofar as it provided for appeal by certiorari under Rule 45 from
the decisions or orders of the Ombudsman in administrative cases, had been
declared unconstitutional by this Court as early as in the case of Fabian v.
Desierto. We ruled in Fabian that appeals from decisions of the Office of the
Ombudsman in administrative disciplinary cases should be taken to the Court of
Appeals under the provisions of Rule 43, in line with the regulatory philosophy
adopted in appeals from quasi-judicial agencies in the 1997 Revised Rules of Civil
Procedure.

Jurisprudence accords a different treatment with respect to an appeal in a


criminal case filed with the Office of the Ombudsman. We made the

Page 217 of 323


pronouncement in Acuña v. Deputy Ombudsman for Luzon that the remedy of an
aggrieved party in criminal complaints before the Ombudsman is to file with this
Court a petition for certiorari under Rule 65.

Considering that the case at bar was a consolidation of an administrative and a


criminal complaint, petitioner had the option to either file a petition for review
under Rule 43 with the Court of Appeals or directly file a certiorari petition
under Rule 65 before this Court. Neither of these two remedies was resorted to by
petitioner.

By availing of a wrong remedy, this petition merits an outright dismissal.

*Gonzales III v. Office of the President, G.R. No. 196231, January 28, 2014.
[IMPORTANT!!!]
(Office of the President; Ombudsman; Office of the Special Prosecutor; Jurisdiction)

Facts:
-This case involves two (2) charges decided by the Office of the President (OP).

-First is the case of Deputy Ombudsman Emilio Gonzales III (Gonzales). The OP
initiated a case against Gonzales for “Gross Neglect of Duty and/or Inefficiency in
the Performance of Official Duty and for Misconduct in Office” allegedly for the
inordinate and unjustified delay in the resolution of a respondent’s Motion for
Reconsideration that spanned nine (9) months. Gonzales was eventually found guilty
by the OP and penalized him with dismissal from office.

-Second is the case of Special Prosecutor Wendell Barreras-Sulit. The OP initiated a


case against Sulit for acts and omissions tantamount to culpable violation of the
Constitution and a betrayal of public trust for allegedly entering into a one-sided plea
bargaining agreement with an official indicted for plunder.

-Section 8(2) of R.A. 6770, otherwise known as AN ACT PROVIDING FOR THE
FUNCTIONAL AND STRUCTURAL ORGANIZATION OF THE OFFICE OF THE
OMBUDSMAN, AND FOR OTHER PURPOSES provides:

Section 8. Removal; Filling of Vacancy. —


xxxx
(2) A Deputy or the Special Prosecutor, may be removed from office by the President for any of the
grounds provided for the removal of the Ombudsman, and after due process. x x x x

-Both Gonzales and Sulit went to the SC to challenge the validity of Section 8(2) of
R.A. 6770, particularly the jurisdiction of the OP over them involving disciplinary
proceedings.

Issues:
(1) Whether or not the Office of the President has disciplinary authority over Deputy
Ombudsman;
(2) Whether or not the Office of the President has disciplinary authority over Special
Prosecutor.

Ruling:
(1) No.

With the advent of the 1987 Constitution, a new Office of the Ombudsman was
created by constitutional fiat. Unlike in the 1973 Constitution, its independence was
expressly and constitutionally guaranteed. x x x x

The Ombudsman’s broad investigative and disciplinary powers include all acts of
malfeasance, misfeasance, and nonfeasance of all public officials, including Members
of the Cabinet and key Executive officers, during their tenure. To support these broad
powers, the Constitution saw it fit to insulate the Office of the Ombudsman from

Page 218 of 323


the pressures and influence of officialdom and partisan politics and from fear of
external reprisal by making it an “independent” office. x x x x

Subjecting the Deputy Ombudsman to discipline and removal by the President,


whose own alter egos and officials in the Executive Department are subject to
the Ombudsman’s disciplinary authority, cannot but seriously place at risk the
independence of the Office of the Ombudsman itself. The Office of the
Ombudsman, by express constitutional mandate, includes its key officials, all of them
tasked to support the Ombudsman in carrying out her mandate. Unfortunately,
intrusion upon the constitutionally-granted independence is what Section 8(2) of RA
No. 6770 exactly did. By so doing, the law directly collided not only with the
independence that the Constitution guarantees to the Office of the Ombudsman, but
inevitably with the principle of checks and balances that the creation of an
Ombudsman office seeks to revitalize. x x x x

In crafting Section 8(2) of RA No. 6770, Congress apparently addressed the concern
that a lack of an external check against the Deputy Ombudsman would result in
mutual protection between the Ombudsman and her Deputies. While the preceding
discussion already suffices to address this concern, it should be added that this
concern stands on shaky grounds since it ignores the existing checks and balances
already in place. On the one hand, the Ombudsman’s Deputies cannot protect the
Ombudsman because she is subject to the impeachment power of Congress. On
the other hand, the Ombudsman’s attempt to cover up the misdeeds of her
Deputies can be questioned before the Court on appeal or certiorari. The same
attempt can likewise subject her to impeachment. x x x x

The mere fact that a statutorily-created sword of Damocles hangs over the Deputy
Ombudsman’s head, by itself, opens up all the channels for external pressures and
influence of officialdom and partisan politics. The fear of external reprisal from the
very office he is to check for excesses and abuses defeats the very purpose of
granting independence to the Office of the Ombudsman. x x x x

The mere filing of an administrative case against the Deputy Ombudsman and
the Special Prosecutor before the OP can already result in their suspension and
can interrupt the performance of their functions, in violation of Section 12,
Article XI of the Constitution. With only one term allowed under Section 11, a
Deputy Ombudsman or Special Prosecutor, if removable by the President, can be
reduced to the very same ineffective Office of the Ombudsman that the framers had
foreseen and carefully tried to avoid by making these offices independent
constitutional bodies. x x x x

(2) Yes.

By vote of 8-7, the Court resolves to maintain the validity of Section 8(2) of RA No.
6770 insofar as Sulit is concerned. The Court does not consider the Office of the
Special Prosecutor to be constitutionally within the Office of the Ombudsman
and is, hence, not entitled to the independence the latter enjoys under the
Constitution.
(Note: There is not much discussion here as to why the Office of the Special Prosecutor
is not entitled to the independence that the Office of the Ombudsman enjoys.)

[IMPORTANT NOTE:
Section 8(2) of R.A. No. 6770 was declared UNCONSTITUTIONAL for granting
disciplinary jurisdiction to the President over a Deputy Ombudsman, in violation of
the independence of the Office of the Ombudsman. However, it was declared
CONSTITUTIONAL insofar as the Office of the Special Prosecutor is concerned.]

*Khan v. Ombudsman, G.R. No. 125296, July 20, 2006.


(Ombudsman; Jurisdiction; GOCCs)

Facts:

Page 219 of 323


-Petitioners are former officers of Philippine Airlines (PAL). In February 1989, private
respondents Rosauro Torralba and Celestino Bandala charged petitioners before the
Deputy Ombudsman (Visayas) for violation of RA 3019, also nown as the Anti-Graft
and Corrupt Practices Act.

-In their complaint, private respondents accused petitioners of using their positions in
PAL to secure a contract for Synergy Services Corporation, a corporation engaged in
hauling and janitorial services in which they were shareholders.

-Petitioners filed an omnibus motion to dismiss the complaint on the following


grounds: (1) the Ombudsman had no jurisdiction over them since PAL was a private
entity and (2) they were not public officers, hence, outside the application of RA 3019.

-The Deputy Ombudsman denied petitioners’ omnibus motion to dismiss,


ratiocinating that: (1) although PAL was originally organized as a private corporation,
its controlling stock was later acquired by the government through the Government
Service Insurance System (GSIS). Therefore, it became a government-owned or
controlled corporation (GOCC); (2) Petitioners were public officers within the definition
of RA 3019, Section 2 (b). Under that provision, public officers included “elective,
appointive officials and employees, permanent or temporary, whether in the classified
or unclassified or exempt service receiving compensation, even nominal, from the
Government.”

-Petitioners then went to the SC via petition for certiorari.

Issues:
(1) Whether or not the Ombudsman has jurisdiction over petitioners;
(2) Whether or not petitioners are public officers.

Ruling:
No.

The 1987 Constitution states the powers and functions of the Office of the
Ombudsman. Specifically, Article XI, Section 13(2) provides:

“Sec. 13. The Office of the Ombudsman shall have the following powers, functions, and duties:

xxx xxx xxx

(2) Direct, upon complaint or at its own instance, any public official or employee of the Government, or
any subdivision, agency or instrumentality thereof, as well as any government-owned or controlled
corporation with original charter, to perform and expedite any act or duty required by law, or to stop,
prevent, and correct any abuse or impropriety in the performance of duties.”

Based on the foregoing provision, the Office of the Ombudsman exercises


jurisdiction over public officials/ employees of GOCCs with original charters.
This being so, it can only investigate and prosecute acts or omissions of the
officials/employees of government corporations. Therefore, although the
government later on acquired the controlling interest in PAL, the fact remains
that the latter did not have an “original charter” and its officers/employees
could not be investigated and/or prosecuted by the Ombudsman.

In Juco v. National Labor Relations Commission, we ruled that the phrase “with
original charter” means “chartered by special law as distinguished from
corporations organized under the Corporation Code.” PAL, being originally a
private corporation seeded by private capital and created under the general
corporation law, does not fall within the jurisdictional powers of the Ombudsman
under Article XI, Section 13(2) of the Constitution. Consequently, the latter is devoid of
authority to investigate or prosecute petitioners.

(2) No.

Page 220 of 323


Neither the 1987 Constitution nor RA 6670 (The Ombudsman Act of 1989) defines
who “public officers” are. Instead, its varied definitions and concepts are found in
different statutes and jurisprudence. Usually quoted in our decisions is Mechem, a
recognized authority on the subject. In the 2002 case of Laurel v. Desierto, the Court
extensively quoted his exposition on the term "public officers":

A public office is the right, authority and duty, created and conferred by law, by
which, for a given period, either fixed by law or enduring at the pleasure of the
creating power, an individual is invested with some portion of the sovereign functions
of the government, to be exercised by him for the benefit of the public. The individual
so invested is a public officer.

The characteristics of a public office, according to Mechem, include the delegation of


sovereign functions, its creation by law and not by contract, an oath, salary,
continuance of the position, scope of duties, and the designation of the position as an
office.

xxx xxx xxx

Mechem describes the delegation to the individual of the sovereign functions of


government as "[t]he most important characteristic" in determining whether a position
is a public office or not.

The most important characteristic which distinguishes an office from an employment


or contract is that the creation and conferring of an office involves a delegation to
the individual of some of the sovereign functions of government to be exercised
by him for the benefit of the public; − that some portion of the sovereignty of the
country, either legislative, executive, or judicial, attaches, for the time being, to
be exercised for the public benefit. Unless the powers conferred are of this nature,
the individual is not a public officer.

From the foregoing, it can be reasonably inferred that "public officers" are those
endowed with the exercise of sovereign executive, legislative or judicial functions. The
explication of the term is also consistent with the Court's pronouncement in Quimpo
that, in the case of officers/employees in GOCCs, they are deemed "public officers"
if their corporations are tasked to carry out governmental functions.

In any event, PAL has since reverted to private ownership and we find it pointless to
scrutinize the implications of a legal issue that technically no longer exists.

*Ombudsman v. Hon. Ibay, G.R. No. 137538, September 3, 2001. [SEVERAL


PRINCIPLES HERE!]
(Ombudsman; Authority; Bank Secrecy Law; Remedial Law: Declaratory Relief)

Facts:
-Sometime in 1998, petitioner Ombudsman conducted an investigation on the alleged
“scam” on the Public Estates Authority-Amari Coastal Bay Development Corporation.

-Initial result of the investigation revealed that the alleged anomaly was committed
through the issuance of checks which were subsequently deposited in several financial
institutions.

-On April 29, 1998, petitioner issued an Order directing private respondent Lourdes
Marquez, branch manager of Union Bank of the Philippines branch at Julia Vargas
Avenue, Pasig City, to produce several bank documents for inspection relative to
Account Nos. 011-37270-5, 240-020718, 245-30317-3 and 245-303318-1, reportedly
maintained in the said branch.

-The inspection would be done “in camera” wherein the bank records would be
examined without bringing the documents outside the bank premises. Its purpose was

Page 221 of 323


to identify the specific bank records prior to the issuance of the required information
not in any manner needed in or relevant to the investigation.

-Marquez failed to comply with petitioner's order, explaining that the subject accounts
pertained to International Corporate Bank (Interbank) which had merged with Union
Bank in 1994. She informed the Ombudsman that she had to first verify from the
Interbank records in its archives the whereabouts of said accounts.

-In view of such failure, the Ombudsman issued an order to Marquez to produce the
requested bank documents for "in camera" inspection. In the event of her failure to
comply as directed, she was ordered to show cause why she should not be cited for
contempt and why she should not be charged for obstruction.

-Instead of complying with the order of petitioner, Marquez filed a petition for
declaratory relief with an application for temporary restraining order and/or
preliminary injunction before the RTC of Makati.

-Petitioner filed a motion to dismiss the aforesaid petition for declaratory relief on the
ground that the RTC has no jurisdiction over the subject matter thereof, which the
RTC later denied in an order, ruling that it has jurisdiction over the case since it is an
action for declaratory relief under Rule 63 of the Rules of Court.

-Petitioner then went to the SC, alleging that the RTC issued the said order with grave
abuse of discretion and clear lack of jurisdiction.

Issue:
(1) Whether or not the RTC has jurisdiction to take cognizance of the petition for
declaratory relief;
(2) Whether or not the Ombudsman has the authority to conduct an in camera
inspection of the subject bank accounts, as an exception to the law on secrecy of bank
deposits.

Ruling:
(1) Yes.

Petitioner contends that the RTC of Makati City lacks jurisdiction over the petition for
declaratory relief. It asserts that respondent judge should have dismissed the petition
outright in view of Section 14 of R.A. 6770.

Section 14 of R.A. 6770 provides:

“Restrictions. – No writ of injunction shall be issued by any court to delay an investigation being
conducted by the Ombudsman under this Act, unless there is a prima facie evidence that the subject
matter of the investigation is outside the jurisdiction of the Office of the Ombudsman.

No court shall hear any appeal or application for remedy against the decision or findings of the
Ombudsman, except the Supreme Court, on pure question of law.”

Petitioner's invocation of the aforequoted statutory provision is misplaced. The special


civil action of declaratory relief falls under the exclusive jurisdiction of the
Regional Trial Court. It is not among the actions within the original jurisdiction
of the Supreme Court even if only questions of law are involved. Similarly, the
Rules of Court is explicit that such action shall be brought before the appropriate
Regional Trial Court. Section 1, Rule 63 of the Rules of Court provides:

“Section 1. Who may file petition. – Any person interested under a deed, will, contract or other written
instrument, whose rights are affected by a statute, executive order or regulation, ordinance, or any other
governmental regulation may, before breach or violation thereof, bring an action in the appropriate
Regional Trial Court to determine any question of construction or validity arising, and for a declaration of
his rights or duties, thereunder. x x x x”

The requisites of an action for declaratory relief are: (1) there must be a justiciable
controversy must be between persons whose interests are adverse; (3) that the party

Page 222 of 323


seeking the relief has a legal interest in the controversy; and (4) that the issue is ripe
for judicial determination.

In this case, the controversy concerns the extent of the power of petitioner to
examine bank accounts under Section 15 (8) of R.A. 6770 vis-à-vis the duty of
banks under Republic Act 1405 not to divulge any information relative to
deposits of whatever nature. The interests of the parties are adverse considering the
antagonistic assertion of a legal right on one hand, that is the power of Ombudsman
to examine bank deposits, and on the other, the denial thereof apparently by private
respondent who refused to allow petitioner to inspect in camera certain bank
accounts. The party seeking relief, private respondent herein, asserts a legal
interest in the controversy. The issue invoked is ripe for judicial determination
as litigation is inevitable. Note that petitioner has threatened private respondent
with “indirect contempt” and “obstruction” charges should the latter not comply with
its order.

(2) No.

Before an in camera inspection of bank accounts may be allowed, there must be


a pending case before a court of competent jurisdiction. Further, the account
must be clearly identified, and the inspection limited to the subject matter of the
pending case before the court of competent jurisdiction. The bank personnel and the
account holder must be notified to be present during the inspection, and such
inspection may cover only the account identified in the pending case.

In Union Bank of the Philippines v. Court of Appeals, we held that, “Section 2 of the Law
on Secrecy of Bank Deposits, as amended, declares bank deposits to be ‘absolutely
confidential’ except:

(a) In an examination made in the course of a special or general examination of a bank


that is specifically authorized by the Monetary Board after being satisfied that there is
reasonable ground to believe that a bank fraud or serious irregularity has been or is
being committed and that it is necessary to look into the deposit to establish such
fraud or irregularity,
(b) In an examination made by an independent auditor hired by the bank to conduct
its regular audit provided that the examination is for audit purposes only and the
results thereof shall be for the exclusive use of the bank,
(c) Upon written permission of the depositor,
(d) In cases of impeachment,
(e) Upon order of a competent court in cases of bribery or dereliction of duty of public
officials, or
(f) In cases where the money deposited or invested is the subject matter of the
litigation.”

In the case at bar, there is yet no pending litigation before any court of
competent authority. What is existing is an investigation by the Office of the
Ombudsman. In short, what the Office of the Ombudsman would wish to do is to fish
for additional evidence to formally charge Amado Lagdameo, et. al., with the
Sandiganbayan. Clearly, there was no pending case in court which would warrant
the opening of the bank account for inspection.

Since there is no pending litigation yet before a court of competent authority, but only
an investigation by the Ombudsman on the so-called “scam,” any order for the
opening of the bank account for inspection is clearly premature and legally
unjustified.

*Ombudsman v. Quimbo, G.R. No. 173277, February 25, 2015.


(Ombudsman; Authority; Remedial Law: Intervention)

Facts:

Page 223 of 323


-Gilda D. Daradal (Daradal), a clerk in the Provincial Engineering Office of Catbalogan,
Samar charged private respondent Engr. Prudencio C. Quimbo (Quimbo), Provincial
Engineer of Samar, with Sexual Harassment and Oppression before the Office of the
Ombudsman-Visayas (Ombudsman-Visayas).

-After due proceedings, the Ombudsman-Visayas issued a resolution dismissing the


case of sexual harassment against Quimbo but finding him guilty of oppression. The
Ombudsman-Visayas imposed the penalty of suspension for six (6) months without
pay.

-Aggrieved, Quimbo elevated the case before the CA by way of a petition for review
under Rule 43 of the Rules of Court. The case, entitled “Prudencio C. Quimbo vs.
Gilda D. Daradal,” was docketed as CA-G.R. SP No. 54737.

-The CA reversed the ruling of the Ombudsman-Visayas, ratiocinating that, “The Office
of the Ombudsman has no power to directly impose sanctions against government
officials and employees who are subject of its investigation as its power is only limited
to recommend the appropriate sanctions but not directly to impose the same.”

-The Ombudsman then filed an omnibus motion for intervention and


reconsideration of the CA decision.

-The CA denied the said motion and explained that, “the person adversely affected by
our ruling in SP No. 54737 is respondent Gilda D. Daradal who opted not to file a
motion for reconsideration thereof. Basic is the rule that every action must be
prosecuted or defended in the name of the real party in interest. x x x x In filing a
motion for intervention and reconsideration, the Ombudsman dangerously departed
from its role as adjudicator and became an advocate.”

-The Ombudsman then went to the SC via Rule 65.

Issues:
(1) Whether or not the Ombudsman has the power to directly impose administrative
penalties against erring public officials or employees;
(2) Whether or not the CA gravely abused its discretion in denying the Ombudsman’s
plea to intervene in its proceedings for lack of legal interest.

Ruling:
(1) Yes.

The Ombudsman has the power to directly impose administrative penalties against
public officials or employees. In the case of Ombudsman v. Apolonio, the Court
categorically delineated the Ombudsman’s power to directly impose, not merely
recommend, administrative sanctions against erring public officials or employees, viz:

The Ombudsman has the power to impose the penalty of removal, suspension,
demotion, fine, censure, or prosecution of a public officer or employee, in the
exercise of its administrative disciplinary authority. The challenge to the
Ombudsman’s power to impose these penalties, on the allegation that the Constitution
only grants it recommendatory powers, had already been rejected by this Court.

Although the tenor of the text in Section 13(3), Article XI15 of the Constitution
merely indicates a “recommendatory” function, this does not divest Congress of
its plenary legislative power to vest the Ombudsman powers beyond those stated
in the Constitutional provision. Pursuant to Republic Act (R.A.) No. 6770,
otherwise known as The Ombudsman Act of 1989, the Ombudsman is legally
authorized to directly impose administrative penalties against errant public
servants. Further, the manifest intent of the lawmakers was to bestow on the
Ombudsman full administrative disciplinary authority in accord with the
constitutional deliberations. Unlike the Ombudsman-like agencies of the past, the
powers of which extend to no more than making findings of fact and

Page 224 of 323


recommendations, and the Ombudsman or Tanodbayan under the 1973 Constitution
who might file and prosecute criminal, civil or administrative cases against public
officials and employees only in cases of failure of justice, the current Ombudsman,
under the 1987 Constitution and R.A. No. 6770, is intended to play a more
active role in the enforcement of laws on anti-graft and corrupt practices and
other offenses committed by public officers and employees. The Ombudsman is to
be an “activist watchman,” not merely a passive one. He is vested with broad powers to
enable him to implement his own actions.

(2) Yes.

Even if the Ombudsman was not impleaded as a party in the proceedings, part of
its broad powers include defending its decisions before the CA. Pursuant to
Section 1 of Rule 19 of the Rules of Court, the Ombudsman may validly intervene in
the said proceedings as its legal interest on the matter is beyond cavil. The Court
elucidated, thus:

x x x the Ombudsman is in a league of its own. It is different from other investigatory


and prosecutory agencies of the government because the people under its jurisdiction
are public officials who, through pressure and influence, can quash, delay or dismiss
investigations directed against them. Its function is critical because public interest (in
the accountability of public officers and employees) is at stake. x x x

It is true that under our rule on intervention, the allowance or disallowance of a


motion to intervene is left to the sound discretion of the court after a consideration of
the appropriate circumstances. However, such discretion is not without limitations.
One of the limits in the exercise of such discretion is that it must not be
exercised in disregard of law and the Constitution. The CA should have considered
the nature of the Ombudsman's powers as provided in the Constitution and RA 6770.
xxxx

Both the CA and respondent likened the Office of the Ombudsman to a judge whose
decision was in question. This was a tad too simplistic (or perhaps even rather
disdainful) of the power, duties and functions of the Office of the Ombudsman. The
Office of the Ombudsman cannot be detached, disinterested and neutral
specially when defending its decisions.

As can be gleaned from the foregoing disquisition, the CA, in the present case, gravely
erred in disallowing the Ombudsman’s motion to intervene. It failed to consider the
essence of the Ombudsman’s constitutionally and statutorily conferred powers
establishing its clear legal interest in ensuring that its directive be implemented.

*Villaseñor v. Ombudsman, G.R. No. 202303, June 4, 2014.


(Ombudsman; Authority; Remedial Law: Execution Pending Appeal)

Facts:
-Petitioners Gerardo R. Villaseñor (Villaseñor) and Rodel A. Mesa (Mesa), along with
several others, were administratively charged in connection with the Manor Hotel fire
tragedy that took place on August 18, 2001, killing 74 people and causing injury to
others. Petitioner Villaseñor was an electrical inspector from the Electrical Division,
and petitioner Mesa was an inspector from the Electrical Engineering Office, both of
Quezon City.

-After due proceedings, Villaseñor was found guilty of conduct prejudicial to the best
interest of the service and gross neglect of duty for which he was meted the penalty of
dismissal from the service. Mesa, meanwhile, was found guilty of conduct prejudicial
to the best interest of the service for which he was meted the penalty of one-year
suspension without pay.

-Villaseñor and Mesa then filed their separate motions for reconsideration of the
Joint Decision.

Page 225 of 323


-On April 18, 2006, Mesa appealed to the CA, which was docketed as CA-G.R. No.
93891. Villaseñor made no appeal, his motion for reconsideration before the
Ombudsman being yet unresolved.

-In the Order dated August 23, 2006, pending resolution of Mesa’s appeal and
Villaseñor’s motion for reconsideration, the Ombudsman directed the Mayor of
Quezon City and the Secretary of the Department of Interior and Local Government to
enforce the Joint Decision immediately upon receipt of the order.

-Villaseñor and Mesa then filed a special civil action for certiorari before the CA,
assailing the August 23, 2006 Order of the Ombudsman ordering the immediate
implementation of the Joint Decision despite the pendency of Villaseñor’s motion for
reconsideration and Mesa’s appeal. The CA dismissed the petition.

-Petitioners then went to the SC via Rule 45.

Issue:
Whether or not the Ombudsman’s order of dismissal from the service and suspension
of one year can be implemented pending resolution of petitioner Villaseñor’s motion for
reconsideration before the Ombudsman, and petitioner Mesa’s appeal before the CA.

Ruling:
Yes.

Section 7, Rule III of the Rules of Procedure of the Office of the Ombudsman, as
amended by A.O. No. 17, dated September 15, 2003, provides:

SEC. 7. Finality and execution of decision.– Where the respondent is absolved of the charge, and in case
of conviction where the penalty imposed is public censure or reprimand, suspension of not more than one
month, or a fine equivalent to one month salary, the decision shall be final, executory and unappealable.
In all other cases, the decision may be appealed to the Court of Appeals on a verified petition for review
under the requirements and conditions set forth in Rule 43 of the Rules of Court, within fifteen (15) days
from receipt of the written Notice of the Decision or Order denying the motion for reconsideration.

An appeal shall not stop the decision from being executory. In case the penalty is suspension or
removal and the respondent wins such appeal, he shall be considered as having been under preventive
suspension and shall be paid the salary and such other emoluments that he did not receive by reason of
the suspension or removal.

A decision of the Office of the Ombudsman in administrative cases shall be executed as a matter of
course. The Office of the Ombudsman shall ensure that the decision shall be strictly enforced and
properly implemented. The refusal or failure by any officer without just cause to comply with an order of
the Office of the Ombudsman to remove, suspend, demote, fine, or censure shall be a ground for
disciplinary action against such officer.

From the above, it can be gleaned that the Ombudsman decisions in administrative
cases may either be unappealable or appealable. Unappealable decisions are final
and executory, and they are as follows: (1) respondent is absolved of the charge; (2)
the penalty imposed is public censure or reprimand; (3) suspension of not more
than one month; and (4) a fine equivalent to one month’s salary. Appealable
decisions, on the other hand, are those which fall outside said enumeration, and may
be appealed to the CA under Rule 43 of the Rules of Court, within 15 days from
receipt of the written notice of the decision or order denying the motion for
reconsideration. Section 7 is categorical in providing that an appeal shall not stop
the decision from being executory, and that such shall be executed as a matter
of course.

Petitioner Mesa was ordered suspended for one year without pay, while petitioner
Villaseñor was ordered dismissed from the service. These are plainly appealable
decisions which are immediately executory pending appeal.

Page 226 of 323


Petitioner Villaseñor argues that the Ombudsman erred in implementing the order of
dismissal against him despite his pending motion for reconsideration with the same
office.

The records show that both petitioners duly filed their respective motions for
reconsideration on December 13, 2004. In the March 2, 2006 Memorandum of the
Ombudsman, Mesa’s motion for reconsideration, among others, was denied. Thus, he
appealed to the CA. A review of the said Memorandum reveals, however, that
Villaseñor’s motion for reconsideration was not enumerated as one of the pleadings
submitted for resolution, and nowhere was his liability discussed or even mentioned
therein. It is, therefore, apparent that Villaseñor’s motion for reconsideration was
never resolved by the Ombudsman, for which reason he has been unable to file an
appeal with the CA.

Nonetheless, Villaseñor’s pending motion for reconsideration cannot stop his


order of dismissal from being executory.

National Economy and Patrimony

*Pilipino Telephone Corporation v. NTC, G.R. No. 138295, August 28, 2003.
(Franchise; Due Process; Remedial Law: Certiorari; Motion for Reconsideration)

*PEZA v. Carantes, G.R. No. 181274, June 23, 2010.


(Ancestral Land; PEZA; CALC; CALT)

*Muller v. Muller, G.R. No. 149615, August 29, 2006.


(Private Lands; Aliens; Ownership and Reimbursement)

Facts:
-Petitioner Elena Muller and respondent Helmut Muller were married in Hamburg,
Germany in 1989.

-In 1992, the couple decided to move and reside permanently in the Philippines.

-By this time, Helmut had inherited the house in Germany from his parents which he
sold and used the proceeds for the purchase of a parcel of land in Antipolo, Rizal at
the cost of P528,000.00 and the construction of a house amounting to P2,300,000.00.

-The Antipolo property was registered in the name of Elena at the Register of Deeds of
Marikina, Metro Manila.

-Due to incompatibilities, the spouses eventually separated. In 1994, Helmut filed a


petition for separation of properties before the RTC of Quezon City.

-The RTC denied Helmut’s prayer to reimburse the acquisition cost of the land and
construction of the house, on the ground that its acquisition was in violation of
Section 7, Article XII of the Constitution. RTC’s judgment was appealed by Helmut to
the CA, which considered Elena’s ownership over the property in trust for Helmut.
The CA then ordered Elena to reimburse the same.

-Elena went to the SC via Rule 45.

-Contention of Helmut: He is not praying for transfer of ownership of the Antipolo


property but merely reimbursement; that the funds paid by him for the said
property were in consideration of his marriage to petitioner; that the funds were given
to petitioner in trust; and that equity demands that respondent should be reimbursed
of his personal funds.

Issue:

Page 227 of 323


Whether or not respondent is entitled to reimbursement of the funds used for the
acquisition of the Antipolo property.

Ruling:
No.

Section 7, Article XII of the 1987 Constitution states:

Save in cases of hereditary succession, no private lands shall be transferred or conveyed except to
individuals, corporations, or associations qualified to acquire or hold lands of the public domain.

Aliens, whether individuals or corporations, are disqualified from acquiring lands of


the public domain. Hence, they are also disqualified from acquiring private lands. The
primary purpose of the constitutional provision is the conservation of the national
patrimony.

Respondent was aware of the constitutional prohibition and expressly admitted his
knowledge thereof to this Court. He declared that he had the Antipolo property titled
in the name of petitioner because of the said prohibition. His attempt at subsequently
asserting or claiming a right on the said property cannot be sustained.

The Court of Appeals erred in holding that an implied trust was created and resulted
by operation of law in view of petitioner’s marriage to respondent. Save for the
exception provided in cases of hereditary succession, respondent’s disqualification
from owning lands in the Philippines is absolute. Not even an ownership in trust
is allowed. Besides, where the purchase is made in violation of an existing statute
and in evasion of its express provision, no trust can result in favor of the party
who is guilty of the fraud. To hold otherwise would allow circumvention of the
constitutional prohibition.

Invoking the principle that a court is not only a court of law but also a court of equity,
is likewise misplaced. It has been held that equity as a rule will follow the law and
will not permit that to be done indirectly which, because of public policy, cannot
be done directly. He who seeks equity must do equity, and he who comes into equity
must come with clean hands.

Thus, in the instant case, respondent cannot seek reimbursement on the ground
of equity where it is clear that he willingly and knowingly bought the property
despite the constitutional prohibition.

Further, the distinction made between transfer of ownership as opposed to recovery of


funds is a futile exercise on respondent’s part. To allow reimbursement would in
effect permit respondent to enjoy the fruits of a property which he is not allowed
to own. Thus, it is likewise proscribed by law.

[NOTE:
Aliens are prohibited to own lands, but they are not prohibited to own houses or
buildings. In the case of Beumer v. Amores, G.R. No. 195670, December 3, 2012,
during the dissolution of the conjugal properties of the couple (wife was Filipino;
husband was Dutch), the six (6) parcels of land were awarded to the wife because of
the constitutional prohibition against the alien. However, the two (2) separate houses
built on two (2) separate lands were declared co-owned by the spouses (not
including the lands where the houses are erected), considering that an alien is allowed
to own houses.]

*Godinez v. Fong Pak Luen, G.R. No. L-36731, January 27, 1983.
(Private Lands; Aliens; Ownership)

Facts:
-Jose Godinez sold a parcel of land under his name to Fong Pak Luen, a Chinese
citizen.

Page 228 of 323


-Subsequently, Fong Pak Luen, sold the land to Trinidad S. Navata, a Filipino citizen,
who was then aware that Fong Pak Luen is a Chinese citizen.

-Godinez died. His heirs then filed a complaint to recover the land from Navata,
arguing that the sale between Godinez and Pak Luen was void ab initio since the
transaction constituted a non-existent contract.

Issue:
Whether or not the heirs of Godinez may recover the subject land.

Ruling:
No.

The litigated property is now in the hands of a Filipino. It is no longer owned by


a disqualified vendee. Navata is constitutionally qualified to own the subject
property. There would be no more public policy to be served in allowing Godinez’s
heirs to recover the land as it is already in the hands of a qualified person.

*Board of Medicine v. Ota, G.R. No. 166097, July 14, 2008.


(Practice of Profession; Aliens; Reciprocity Rule)

Facts:
-Yasuyuki Ota (respondent) is a Japanese national (married to a Filipina), who has
continuously resided in the Philippines for more than 10 years. He graduated from
Bicol Christian College of Medicine in 1991 with a degree of Doctor of Medicine.

-After successfully completing a one-year post graduate internship training, he filed an


application to take the medical board examinations. He was required by the
Professional Regulation Commission (PRC) to submit an affidavit of undertaking,
stating among others that should he successfully pass the same, he would not
practice medicine until he submits proof that reciprocity exists between Japan and the
Philippines in admitting foreigners into the practice of medicine.

-Ota submitted a duly notarized English translation of the Medical Practitioners Law of
Japan duly authenticated by the Consul General of the Philippine Embassy to Japan;
thus, he was allowed to take the Medical Board Examinations in1992, which he
subsequently passed.

-Despite all these, the Board of Medicine (Board) of the PRC denied Ota’s request for a
license to practice medicine in the Philippines on the ground that the Board “believes
that no genuine reciprocity can be found in the law of Japan as there is no Filipino or
foreigner who can possibly practice there.”

-Ota then filed a Petition for Certiorari and Mandamus against the Board before the
RTC of Manila.

-Contention of the PRC: Ota has not established by competent and conclusive
evidence that reciprocity in the practice of medicine exists between the Philippines and
Japan. While documents state that foreigners are allowed to practice medicine in
Japan, they do not similarly show that the conditions for the practice of medicine in
said country are practical and attainable by a foreign applicant. There is no reciprocity
in this case, as the requirements to practice medicine in Japan are practically
impossible for a Filipino to comply with.

Likewise, PRC argues that its power to regulate and control the practice of medicine
includes the power to regulate admission to the ranks of those authorized to practice
medicine, which power is discretionary and not ministerial, hence, not compellable by
a writ of mandamus.

Issues:

Page 229 of 323


(1) Whether or not respondent Ota, under the circumstances, has sufficiently complied
with the requirements of the law;
(2) Whether or not the issuance of a license by the PRC is compellable by mandamus
under the circumstances.

Ruling:
(1) Yes.

The power to regulate the exercise of a profession or pursuit of an occupation cannot


be exercised by the State or its agents in an arbitrary, despotic, or oppressive manner.
x x x x As the legislature cannot validly bestow an arbitrary power to grant or refuse a
license on a public agency or officer, courts will generally strike down license
legislation that vests in public officials discretion to grant or refuse a license to carry
on some ordinarily lawful business, profession, or activity without prescribing definite
rules and conditions for the guidance of said officials in the exercise of their power.

As required by law, respondent Ota submitted a copy of the Medical Practitioners Law
of Japan, duly authenticated by the Consul General of the Embassy of the Philippines
in Japan. x x x x

R.A. No. 2382, which provides who may be candidates for the medical board
examinations, merely requires a foreign citizen to submit competent and conclusive
documentary evidence, confirmed by the Department of Foreign Affairs (DFA), showing
that his country’s existing laws permit citizens of the Philippines to practice medicine
under the same rules and regulations governing citizens thereof. x x x x

Nowhere in the said law is it stated that the foreign applicant must show that
the conditions for the practice of medicine in said country are practical and
attainable by Filipinos. Neither is it stated that it must first be proven that a
Filipino has been granted license and allowed to practice his profession in said
country before a foreign applicant may be given license to practice in the
Philippines. Indeed, the phrase used in both R.A. No. 2382 and P.D. No. 223 is that:

“The applicant shall submit competent and conclusive documentary evidence, confirmed by the
Department of Foreign Affairs, showing that his country's existing laws permit citizens of the Philippines
to practice the profession [of medicine] under the [same] rules and regulations governing citizens thereof.
x x x”

It is enough that the laws in the foreign country permit a Filipino to get license and
practice therein. Requiring respondent to prove first that a Filipino has already
been granted license and is actually practicing therein unduly expands the
requirements provided for under R.A. No. 2382. x x x x

Granting that there is still no Filipino who has been given license to practice medicine
in Japan, it does not mean that no Filipino will ever be able to be given one.

(2) Yes.

Indeed, to be granted the privilege to practice medicine, the applicant must show
that he possesses all the qualifications and none of the disqualifications. It must
also appear that he has fully complied with all the conditions and requirements
imposed by the law and the licensing authority.

In this case, there is no doubt as to the competence and qualifications of respondent.


He finished his medical degree from Bicol Christian College of Medicine. He completed
a one-year post graduate internship training at the Jose Reyes Memorial Medical
Center, a government hospital. Then he passed the Medical Board Examinations
which was given on August 8, 1992 with a general average of 81.83, with scores higher
than 80 in 9 of the 12 subjects.

In fine, the only matter being questioned by petitioners is the alleged failure of
respondent to prove that there is reciprocity between the laws of Japan and the

Page 230 of 323


Philippines in admitting foreigners into the practice of medicine. Respondent has
satisfactorily complied with the said requirement and the CA has not committed any
reversible error in ordering the Board to issue in favor of respondent the
corresponding Certificate of Registration and/or license to practice medicine in the
Philippines.

*Kalipunan ng Damayang Mahihirap, Inc. v. Roberdo, G.R. No. 200903, July 22,
2014.
(Urban Land & Housing Reform; Demolition; Urban Development Housing Act;
Remedial Law: Eviction;)

*EPZA v. CHR, G.R. No. 101476, April 14, 1992.


(CHR; Authority; Injunction Order)

*Simon v. CHR, G.R. No. 100150, January 5, 1994.


(CHR; Demolition; Contempt; Quasi-judicial Body)

*Cudia v. PMA, G.R. No. 211362, February 24, 2015.


(CHR; Investigative Body; Honor Code

*DECS v. San Diego, G.R. No. 89572, December 21, 1989.


(Quality Education; Entrance Exam; Medical Profession)

Facts:
-Petitioner took the National Medical Admission Test (NMAT) three times and flunked
it as many times.

-When he applied to take it again, the petitioner rejected his application on the basis
of the NMAT rule stating that, “after three (3) successive failures, a student shall not
be allowed to take the NMAT for the fourth time.”

-He then went to the RTC of Valenzuela, Metro Manila for petition for mandamus, to
compel his admission to the test and to challenged the constitutionality of the said
rule. He invoked his constitutional rights to academic freedom and quality education.

Issue:
Whether or not a person who has thrice failed the National Medical Admission Test
(NMAT) is entitled to take it again, anchored on his right to quality education.

Ruling:
No.

That the power to regulate and control the practice of medicine includes the power to
regulate admission to the ranks of those authorized to practice medicine, is well
recognized. Thus, legislation and administrative regulations requiring those who
wish to practice medicine first to take and pass medical board examinations
have long ago been recognized as valid exercises of governmental power.
Similarly, the establishment of minimum medical educational requirements-i.e., the
completion of prescribed courses in a recognized medical school-for admission to the
medical profession, has also been sustained as a legitimate exercise of the regulatory
authority of the state. x x x x

Police power is validly exercised if (a) the interests of the public generally, as
distinguished from those of a particular class, require the interference of the State,
and (b) the means employed are reasonably necessary to the attainment of the object
sought to be accomplished and not unduly oppressive upon individuals.

Page 231 of 323


The subject of the challenged regulation is certainly within the ambit of the police
power. It is the right and indeed the responsibility of the State to insure that the
medical profession is not infiltrated by incompetents to whom patients may
unwarily entrust their lives and health.

The method employed by the challenged regulation is not irrelevant to the purpose of
the law nor is it arbitrary or oppressive. The three-flunk rule is intended to insulate
the medical schools and ultimately the medical profession from the intrusion of
those not qualified to be doctors.

While every person is entitled to aspire to be a doctor, he does not have a


constitutional right to be a doctor. This is true of any other calling in which the
public interest is involved; and the closer the link, the longer the bridge to one’s
ambition. The State has the responsibility to harness its human resources and to see
to it that they are not dissipated or, no less worse, not used at all. These resources
must be applied in a manner that will best promote the common good while also giving
the individual a sense of satisfaction.

The right to quality education invoked by the private respondent is not absolute. The
Constitution also provides that “every citizen has the right to choose a profession
or course of study, subject to fair, reasonable and equitable admission and
academic requirements. x x x x”

There can be no question that a substantial distinction exists between medical


students and other students who are not subjected to the NMAT and the three-flunk
rule. The medical profession directly affects the very lives of the people, unlike
other careers which, for this reason, do not require more vigilant regulation. The
accountant, for example, while belonging to an equally respectable profession, does
not hold the same delicate responsibility as that of the physician and so need not be
similarly treated.

***ADMINISTRATIVE LAW***

*Regino v. Pangasinan Colleges of Science and Technology, G.R. No. 156109,


November 18, 2004.
(Exhaustion of Administrative Remedies; Doctrine of Prior Resort; Academic Freedom)

Facts:
-Regino was prevented by her two (2) teachers from taking the final examinations due
to her failure to pay for the tickets for her school-fund raising activity. The teachers
insisted that it was pursuant to the school’s policy.

-Regino filed, as a pauper litigant, a Complaint for damages against the school
(PCST). In her Complaint, she prayed for P500,000 as nominal damages; P500,000 as
moral damages; at least P1,000,000 as exemplary damages; P250,000 as actual
damages; plus the costs of litigation and attorney's fees.

-PCST filed a Motion to Dismiss on the ground of Regino’s failure to exhaust


administrative remedies.

-PCST’s contention: The question raised involved the determination of the wisdom of
an administrative policy of the PCST; hence, the case should have been initiated before
the proper administrative body, the Commission of Higher Education (CHED).

Issues:
(1) Whether or not prior recourse to the CHED, under the circumstances, was
required;
(1) Whether or not PCST may invoke academic freedom under the circumstances.

Ruling:
(1) No.

Page 232 of 323


First, Regino is not asking for the reversal of the policies of PCST. Neither is she
demanding it to allow her to take her final examinations, as she was already enrolled
in another educational institution. A reversal of the acts complained of would not
adequately redress her grievances. Under the circumstances, the consequences of
PCST’s acts could no longer be undone or rectified.

Second, exhaustion of administrative remedies is applicable when there is


competence on the part of the administrative body to act upon the matter
complained of. Administrative agencies are not courts; they are neither part of the
judicial system, nor are they deemed judicial tribunals. Specifically, the CHED does
not have the power to award damages. Hence, petitioner could not have
commenced her case before the Commission.

Third, the exhaustion doctrine admits of exceptions, one of which arises when
the issue is purely legal and well within the jurisdiction of the trial court.
Regino’s action for damages inevitably calls for the application and the
interpretation of the Civil Code, a function that falls within the jurisdiction of
the courts.

[NOTE:
In the present case, PCST imposed the assailed revenue-raising measure belatedly, in
the middle of the semester. It exacted the dance party fee as a condition for the
students' taking the final examinations, and ultimately for its recognition of their
ability to finish a course. The fee, however, was not part of the school-student
contract entered into at the start of the school year. Hence, it could not be
unilaterally imposed to the prejudice of the enrollees.

Such contract is by no means an ordinary one. In Non, we stressed that the school-
student contract “is imbued with public interest, considering the high priority given by
the Constitution to education and the grant to the State of supervisory and regulatory
powers over all educational institutions.”]

(2) No.

According to present jurisprudence, academic freedom encompasses the


independence of an academic institution to determine for itself (1) who may
teach, (2) what may be taught, (3) how it shall teach, and (4) who may be
admitted to study. x x x x

In Tangonan v. Paño, the Court upheld, in the name of academic freedom, the right of
the school to refuse readmission of a nursing student who had been enrolled on
probation, and who had failed her nursing subjects. These instances notwithstanding,
the Court has emphasized that once a school has, in the name of academic freedom,
set its standards, these should be meticulously observed and should not be used to
discriminate against certain students. After accepting them upon enrollment, the
school cannot renege on its contractual obligation on grounds other than those made
known to, and accepted by, students at the start of the school year.

*MIAA v. Blancaflor, G.R. No. 157581, December 1, 2004.


(Administrative Code: Prior notice and hearing; Rate increases)

Facts:
-Petitioner Manila International Airport Authority (MIAA) is a government-owned and
controlled corporation that owns, operates, and manages the Ninoy Aquino
International Airport (NAIA). Petitioner's properties, facilities, and services are
available for public use subject to such fees, charges, and rates as may be fixed in
accordance with law. Herein respondents are the users, lessees and occupants of
petitioner's properties, facilities, and services.

Page 233 of 323


-The schedule of aggregate dues collectible for the use of petitioner's properties,
facilities, and services are divided into: (1) aeronautical fees; (2) rentals; (3) business
concessions; (4) other airport fees and charges; and (5) utilities.

-On May 19, 1997, petitioner issued Resolution No. 97-513 announcing an increase in
the rentals of its terminal buildings, VIP lounge, other airport buildings and land, as
well as check-in and concessions counters. Business concessions, particularly
concessionaire privilege fees, were also increased.

-On April 2, 1998, petitioner passed Resolution No. 98-304 adopting twenty percent
(20%) of the increase recommended by Punongbayan and Araullo, to take effect
immediately on June 1, 1998. Thus, petitioner issued the corresponding
Administrative Order No. 1, Series of 1998 to reflect the new schedule of fees, charges,
and rates.

-On February 5, 1999, petitioner issued Resolution No. 99-11, which further increased
the other airport fees and charges, specifically for parking and porterage services, and
the rentals for hangars. Accordingly, petitioner amended Administrative Order No. 1,
Series of 1998.

-Respondents requested that the implementation of the new fees, charges, and rates
be deferred due to lack of prior notice and hearing. The request was denied.
Petitioner likewise refused to renew the identification cards of respondents' personnel,
and vehicle stickers to prevent entry to the premises.

-In view of this, respondents filed with the RTC of Makati City a Complaint for
Injunction with Application for a Writ of Preliminary Injunction and/or Temporary
Restraining Order.

-After due hearing, the RTC issued a Writ of Preliminary Injunction.

-MIAA then went to the SC via review on certiorari.

-MIAA’s contentions: (1) Its charter authorizes it to increase its fees, charges, and
rates without need of public hearing; (2) Its fees, charges, and rates are contractual
in nature such that if respondents are not amenable to any increase, they are free to
terminate the lease; (3) The charter which created it, being a special law, prevails over
the Public Service Act and the Administrative Code, which are laws of general
application.

Issues:
(1) Whether or not prior notice and conduct of public hearing are required before MIAA
can increase its rates and charges for the use of its facilities;
(2) Whether or not rates herein are contractual in nature and respondents are free to
terminate the lease contracts should they be unable to pay the increased dues.

Ruling:
(1) Yes.

B.P. Blg. 325,21 provides:

“SEC. 2. Determination of rates. – The fees and charges shall be revised at just and reasonable rates
sufficient to cover administrative costs and, wherever practicable, be uniform for similar or comparable
services and functions. The revision of rates shall be determined by the respective ministry heads or
equivalent functionaries conformably with the rules and regulations of the Ministry of Finance issued
pursuant to Section 4 hereof, upon recommendation of the imposing and collecting authorities concerned,
subject to the approval of the Cabinet.”

MIAA’s Charter established MIAA as an attached agency of the Department of


Transportation and Communications. Hence, the “ministry head” who has the
power to determine the revision of fees, charges, and rates of the MIAA is now
the DOTC Secretary. Clearly, petitioner has no authority to increase its fees, charges,

Page 234 of 323


or rates as the power to do so is vested solely in the DOTC Secretary, although
petitioner’s prerogative to recommend possible increases thereon is of course
recognized.

As an attached agency of the DOTC, the MIAA is governed by the Administrative


Code of 1987. The Administrative Code specifically requires notice and public
hearing in the fixing of rates:

BOOK VII. – Administrative Procedure

“SEC. 9. Public Participation. - … (2) In the fixing of rates, no rule or final order shall be valid unless
the proposed rates shall have been published in a newspaper of general circulation at least two (2)
weeks before the first hearing thereon.”

It follows that the rate increases imposed by petitioner are invalid for lack of the
required prior notice and public hearing. They are also ultra vires because, to begin
with, petitioner is not the official authorized to increase the subject fees, charges, or
rates, but rather the DOTC Secretary.

(2) Petitioner’s theory that its fees, charges, and rates are contractual in nature and
thus, respondents are free to terminate the lease contracts should they be unable to
pay the increased dues is unacceptable.

As the country's principal airport for both international and domestic air
transport, petitioner's properties, facilities, and services are imbued with
paramount public and even national interest. Petitioner is not at liberty to
increase fees, charges, or rates at will, without due regard to parameters set by
laws and regulations. Among the considerations mentioned in E.O. No. 903 are that
fees and charges should reflect adequately the costs and increases in price levels and
the volume of traffic. For any change in its fees, charges, or rates without due regard
to valid limitations can create a profound impact on the country's economy in general
and air transport in particular.

In the same vein, we are unable to share petitioner's claim that the specified increases
in fees, charges and rates would necessarily redound to the benefit of the country.
Needless to stress, in our view, such increases will ultimately be passed on to the
ordinary Filipino, either directly or indirectly. Conceivably, in extreme instances, the
lessee corporations who are unable to pay exorbitant fees, charges, and rates imposed
by petitioner could be left with no choice but to close shop leaving hundreds if not
thousands of Filipinos jobless. No one needs reminding that higher prices and more
unemployment are the last things our country's challenged economy needs at this
time. Balancing of interests among the parties concerned, in a public hearing, is
obviously called for.

*Pascual v. Board of Medical Examiners, G.R. No. L-25018, May 26, 1969.
(Right against self-incrimination in administrative proceedings)

Facts:
-An administrative case for alleged malpractice was filed against petitioner Arsenio
Pascual, Jr before the Board of Medical Examiners (Board). At the initial hearing,
counsel for complainants announced that he would present as his first witness herein
petitioner, who was the respondent in such malpractice charge.

-Petitioner then went to the RTC, praying that a writ of preliminary injunction be
issued against the respondent Board. The RTC granted the same.

-Board’s contention: The right against self-incrimination is available only when a


question calling for an incriminating answer is asked of a witness. Petitioner’s remedy
is to object once he is in the witness stand.

Issue:

Page 235 of 323


Whether or not a respondent in administrative proceedings may refuse, not only to
answer incriminatory questions, but also to take the witness stand.

Ruling:
Yes.

Although an administrative case, there is clearly the imposition of a penalty. Thus, the
proceeding for a malpractice case while administrative in character, possesses a
criminal or penal aspect. Petitioner could suffer not the forfeiture of property but the
revocation of his license as a medical practitioner, for some an even greater
deprivation. x x x x

We hold that in an administrative hearing against a medical practitioner for alleged


malpractice, respondent Board of Medical Examiners cannot, consistently with the
self-incrimination clause, compel the person proceeded against to take the
witness stand without his consent.

*United Pepsi-Cola Supervisory Union v. Hon. Laguesma, G.R. No. 122226,


March 25, 1998.
(Remedial Law: Res Judicata; Labor Law: Certification Election; Quasi-judicial
proceedings)

Facts:
-Petitioner is a union of supervisory employees. In 1995, the union filed a petition for
certification election on behalf of the route managers at Pepsi-Cola Products
Philippines, Inc. However, its petition was denied by the med-arbiter, on the ground
that the route managers are managerial employees and, therefore, ineligible for
union membership under the first sentence of Art. 245 of the Labor Code, which
provides:

“Ineligibility of managerial employees to join any labor organization; right of supervisory employees. —
Managerial employees are not eligible to join, assist or form any labor organization. Supervisory
employees shall not be eligible for membership in a labor organization of the rank-and-file employees but
may join, assist or form separate labor organizations of their own.”

-The med-arbiter based his ruling that “route managers are managerial employees” on
the two previous determinations by the Secretary of Labor and Employment. In Case
No. OS-MA-10-318-91, entitled Worker's Alliance Trade Union (WATU) v. Pepsi-Cola
Products Philippines, Inc., decided on November 13, 1991, the Secretary of Labor found
that route managers therein are managerial employees. Meanwhile, on July 6, 1992,
said finding was reiterated in Case No. OS-A-3-71-92 entitled In Re: Petition for Direct
Certification and/or Certification Election-Route Managers/Supervisory Employees of
Pepsi-Cola Products Phils. Inc., where the Secretary of Labor ruled that, “route
managers are, by the very nature of their functions and the authority they wield over
their subordinates, managerial employees.”

-On appeal, the Secretary of Labor and Employment affirmed the ruling of the med-
arbiter.

-Petitioners then went to the SC.

-Petitioners’ contention: Previous administrative determinations do not have the


effect of res judicata in the present case, because “labor relations proceedings” are
“non-litigious and summary in nature without regard to legal technicalities.”

Issue:
Whether or not the principle of res judicata applies to the present case.

Ruling:
Yes.

Page 236 of 323


The doctrine of res judicata certainly applies to adversary administrative
proceedings. As early as 1956, in Brillantes v. Castro, we sustained the dismissal of
an action by a trial court on the basis of a prior administrative determination of the
same case by the Wage Administration Service, applying the principle of res judicata.
Recently, in Abad v. NLRC, we applied the related doctrine of stare decisis in holding
that the prior determination that certain jobs at the Atlantic Gulf and Pacific Co., were
project employments was binding in another case involving another group of
employees of the same company. Indeed, in Nasipit Lumber Co., this Court clarified
toward the end of its opinion that “the doctrine of res judicata applies . . . to
judicial or quasi-judicial proceedings and not to the exercise of administrative
powers.” Now proceedings for certification election, such as those involved in Case
No. OS-M-A-10-318-91 and Case No. OS-A-3-71-92, are quasi-judicial in nature
and, therefore, decisions rendered in such proceedings can attain finality. x x x x

*Encinas v. Agustin, G.R. No. 187317, April 11, 2013.


(Affidavit of Desistance in Administrative Proceedings)

-Petitioner was held administratively liable for grave misconduct and conduct
prejudicial to the best interest of service and was ordered dismissed.

-One of his arguments on appeal was the fact that the complainants had already
executed an affidavit of desistance in his favor.

Issue:
Whether or not the affidavit of desistance executed by the complainants herein
exonerates petitioner from administrative liability.

Ruling:
No.

Even assuming that an Affidavit of Desistance was indeed executed by respondents,


petitioner is still not exonerated from liability. The subsequent reconciliation of the
parties to an administrative proceeding does not strip the court of its
jurisdiction to hear the administrative case until its resolution. Atonement, in
administrative cases, merely obliterates the personal injury of the parties and
does not extend to erase the offense that may have been committed against the
public service. The subsequent desistance by respondents does not free petitioner
from liability, as the purpose of an administrative proceeding is to protect the
public service based on the time-honored principle that a public office is a public
trust. A complaint for malfeasance or misfeasance against a public servant of
whatever rank cannot be withdrawn at any time for whatever reason by a
complainant, as a withdrawal would be anathema to the preservation of the faith and
confidence of the citizenry in their government, its agencies and instrumentalities.
Administrative proceedings should not be made to depend on the whims and
caprices of complainants who are, in a real sense, only witnesses therein.

*Smart v. NTC, G.R. No. 151908, August 12, 2003.


(Jurisdiction; Quasi-legislative functions; Exhaustion of administrative remedies)

Facts:
-Pursuant to its rule-making and regulatory powers, the National Telecommunications
Commission (NTC) issued Memorandum Circular No. 13-6-2000, promulgating rules
and regulations on the billing of telecommunications services.

-The said circular provided, among others, that: (1) There shall be no charge for calls
that are diverted to a voice mailbox, voice prompt, recorded message or similar facility
excluding the customer's own equipment; (2) Public telecommunications entities
(PTEs) shall verify the identification and address of each purchaser of prepaid SIM
cards; (3) The unit of billing for the cellular mobile telephone service whether postpaid
or prepaid shall be reduced from 1 minute per pulse to 6 seconds per pulse. The
authorized rates per minute shall thus be divided by 10.

Page 237 of 323


-In view of the said circular, petitioners Isla Communications Co., Inc. and Pilipino
Telephone Corporation filed against the NTC an action for declaration of nullity of the
same, with prayer for the issuance of a writ of preliminary injunction and temporary
restraining order.

-Petitioners’ contentions: (1) The NTC has no jurisdiction to regulate the sale of
consumer goods such as the prepaid call cards since such jurisdiction belongs to the
Department of Trade and Industry under the Consumer Act of the Philippines; (2) The
Billing Circular is oppressive, confiscatory and violative of the constitutional
prohibition against deprivation of property without due process of law. Hence, they
prayed that the Billing Circular be declared null and void ab initio.

-Respondent NTC and its co-defendants filed with the RTC a motion to dismiss the
case on the ground of petitioners’ failure to exhaust administrative remedies.

-After due proceedings, the RTC ruled in favor of petitioners and granted the writ of
injunction.

-NTC thus filed a special civil action for certiorari and prohibition with the Court of
Appeals. The CA reversed the ruling of the RTC, ratiocinating that petitioners failed to
exhaust administrative remedies as it was the NTC and not the RTC that has the
primary jurisdiction over the dispute.

-The petitioners then went to the SC via Rule 45.

Issues:
(1) Whether or not petitioners failed to exhaust administrative remedies;
(2) Whether or not the RTC has jurisdiction over the case;

Ruling:
(1) No.

Administrative agencies possess quasi-legislative or rule-making powers and quasi-


judicial or administrative/adjudicatory powers. Quasi-legislative or rule-making power
is the power to make rules and regulations which results in delegated legislation that
is within the confines of the granting statute and the doctrine of non-delegability and
separability of powers.

In questioning the validity or constitutionality of a rule or regulation issued by


an administrative agency, a party need not exhaust administrative remedies
before going to court. The principle of exhaustion of administrative remedies
applies only where the act of the administrative agency concerned was
performed pursuant to its quasi-judicial function, and not when the assailed act
pertained to its rule-making or quasi-legislative power.

(2) Yes.

Where what is assailed is the validity or constitutionality of a rule or regulation


issued by the administrative agency in the performance of its quasi-legislative
function, the regular courts have jurisdiction to pass upon the same. The
determination of whether a specific rule or set of rules issued by an administrative
agency contravenes the law or the constitution is within the jurisdiction of the regular
courts. Indeed, the Constitution vests the power of judicial review or the power to
declare a law, treaty, international or executive agreement, presidential decree,
order, instruction, ordinance, or regulation in the courts, including the regional
trial courts.

This is within the scope of judicial power, which includes the authority of the
courts to determine in an appropriate action the validity of the acts of the
political departments. Judicial power includes the duty of the courts of justice to

Page 238 of 323


settle actual controversies involving rights which are legally demandable and
enforceable, and to determine whether or not there has been a grave abuse of
discretion amounting to lack or excess of jurisdiction on the part of any branch or
instrumentality of the Government.

In the case at bar, the issuance by the NTC of the subject circular was pursuant to its
quasi-legislative or rule-making power. As such, petitioners were justified in
invoking the judicial power of the Regional Trial Court to assail the
constitutionality and validity of the said issuance.

In their complaint before the Regional Trial Court, petitioners averred that the Circular
contravened Civil Code provisions on sales and violated the constitutional
prohibition against the deprivation of property without due process of law. These are
within the competence of the trial judge. Contrary to the finding of the Court of
Appeals, the issues raised in the complaint do not entail highly technical
matters.

***LAW OF PUBLIC OFFICERS***

*Romualdez III v. CSC, G.R. Nos. 94878-94881, May 15, 1991.


(Appointment; Permanent and Temporary; Security of Tenure; Remedial Law:
Mandamus)

Facts:
-Petitioner was extended a permanent appointment as a Commercial Attache of the
Department of Trade continuously for twelve years from September, 1975 to August
30, 1987. His civil service eligibilities are: Patrolman of the City of Manila (1963 CS
Exam) and a Commercial Attache (1973 CS Exam).

-On September 1, 1987, he was transferred to the respondent Philippine Coconut


Authority (PCA) whereby he was extended an appointment as Deputy Administrator for
Industrial Research and Market Development. The nature of his employment status
was “temporary,” for the period covering September 1, 1987 to August 30, 1988.

-His appointment was renewed for another six months from September 1, 1988 to
February 28, 1989 also on a “temporary” status and subject to certain conditions to
which petitioner agreed.

-When his appointment expired on February 28, 1989, the Governing Board did not
renew the same so he was promptly informed thereof by the Acting Chairman of the
Board.

-Petitioner then appealed to respondent Civil Service Commission, requesting


reinstatement to his previous position in the PCA. The CSC denied his petitioner’s
request.

-Petitioner then went to the SC via petition for mandamus under Rule 65.

Issues:
(1) Whether or not petitioner’s right to security of tenure was violated;
(2) Whether or not mandamus should issue under the circumstances.

Ruling:
(1) No.

No doubt the appointment extended to petitioner by respondent PCA as PCA Deputy


Administrator for Industrial Research and Market Development was temporary.
Although petitioner was formerly holding a permanent appointment as a commercial
attache, he sought and accepted this temporary appointment to respondent PCA.

Page 239 of 323


His temporary appointment was for a definite period and when it lapsed and was not
renewed on February 28, 1987, he complains that there was a denial of due process.
This is not a case of removal from office. Indeed, when he accepted this
temporary appointment he was thereby effectively divested of security of tenure.
A temporary appointment does not give the appointee any definite tenure of
office but makes it dependent upon the pleasure of the appointing power.

(2) No.

The matter of converting such a temporary appointment to a permanent one is


addressed to the sound discretion of the appointing authority. Respondent CSC
cannot direct the appointing authority to make such an appointment if it is not
so disposed. x x x x

It is recognized that, “the appointing authority is given a wide latitude of


discretion in the selection of personnel of his department or agency.” Respondent
PCA exercised its discretion and opted not to extend the appointment of petitioner. It
cannot be compelled to extend petitioner's appointment, much less can it be
directed to extend a permanent appointment to petitioner. A discretionary duty
cannot be compelled by mandamus. More so when as in this case petitioner has not
shown a lawful right to the position. If the legal rights of the petitioner are not well-
defined, clear and certain, the petition must be dismissed.

*Camarines Norte v. Gonzales, G.R. No. 185740, July 23, 2013. [IMPORTANT!!!]
(Security of Tenure; Primarily Confidential positions; CSC)

Facts:
-Respondent Beatriz O. Gonzales (Gonzales) was appointed as the provincial
administrator of the Province of Camarines Norte by then Governor Roy A. Padilla, Jr.
on April 1, 1991. Her appointment was on a permanent capacity.

-At that time, the position of the provincial administrator was classified as a career
service position, and hence, Gonzales could not be removed from her position
without just cause.

-Eventually, an administrative case was filed against Gonzales and after due
proceedings, the CSC found her guilty of insubordination and suspended her for six
months.

-At this time, the position of provincial administrator has been re-classified into a
non-career service position—which is highly confidential, coterminous position
by R.A. 7160.

-After serving her six-month suspension, the CSC issued a resolution directing
Gonzales’ reinstatement.

-Governor Pimentel reinstated Gonzales as provincial administrator on October 12,


2000, but terminated her services the next day for loss of confidence. He then wrote
a letter to the CSC reporting his compliance with its order, and Gonzales’ subsequent
dismissal as a confidential employee.

-The CSC responded through a resolution directing Gonzales’ reinstatement as


provincial administrator. It clarified that while the Local Government Code of 1991
(Republic Act No. RA 7160) made the provincial administrator position coterminous
and highly confidential in nature, this conversion cannot retroactively operate to
prejudice officials who were already issued permanent appointments as administrators
prior to the new law’s effectivity.

-The governor (petitioner) then filed a petition for review before the CA, which affirmed
CSC’s ruling.

Page 240 of 323


-Petitioner then went to the SC via petition for review on certiorari.

Issue:
Whether or not Gonzales’ security of tenure was violated under the circumstances.

Ruling:
No.

In the current case, Congress, through R.A. 7160, did not abolish the provincial
administrator position but significantly modified many of its aspects. It is now a
primarily confidential position under the non-career service tranche of the civil
service. This change could not have been aimed at prejudicing Gonzales, as she was
not the only provincial administrator incumbent at the time RA 7160 was enacted.
Rather, this change was part of the reform measures that R.A. 7160 introduced to
further empower local governments and decentralize the delivery of public service. x x
xx

Thus, Gonzales’ permanent appointment as provincial administrator prior to the


enactment of R.A. 7160 is immaterial to her removal as provincial
administrator. For purposes of determining whether Gonzales’ termination
violated her right to security of tenure, the nature of the position she occupied
at the time of her removal should be considered, and not merely the nature of
her appointment at the time she entered government service. x x x x

Moreover, it is a basic tenet in the country's constitutional system that, “public office
is a public trust,” and that there is no vested right in public office, nor an absolute
right to hold office. No proprietary title attaches to a public office, as public service is
not a property right. Excepting constitutional offices which provide for special
immunity as regards salary and tenure, no one can be said to have any vested right in
an office. The rule is that offices in government, except those created by the
constitution, may be abolished, altered, or created anytime by statute. Any
issues on the classification for a position in government may be brought to and
determined by the courts. x x x x

To be sure, both career and non-career service employees have a right to security of
tenure. All permanent officers and employees in the civil service, regardless of whether
they belong to the career or non-career service category, are entitled to this guaranty;
they cannot be removed from office except for cause provided by law and after
procedural due process. The concept of security of tenure, however, labors under a
variation for primarily confidential employees due to the basic concept of a “primarily
confidential” position. Serving at the confidence of the appointing authority, the
primarily confidential employee’s term of office expires when the appointing
authority loses trust in the employee. When this happens, the confidential
employee is not “removed” or “dismissed” from office; his term merely “expires”
and the loss of trust and confidence is the “just cause” provided by law that
results in the termination of employment. In the present case where the trust and
confidence has been irretrievably eroded, we cannot fault Governor Pimentel’s exercise
of discretion when he decided that he could no longer entrust his confidence in
Gonzales. x x x x

Thus, Gonzales’ termination for loss of confidence was lawful. She could no longer
be reinstated as provincial administrator of Camarines Norte or to any other
comparable position. This conclusion, however, is without prejudice to Gonzales’
entitlement to retirement benefits, leave credits, and future employment in
government service.

*Uy v. Hon. Flores, A.M. No. RTJ-12-2332, June 25, 2014.


(CSC; Jurisdiction; Transfer/Reassignment)

Facts:

Page 241 of 323


-In a Revenue Travel Assignment Order, Commissioner of Internal Revenue Lilian B.
Hefti relieved Mustapha M. Gandarosa as Regional Director of Revenue Region No. 16,
Bureau of Internal Revenue, Cagayan de Oro City. Hefti reassigned Gandarosa as
Chief of Staff of the Special Concerns Group at the Bureau’s Head Office in Quezon
City. Secretary of Finance Margarito B. Teves approved Hefti's order.

-Gandarosa then filed a Rule 65 petition for certiorari and/or prohibition with prayer
for a temporary restraining order before the Regional Trial Court, Branch 7, Tubod,
Lanao del Norte, presided by respondent Judge Flores, praying that Hefti’s order be
declared void and that a writ of injunction be issued prohibiting the Secretary of
Finance and the new Commissioner of Internal Revenue from enforcing Hefti’s order
and from replacing or reassigning him.

-Judge Flores granted a temporary restraining order and writ of preliminary


injunction in favor of Gandarosa.

-Meanwhile, the new Commissioner of Internal Revenue, Sixto S. Esquivias IV, issued
a new Revenue Travel Assignment Order reiterating Hefti’s order. Secretary Teves also
approved Esquivias’s order. Gandarosa thus filed a petition for indirect contempt
against Secretary Teves and Commissioner Esquivias.

-Judge Flores then ordered Secretary Teves, Commissioner Esquivias and their
subordinate officials to maintain the status quo and retain Gandarosa as Regional
Director of Revenue Region No. 16.

-On appeal, the CA annulled the ruling of Judge Flores, and ordered the latter to
dismiss Gandarosa’s Rule 65 petititon. The CA ruled that the trial court lacks
jurisdiction over the Rule 65 petition. Said CA Decision attained finality and entry of
judgment was made.

-Complainant Uy then filed the present administrative case alleging that Judge Flores
exhibited gross ignorance of the law when he assumed jurisdiction over the Rule 65
petition as it is the Civil Service Commission which has jurisdiction over the issue of
Gandarosa’s reassignment.

Issue:
Whether or not Judge Flores is guilty of gross ignorance of the law.

Ruling:
Yes.

When a law or a rule is basic, judges owe it to their office to simply apply the law.
Anything less is gross ignorance of the law. There is gross ignorance of the law when
an error committed by the judge was gross or patent, deliberate or malicious. Gross
ignorance of the law or incompetence cannot be excused by a claim of good faith.
When an error is so gross and patent, such error produces an inference of bad
faith, making the judge liable for gross ignorance of the law.

Here, Judge Flores assumed jurisdiction over the Rule 65 petition assailing Hefti’s
order when he should have dismissed the petition for Gandarosa’s failure to
exhaust administrative remedies. An employee who questions the validity of his
transfer should appeal to the Civil Service Commission per Section 26(3), Chapter
5, Subtitle A, Book V of the Administrative Code of 1987, which reads:

“SEC. 26. Personnel Actions. – x x x


xxxx
(3) Transfer. x x x
x x x. If the employee believes that there is no justification for the transfer, he may appeal his case to
the [Civil Service] Commission.”

Page 242 of 323


Moreover, under the law, any employee who questions the validity of his transfer
should appeal to the Civil Service Commission. Respondent judge should have
dismissed the action below for failure of private respondent to exhaust administrative
remedies.

The law is basic and jurisprudence is clear but Judge Flores failed to apply them.
Judge Flores committed a gross and patent error which makes him liable for gross
ignorance of the law notwithstanding his claim of good faith. Judge Flores even
mentioned in the Order dated November 21, 2008 the contention of the Office of the
Solicitor General that the trial court lacks jurisdiction over the case. Judge Flores’s
gross and patent error produces an inference of bad faith on his part, considering that
the issue of jurisdiction was raised.

Even if we assume that the trial court has jurisdiction over Gandarosa’s Rule 65
petition, Section 4, Rule 65 of the Rules of Court requires that the petition must be
filed in the Regional Trial Court exercising jurisdiction over the territorial area as
defined by the Supreme Court. But the trial court presided by Judge Flores is within
the 12th Judicial Region while the Head Office and Regional Office, Revenue Region
No. 16, of the Bureau of Internal Revenue are respectively located in Metro Manila,
National Capital Judicial Region,and Cagayan de Oro City, 10th Judicial Region.
Judge Flores issued a temporary restraining order and writ of preliminary injunction
against the Secretary of Finance and Commissioner of Internal Revenue who both
hold office in Metro Manila, outside the territorial area where his court can
exercise its jurisdiction.

[NOTE: The SC suspended Judge Flores for three months and one day without pay.]

*Ombudsman v. Racho, G.R. No. 185685, January 31, 2011.


(SALN; Dishonesty; Negligence)

-DYHP Balita Action Team (DYHP) reported to Deputy Ombudsman for the Visayas,
Primo Miro, a concerned citizen’s complaint regarding the alleged unexplained wealth
of Racho, then Chief of the Special Investigation Division of the Bureau of Internal
Revenue (BIR), Cebu City.

-To support the allegation, the complainant attached copies of bank certifications, all
issued in June of 1999, by Metrobank Cebu (Tabunok Branch), BPI Cebu (Mango
Branch), and PCI Bank (Magallanes Branch). In total, Racho appeared to have an
aggregate bank deposit of ₱5,798,801.39.

-Acting on the letter, the Ombudsman launched a fact-finding investigation and


directed the BIR to submit Racho’s Statements of Assets, Liabilities and Net Worth
(SALN) from 1995 to 1999. BIR complied with the order and gave copies of Racho’s
SALN.

-Rancho admitted the bank accounts but he explained that the deposits reflected
therein were not entirely his. Racho proffered that some of the money came from his
brothers and nephew as part of their contribution to the business that they had
planned to put up.

-Eventually, the Ombudsman found respondent Nieto A. Racho (Racho) guilty of


dishonesty and ordered him dismissed from the service with forfeiture of all benefits
and perpetual disqualification from public office.

-Upon appeal, however, the CA found Racho guilty of negligence only and reduced the
penalty to suspension from office for six months, without pay.

-The Ombudsman then went to the SC via petition for review on certiorari under Rule
45.

Issue:

Page 243 of 323


Whether or not Racho’s non-disclosure of the bank deposits in his SALN constitutes
dishonesty.

Ruling:
Yes.

Section 7 and Section 8 of Republic Act (R.A.) 301938 explain the nature and
importance of accomplishing a true, detailed and sworn SALN, thus:

“Sec. 7. Statement of Assets and Liabilities. — Every public officer, within thirty days after assuming
office, and thereafter, on or before the fifteenth day of April following the close of every calendar year, as
well as upon the expiration of his term of office, or upon his resignation or separation from office, shall
prepare and file with the office of corresponding Department Head, or in the case of a Head Department
or chief of an independent office, with the Office of the President, a true, detailed and sworn statement of
the amounts and sources of his income, the amounts of his personal and family expenses and the
amount of income taxes paid for the next preceding calendar year: Provided, That public officers
assuming office less than two months before the end of the calendar year, may file their first statement on
or before the fifteenth day of April following the close of said calendar year.

Sec. 8. Prima Facie Evidence of and Dismissal Due to Unexplained Wealth. — If in accordance with the
provisions of Republic Act Numbered One Thousand Three Hundred Seventy-Nine, a public official has
been found to have acquired during his incumbency, whether in his name or in the name of other
persons, an amount of property and/or money manifestly out of proportion to his salary and to his other
lawful income, that fact shall be ground for dismissal or removal. Properties in the name of the spouse
and dependents of such public official may be taken into consideration, when their acquisition through
legitimate means cannot be satisfactorily shown. Bank deposits in the name of or manifestly excessive
expenditures incurred by the public official, his spouse or any of their dependents including but not
limited to activities in any club or association or any ostentatious display of wealth including frequent
travel abroad of a non-official character by any public official when such activities entail expenses
evidently out of proportion to legitimate income, shall likewise be taken into consideration in the
enforcement of this Section, notwithstanding any provision of law to the contrary. The circumstances
hereinabove mentioned shall constitute valid ground for the administrative suspension of the public
official concerned for an indefinite period until the investigation of the unexplained wealth is completed.”

By mandate of law, every public official or government employee is required to make a


complete disclosure of his assets, liabilities and net worth in order to suppress any
questionable accumulation of wealth because the latter usually results from non-
disclosure of such matters. Hence, a public official or employee who has acquired
money or property manifestly disproportionate to his salary or his other lawful
income shall be prima facie presumed to have illegally acquired it.

It should be understood that what the law seeks to curtail is “acquisition of


unexplained wealth.” Where the source of the undisclosed wealth can be properly
accounted, then it is “explained wealth” which the law does not penalize.

In this case, Racho not only failed to disclose his bank accounts containing
substantial deposits but he also failed to satisfactorily explain the accumulation
of his wealth or even identify the sources of such accumulated wealth. The
documents that Racho presented, like those purportedly showing that his brothers
and nephew were financially capable of sending or contributing large amounts of
money for their business, do not prove that they did contribute or remit money for
their supposed joint business venture. Equally, the Special Power of Attorney that was
supposedly issued by Vieto, Dido and Henry Racho in favor of Racho on January 28,
1993 to show their business plans, contained a glaringly inconsistent statement that
belies the authenticity of the document.

More important, the Joint Affidavits allegedly executed by Racho’s siblings and
nephew to corroborate his story were later disowned and denied by his nephew,
Henry, and brother, Vieto, as shown by their Counter-Affidavits. Henry averred that he
was out of the country at the time of the alleged execution of the Joint Affidavit on
December 18, 2004 and he arrived in Manila only on September 16, 2005. Vieto, on
the other hand, denied having signed the Joint Affidavit. He disclosed that as a left-
handed person, he pushes the pen instead of pulling it. He concluded that the
signature on the Joint Affidavit was made by a right-handed person. He likewise
included a copy of his passport containing his real signature for comparison.

Page 244 of 323


Thus, the SPA and Joint Affidavits which should explain the sources of Racho’s wealth
are dubious and merit no consideration. x x x x

[IMPORTANT NOTE FROM THIS CASE:

“It should be emphasized, however, that mere misdeclaration of the SALN does not
automatically amount to dishonesty. Only when the accumulated wealth
becomes manifestly disproportionate to the employee’s income or other sources
of income and the public officer/employee fails to properly account or explain
his other sources of income, does he become susceptible to dishonesty because
when a public officer takes an oath or office, he or she binds himself or herself to
faithfully perform the duties of the office and use reasonable skill and diligence, and to
act primarily for the benefit of the public. Thus, in the discharge of duties, a public
officer is to use that prudence, caution and attention which careful persons use in the
management of their affairs.”

*Civil Service Commission v. Cortes, G.R. No. 200103, April 23, 2014.
(Prohibitions; Nepotism; CSC)

Facts:
-In 2008, the Commission En Banc of the Commission on Human Rights (CHR) issued
a Resolution approving the appointment to the position of Information Officer V (IO V)
of respondent Maricelle M. Cortes (Cortes).

-Cortes is the daughter of CHR Commissioner Eligio P. Mallari, who abstained from
voting and requested the CHR to render an opinion on the legality of the respondent's
appointment.

-In a Memorandum, CHR Legal Division Chief Atty. Efren Ephraim G. Lamorena
rendered an opinion that respondent Cortes’ appointment is not covered by the rule on
nepotism because the appointing authority, the Commission En Banc, has a
personality distinct and separate from its members.

-Thereafter, the Civil Service Commission-NCR (CSC-NCR) Field Office informed CHR
Chairperson Quisumbing that it would conduct an investigation on the appointment of
respondent Cortes.

-The Director of the CSC-NCR Field Office then informed CHR Chairperson
Quisumbing that the appointment of respondent Cortes is not valid because it is
covered by the rule on nepotism under Section 9 of the Revised Omnibus Rules on
Appointments and Other Personnel Actions.

-Respondent Cortes then filed a petition for review before the CSC, which denied the
same.

-Consequently, CHR Commissioner and Officer-in-Charge Ma. Victoria V. Cardona


terminated respondent’s services.

-Respondent Cortes then filed a Petition for Review under Rule 43 with Prayer for
Issuance of Temporary Restraining Order and/or Writ of Preliminary Injunction before
the Court of Appeals (CA), which later granted Cortes’ petition and reinstated her to
her position as IO V in the CHR.

-The CSC then went to the SC via petition for review on certiorari under Rule 45.

-Contentions of Cortes: (1) The appointing authority referred to in Section 59 of the


Administrative Code is the Commission En Banc and not the individual
Commissioners who compose it; (2) Commissioner Mallari, father of respondent Cortes
abstained from voting, making the appointment not nepotistic.

Page 245 of 323


Issue:
Whether or not the appointment of Cortes is covered by the prohibition against
nepotism.

Ruling:
Yes.

Nepotism is defined as an appointment issued in favor of a relative within the


third civil degree of consanguinity or affinity of any of the following: (1)
appointing authority; (2) recommending authority; (3) chief of the bureau or
office; and (4) person exercising immediate supervision over the appointee. Here,
it is undisputed that respondent Cortes is a relative of Commissioner Mallari in the
first degree of consanguinity, as in fact Cortes is the daughter of Commissioner
Mallari.

By way of exception, the following shall not be covered by the prohibition: (1)
persons employed in a confidential capacity; (2) teachers; (3) physicians; and (4)
members of the Armed Forces of the Philippines. In the present case, however, the
appointment of respondent Cortes as IO V in the CHR does not fall to any of the
exemptions provided by law.

The purpose of Section 59 on the rule against nepotism is to take out the discretion of
the appointing and recommending authority on the matter of appointing or
recommending for appointment a relative. The rule insures the objectivity of the
appointing or recommending official by preventing that objectivity from being in fact
tested. Clearly, the prohibition against nepotism is intended to apply to natural
persons. It is one pernicious evil impeding the civil service and the efficiency of its
personnel.

Moreover, basic rule in statutory construction is the legal maxim that "we must
interpret not by the letter that killeth, but by the spirit that giveth life." To rule that
the prohibition applies only to the Commission, and not to the individual
members who compose it, will render the prohibition meaningless. Apparently,
the Commission En Banc, which is a body created by fiction of law, can never
have relatives to speak of.

Indeed, it is absurd to declare that the prohibitive veil on nepotism does not
include appointments made by a group of individuals acting as a body. What
cannot be done directly cannot be done indirectly. This principle is elementary and
does not need explanation. Certainly, if acts that cannot be legally done directly can be
done indirectly, then all laws would be illusory.

In the present case, respondent Cortes' appointment as IO V in the CHR by the


Commission En Banc, where his father is a member, is covered by the prohibition.
Commissioner Mallari's abstention from voting did not cure the nepotistic
character of the appointment because the evil sought to be avoided by the
prohibition still exists. His mere presence during the deliberation for the
appointment of IO V created an impression of influence and cast doubt on the
impartiality and neutrality of the Commission En Banc.

*Rama v. Court of Appeals, G.R. No. L-44484, March 16, 1987.


(Liability for Tort; Partisan Politics; Personal Liability)

-The Sangguniang Panlalawigan of Cebu issued a resolution “to economize the


expenditure of its Road and Bridge Fund for the maintenance and repair of provincial
roads and bridges receiving national aid and to adopt a more comprehensive,
systematic, efficient, progressive and orderly operation and maintenance of the Office
of the Provincial Engineer."

-To implement said policy, the provincial board resolved to abolish around thirty
positions the salaries of which were paid from the "JJ" Road and Bridge Fund.

Page 246 of 323


-Consequently, about 200 employees of the province were eased out of their
respective jobs and, to implement the mechanization program in the maintenance of
roads and bridges, the provincial government purchased heavy equipment worth
P4,000,000.00.

-However, contrary to its declared policy to economize, the provincial administration


later on hired around one thousand new employees, renovated the office of the
provincial engineer and provided the latter with a Mercedes-Benz car.

-Aggrieved by these turn of events, the employees whose positions were abolished filed
separate petitions for mandamus, damages and attorneys fees aimed at the
annulment of the said resolution, their reinstatement and the recovery of damages.

-The provincial governor, vice-governor, members of the Sangguniang Panlalawigan,


provincial auditor, provincial treasurer and provincial engineer were named
respondents in said action and were sued “both in their official and personal
capacities” as a result of their alleged unjust, oppressive, illegal and malicious’ acts.

Issue:
Whether or not the said provincial officers are personally liable for damages for
adopting the subject resolution.

Ruling:
Yes.

A public officer by virtue of his office alone, is not immune from damages in his
personal capacity arising from illegal acts done in bad faith. A different rule
would sanction the use of public office as a tool of oppression.

Thus, in Correa vs. CFI of Bulacan, We held personally liable a mayor who illegally
dismissed policemen even if he had relinquished his position. Therein, We held that:

“A public officer who commits a tort or other wrongful act, done in excess or
beyond the scope of his duty, is not protected by his office and is personally
liable therefor like any private individual. This principle of personal liability has
been applied to cases where a public officer removes another officer or discharges an
employee wrongfully, the reported cases saying that by reason of non-compliance with
the requirements of law in respect to removal from office, the officials were acting
outside of their official authority.”

We hold that the petitioners in the instant three cases are personally liable for
damages because of their precipitate dismissal of provincial employees through an
ostensibly legal means.

Records show that the provincial employees concerned were "eased out because of
their party affiliation." i.e., they belonged to the Liberal Party whose presidential
candidate then was Sergio Osmena Jr. Such act of the petitioners reflected their
malicious intent to do away with the followers of the rival political party so as to
accommodate their own proteges who, it turned out, even outnumbered the
dismissed employees.

Indeed, municipal officers are liable for damages if they act maliciously or
wantonly and if the work which they perform is done rather to injure an
individual than to discharge a public duty. As we have held in Vda de Laig vs. Court
of Appeals, a public officer is civilly liable for failure to observe honesty and good faith
in the performance of their duties as public officers or for wilfully or negligently
causing damage to another (Article 20, Civil Code) or for wilfully causing loss or injury
to another in a manner that is contrary to morals, good customs and/or public policy.
xxxx

Page 247 of 323


It is an undeniable fact that the dismissed employees who were holding such
positions as foremen, watchmen and drivers, suffered the uncertainties of the
unemployed when they were plucked out of their positions. That not all of them
testified as to the extent of damages they sustained on account of their separation
from their government jobs, cannot be used as a defense by the petitioners. Suffice it
to state that considering the positions they were holding, the dismissed employees
concerned belong to a low-salaried group, who, if deprived of wages would generally
incur considerable economic hardships.

*Ombudsman v. Andutan Jr., G.R. No. 164679, July 27, 2011.


(CSC; Administrative Case; Resignation; Ombudsman’s Authority)

Facts:
-Uldarico P. Andutan, Jr. was formerly the Deputy Director of the One-Stop Shop Tax
Credit and Duty Drawback Center of the Department of Finance (DOF). On June 30,
1998, then Executive Secretary Ronaldo Zamora issued a Memorandum directing all
non-career officials or those occupying political positions to vacate their positions
effective July 1, 1998.

-On July 1, 1998, pursuant to the Memorandum, Andutan resigned from the DOF.

-On September 1, 1999, Andutan, together with Rowena P. Malonzo and several
other persons, was criminally charged by the Fact Finding and Intelligence Bureau
(FFIB) of the Ombudsman with Estafa through Falsification of Public Documents, and
violations of Section 3(a), (e) and (j) of Republic Act No. (R.A.) 3019, otherwise known
as the Anti-Graft and Corrupt Practices Act.

-As government employees at the time of the alleged commission of the offense,
Andutan, Belicena and Malonzo were likewise administratively charged of Grave
Misconduct, Dishonesty, Falsification of Official Documents and Conduct Prejudicial
to the Best Interest of the Service.

-After due proceedings, the Ombudsman found the respondents guilty of Gross
Neglect of Duty. Having been separated from the service, Andutan was imposed the
penalty of forfeiture of all leaves, retirement and other benefits and privileges, and
perpetual disqualification from reinstatement and/or reemployment in any branch or
instrumentality of the government, including government owned and controlled
agencies or corporations.

-Andutan then filed a petition for review on certiorari before the CA, which annulled
and set aside the decision of the Ombudsman. The CA ruled that: (1) the Ombudsman
may not conduct the necessary investigation of any administrative act or omission
complained of if it the complaint was filed after one year from the occurrence of the act
or omission complained of; and (2) the administrative case was filed after Andutan’s
forced resignation.

-Section 20 (5) of R.A. No. 6770 provides that:

SEC. 20. Exceptions. – The Office of the Ombudsman may not conduct the necessary investigation
of any administrative act or omission complained of if it believes that:
xxxx
(5) The complaint was filed after one year from the occurrence of the act or omission complained
of.

-Aggrieved by the CA’s decision, the Ombudsman then went to the SC via petition for
review on certiorari.

-Contentions of the Ombudsman: (1) Administrative offenses do not prescribe after


one year from their commission; and (2) in cases of “capital” administrative offenses,
resignation or optional retirement cannot render administrative proceedings moot and

Page 248 of 323


academic, since accessory penalties such as perpetual disqualification and the
forfeiture of retirement benefits may still be imposed.

Issues:
(1) Whether or not Section 20(5) of R.A. 6770 prohibit the Ombudsman from
conducting an administrative investigation a year after the act was committed;
(2) Whether or not Ombudsman retains its authority to institute an administrative
complaint against a government employee who had already resigned.

Ruling:
(1) No.

The provisions of Section 20(5) are merely directory; the Ombudsman is not
prohibited from conducting an investigation a year after the supposed act was
committed.

The issue of whether Section 20(5) of R.A. 6770 is mandatory or discretionary has
been settled by jurisprudence. In Office of the Ombudsman v. De Sahagun, the Court
held:

Well-entrenched is the rule that administrative offenses do not prescribe.


Administrative offenses by their very nature pertain to the character of public officers
and employees. In disciplining public officers and employees, the object sought is not
the punishment of the officer or employee but the improvement of the public service
and the preservation of the public’s faith and confidence in our government. x x x x

The period stated in Section 20(5) of R.A. No. 6770 does not refer to the
prescription of the offense but to the discretion given to the Ombudsman on
whether it would investigate a particular administrative offense. The use of the
word “may” in the provision is construed as permissive and operating to confer
discretion. x x x x

Clearly, Section 20 of R.A. 6770 does not prohibit the Ombudsman from
conducting an administrative investigation after the lapse of one year, reckoned
from the time the alleged act was committed. Without doubt, even if the
administrative case was filed beyond the one (1) year period stated in Section 20(5),
the Ombudsman was well within its discretion to conduct the administrative
investigation.

(2) No.

Andutan’s prior resignation divests the Ombudsman of its right to institute an


administrative complaint against him.

Although the Ombudsman is not precluded by Section 20(5) of R.A. 6770 from
conducting the investigation, the Ombudsman can no longer institute an
administrative case against Andutan because the latter was not a public servant
at the time the case was filed.

The Ombudsman argued – in both the present petition and in the petition it filed with
the CA – that Andutan’s retirement from office does not render moot any
administrative case, as long as he is charged with an offense he committed while in
office. x x x x

To recall, we have held in the past that a public official’s resignation does not render
moot an administrative case that was filed prior to the official’s resignation. In Pagano
v. Nazarro, Jr., we held that:

In Office of the Court Administrator v. Juan, this Court categorically ruled that the
precipitate resignation of a government employee charged with an offense

Page 249 of 323


punishable by dismissal from the service does not render moot the
administrative case against him. Resignation is not a way out to evade
administrative liability when facing administrative sanction. The resignation of a
public servant does not preclude the finding of any administrative liability to
which he or she shall still be answerable.

However, the facts of the above-mentioned case are not entirely applicable to the
present case. In the above-cited case, the Court found that the public officials –
subject of the administrative cases – resigned, either to prevent the continuation
of a case already filed or to pre-empt the imminent filing of one. Here, neither
situation obtains.

The Ombudsman’s general assertion that Andutan pre-empted the filing of a case
against him by resigning, since he "knew for certain that the investigative and
disciplinary arms of the State would eventually reach him" is unfounded. First,
Andutan’s resignation was neither his choice nor of his own doing; he was forced
to resign. Second, Andutan resigned from his DOF post on July 1, 1998, while the
administrative case was filed on September 1, 1999, exactly one (1) year and two
(2) months after his resignation. The Court struggles to find reason in the
Ombudsman’s sweeping assertions in light of these facts.

What is clear from the records is that Andutan was forced to resign more than a
year before the Ombudsman filed the administrative case against him.
Additionally, even if we were to accept the Ombudsman’s position that Andutan
foresaw the filing of the case against him, his forced resignation negates the claim that
he tried to prevent the filing of the administrative case.

*Del Castillo v. CSC, G.R. No. 112513, February 14, 1995.


(Right to Back Salaries; Illegal Dismissal)

Facts:
-Petitioner Del Castillo was an employee of the respondent Professional Regulation
Commission (PRC). On July 16-17, 1990, he was assigned as a watcher in the
Optometry Licensure Examination.

-Later, on July 24, 1990 and July 27, 1990, Susan O. Corpuz and Enriquieta Flores,
also watchers in the said licensure examination, respectively wrote the CSC stating
that they saw Del Castillo answering the test papers submitted by an examinee.

-On August 1, 1990, Del Castillo was charged with grave misconduct by PRC
Commissioner Julio B. Francia, Jr.

-Del Castillo was eventually found guilty as charged and was dismissed from service.

-Petitioner appealed to the Merit System Protection Board (MSPB), which then handed
down its decision exonerating Del Castillo, there being no substantial evidence
adduced to support the conviction or finding of guilt. The MSPB then ordered the
reinstatement of Del Castillo, but its decision was silent on the award of back
salaries.

-The PRC appealed the decision of the MSPB to the respondent Civil Service
Commission (CSC), which set aside the ruling of the MSPB and ordered the dismissal
of Del Castillo.

-Petitioner then went to the SC via Petition for Certiorari under Rule 65.

Issues:
(1) Whether or not the CSC gravely abused its discretion when it gave due course to
the appeal of PRC from MSPB’s decision;
(2) Whether or not Del Castillo is entitled to back salaries despite absence of the award
thereof in the decision reinstating him.

Page 250 of 323


Ruling:
(1) Yes.

Section 37, paragraph (a), of PD 807, the Philippine Civil Service Law, provides:

“(a.) The Commission shall decide upon appeal all administrative disciplinary cases involving the
imposition of a penalty of suspension for more than thirty days, or fine in an amount exceeding
thirty days’ salary, demotion in rank or salary or transfer, removal or dismissal from office. x x x x”

Interpreting the above provision, we held in Mendez v. CSC that:


xxx xxx xxx
It is axiomatic that the right to appeal is merely a statutory privilege and may be
exercised only in the manner and in accordance with the provision of law. (Victorias
Milling Co., Inc. vs. Office of the Presidential Assistant for Legal Affairs, 153 SCRA
318).

A cursory reading of P.D. 807, otherwise known as “The Philippine Civil Service Law”
shows that said law does not contemplate a review of decisions exonerating
officers or employees from administrative charges. x x x x

By inference or implication the remedy of appeal may be availed of only in a case


where the respondent is found guilty of the charges filed against him. But when the
respondent is exonerated of said charges, as in this case, there is no occasion for
appeal.

In University of the Philippines v. Civil Service Commission, et al., we again ruled that:

Section 37 of Presidential Decree No. 807, or the Civil Service Decree, provides that
the Civil Service Commission shall have appellate jurisdiction over all
administrative disciplinary cases involving the imposition of a penalty of
suspension for more than thirty (30) days’ or fine in an amount exceeding thirty
(30) days’ salary, demotion in rank or salary, or transfer, removal or dismissal
from office. The inescapable conclusion is that in an administrative case, where
the penalty imposed is not one of those covered by or is less than those
enumerated under Section 37, the decision of the disciplining authority shall be
final and inappealable. Respondent CSC has no jurisdiction to review the same
on appeal.

(2) Yes.

Having been exonerated of the charges against him, petitioner should clearly be
awarded back salaries, the silence of the MSPB's decision notwithstanding.

When an official or employee was illegally dismissed and his reinstatement has later
been ordered, for all legal purposes he is considered as not having left his office.
Therefore, he is entitled to all the rights and privileges that accrue to him by
virtue of the office he held.

As likewise reaffirmed by the Court in Perez vs. Evite, “under Section 45 of Rule 39,
Rules of Court, a judgment is not confined to what appears upon the face of the
decision, but also those necessarily included therein or necessary thereto.” The
late Chief Justice Fred Ruiz Castro stressed for the Court in Padua vs. Robles, that the
sufficiency and efficacy of a judgment must be tested by its substance rather than its
form. In construing a judgment, its legal effects including such effects that necessarily
follow because of legal implications, rather than the language used, govern. Also, its
meaning, operation, and consequences must be ascertained like any other written
instrument. Thus, a judgment rests on the intention of the court as gathered from
every part thereof, including the situation to which it applies and the attendant
circumstances.

Page 251 of 323


*Miranda v. Carreon, G.R. No. 143540, April 11, 2003.
(Civil Service; Probationary Period; Administrative Code; Poor Performance; Illegal
Dismissal)

-In the early part of 1988, Vice Mayor Amelita Navarro, while serving as Acting Mayor
of the City of Santiago because of the suspension of Mayor Jose Miranda, appointed
respondents to various positions in the city government.

-Their appointments were with permanent status and based on the evaluation made
by the City Personnel Selection and Promotion Board (PSPB) created pursuant to
Republic Act No. 7160. The Civil Service Commission (CSC) approved the
appointments.

-When Mayor Jose Miranda reassumed his post on March 5, 1998 after his
suspension, he considered the composition of the PSPB irregular since the majority
party, to which he belongs, was not properly represented. He then formed a three-man
special performance audit team to conduct a personnel evaluation audit of those who
were previously screened by the PSPB and those on probation. After conducting the
evaluation, the audit team submitted to him a report dated June 8, 1998 stating that
the respondents were found “wanting in their performance.”

-On June 10, 1998, or three months after Mayor Miranda reassumed his post, he
issued an order terminating respondents’ services effective June 15, 1998 because
they “performed poorly” during the probationary period.

-Respondents appealed to the CSC, contending that being employees on probation,


they can be dismissed from the service on the ground of poor performance only after
their probationary period of six months, not after three (3) months.

-On October 19, 1998, the CSC issued a resolution reversing the order of Mayor
Miranda and ordering that respondents be reinstated to their former positions with
payment of backwages.

-Petitioner then filed with the Court of Appeals a petition for review on certiorari.

Issue:
Whether or not respondents’ services were illegally terminated by former Mayor Jose
Miranda.

Ruling:
Yes.

The 1987 Constitution provides that, “no officer or employee of the civil service
shall be removed or suspended except for cause provided by law.”

Under the Revised Administrative Code of 1987, a government officer or employee


may be removed from the service on two (2) grounds: (1) unsatisfactory conduct
and (2) want of capacity. While the Code does not define and delineate the concepts
of these two grounds, however, the Civil Service Law (Presidential Decree No. 807, as
amended) provides specific grounds for dismissing a government officer or employee
from the service. Among these grounds are inefficiency and incompetence in the
performance of official duties. In the case at bar, respondents were dismissed on the
ground of poor performance. Poor performance falls within the concept of inefficiency
and incompetence in the performance of official duties which, as earlier mentioned,
are grounds for dismissing a government official or employee from the service.

But inefficiency or incompetence can only be determined after the passage of


sufficient time, hence, the probationary period of six (6) months for the
respondents. Indeed, to be able to gauge whether a subordinate is inefficient or
incompetent requires enough time on the part of his immediate superior within

Page 252 of 323


which to observe his performance. This condition, however, was not observed in this
case. As aptly stated by the CSC, it is quite improbable that Mayor Jose Miranda
could finally determine the performance of respondents for only the first three
months of the probationary period.

Not only that, we find merit in respondents’ claim that they were denied due process.
They cited Item 2.2 (b), Section VI of the Omnibus Guidelines on Appointments and
Other Personnel Actions (CSC Memorandum Circular No. 38, Series of 1993, as
amended by CSC Memorandum Circular No. 12, Series of 1994) which provides:

“2.2. Unsatisfactory or Poor Performance


xxx
b. An official or employee who, for one evaluation period, is rated poor in performance, may be dropped
from the rolls after due notice. Due notice shall mean that the officer or employee is informed in writing of
the status of his performance not later than the fourth month of that rating period with sufficient
warning that failure to improve his performance within the remaining period of the semester shall
warrant his separation from the service. Such notice shall also contain sufficient information which
shall enable the employee to prepare an explanation.”

Respondents vehemently assert that they were never notified in writing regarding
the status of their performance, neither were they warned that they will be
dismissed from the service should they fail to improve their performance.
Significantly, petitioner did not refute respondents’ assertion. The records show that
what respondents received was only the termination order from Mayor Jose
Miranda. Obviously, respondents’ right to due process was violated.

Moreover, respondents contend that the only reason behind their arbitrary dismissal
was Mayor Jose Miranda’s perception that they were not loyal to him, being
appointees of then Acting Mayor Navarro. This contention appears to be true
considering that all those who were accepted and screened by the PSPB during the
incumbency of Acting Mayor Navarro were rated to have performed poorly by an audit
team whose three members were personally picked by Mayor Jose Miranda.

*Quinto v. Comelec, G.R. No. 189698, February 22, 2010.


(Ipso Facto Resignation; Appointive Officers; Elective Officers; Equal Protection Clause)

Facts:
-In view of the May 10, 2010 National and Local Elections, the Comelec issued
Resolution No. 8678.

-Sections 4 and 5 of Comelec Resolution No. 8678 provide:

SEC. 4. Effects of Filing Certificates of Candidacy. -


a) Any person holding a public appointive office or position including active members of the Armed
Forces of the Philippines, and other officers and employees in government-owned or controlled
corporations, shall be considered ipso facto resigned from his office upon the filing of his certificate
of candidacy.

b) Any person holding an elective office or position shall not be considered resigned upon the filing
of his certificate of candidacy for the same or any other elective office or position.

SEC. 5. Period for filing Certificate of Candidacy. - The certificate of candidacy shall be filed on regular
days, from November 20 to 30, 2009, during office hours, except on the last day, which shall be until
midnight.

-Alarmed that they would be deemed ipso facto resigned from their offices the moment
they file their CoCs, petitioners Eleazar P. Quinto and Gerino A. Tolentino, Jr., who
hold appointive positions in the government and who intend to run in the coming
elections, filed the instant petition for prohibition and certiorari, seeking the
declaration of the afore-quoted Section 4(a) of Resolution No. 8678 as null and void.

-Petitioners’ contentions: (1) Appointive officials should not be deemed ipso facto
resigned from their government offices when they file their CoCs, because at such time
they are not yet treated by law as candidates. They should be considered resigned

Page 253 of 323


from their respective offices only at the start of the campaign period when they are,
by law, already considered as candidates; (2) The provision considering appointive
officials as ipso facto resigned from office upon the filing of their CoCs is
discriminatory and violates the equal protection clause in the Constitution.

Issue:
Whether or not Section 4(a) of Comelec Resolution 8678 is unconstitutional.

Ruling:
No. It is valid.

Section 2(4), Article IX-B of the 1987 Constitution (“No officer or employee in the civil
service shall engage, directly or indirectly, in any electioneering or partisan political
campaign.”) and the implementing statutes apply only to civil servants holding
apolitical offices. Stated differently, the constitutional ban does not cover elected
officials, notwithstanding the fact that “the civil service embraces all branches,
subdivisions, instrumentalities, and agencies of the Government, including
government-owned or controlled corporations with original charters.” This is because
elected public officials, by the very nature of their office, engage in partisan
political activities almost all year round, even outside of the campaign period.
Political partisanship is the inevitable essence of a political office, elective positions
included. x x x x

The petitioners’ contention, that the repeal of Section 67 of the Omnibus Election Code
pertaining to elective officials gives undue benefit to such officials as against the
appointive ones and violates the equal protection clause of the constitution, is
tenuous.

The equal protection of the law clause in the Constitution is not absolute, but is
subject to reasonable classification. If the groupings are characterized by
substantial distinctions that make real differences, one class may be treated and
regulated differently from the other. The Court has explained the nature of the equal
protection guarantee in this manner:

The equal protection of the law clause is against undue favor and individual or class
privilege, as well as hostile discrimination or the oppression of inequality. It is not
intended to prohibit legislation which is limited either in the object to which it is
directed or by territory within which it is to operate. It does not demand absolute
equality among residents; it merely requires that all persons shall be treated alike,
under like circumstances and conditions both as to privileges conferred and
liabilities enforced. The equal protection clause is not infringed by legislation which
applies only to those persons falling within a specified class, if it applies alike to all
persons within such class, and reasonable grounds exist for making a distinction
between those who fall within such class and those who do not.

Substantial distinctions clearly exist between elective officials and appointive


officials. The former occupy their office by virtue of the mandate of the
electorate. They are elected to an office for a definite term and may be removed
therefrom only upon stringent conditions. On the other hand, appointive officials
hold their office by virtue of their designation thereto by an appointing
authority. Some appointive officials hold their office in a permanent capacity
and are entitled to security of tenure while others serve at the pleasure of the
appointing authority.

Another substantial distinction between the two sets of officials is that under Section
55, Chapter 8, Title I, Subsection A. Civil Service Commission, Book V of the
Administrative Code of 1987 (Executive Order No. 292), appointive officials, as
officers and employees in the civil service, are strictly prohibited from engaging
in any partisan political activity or taking part in any election except to vote.
Under the same provision, elective officials, or officers or employees holding

Page 254 of 323


political offices, are obviously expressly allowed to take part in political and
electoral activities.

By repealing Section 67 but retaining Section 66 of the Omnibus Election Code, the
legislators deemed it proper to treat these two classes of officials differently with
respect to the effect on their tenure in the office of the filing of the certificates of
candidacy for any position other than those occupied by them. Again, it is not within
the power of the Court to pass upon or look into the wisdom of this
classification.

Since the classification justifying Section 14 of Rep. Act No. 9006, i.e., elected
officials vis-à-vis appointive officials, is anchored upon material and significant
distinctions and all the persons belonging under the same classification are
similarly treated, the equal protection clause of the Constitution is, thus, not
infringed. x x x x

An election is the embodiment of the popular will, perhaps the purest expression of
the sovereign power of the people. It involves the choice or selection of candidates to
public office by popular vote. Considering that elected officials are put in office by
their constituents for a definite term, it may justifiably be said that they were
excluded from the ambit of the deemed resigned provisions in utmost respect for
the mandate of the sovereign will. In other words, complete deference is accorded
to the will of the electorate that they be served by such officials until the end of
the term for which they were elected. In contrast, there is no such expectation
insofar as appointed officials are concerned.

The dichotomized treatment of appointive and elective officials is therefore


germane to the purposes of the law. For the law was made not merely to preserve
the integrity, efficiency, and discipline of the public service; the Legislature, whose
wisdom is outside the rubric of judicial scrutiny, also thought it wise to balance this
with the competing, yet equally compelling, interest of deferring to the sovereign will.

***SUFFRAGE AND ELECTION LAWS***

*Yap v. Comelec, G.R. No. 150946, October 23, 2003.


(The Commission on Elections; En Banc; Division; Quasi-judicial Proceedings;
Jurisdiction)

Facts:
-Flora L. Benzonan (respondent) was a mayoralty candidate in the Municipality of
Glan, Sarangani during the May 14, 2001 elections.

-After the casting of ballots, petitioners Enrique B. Yap, Jr., Venancio S. Wata, Jr.,
Gildo Villorente, Sr., Ting Musa, Benedicto L. Ruiz, Ananias S. Emnace, Vannevar B.
Alegado, Alito Arnold Carino, Saturnino Bag, Jr. and Federico J. Tangan, were
proclaimed as duly elected Mayor, Vice-Mayor and members of the Sangguniang
Bayan of Glan, Sarangani, respectively.

-Benzonan then instituted a pre-proclamation controversy case with the


Commission on Election (COMELEC) en banc.

-Benzonan argued her pre-proclamation case on the grounds that, among other: (1)
the canvassing proceedings, which were initially held in the Session Hall of the
Sangguniang Bayan of Glan, were later transferred to the Provincial Capitol of Danao
Province, contrary to COMELEC Resolution No. 3848; (2) neither Benzonan nor her
representatives were notified of the last three days of the canvassing proceedings and,
consequently, they were not able to participate therein; (3) the third MBC was illegally
constituted as its Chairman, Vice-Chairman and Secretary are not qualified under the
Omnibus Election Code; (4) the MBC had falsified the certificate of canvass votes.

Page 255 of 323


-[IMPORTANT NOTE:
A “Pre-Proclamation Controversy” covers only two issues:
(1) Illegal composition of the Board of Canvassers (BOC);
(2) Illegal proceedings of the BOC.

*There is illegal composition of the BOC when any of the members do not possess legal
qualifications and appointments.

*There is illegal proceedings of the BOC when any of the following circumstances are
present:
(a) precipitate canvassing;
(b) terrorism;
(c) lack of sufficient notice to the members of the BOC's;
(d) Improper venue.]

-After due hearing, the COMELEC en banc issued a resolution finding that, based on
the evidence presented, the canvass of votes had been conducted in a place other than
the previous venue at the inception of the proceedings to which all were notified. Thus,
the proclamations of the winning candidates were declared null and void and a re-
canvass of the election returns was ordered.

-Petitioners then filed with the SC a petition for certiorari under Rule 65 with a prayer
for a temporary restraining order and preliminary prohibitory injunction.

Issue:
Whether or not the COMELEC en banc has jurisdiction over the case.

Ruling:
No.

Section 3 (c) of Article IX-C of the Constitution reads:

The Commission on Elections may sit en banc or in two divisions, and shall promulgate its rules of
procedure in order to expedite the disposition of election cases, including pre-proclamation controversies.
All such election cases shall be heard and decided in division, provided that motions for
reconsideration of decisions shall be decided by the Commission en banc.

Beginning with Sarmiento v. COMELEC and reiterated in subsequent cases, the most
recent being Balindong v. COMELEC, the Court has upheld this constitutional
mandate and consistently ruled that the COMELEC sitting en banc does not have
the requisite authority to hear and decide election cases in the first instance.
This power pertains to the divisions of the Commission and any decision by the
Commission en banc as regards election cases decided by it in the first instance
is null and void for lack of jurisdiction.

It is important to clarify, however, that not all cases relating to election laws filed
before the COMELEC are required to be first heard by a division. Under the
Constitution, the COMELEC exercises both administrative and quasi-judicial powers.
The COMELEC en banc can act directly on matters falling within its
administrative powers. It is only when the exercise of quasi-judicial powers are
involved that the COMELEC is mandated to decide cases first in division, and
then, upon motion for reconsideration, en banc.

It is clear that the present case is one that involves a pre-proclamation


controversy that requires the exercise of the COMELEC’s quasi-judicial powers,
as the illegality of the composition and proceedings of the MBC, including the
falsification of election returns and certificate of canvass, were alleged to be in issue.
Furthermore, in her comment to the petition dated January 9, 2000, Benzonan
categorically stated that it is not disputed that what is involved here is a pre-
proclamation controversy.

Page 256 of 323


Also undisputed is the fact that Benzonan filed her pre-proclamation case directly with
the COMELEC en banc and that the case was subsequently decided by the COMELEC,
sitting en banc. As aforestated, the COMELEC en banc is without jurisdiction to
decide cases involving these types of controversies in the first instance. Thus, the
procedure taken by Benzonan resulted in a resolution in her favor that the Court
must declare null and void and set aside.

*Jaramilla v. Comelec, G.R. No. 155717, October 23, 2003. [IMPORTANT!!!]


(The Commission on Elections; En Banc; Division; Administrative Proceedings;
Jurisdiction)

Facts:
-Antonio Suyat (respondent) and Alberto J. Jaramilla (petitioner) both ran for the
position of Member of the Sangguniang Bayan in the Municipality of Sta. Cruz, Ilocos
Sur in the May 14, 2001 elections.

-On May 16, 2001, the Municipal Board of Canvassers of Sta. Cruz, proclaimed the
winning candidates for the offices of Mayor, Vice-Mayor and eight (8) members of the
Sangguniang Bayan. Jaramilla ranked #7 while Suyat ranked #9.

-Upon review of the tabulations by Suyat, he discovered that when the figures were
forwarded to the Statement of Votes by Precinct, Jaramilla was credited with seventy
three (73) votes (instead of 23 votes) for Precinct No. 34A1 or fifty (50) votes more than
what Jaramilla actually obtained.

-If the entry were to be corrected, hence, Suyat would rank #8 while Jaramilla would
rank #9.

-On June 13, 2001, respondent Suyat filed before the COMELEC en banc an Urgent
Motion for Issuance of Order to Reconvene, which the latter treated as a Petition for
Correction of Manifest Error.

-Petitioner countered in his Answer that said petition should be dismissed for having
been filed out of time and for lack of the required certification of non-forum shopping.

-Comelec en banc then ruled in favor of Suyat and annulled the proclamation of
Jaramilla. It then constituted a new Municipal Board Canvassers and ordered the
same to immediately convene to effect a correction in the entry in the Statement of
Votes by Precinct.

Issues:
(1) Whether or not the Comelec en banc has jurisdiction over the petition.

(2) Whether or not the Comelec en banc should have dismissed Suyat’s petition
considering that it was filed beyond the five-day reglementary period fixed in the
COMELEC Rules of Procedureand it lacked certification against forum shopping;

(3) Whether or not the Comelec en banc should have dismissed Suyat’s petition for
failure to pay the docket fees on time.

Ruling:
(1) Yes.

Article IX-C of the Constitution states in part that:

Sec. 3. The Commission on Elections may sit en banc or in two divisions, and shall promulgate its rules
of procedure in order to expedite disposition of election cases, including pre-proclamation controversies.
All such election cases shall be heard and decided in division, provided that motions for reconsideration
of decisions shall be decided by the Commission en banc.

As stated in the provision, and in line with the Court’s recent pronouncement in Milla
v. Balmores-Laxa, election cases including pre-proclamation controversies should

Page 257 of 323


first be heard and decided by a division of the COMELEC, and then by the
commission en banc if a motion for reconsideration of the division is filed.

It must be noted however that this provision applies only in cases where the
COMELEC exercises its adjudicatory or quasi-judicial powers, and not when it
merely exercises purely administrative functions. Accordingly, when the case
demands only the exercise by the COMELEC of its administrative functions,
such as the correction of a manifest mistake in the addition of votes or an
erroneous tabulation in the statement of votes, the COMELEC en banc can
directly act on it in the exercise of its constitutional function to decide
questions affecting elections.

The Petition for Correction of Manifest Errors in the case at bar alleges an erroneous
copying of figures from the election return to the Statement of Votes by Precinct. Such
an error in the tabulation of the results, which merely requires a clerical
correction without the necessity of opening ballot boxes or examining ballots,
demands only the exercise of the administrative power of the COMELEC. Hence,
the Commission en banc properly assumed original jurisdiction over the aforesaid
petition.

(2) No.

The COMELEC has the discretion to suspend its rules or any portion thereof in the
interest of justice. Section 4, Rule 1 of the COMELEC Rules expressly provides that:

SEC. 4. Suspension of the Rules – In the interest of justice and in order to obtain speedy disposition of all
matters pending before the commission, these rules or any portion thereof may be suspended by the
Commission.

The COMELEC therefore has authority to suspend the reglementary periods provided
by the rules, or the requirement of certification of non-forum shopping for that matter,
in the interest of justice and speedy resolution of the cases before it.

(3) No.

The COMELEC is not constrained to dismiss a case before it by reason of non-


payment of filing fees. Section 18, Rule 40 the COMELEC Rules of Procedure states:

SEC 18. Nonpayment of Prescribed Fees – If the fees above prescribed are not paid, the Commission may
refuse to take action thereon until they are paid and may dismiss the action or the proceeding.

The use of the word “may” in the aforecited provision readily shows that the
COMELEC is conferred the discretion whether to entertain the petition or not in
case of non-payment of legal fees. And even if it were not afforded such discretion,
as discussed above, it is authorized to suspend its rules or any portion thereof in
the interest of justice. x x x x

Laws governing election contests must be liberally construed to the end that the
will of the people in the choice of public officials may not be defeated by mere
technical objections. Adherence to technicality that would put a stamp on a palpably
void proclamation, with the inevitable result of frustrating the people’s will, can never
be countenanced.

*Cayetano v. Comelec, G.R. No. 193846, April 12, 2011.


(The Commission on Elections; En Banc; Division; Jurisdiction)

Facts:
-Maria Laarni Cayetano (petititoner) and Dante O. Tinga (respondent) were candidates
for the position of Mayor of Taguig City.

Page 258 of 323


-Cayetano was proclaimed the winner thereof on May 12, 2010, receiving a total of
Ninety-Five Thousand Eight Hundred Sixty-Five (95,865) votes as against the Ninety-
Three Thousand Four Hundred Forty-Five (93,445) votes received by Tinga.

-On May 24, 2010, Tinga filed an Election Protest against petitioner before the
COMELEC Second Division. Tinga’s protest listed election frauds and irregularities
allegedly committed by petitioner, which translated to the latter’s ostensible win as
Mayor of Taguig City. On the whole, Tinga claims that he is the actual winner of the
mayoralty elections in Taguig City.

-Cayetano filed her Answer with Counter-Protest and Counterclaim on June 7, 2010.
Petitioner raised, among others, the affirmative defense of insufficiency in form and
content of the Election Protest and prayed for the immediate dismissal thereof.

-The COMELEC Second Division issued a Preliminary Conference Order dated August
23, 2010, finding the protest filed by Tinga and counter-protest filed by Cayetano to be
sufficient in form and substance.

-Petitioner then filed a Motion for Reconsideration of the Preliminary Conference Order
relative to the denial of her affirmative defenses, which the Comelec likewise denied.

-Cayetano then went to the SC via petition for certiorari under Rule 64, in relation to
Rule 65 imputing grave abuse of discretion.

Issue:
Whether or not such petition shall prosper.

Ruling:
No.

The Supreme Court has no power to review via certiorari an interlocutory order or
even a final resolution of a Division of the COMELEC. Failure to abide by this
procedural requirement constitutes a ground for dismissal of the petition.

The general rule is that a decision or an order of a COMELEC Division cannot be


elevated directly to this Court through a special civil action for certiorari.
Furthermore, a motion to reconsider a decision, resolution, order, or ruling of a
COMELEC Division shall be elevated to the COMELEC En Banc. However, a motion
to reconsider an interlocutory order of a COMELEC Division shall be resolved by
the division which issued the interlocutory order, except when all the members of
the division decide to refer the matter to the COMELEC En Banc.

Thus, in general, interlocutory orders of a COMELEC Division are not appealable,


nor can they be proper subject of a petition for certiorari. To rule otherwise
would not only delay the disposition of cases but would also unnecessarily clog
the Court docket and unduly burden the Court.

This does not mean that the aggrieved party is without recourse if a COMELEC
Division denies the motion for reconsideration. The aggrieved party can still assign
as error the interlocutory order if in the course of the proceedings he decides to
appeal the main case to the COMELEC En Banc. The exception enunciated in Kho
case and Repol case is when the interlocutory order of a COMELEC Division is a
patent nullity because of absence of jurisdiction to issue the interlocutory order, as
where a COMELEC Division issued a temporary restraining order without a time limit,
which is the Repol case, or where a COMELEC Division admitted an answer with
counter-protest which was filed beyond the reglementary period, which is the Kho
case.

This Court has already ruled in Reyes v. RTC of Oriental Mindoro, that “it is the
decision, order or ruling of the COMELEC En Banc that, in accordance with
Section 7, Art. IX-A of the Constitution, may be brought to the Supreme Court

Page 259 of 323


on certiorari.” The exception provided in Kho and Repol is unavailing in this case
because unlike in Kho and Repol, the assailed interlocutory orders of the COMELEC
First Division in this case are not a patent nullity.

X *Buac v. Comelec, G.R. No. 155855, January 26, 2004.


(Comelec; Plebiscite; Jurisdiction)

Facts:
-In April 1988, a plebiscite was held in Taguig for the ratification of the Taguig
Cityhood Law (Republic Act No. 8487) proposing the conversion of Taguig from a
municipality into a city.

-Without completing the canvass of sixty-four (64) other election returns, the Plebiscite
Board of Canvassers declared that the "NO" votes won and that the people rejected the
conversion of Taguig to a city.

-The Board of Canvassers was, however, ordered by the COMELEC en banc to


reconvene and complete the canvass. The Board did and in due time issued an Order
proclaiming that the negative votes prevailed in the plebiscite conducted.

-Petitioners then filed with the COMELEC a petition to annul the results of the
plebiscite with a prayer for revision and recount of the ballots cast therein. They
alleged that fraud and irregularities attended the casting and counting of votes.

-The case was docketed as an election protest and raffled to the COMELEC Second
Division.

-Respondent Cayetano intervened and moved to dismiss the petition on the ground of
lack of jurisdiction of the COMELEC. He claimed that a plebiscite cannot be the
subject of an election protest. He averred that the jurisdiction to hear a complaint
involving the conduct of a plebiscite is lodged with the RTC.

-Comelec Second Division eventually dismissed the petition and ruled that the
COMELEC has no jurisdiction over said case as it involves an exercise of quasi-judicial
powers not contemplated under Section 2 (2), Article IX (C) of the 1987 Constitution.

-On appeal, the COMELEC en banc affirmed the ruling of its 2nd Division. It
concluded that the jurisdiction over the petition to annul the Taguig plebiscite results
is lodged with the RTC under Section 19 (6) of Batas Pambansa Big. 129 which
provides that the RTC shall have exclusive original jurisdiction in cases not within the
exclusive jurisdiction of any court or body exercising judicial or quasi-judicial
functions.

-Petitioners then went to the SC via petition for certiorari and mandamus.

Issue:
Whether or not the Comelec has jurisdiction to annul the results of the plebiscite.

Ruling:
Yes.

We hold that the invocation of judicial power to settle disputes involving the conduct
of a plebiscite is misplaced. Section 1, Article VIII of the Constitution defines judicial
power as including “the duty of the courts of justice to settle actual controversies
involving rights which are legally demandable and enforceable and to determine
whether or not there has been a grave abuse of discretion amounting to lack or excess
of jurisdiction on the part of any branch or instrumentality of the Government.”

The case at bar assailing the regularity of the conduct of the Taguig plebiscite does not
fit the kind of a case calling for the exercise of judicial power. It does not involve the
violation of any legally demandable right and its enforcement. There is no

Page 260 of 323


plaintiff or defendant in the case at bar for it merely involves the ascertainment
of the vote of the electorate of Taguig whether they approve or disapprove the
conversion of their municipality to a highly urbanized city. There is no invocation
of a private right conferred by law that has been violated and which can be vindicated
alone in our courts of justice in an adversarial proceeding. Rather, the issue in the
case at bar is the determination of the sovereign decision of the electorate of Taguig.
The purpose of this determination is more to protect the sovereignty of the people and
less to vindicate the private interest of any individual. Such a determination does not
contemplate the clash of private rights of individuals and hence cannot come under
the traditional jurisdiction of courts. x x x x

In the case at bar, the conduct of the Taguig plebiscite is the core of the controversy.
This is a matter that involves the enforcement and administration of a law
relative to a plebiscite. It falls under the jurisdiction of the COMELEC under Section
2(1), Article IX (C) of the Constitution which gives it the power “to enforce and
administer all laws and regulations relative to the conduct of a x x x plebiscite x x x.”

While the jurisdiction of the COMELEC is most commonly invoked over popular
elections that which involves the choice or selection ' of candidates to public office by
popular vote, the same may likewise be invoked in connection with the conduct of
plebiscite.

Article LX-C, Section 2(1) is very explicit that the COMELEC has the power to
“enforce administer all laws and regulations relative to the conduct of an
election, plebiscite, initiative, referendum and recall.” To enforce means to cause
to take effect or to cause the performance of such act or acts necessary to bring into
actual effect or operation, a plan or measure. When we say the COMELEC has the
power to enforce all laws relative to the conduct of a plebiscite, it necessarily
entails all the necessary and incidental power for it to achieve the holding of an
honest and credible plebiscite.

Obviously, the power of the COMELEC is not limited to the mere administrative
function of conducting the plebiscite. The law is clear. It is also mandated to enforce
the laws relative to the conduct of the plebiscite. Hence, the COMELEC, whenever it is
called upon to correct or check what the Board of Canvassers erroneously or
fraudulently did during the canvassing, can verify or ascertain the true results of the
plebiscite either through a pre-proclamation case or through revision of ballots. To
remove from the COMELEC the power to ascertain the true results of the plebiscite
through revision of ballots is to render nugatory its constitutionally mandated power
to "enforce" laws relative to the conduct of plebiscite.

If the COMELEC has quasi-judicial power to enforce laws relating to elective


officials then there is no reason why it cannot exercise the same power to
ascertain the true results of a plebiscite. All that the Constitution provides is that
the COMELEC shall exercise exclusive jurisdiction over all contests relating to elective
officials. The provision is not a limiting provision in the sense that it only limits the
quasi-judicial power of the COMELEC to said cases. To repeat, the power of the
COMELEC to ascertain the true results of the plebiscite is implicit in its power to
enforce all laws relative to the conduct of plebiscite.

*Philippine Press Institute v. Comelec, G.R. No. L-119694, May 22, 1995.
(Comelec; Police Power; Eminent Domain)

Facts:
-The Comelec promulgated Resolution No. 2772, directing publishers of newspapers
like the Business World, the Philippine Star, the Malaya and the Philippine Times
Journal to provide free print space of not less than one half (1/2) page for use as
“Comelec Space.”

Page 261 of 323


-Philippine Press Institute, Inc. ("PPI”), a non-stock, non-profit organization of
newspaper and magazine publishers, then went to the SC via a Petition for Certiorari
and Prohibition.

Issue:
Whether or not Resolution No. 2772 is constitutional.

Ruling:
No.

To compel print media companies to donate “Comelec-space” of the dimensions


specified in Section 2 of Resolution No. 2772 (not less than one-half page),
amounts to “taking” of private personal property for public use or purposes.
Section 2 failed to specify the intended frequency of such compulsory "donation:" only
once during the period from 6 March 1995 (or 21 March 1995) until 12 May 1995? or
everyday or once a week? or as often as Comelec may direct during the same period?
The extent of the taking or deprivation is not insubstantial; this is not a case of a de
minimis temporary limitation or restraint upon the use of private property. The
monetary value of the compulsory "donation," measured by the advertising rates
ordinarily charged by newspaper publishers whether in cities or in non-urban areas,
may be very substantial indeed.

The taking of private property for public use is, of course, authorized by the
Constitution, but not without payment of "just compensation" (Article III, Section
9). x x x x

As earlier noted, the Solicitor General also contended that Section 2 of Resolution No.
2772, even if read as compelling publishers to "donate" "Comelec space," may be
sustained as a valid exercise of the police power of the state. However, Section 2 of
Resolution No. 2772 is a blunt and heavy instrument that purports, without a
showing of existence of a national emergency or other imperious public
necessity, indiscriminately and without regard to the individual business condition of
particular newspapers or magazines located in differing parts of the country, to take
private property of newspaper or magazine publishers. No attempt was made to
demonstrate that a real and palpable or urgent necessity for the taking of print
space confronted the Comelec and that Section 2 of Resolution No. 2772 was
itself the only reasonable and calibrated response to such necessity available to
the Comelec. Hence, Section 2 does not constitute a valid exercise of the police
power of the State.

X *Sahali v. Comelec, G.R. No. 201796, January 15, 2013.


(Comelec; Scope of Authority; Technical Examination; Interlocutory Order)

Facts:
-During the May 10, 2010 elections, Sadikul and private respondent Rashidin H.
Matba (Matba) were two of the four candidates who ran for the position of governor in
the Province of Tawi-Tawi while Ruby and private respondent Jilkasi J. Usman
(Usman) ran for the position of Vice-Governor.

-On May 14, 2010, the Provincial Board of Canvassers (PBOC) proclaimed petitioners
Sadikul and Ruby as the duly elected governor and vice-governor, respectively, of the
province of Tawi-Tawi.

-Alleging that the said elections in the Province of Tawi-Tawi were attended by
massive and wide-scale irregularities, Matba filed an Election Protest Ad
Cautelam with the COMELEC. The said election protest filed by Matba was raffled to
the First Division of the COMELEC.

-Usman also filed an Election Protest Ad Cautelam with the COMELEC, contesting the
results in 39 out of the 282 clustered precincts in the Province of Tawi-Tawi. Usman’s
election protest was likewise raffled to the First Division of the COMELEC.

Page 262 of 323


-The respective election protests filed by private respondents Matba and Usman
prayed, inter alia, for the technical examination of the ballots, Election Day
Computerized Voters List (EDCVL), the Voters Registration Record (VRR), and the
Book of Voters in all the protested precincts of the province of Tawi-Tawi.

-Eventually, the COMELEC First Division ordered the recount of the contested ballots,
directing the creation of five recount committees for the said purpose.

-In the course of the proceedings, Matba and Usman filed a Manifestation and Ex-
Parte Motion praying for a technical examination of the EDCVL, the VRR and the Book
of Voters for the contested precincts in the province of Tawi-Tawi by comparing the
signature and the thumbmarks appearing on the EDCVL as against those appearing
on the VRRs and the Book of Voters. Matba and Usman averred that, instead of
recounting the ballots in the pilot precincts constituting 20% of the protested
precincts, the COMELEC First Division should order the technical examination of the
said election paraphernalia from the 38 clustered precincts that are the subject of
both election protests filed by them.

-The COMELEC First Division granted the said motion and directed its Election
Records and Statistics Department (ERSD) to conduct a technical examination of the
said election paraphernalia.

-Sadikul and Ruby jointly filed with the COMELEC First Division a Strong
Manifestation of Grave Concern and Motion for Reconsideration, arguing that: (1) the
said Order was issued without due process since the COMELEC First Division did not
allow them to oppose the said ex-parte motion; (2) the COMELEC First Division cannot
just order a technical examination in the absence of published rules on the matter.

-The COMELEC First Division denied the MR.

-Feeling aggrieved, Sadikul and Ruby wne to the SC via Petition for Certiorari under
Rule 65 in relation to Rule 64.

Issue:
Whether or not the COMELEC First Division committed grave abuse of discretion
amounting to lack or excess of jurisdiction when: first, it did not give petitioners the
opportunity to oppose the motion for technical examination filed by Matba and
Usman; and second, it ordered the technical examination of the said election
paraphernalia despite the lack of sanction and published rules governing such
examination.

Ruling:
No.

A. Procedural

The petitioners’ resort to the extraordinary remedy of certiorari to assail an


interlocutory order issued by the COMELEC First Division is amiss. "A party
aggrieved by an interlocutory order issued by a Division of the COMELEC in an
election protest may not directly assail the order in this Court through a special
civil action for certiorari. The remedy is to seek the review of the interlocutory order
during the appeal of the decision of the Division in due course."

Under the Constitution, the power of the Supreme Court to review election cases
falling within the original exclusive jurisdiction of the COMELEC only extends to
final decisions or resolutions of the COMELEC en banc, not to interlocutory
orders issued by a Division thereof.

Rule 65, Section 1, 1997 Rules of Civil Procedure, as amended, requires that there be
no appeal, or any plain, speedy and adequate remedy in the ordinary course of law. A

Page 263 of 323


motion for reconsideration is a plain and adequate remedy provided by law.
Failure to abide by this procedural requirement constitutes a ground for dismissal of
the petition.

In like manner, a decision, order or resolution of a division of the Comelec must


be reviewed by the Comelec en banc via a motion for reconsideration before the
final en banc decision may be brought to the Supreme Court on certiorari. The
pre-requisite filing of a motion for reconsideration is mandatory.

Here, the Orders dated March 5, 2012 and May 3, 2012 issued by the First Division of
the COMELEC were merely interlocutory orders since they only disposed of an
incident in the main case i.e. the propriety of the technical examination of the said
election paraphernalia. Thus, the proper recourse for the petitioners is to await the
decision of the COMELEC First Division in the election protests filed by Matba and
Usman, and should they be aggrieved thereby, to appeal the same to the COMELEC en
banc by filing a motion for reconsideration.

B. Substantive

(1) As to the allegation by petitioners that they were denied due process when the
COMELEC granted the motion for technical examination filed by Matba and Usman
without giving them the opportunity to oppose the said motion:

It bears stressing that the COMELEC, in election disputes, is not duty-bound to


notify and direct a party therein to file an opposition to a motion filed by the
other party. It is incumbent upon the party concerned, if he/she deems it necessary,
to file an opposition to a motion within five days from receipt of a copy of the same
without awaiting for the COMELEC’s directive to do so. On this score, Section 3, Rule
9 of COMELEC Resolution No. 880429 clearly provides that:

Sec. 3. No hearings on motions. – Motions shall not be set for hearing unless the Commission directs
otherwise. Oral argument in support thereof shall be allowed only upon the discretion of the
Commission. The adverse party may file opposition five days from receipt of the motion, upon the
expiration of which such motion is deemed submitted for resolution. The Commission shall resolve
the motion within five days.

If the party concerned, despite receipt of a copy of the motion that was filed with the
COMELEC, did not file an opposition to the said motion, the motion would be deemed
submitted for resolution upon the expiration of the period to file an opposition thereto.

Here, the petitioners did not file an opposition to the said motion for technical
examination that was filed by Matba and Usman on February 24, 2012. It was
only after the COMELEC First Division issued its March 5, 2012 Order that the
petitioners decided to register their opposition to the intended technical examination,
albeit in the form of a motion for reconsideration of the said Order. Contrary to the
petitioners’ claim, Section 3, Rule 9 of COMELEC Resolution No. 8804 gave them the
opportunity to raise their objections to the said motion for technical examination.
However, for reasons known only to them, petitioners did not file any opposition to the
said motion. Accordingly, it is the petitioners themselves and not the COMELEC
First Division who should be faulted for their predicament.

(2) As to petitioners’ allegation that the COMELEC First Division cannot order a
technical examination of the said election paraphernalia since there is as yet no
published rule therefor:

While Section 1, Rule 18 of COMELEC Resolution No. 8804 does not explicitly provide
for the rule on the technical examination of election paraphernalia, it does not mean,
however, that the COMELEC First Division does not have the power to order the
conduct of such technical examination.

The absence of a rule which specifically mandates the technical examination of


the said election paraphernalia does not mean that the COMELEC First Division

Page 264 of 323


is barred from issuing an order for the conduct thereof. The power of the
COMELEC First Division to order the technical examination election
paraphernalia in election protest cases stems from its “exclusive original
jurisdiction over all contest relating to the elections, returns and qualifications
of all elective regional, provincial and city officials.”

Otherwise stated, the express grant of power to the COMELEC to resolve election
protests carries with it the grant of all other powers necessary, proper, or
incidental to the effective and efficient exercise of the power expressly granted.
Verily, the exclusive original jurisdiction conferred by the constitution to the
COMELEC to settle said election protests includes the authority to order a technical
examination of relevant election paraphernalia, election returns and ballots in order to
determine whether fraud and irregularities attended the canvass of the votes. x x x x

Concomitant to the COMELEC’s duty to expeditiously resolve election cases is the


authority to resort to every reasonable and efficient means available to it to settle the
controversy. The COMELEC is thus enjoined, "not only to maintain its sense of
urgency in resolving these cases, but also to explore every reasonable and feasible
means of ascertaining which candidate was duly elected."

Here, the technical examination ordered by the COMELEC First Division, by


comparing the signature and the thumbmarks appearing on the EDCVL as against
those appearing on the VRRs and the Book of Voters, is a reasonable, efficient and
expeditious means of determining the truth or falsity of the allegations of fraud
and irregularities in the canvass of the votes in the province of Tawi-Tawi.
Accordingly, the COMELEC First Division did not commit any abuse of discretion
when it allowed the technical examination of the said election paraphernalia.

*Akbayan v. Comelec, G.R. No. 147066, March 26, 2001.


(Comelec; Voters’ Registration; Discretion; Mandamus)

-The Comelec set the deadline for the continuing voters registration under R.A. 8189
on December 27, 2000.

-According to petitioners (all youth organizations), around four million youth failed
to register on or before the December 27, 2000 deadline set by the respondent
COMELEC.

-Petitioners then requested the Comelec to hold additional two days of registration,
to be conducted preferably on February 17 and 18, 2001 nationwide.

-The Comelec denied the said request in a Resolution dated February 8, 2001.

-Aggrieved by the denial, petitioners went to the SC via Petition for Certiorari and
Mandamus.

Issues:
(1) Whether or not the COMELEC committed grave abuse of discretion in issuing
COMELEC Resolution dated February 8, 2001, denying the request to conduct a two-
day additional registration of new voters on February 17 and 18, 2001;

(2) Whether or not the SC can compel the COMELEC, through the extraordinary writ
of mandamus, to conduct a special registration of new voters during the period
between the COMELEC's imposed December 27, 2000 deadline and the May 14, 2001
general elections.

Ruling:
(1) No.

Grave abuse of discretion implies a capricious and whimsical exercise of judgment as


is equivalent to lack of jurisdiction, or, when the power is exercised in an arbitrary or

Page 265 of 323


despotic manner by reason of passion or personal hostility, and it must be so patent
and gross as to amount to an evasion of positive duty enjoined or to act at all in
contemplation of laws.

Under these circumstances, we rule that the COMELEC in denying the request of
petitioners to hold a special registration, acted within the bounds and confines
of the applicable law on the matter - Section 8 of R.A. 8189. In issuing the assailed
Resolution, respondent COMELEC simply performed its constitutional task to
enforce and administer all laws and regulations relative to the conduct of an
election, inter alia, questions relating to the registration of voters; evidently,
respondent COMELEC merely exercised a prerogative that chiefly pertains to it
and one which squarely falls within the proper sphere of its constitutionally
mandated powers. Hence, whatever action the COMELEC takes in the exercise of
its wide latitude of discretion, specifically on matters involving voters’
registration, pertains to the wisdom rather than the legality of the act.
Accordingly, in the absence of clear showing of grave abuse of power or discretion on
the part of respondent COMELEC, this Court may not validly conduct an incursion
and meddle with affairs exclusively within the province of respondent COMELEC - a
body accorded by no less than the fundamental law with independence.

(2) No.

As an extraordinary writ, the remedy of mandamus lies only to compel an officer


to perform a ministerial duty, not a discretionary one; mandamus will not issue to
control the exercise of discretion of a public officer where the law imposes upon him
the duty to exercise his judgment in reference to any manner in which he is required
to act, because it is his judgment that is to be exercised and not that of the court.

Considering the circumstances where the writ of mandamus lies and the peculiarities
of the present case, we are of the firm belief that petitioners failed to establish, to the
satisfaction of this Court, that they are entitled to the issuance of this extraordinary
writ so as to effectively compel respondent COMELEC to conduct a special registration
of voters. For the determination of whether or not the conduct of a special
registration of voters is feasible, possible or practical within the remaining
period before the actual date of election, involves the exercise of discretion and
thus, cannot be controlled by mandamus.

*Vidal v. Comelec, G.R. No. 206666; January 21, 2015.


(Comelec; Disqualification; Pardon)

-On September 12, 2007, the Sandiganbayan convicted former President Estrada, a
former President of the Republic of the Philippines, for the crime of plunder.

-On October 25, 2007, however, former President Gloria Macapagal Arroyo (former
President Arroyo) extended executive clemency, by way of pardon, to former President
Estrada, to wit:

“WHEREAS, this Administration has a policy of releasing inmates who have reached the age of seventy
(70),

WHEREAS, Joseph Ejercito Estrada has been under detention for six and a half years,

WHEREAS, Joseph Ejercito Estrada has publicly committed to no longer seek any elective position
or office,

IN VIEW HEREOF and pursuant to the authority conferred upon me by the Constitution, I hereby grant
executive clemency to JOSEPH EJERCITO ESTRADA, convicted by the Sandiganbayan of Plunder and
imposed a penalty of Reclusion Perpetua. He is hereby restored to his civil and political rights.

The forfeitures imposed by the Sandiganbayan remain in force and in full, including all writs and
processes issued by the Sandiganbayan in pursuance hereof, except for the bank account(s) he owned
before his tenure as President.

Upon acceptance of this pardon by JOSEPH EJERCITO ESTRADA, this pardon shall take effect. x x x x”

Page 266 of 323


-On October 26, 2007, at 3:35 p.m., former President Estrada "received and
accepted" the pardon by affixing his signature beside his handwritten notation
thereon.

-On November 30, 2009, former President Estrada filed a Certificate of Candidacy for
the position of President.

-Erap then ran for president during the 2010 synchronized elections, but lost.

-On October 2, 2012, former President Estrada once more ventured into the political
arena, and filed a Certificate of Candidacy, this time vying for a local elective post,
that ofthe Mayor of the City of Manila.

-On January 24, 2013, herein petitioner Risos-Vidal filed a Petition for
Disqualification against former President Estrada before the COMELEC. Eventually,
the COMELEC Second Division dismissed the petition for disqualification, on the
ground President Estrada’s right to seek public office has been effectively restored by
the pardon vested upon him by former President Arroyo.

-Petitioner then went to the SC via a Petition for Certiorari filed under Rule 64, in
relation to Rule 65.

-Petitioner’s contentions: The pardon extended to Erap was conditional as embodied in


the third Whereas Clause of the pardon, i.e., “WHEREAS, Joseph Ejercito Estrada has
publicly committed to no longer seek any elective position or office.” The
aforementioned commitment was what impelled former President Arroyo to pardon
former President Estrada, without it, the clemency would not have been extended. It
is not enough that a pardon makes a general statement that such pardon carries with
it the restoration of civil and political rights.

Issue:
Whether or not the COMELEC committed grave abuse of discretion amounting to lack
or excess of jurisdiction in ruling that former President Estrada is qualified to vote and
be voted for in public office as a result of the pardon granted to him by former
President Arroyo.

Ruling:
No.

Former President Estrada was granted an absolute pardon that fully restored all
his civil and political rights, which naturally includes the right to seek public
elective office. The wording of the pardon extended to former President Estrada
is complete, unambiguous, and unqualified. It is likewise unfettered by Articles 36
and 41 of the Revised Penal Code. The only reasonable, objective, and constitutional
interpretation of the language of the pardon is that the same in fact conforms to
Articles 36 and 41 of the Revised Penal Code. x x x x

It is apparent from constitutional provisions that the only instances in which the
President may not extend pardon remain to be in: (1) impeachment cases; (2) cases
that have not yet resulted in a final conviction; and (3) cases involving violations of
election laws, rules and regulations in which there was no favorable recommendation
coming from the COMELEC. Therefore, it can be argued that any act of Congress by
way of statute cannot operate to delimit the pardoning power of the President. x x x x
Articles 36 and 41 of the Revised Penal Code cannot, in any way, serve to abridge or
diminish the exclusive power and prerogative of the President to pardon persons
convicted of violating penal statutes.

A close scrutiny of the text of the pardon extended to former President Estrada
shows that both the principal penalty of reclusion perpetua and its accessory
penalties are included in the pardon. The first sentence refers to the executive

Page 267 of 323


clemency extended to former President Estrada who was convicted by the
Sandiganbayan of plunder and imposed a penalty of reclusion perpetua. The latter is
the principal penalty pardoned which relieved him of imprisonment. The sentence that
followed, which states that "(h)e is hereby restored to his civil and political rights,"
expressly remitted the accessory penalties that attached to the principal penalty of
reclusion perpetua. Hence, even if we apply Articles 36 and 41 of the Revised Penal
Code, it is indubitable from the text of the pardon that the accessory penalties of civil
interdiction and perpetual absolute disqualification were expressly remitted together
with the principal penalty of reclusion perpetua. x x x x

Contrary to Risos-Vidal’s declaration, the third preambular clause of the pardon,


i.e., "[w]hereas, Joseph Ejercito Estrada has publicly committed to no longer
seek any elective position or office," neither makes the pardon conditional, nor
militate against the conclusion that former President Estrada’s rights to suffrage and
to seek public elective office have been restored.

This is especially true as the pardon itself does not explicitly impose a condition
or limitation, considering the unqualified use of the term “civil and political
rights” as being restored. Jurisprudence educates that a preamble is not an
essential part of an act as it is an introductory or preparatory clause that
explains the reasons for the enactment, usually introduced by the word
"whereas." Whereas clauses do not form part of a statute because, strictly
speaking, they are not part of the operative language of the statute. In this case, the
whereas clause at issue is not an integral part of the decree of the pardon, and
therefore, does not by itself alone operate to make the pardon conditional or to make
its effectivity contingent upon the fulfilment of the aforementioned commitment nor to
limit the scope of the pardon.

If former President Arroyo intended for the pardon to be conditional on Estrada’s


promise never to seek a public office again, the former ought to have explicitly
stated the same in the text of the pardon itself. Since former President Arroyo did
not make this an integral part of the decree of pardon, the 3rd preambular clause
cannot be interpreted as a condition to the pardon extended to former President
Estrada.

*Moreno v. Comelec, G.R. No. 168550, August 10, 2006.


(Comelec; Disqualification; Probation)

Facts:
-In 1998, petitioner Urbano M. Moreno (Moreno) was convicted by final judgment of
the crime of Arbitrary Detention and was sentenced to suffer imprisonment of Four
(4) Months and One (1) Day to Two (2) Years and Four (4) Months.

-Moreno was eventually granted probation by the court, and in 2000, his probation
was officially terminated.

-In 2002, Moreno decided to run for the elective office of Punong Barangay of
Barangay Cabugao, Daram, Samar.

-Norma L. Mejes (Mejes) filed a petition to disqualify Moreno based on his


aforementioned conviction.

-The Comelec First Division ruled that Moreno was disqualified from running for
Punong Barangay. The Comelec en banc then affirmed the same, ruling that Sec. 40(a)
of the Local Government Code provides that those sentenced by final judgment for an
offense involving moral turpitude or for an offense punishable by one (1) year or more
of imprisonment, within two (2) years after serving sentence, are disqualified from
running for any elective local position. Since Moreno was released from probation on
December 20, 2000, disqualification shall commence on this date and end two (2)
years thence.

Page 268 of 323


-Moreno then went to the SC.

Issue:
Whether or not Moreno is disqualified to run under the circumstances.

Ruling:
No.

Sec. 40(a) of the Local Government Code reads:

Sec. 40. Disqualifications. – The following persons are disqualified from running for any elective local
position:

(a) Those sentenced by final judgment for an offense involving moral turpitude or for an offense
punishable by one (1) year or more of imprisonment, within two (2) years after serving sentence x x x
x.

The phrase "service of sentence," understood in its general and common sense, means
the confinement of a convicted person in a penal facility for the period adjudged by the
court.

In Baclayon v. Mutia, the Court declared that an order placing defendant on


probation is not a sentence but is rather, in effect, a suspension of the
imposition of sentence. We held that the grant of probation to petitioner suspended
the imposition of the principal penalty of imprisonment, as well as the accessory
penalties of suspension from public office and from the right to follow a profession or
calling, and that of perpetual special disqualification from the right of suffrage. We
thus deleted from the order granting probation the paragraph which required that
petitioner refrain from continuing with her teaching profession.

Applying this doctrine to the instant case, the accessory penalties of suspension
from public office, from the right to follow a profession or calling, and that of
perpetual special disqualification from the right of suffrage, attendant to the
penalty of arresto mayor in its maximum period to prision correccional in its minimum
period imposed upon Moreno were similarly suspended upon the grant of
probation.

During the period of probation, the probationer is not even disqualified from
running for a public office because the accessory penalty of suspension from
public office is put on hold for the duration of the probation.

Clearly, the period within which a person is under probation cannot be equated
with service of the sentence adjudged. Sec. 4 of the Probation Law specifically
provides that the grant of probation suspends the execution of the sentence.
During the period of probation, the probationer does not serve the penalty
imposed upon him by the court but is merely required to comply with all the
conditions prescribed in the probation order.

It is regrettable that the Comelec and the OSG have misapprehended the real issue in
this case. They focused on the fact that Moreno’s judgment of conviction attained
finality upon his application for probation instead of the question of whether his
sentence had been served.

This is as good a time as any to clarify that those who have not served their sentence
by reason of the grant of probation which, we reiterate, should not be equated with
service of sentence, should not likewise be disqualified from running for a local elective
office because the two (2)-year period of ineligibility under Sec. 40(a) of the Local
Government Code does not even begin to run.

The fact that the trial court already issued an order finally discharging Moreno fortifies
his position. Sec. 16 of the Probation Law provides that "the final discharge of the
probationer shall operate to restore to him all civil rights lost or suspended as a

Page 269 of 323


result of his conviction and to fully discharge his liability for any fine imposed
as to the offense for which probation was granted." Thus, when Moreno was finally
discharged upon the court’s finding that he has fulfilled the terms and conditions of
his probation, his case was deemed terminated and all civil rights lost or suspended
as a result of his conviction were restored to him, including the right to run for
public office.

We agree with Moreno that the Probation Law should be construed as an exception
to the Local Government Code. While the Local Government Code is a later law
which sets forth the qualifications and disqualifications of local elective officials, the
Probation Law is a special legislation which applies only to probationers. It is a canon
of statutory construction that a later statute, general in its terms and not
expressly repealing a prior special statute, will ordinarily not affect the special
provisions of such earlier statute.

*Condon v. Comelec, G.R. No. 198742, August 10, 2012. [IMPORTANT!]


(Disqualification; Citizenship; Estoppel; Oath of Renunciation)

Facts:
-Teodora Sobejana-Condon (petitioner) is a natural-born Filipino citizen having been
born to Filipino parents. On December 13, 1984, she became a naturalized
Australian citizen owing to her marriage to a certain Kevin Thomas Condon.

-On December 2, 2005, she filed an application to re-acquire Philippine citizenship


before the Philippine Embassy in Canberra, Australia pursuant to Section 3 of R.A.
No. 9225 otherwise known as the "Citizenship Retention and Re-Acquisition Act of
2003." The application was approved and the petitioner took her oath of allegiance
to the Republic of the Philippines on December 5, 2005.

-On September 18, 2006, the petitioner filed an unsworn Declaration of


Renunciation of Australian Citizenship before the Department of Immigration and
Indigenous Affairs, Canberra, Australia, which in turn issued the Order dated
September 27, 2006 certifying that she has ceased to be an Australian citizen.

-The petitioner ran for Mayor in her hometown of Caba, La Union in the 2007
elections. She lost in her bid. She again sought elective office during the May 10,
2010 elections this time for the position of Vice-Mayor. She obtained the highest
numbers of votes and was proclaimed as the winning candidate. She took her oath of
office on May 13, 2010.

-Soon thereafter, private respondents Robelito V. Picar, Wilma P. Pagaduan and Luis
M. Bautista, (private respondents) all registered voters of Caba, La Union, filed
separate petitions for quo warranto questioning the petitioner’s eligibility before the
RTC. The petitions similarly sought the petitioner’s disqualification from holding her
elective post on the ground that she is a dual citizen and that she failed to execute a
"personal and sworn renunciation of any and all foreign citizenship before any
public officer authorized to administer an oath" as imposed by Section 5(2) of R.A.
No. 9225.

-Petitioner herself admitted during trial that the personal declaration of


renunciation she filed in Australia was not under oath.

-The RTC held that the petitioner’s failure to comply with Section 5(2) of R.A. No. 9225
rendered her ineligible to run and hold public office. Upon appeal, the Comelec
affirmed the ruling of the RTC.

-Candon then went to the SC via certiorari under Rule 64 in relation to Rule 65.

-Petitioner’s contentions: (1) Since she ceased to be an Australian citizen on


September 27, 2006, she no longer held dual citizenship and was only a Filipino
citizen when she filed her certificate of candidacy as early as the 2007 elections.

Page 270 of 323


Hence, the “personal and sworn renunciation of foreign citizenship” imposed by
Section 5(2) of R.A. No. 9225 to dual citizens seeking elective office does not apply to
her; (2) a sworn renunciation is a mere formal and not a mandatory requirement;
and (3) the private respondents are estopped from questioning her eligibility since
they failed to do so when she filed certificates of candidacy for the 2007 and 2010
elections.

Issues:
(1) Whether or not the “sworn renunciation of foreign citizenship” in Section 5(2) of
R.A. No. 9225 is a mere formal and not a mandatory requirement R.A. No. 9225;

(2) Whether or not private respondents are estopped from questioning petitioner’s
eligibility considering that respondents failed to challenge her qualifications when she
filed her COC;

(3) Whether or not petitioner is disqualified to run for office.

Ruling:
(1) No.

The language of Section 5(2) is free from any ambiguity. In Lopez v. COMELEC, we
declared its categorical and single meaning: a Filipino American or any dual citizen
cannot run for any elective public position in the Philippines unless he or she
personally swears to a renunciation of all foreign citizenship at the time of filing
the certificate of candidacy. We also expounded on the form of the renunciation and
held that to be valid, the renunciation must be contained in an affidavit duly
executed before an officer of the law who is authorized to administer an oath
stating in clear and unequivocal terms that affiant is renouncing all foreign
citizenship.

The language of the provision is plain and unambiguous. It expresses a single,


definite, and sensible meaning and must thus be read literally. The foreign
citizenship must be formally rejected through an affidavit duly sworn before an
officer authorized to administer oath. x x x x

An oath is a solemn declaration, accompanied by a swearing to God or a revered


person or thing, that one’s statement is true or that one will be bound to a promise.
The person making the oath implicitly invites punishment if the statement is untrue or
the promise is broken. The legal effect of an oath is to subject the person to
penalties for perjury if the testimony is false.

Indeed, the solemn promise, and the risk of punishment attached to an oath ensures
truthfulness to the prospective public officer’s abandonment of his adopted state and
promise of absolute allegiance and loyalty to the Republic of the Philippines.

To hold the oath to be a mere pro forma requirement is to say that it is only for
ceremonial purposes; it would also accommodate a mere qualified or temporary
allegiance from government officers when the Constitution and the legislature
clearly demand otherwise. x x x x

The fact that petitioner won the elections cannot cure the defect of her candidacy.
Garnering the most number of votes does not validate the election of a disqualified
candidate because the application of the constitutional and statutory provisions on
disqualification is not a matter of popularity. x x x x

(2) No.

The fact that the petitioner’s qualifications were not questioned when she filed
certificates of candidacy for 2007 and 2010 elections cannot operate as an estoppel to
the petition for quo warranto before the RTC.

Page 271 of 323


Under the Batas Pambansa Bilang 881 (Omnibus Election Code), there are two
instances where a petition questioning the qualifications of a registered
candidate to run for the office for which his certificate of candidacy was filed can be
raised, to wit:

(a) Before election, pursuant to Section 78 thereof which provides that:

Sec. 78. Petition to deny due course or to cancel a certificate of candidacy. – A verified petition seeking to
deny due course or to cancel a certificate of candidacy may be filed by any person exclusively on the
ground that any material representation contained therein as required under Section 74 hereof is
false. The petition may be filed at any time not later than twenty-five (25) days from the time of the
filing of the certificate of candidacy and shall be decided, after due notice and hearing, not later than
fifteen days before the election; and

(b) After election, pursuant to Section 253 thereof, viz:

Sec. 253. Petition for quo warranto. – Any voter contesting the election of any Member of the Batasang
Pambansa, regional, provincial, or city officer on the ground of ineligibility or of disloyalty to the
Republic of the Philippines shall file a sworn petition for quo warranto with the Commission within ten
(10) days after the proclamation of the results of the election.

Hence, if a person qualified to file a petition to disqualify a certain candidate fails to


file the petition within the twenty-five (25)-day period prescribed by Section 78 of the
Omnibus Election Code for whatever reasons, the elections laws do not leave him
completely helpless as he has another chance to raise the disqualification of the
candidate by filing a petition for quo warranto within ten (10) days from the
proclamation of the results of the election, as provided under Section 253 of the
Omnibus Election Code.

(3) Yes.

In fine, R.A. No. 9225 categorically demands natural-born Filipinos who re-
acquire their citizenship and seek elective office, to execute a personal and
sworn renunciation of any and all foreign citizenships before an authorized
public officer prior to or simultaneous to the filing of their certificates of
candidacy, to qualify as candidates in Philippine elections. The rule applies to all
those who have re-acquired their Filipino citizenship, like petitioner, without regard as
to whether they are still dual citizens or not. It is a pre-requisite imposed for the
exercise of the right to run for public office.

Stated differently, it is an additional qualification for elective office specific only to


Filipino citizens who re-acquire their citizenship under Section 3 of R.A. No. 9225. It
is the operative act that restores their right to run for public office. The
petitioner's failure to comply therewith in accordance with the exact tenor of
the law, rendered ineffectual the Declaration of Renunciation of Australian
Citizenship she executed on September 18, 2006. As such, she is yet to regain her
political right to seek elective office. Unless she executes a sworn renunciation of
her Australian citizenship, she is ineligible to run for and hold any elective office
in the Philippines.

*Maquiling v. Comelec, G.R. No. 195649, April 16, 2013. [IMPORTANT!!!]


(Disqualification; Dual Citizenship; Naturalization; US Passport)

Facts:
-Rommel Arnado (respondent) is a natural born Filipino citizen. However, as a
consequence of his subsequent naturalization as a citizen of the United States of
America, he lost his Filipino citizenship.

-Arnado applied for repatriation under Republic Act (R.A.) No. 9225 before the
Consulate General of the Philippines in San Franciso, USA and took the Oath of
Allegiance to the Republic of the Philippines on 10 July 2008. On the same day an
Order of Approval of his Citizenship Retention and Re-acquisition was issued in his
favor.

Page 272 of 323


-On 3 April 2009 Arnado again took his Oath of Allegiance to the Republic and
executed an Affidavit of Renunciation of his foreign citizenship.

-On 30 November 2009, Arnado filed his Certificate of Candidacy for Mayor of
Kauswagan, Lanao del Norte, which contained, among others, the statement that he
was not a permanent resident of, or immigrant to, a foreign country.

-On 28 April 2010, respondent Linog C. Balua (Balua), another mayoralty candidate,
filed a petition to disqualify Arnado and/or to cancel his certificate of candidacy for
municipal mayor of Kauswagan, Lanao del Norte.

-On 30 April 2010, the COMELEC (First Division) issued an Order requiring the
respondent to personally file his answer and memorandum within three (3) days from
receipt thereof.

-After Arnado failed to answer the petition, Balua moved to declare him in default and
to present evidence ex-parte. Neither motion was acted upon, having been overtaken
by the 2010 elections where Arnado garnered the highest number of votes and was
subsequently proclaimed as the winning candidate for Mayor of Kauswagan, Lanao
del Norte.

-It was only after his proclamation that Arnado filed his verified answer.

-Instead of treating the Petition as an action for the cancellation of a certificate of


candidacy based on misrepresentation, the COMELEC First Division considered it as
one for disqualification.

-After due proceedings, the Comelec First Division granted the disqualification
petition and ruled that Arnado’s continued use of his US passport is a strong
indication that Arnado had no real intention to renounce his US citizenship and that
he only executed an Affidavit of Renunciation to enable him to run for office.

-Arnado then sought reconsideration of the resolution before the COMELEC En Banc.

-Arnado’s contentions: (1) his US passport subsequent to his renunciation of his


American citizenship is not tantamount to a repudiation of his Filipino citizenship, as
he did not perform any act to swear allegiance to a country other than the Philippines;
(2) his proclamation as the winning candidate ousted the COMELEC from jurisdiction
over the case and the proper recourse was a petition for quo warranto, which should
have been filed within ten days from his proclamation.

-Petitioner Casan Macode Maquiling (Maquiling), another candidate for mayor of


Kauswagan, and who garnered the second highest number of votes in the 2010
elections, intervened in the case. Arnado opposed all motions filed by Maquiling,
claiming that intervention is prohibited after a decision has already been rendered,
and that as a second-placer, Maquiling undoubtedly lost the elections and thus does
not stand to be prejudiced or benefitted by the final adjudication of the case.

-After due proceeedings, the COMELEC En Banc reversed and set aside the ruling of
the First Division and granted Arnado’s Motion for Reconsideration.

-Maquiling then went to the SC via Petition for Certiorari ender Rule 64 in
conjunction with Rule 65.

Issues:
(1) Whether or not intervention is allowed in a disqualification case;
(2) Whether or not it was proper for Comelec to treat the petition for cancellation of
Arnado’s COC as a disqualification case;
(3) Whether or not the use of a foreign passport after renouncing foreign citizenship
affects one’s qualifications to run for public office;

Page 273 of 323


(4) Whether or not the rule on succession in the Local Government Code is applicable
to this case.

Ruling:
(1) Yes.

Section 6 of R.A. No. 6646 states:

Sec. 6. Effect of Disqualification Case. - Any candidate who has been declared by final judgment to be
disqualified shall not be voted for, and the votes cast for him shall not be counted. If for any reason a
candidate is not declared by final judgment before an election to be disqualified and he is voted for and
receives the winning number of votes in such election, the Court or Commission shall continue with
the trial and hearing of the action, inquiry, or protest and, upon motion of the complainant or any
intervenor, may during the pendency thereof order the suspension of the proclamation of such candidate
whenever the evidence of his guilt is strong.

Under this provision, intervention may be allowed in proceedings for


disqualification even after election if there has yet been no final judgment
rendered.

Clearly then, Maquiling has the right to intervene in the case. The fact that the
COMELEC En Banc has already ruled that Maquiling has not shown that the
requisites for the exemption to the second-placer rule set forth in Sinsuat v. COMELEC
are present and therefore would not be prejudiced by the outcome of the case, does
not deprive Maquiling of the right to elevate the matter before this Court.

Arnado’s claim that the main case has attained finality as the original petitioner and
respondents therein have not appealed the decision of the COMELEC En Banc, cannot
be sustained. The elevation of the case by the intervenor prevents it from attaining
finality. It is only after this Court has ruled upon the issues raised in this instant
petition that the disqualification case originally filed by Balua against Arnado will
attain finality.

(2) Yes. (See above provision. “the Court or Commission shall continue with the trial and
hearing of the action…”)

(3) Yes.

The use of foreign passport after renouncing one’s foreign citizenship is a


positive and voluntary act of representation as to one’s nationality and
citizenship; it does not divest Filipino citizenship regained by repatriation but it
recants the Oath of Renunciation required to qualify one to run for an elective
position.

Arnado took all the necessary steps to qualify to run for a public office. He took the
Oath of Allegiance and renounced his foreign citizenship. There is no question that
after performing these twin requirements required under Section 5(2) of R.A. No. 9225
or the Citizenship Retention and Re-acquisition Act of 2003, he became eligible to run
for public office.

By renouncing his foreign citizenship, he was deemed to be solely a Filipino citizen,


regardless of the effect of such renunciation under the laws of the foreign country.

However, this legal presumption does not operate permanently and is open to attack
when, after renouncing the foreign citizenship, the citizen performs positive acts
showing his continued possession of a foreign citizenship.

Arnado himself subjected the issue of his citizenship to attack when, after renouncing
his foreign citizenship, he continued to use his US passport to travel in and out of the
country before filing his certificate of candidacy on 30 November 2009. The pivotal
question to determine is whether he was solely and exclusively a Filipino citizen at the

Page 274 of 323


time he filed his certificate of candidacy, thereby rendering him eligible to run for
public office.

Between 03 April 2009, the date he renounced his foreign citizenship, and 30
November 2009, the date he filed his COC, he used his US passport four times,
actions that run counter to the affidavit of renunciation he had earlier executed.
By using his foreign passport, Arnado positively and voluntarily represented
himself as an American, in effect declaring before immigration authorities of both
countries that he is an American citizen, with all attendant rights and privileges
granted by the United States of America.

The renunciation of foreign citizenship is not a hollow oath that can simply be
professed at any time, only to be violated the next day. It requires an absolute and
perpetual renunciation of the foreign citizenship and a full divestment of all civil and
political rights granted by the foreign country which granted the citizenship.

While the act of using a foreign passport is not one of the acts enumerated in
Commonwealth Act No. 63 constituting renunciation and loss of Philippine citizenship,
it is nevertheless an act which repudiates the very oath of renunciation required for a
former Filipino citizen who is also a citizen of another country to be qualified to run for
a local elective position.

When Arnado used his US passport on 14 April 2009, or just eleven days after he
renounced his American citizenship, he recanted his Oath of Renunciation that he
"absolutely and perpetually renounce(s) all allegiance and fidelity to the UNITED
STATES OF AMERICA" and that he "divest(s) himself of full employment of all civil and
political rights and privileges of the United States of America."

We agree with the COMELEC En Banc that such act of using a foreign passport
does not divest Arnado of his Filipino citizenship, which he acquired by
repatriation. However, by representing himself as an American citizen, Arnado
voluntarily and effectively reverted to his earlier status as a dual citizen. Such
reversion was not retroactive; it took place the instant Arnado represented himself as
an American citizen by using his US passport.

This act of using a foreign passport after renouncing one’s foreign citizenship is
fatal to Arnado’s bid for public office, as it effectively imposed on him a
disqualification to run for an elective local position.

By the time he filed his certificate of candidacy on 30 November 2009, Arnado


was a dual citizen enjoying the rights and privileges of Filipino and American
citizenship. He was qualified to vote, but by the express disqualification under
Section 40(d) of the Local Government Code, he was not qualified to run for a
local elective position.

In effect, Arnado was solely and exclusively a Filipino citizen only for a period of eleven
days, or from 3 April 2009 until 14 April 2009, on which date he first used his
American passport after renouncing his American citizenship. x x x x

Qualifications for public office are continuing requirements and must be possessed
not only at the time of appointment or election or assumption of office but during the
officer's entire tenure. Once any of the required qualifications is lost, his title may be
seasonably challenged.

The purpose of the Local Government Code in disqualifying dual citizens from running
for any elective public office would be thwarted if we were to allow a person who has
earlier renounced his foreign citizenship, but who subsequently represents himself as
a foreign citizen, to hold any public office.

Arnado’s subsequent use of his Philippine passport does not correct the fact that
after he renounced his foreign citizenship and prior to filing his certificate of

Page 275 of 323


candidacy, he used his US passport. In the same way that the use of his foreign
passport does not undo his Oath of Renunciation, his subsequent use of his Philippine
passport does not undo his earlier use of his US passport.

Citizenship is not a matter of convenience. It is a badge of identity that comes with


attendant civil and political rights accorded by the state to its citizens. It likewise
demands the concomitant duty to maintain allegiance to one’s flag and country. While
those who acquire dual citizenship by choice are afforded the right of suffrage, those
who seek election or appointment to public office are required to renounce their
foreign citizenship to be deserving of the public trust. Holding public office demands
full and undivided allegiance to the Republic and to no other.

(4) No.

With Arnado’s disqualification, Maquiling then becomes the winner in the


election as he obtained the highest number of votes from among the qualified
candidates.

We have ruled in the recent cases of Aratea v. COMELEC and Jalosjos v. COMELEC
that a void COC cannot produce any legal effect. Thus, the votes cast in favor of
the ineligible candidate are not considered at all in determining the winner of an
election. x x x x

Even when the votes for the ineligible candidate are disregarded, the will of the
electorate is still respected, and even more so. The votes cast in favor of an ineligible
candidate do not constitute the sole and total expression of the sovereign voice. The
votes cast in favor of eligible and legitimate candidates form part of that voice and
must also be respected.

As in any contest, elections are governed by rules that determine the


qualifications and disqualifications of those who are allowed to participate as
players. When there are participants who turn out to be ineligible, their victory is
voided and the laurel is awarded to the next in rank who does not possess any of the
disqualifications nor lacks any of the qualifications set in the rules to be eligible as
candidates. x x x x

The electorate’s awareness of the candidate’s disqualification is not a prerequisite for


the disqualification to attach to the candidate. The very existence of a disqualifying
circumstance makes the candidate ineligible. Knowledge by the electorate of a
candidate’s disqualification is not necessary before a qualified candidate who placed
second to a disqualified one can be proclaimed as the winner. The second-placer in the
vote count is actually the first-placer among the qualified candidates.

That the disqualified candidate has already been proclaimed and has assumed office is
of no moment. The subsequent disqualification based on a substantive ground
that existed prior to the filing of the certificate of candidacy voids not only the
COC but also the proclamation. x x x x

The disqualifying circumstance surrounding Arnado’s candidacy involves his


citizenship. It does not involve the commission of election offenses as provided for in
the first sentence of Section 68 of the Omnibus Election Code, the effect of which is to
disqualify the individual from continuing as a candidate, or if he has already been
elected, from holding the office. x x x x The affirmation of Arnado's disqualification,
although made long after the elections, reaches back to the filing of the
certificate of candidacy. Arnado is declared to be not a candidate at all in the
May 2010 elections.

Arnado being a non-candidate, the votes cast in his favor should not have been
counted. This leaves Maquiling as the qualified candidate who obtained the
highest number of votes. Therefore, the rule on succession under the Local
Government Code will not apply.

Page 276 of 323


[IMPORTANT NOTE: This case has to be contrasted with the case of Grace Poe-
Llamanzares. Poe was not disqualified therein because while it is true that she still
used her US passport AFTER her REPATRIATION in 2006, she no longer used her US
passport after her RENUNCIATION of allegiance to the United States of America in
2010. In Maquiling case, he used his US passport even after he renounced his US
allegiance. Hence, the disqualification.

[ANOTHER IMPORTANT NOTE: Contrast this case with Ejercito v. Comelec, G.R. No.
212398, November 25, 2014, wherein former governor E.R. Ejercito was later on
disqualified for having exceeded the total allowable election campaign expenditures
mandated by law. The second placer did not take his place. It was the vice governor
who replaced E.R. Ejercito pursuant to the provisions of the Local Government Code.
See the digest of that case to see why.]

*Ugdoracion v. Comelec, G.R. No. 179851, April 18, 2008.


(Disqualification; Residency; Green Card Holder)

Facts:
-Jose Ugdoracion, Jr., (petitioner) and Ephraim Tungol (respondent) were rival
mayoralty candidates in the Municipality of Albuquerque, Province of Bohol in the
May 14, 2007 elections. Both filed their respective Certificates of Candidacy (COC).

-On April 11, 2007, Tungol filed a Petition to Deny Due Course or Cancel the
Certificate of Candidacy of Jose Ugdoracion, Jr., contending that Ugdoracion's
declaration of eligibility for Mayor constituted material misrepresentation because
Ugdoracion is actually a "green card" holder or a permanent resident of the United
States of America (USA). Specifically, Ugdoracion stated in his COC that he had
resided in Albuquerque, Bohol, Philippines for forty-one years before May 14, 2007
and he is not a permanent resident or an immigrant to a foreign country.

-Tungol’s contention: In our jurisdiction, domicile is equivalent to residence, and he


retained his domicile of origin (Albuquerque, Bohol) notwithstanding his ostensible
acquisition of permanent residency in the USA. Also, his status as a “green card”
holder was not of his own making but a mere offshoot of a petition filed by his sister.
He admitted his intermittent travels to the USA, but only to visit his siblings, and
short working stint thereat to cover his subsistence for the duration of his stay.

-On May 8, 2007, the COMELEC First Division promulgated ruled to cancel
Ugdoracion's COC and remove his name from the certified list of candidates for the
position of Mayor of Albuquerque, Bohol.

Issue:
Whether or not the COMELEC committed grave abuse of discretion in cancelling
Ugdoracion's COC for material misrepresentation.

Ruling:
No.

The dust had long settled over the implications of a “green card” holder status on an
elective official's qualification for public office. We ruled in Caasi v. Court of Appeals
that a Filipino citizen's acquisition of a permanent resident status abroad
constitutes an abandonment of his domicile and residence in the Philippines. In
short, the “green card” status in the USA is a renunciation of one's status as a
resident of the Philippines. x x x x

Domicile is classified into (1) domicile of origin, which is acquired by every


person at birth; (2) domicile of choice, which is acquired upon abandonment of
the domicile of origin; and (3) domicile by operation of law, which the law
attributes to a person independently of his residence or intention. x x x x

Page 277 of 323


The general rule is that the domicile of origin is not easily lost; it is lost only when
there is an actual removal or change of domicile, a bona fide intention of abandoning
the former residence and establishing a new one, and acts which correspond with
such purpose. In the instant case, however, Ugdoracion's acquisition of a lawful
permanent resident status in the United States amounted to an abandonment
and renunciation of his status as a resident of the Philippines; it constituted a
change from his domicile of origin, which was Albuquerque, Bohol, to a new
domicile of choice, which is the USA.

The contention that Ugdoracion’s USA resident status was acquired involuntarily, as it
was simply the result of his sister's beneficence, does not persuade. Although
immigration to the USA through a petition filed by a family member (sponsor) is
allowed by USA immigration laws, the petitioned party is very much free to accept
or reject the grant of resident status. Permanent residency in the USA is not
conferred upon the unwilling; unlike citizenship, it is not bestowed by operation of
law. To reiterate, a person can have only one residence or domicile at any given time. x
xxx

We are not unmindful of the fact that Ugdoracion appears to have won the election as
Mayor of Albuquerque, Bohol. Sadly, winning the election does not substitute for
the specific requirements of law on a person’s eligibility for public office which
he lacked, and does not cure his material misrepresentation which is a valid ground
for the cancellation of his COC.

*Ejercito v. Comelec, G.R. No. 212398, November 25, 2014.


(Election Offenses; Remedial Law: Formal Offer; Comelec; Disqualification;
Overspending; Succession)

Facts:
-Emilio Ramon "E.R." P. Ejercito (petitioner) and Edgar "Egay" S. San Luis
(respondent) were both running for governor for the May 13, 2013 National and Local
Elections.

-Three days prior to the elections, San Luis filed a petition for disqualification before
the Office of the COMELEC Clerk in Manila against Ejercito.

-The petition alleged the following:

*Based on the records of the Provincial COMELEC, the Province of Laguna has a total
of 1,525,522 registered electorate.

*Par. (a), Section 5 of COMELEC Resolution No. 9615, otherwise known as the Rules
and Regulations Implementing FAIR ELECTION ACT provides that “the aggregate
amount that a candidate may spent for election campaign is three pesos (₱3.00) for
every voter currently registered in the constituency where the candidate filed his
certificate of candidacy.”

*Accordingly, a candidate for the position of Provincial Governor of Laguna is only


authorized to incur an election expense amounting to FOUR MILLION FIVE HUNDRED
SEVENTY-SIX THOUSAND FIVE HUNDRED SIXTY-SIX (₱4,576,566.00) PESOS.

*However, in violation of the afore-quoted provision of law, Ejercito exceeded his


expenditures in relation to his campaign for the 2013 election. For television
campaign commercials alone, he already spent the sum of Ph₱23,730,784.00 based
on LP’s official monitoring on the following dates: April 28, May 4 & May 5, 2013.

-Elections were then held. Ejercito ranked first while San Luis ranked second.

-Subsequently, on May 16, 2013, San Luis filed a Very Urgent Ex-Parte Motion to
Issue Suspension of Possible Proclamation of Respondent and Supplemental to the
Very Urgent Ex-Parte Motion to Issue Suspension of Possible Proclamation of

Page 278 of 323


Respondent. However, these were not acted upon by the COMELEC. The next day,
Ejercito and Ramil L. Hernandez were proclaimed by the Provincial Board of
Canvassers as the duly-elected Governor and Vice-Governor, respectively, of Laguna.

-The COMELEC First Division issued a Summons with Notice of Conference on June
4, 2013. Ejercito then filed his Verified Answer on June 13, 2013 that prayed for the
dismissal of the petition.

-In his Memorandum, San Luis further alleged that he was able to secure documents
from the Information and Education Department of the Commission on Elections
showing that Ejercito incurred advertising expenses with ABS-CBN in the amount of
₱20,197,170.25], not to mention his advertisement with GMA 7. Copies of the
summary report, media purchase order, advertising contract and official receipt were
marked as EXHS. "B-1", "B-2", "B-3", and"B-4" (Annexes "A", "B", "C", and "D",
supplemental to the very urgent ex-parte motion).

-Due proceedings ensued, and on September 26, 2013, the COMELEC First Division
promulgated a Resolution granting the Petition for Disqualification filed against
Ejercito and declaring permanent vacancy in the Office of the Provincial Governor of
Laguna.

-Therafter, Vice-Governor Ramil Hernandez was sworn in as the Governor of


Laguna at the COMELEC Main Office in Manila.

-Ejercito then went to the SC via petition for certiorari under Rule 64, in relation to
Rule 65.

-Ejercito’s contentions: (1) The accusation deserves no consideration for being


speculative, self-serving, and uncorroborated by any other substantial evidence; (2)
Considering that San Luis’ petition partakes of the nature of a complaint for election
offenses, the COMELEC First Division has no jurisdiction over the same based on
COMELEC Resolution No. 938651 and Section 26552 of the OEC; (3) The conduct of
preliminary investigation to determine whether the acts enumerated under Section 68
of the OEC were indeed committed is a requirement prior to actual disqualification; (4)
The Advertising Contract dated May 8, 2013 should not have been relied upon by the
COMELEC. First, it was not formally offered in evidence pursuant to Section 34,
Rule 13268 of the Rules and he was not even furnished with a copy thereof,
depriving him of the opportunity to examine its authenticity and due execution and
object to its admissibility; (5) The advertising contracts between ABS-CBN
Corporation and Scenema Concept International, Inc. were executed by an identified
supporter without his knowledge and consent as, in fact, his signature thereon was
obviously forged; and (6) the petition questioning his qualification was rendered moot
and academic by his proclamation as the duly-elected Provincial Governor of Laguna
for the term 2013-2016.

Issues:
(1) Whether or not prior conduct of preliminary investigation is required in the
resolution of the electoral aspect of a disqualification case;
(2) Whether or not the Advertising Contract dated May 8, 2013 should not have been
relied upon by the COMELEC, considering that the same was not formally offered in
evidence;
(3) Whether or not Ejercito should be disqualified;
(4) Whether or not petitioner San Luis, as the second placer, should replace Ejercito.

Ruling:
(1) No.

Sec. 6 of RA No. 6646 provides:

“SEC. 6. Effects of Disqualification Case. - Any candidate who has been declared by final judgment to be
disqualified shall not be voted for, and the votes cast for him shall not be counted. If for any reason a

Page 279 of 323


candidate is not declared by final judgment before an election to be disqualified and he is voted for and
receives the winning number of votes in such election, the Court or Commission shall continue
with the trial and hearing of the action, inquiry or protestand, upon motion of the complainant or any
intervenor, may during the pendency thereof order the suspension of the proclamation of such candidate
whenever the evidence of his guilt is strong.”

Clearly, the legislative intent is that the COMELEC should continue the trial and
hearing of the disqualification case to its conclusion, i.e., until judgment is rendered
thereon. The word “shall” signifies that this requirement of the law is mandatory,
operating to impose a positive duty which must be enforced. The implication is that
the COMELEC is left with no discretion but to proceed with the disqualification case
even after the election. x x x x

The “exclusive power of the COMELEC to conduct a preliminary investigation of


all cases involving criminal infractions of the election laws” stated in Par. 1 of
COMELEC Resolution No. 2050 pertains to the criminal aspect of a
disqualification case. It has been repeatedly underscored that an election offense
has its criminal and electoral aspects. While its criminal aspect to determine the
guilt or innocence of the accused cannot be the subject of summary hearing, its
electoral aspect to ascertain whether the offender should be disqualified from
office can be determined in an administrative proceeding that is summary in
character.

This Court said in Sunga case:

“It is worth to note that an election offense has criminal as well as electoral aspects.
Its criminal aspect involves the ascertainment of the guilt or innocence of the accused
candidate. Like in any other criminal case, it usually entails a full-blown hearing and
the quantum of proof required to secure a conviction is beyond reasonable doubt. Its
electoral aspect, on the other hand, is a determination of whether the offender
should be disqualified from office. This is done through an administrative
proceeding which is summary in character and requires only a clear preponderance of
evidence. Thus, under Sec. 4 of the COMELEC Rules of Procedure, petitions for
disqualification “shall be heard summarily after due notice.” It is the electoral
aspect that we are more concerned with, under which an erring candidate may
be disqualified even without prior criminal conviction.”

and equally in Lanot case:

“The electoral aspect of a disqualification case determines whether the offender


should be disqualified from being a candidate or from holding office. Proceedings
are summary in character and require only clear preponderance of evidence. An
erring candidate may be disqualified even without prior determination of
probable cause in a preliminary investigation. The electoral aspect may proceed
independently of the criminal aspect, and vice-versa.

The criminal aspect of a disqualification case determines whether there is


probable cause to charge a candidate for an election offense. The prosecutor is
the COMELEC, through its Law Department, which determines whether probable
cause exists. If there is probable cause, the COMELEC, through its Law
Department, files the criminal information before the proper court. Proceedings
before the proper court demand a full-blown hearing and require proof beyond
reasonable doubt to convict. A criminal conviction shall result in the disqualification of
the offender, which may even include disqualification from holding a future public
office.

(2) No.

Contrary to Ejercito’s claim, Section 34, Rule 132 of the Rules is inapplicable. Section
4, Rule 171 of the Rules of Court is clear enough in stating that it shall not
apply to election cases except by analogy or in a suppletory character and
whenever practicable and convenient. In fact, nowhere from COMELEC

Page 280 of 323


Resolution No. 9523 requires that documentary evidence should be formally
offered in evidence. We remind again that the electoral aspect of a disqualification
case is done through an administrative proceeding which is summary in
character. x x x x

Further, Ejercito’s dependence on Ang Tibay is weak. The essence of due process is
simply an opportunity to be heard, or, as applied to administrative proceedings, an
opportunity to explain one's side or an opportunity to seek for a reconsideration of the
action or ruling complained of. Any seeming defect in its observance is cured by the
filing of a motion for reconsideration and denial of due process cannot be successfully
invoked by a party who had the opportunity to be heard thereon. In this case, it is
undisputed that Ejercito filed a motion for reconsideration before the COMELEC
En Banc. Despite this, he did not rebut the authenticity and due execution of the
advertising contracts when he decided not to discuss the factual findings of the
COMELEC First Division on the alleged ground that it may be construed as a waiver
of the jurisdictional issues that he raised.

Also, The COMELEC may properly take and act on the advertising contracts without
further proof from the parties herein. Aside from being considered as an admission
and presumed to be proper submissions from them, the COMELEC already has
knowledge of the contracts for being ascertainable from its very own records.
Said contracts are ought to be known by the COMELEC because of its statutory
function as the legal custodian of all advertising contracts promoting or
opposing any candidate during the campaign period. As what transpired in this
case, the COMELEC has the authority and discretion to compare the submitted
advertising contracts with the certified true copies of the broadcast logs, certificates of
performance or other analogous records which a broadcast station or entity is required
to submit for the review and verification of the frequency, date, time and duration of
advertisements aired.

(3) Yes.

We refuse to believe that the advertising contracts between ABS-CBN Corporation and
Scenema Concept International, Inc. were executed without Ejercito’s knowledge and
consent.

Notably, R.A. No. 9006 explicitly directs that broadcast advertisements donated
to the candidate shall not be broadcasted without the written acceptance of the
candidate, which shall be attached to the advertising contract and shall be submitted
to the COMELEC, and that, in every case, advertising contracts shall be signed by the
donor, the candidate concerned or by the duly-authorized representative of the
political party.

COMELEC Resolution No. 9615 also unambiguously states that it shall be unlawful to
broadcast any election propaganda donated or given free of charge by any person or
broadcast entity to a candidate withoutthe written acceptance of the said candidate
and unless they bear and be identified by the words "airtime for this broadcast was
provided free of charge by" followed by the true and correct name and address of the
donor.

This Court cannot give weight to Ejercito’s representation that his signature on the
advertising contracts was a forgery. The issue is a belated claim, raised only for the
first time in this petition for certiorari. It is a rudimentary principle of law that matters
neither alleged in the pleadings nor raised during the proceedings below cannot be
ventilated for the first time on appeal before the Supreme Court. It would be offensive
to the basic rules of fair play and justice to allow Ejercito to raise an issue that was
not brought up before the COMELEC. While it is true that litigation is not a game of
technicalities, it is equally truethat elementary considerations of due process require
that a party be duly apprised of a claim against him before judgment may be rendered.

Page 281 of 323


Likewise, whether the advertising contracts were executed without Ejercito’s
knowledge and consent, and whether his signatures thereto were fraudulent, are
issues of fact. Any factual challenge has no place in a Rule 65 petition. This Court is
not a trier of facts and is not equipped to receive evidence and determine the truth of
factual allegations. x x x x

It is important to note that Ejercito deliberately did not tackle the merit and substance
of the charges against him. He limited himself to raising procedural issues. The Court
finds that it has been sufficiently established that Ejercito had spent in his election
campaign an amount in excess of that allowed by law.

(4) No.

The votes for the disqualified winning candidate remained valid. The second placer is
just that, a second placer. He lost the elections. He was repudiated by either a
majority or plurality of voters.

San Luis could not be considered the first among qualified candidates, considering
that Ejercito was a qualified candidate who has later on been disqualified for
overspending.

With the disqualification of the winning candidate and the application of the doctrine
of rejection of the second placer, the rules on succession under the law accordingly
apply. Here, the vice governor takes the place of the disqualified governor.

*Peñera v. Comelec, G.R. No. 181613, November 25, 2009. [IMPORTANT!!!


DOCTRINAL!!!]
(Prohibitions; Premature Campaigning)

Facts:
-Petitioner Rosalinda A. Penera (Penera) and private respondent Edgar T. Andanar
(Andanar) were mayoralty candidates in Sta. Monica during the 14 May 2007
elections.

-On 2 April 2007, Andanar filed before the Office of the Regional Election Director
(ORED), Caraga Region (Region XIII), a Petition for Disqualification against Penera,
as well as the candidates for Vice-Mayor and Sangguniang Bayan who belonged to her
political party, for premature campaigning.

-Andanar claimed that on 29 March 2007 – a day before the start of the authorized
campaign period on 30 March 2007 – Penera and her partymates went around the
different barangays in Sta. Monica, announcing their candidacies and requesting the
people to vote for them on the day of the elections. Attached to the Petition were the
Affidavits of individuals who witnessed the said incident.

-After the parties filed their respective position papers, the records of the case were
transmitted to the COMELEC main office in Manila for adjudication. It was
subsequently raffled to the COMELEC Second Division.

-While the disqualification case was pending before the COMELEC Second Division,
the 14 May 2007 elections took place and, as a result thereof, Penera was proclaimed
the duly elected Mayor of Sta. Monica. Penera soon assumed office on 2 July 2002.

-After due proceedings, the COMELEC Second Division issued a resolution, which
disqualified Penera from continuing as a mayoralty candidate in Sta. Monica, for
engaging in premature campaigning, in violation of Sections 80 and 68 of the
Omnibus Election Code.

-Upon elevation to the Comelec En Banc, the resolution of the COMELEC Second
Division was affirmed.

Page 282 of 323


-Penera then went to the SC via Petition for Certiorari with Prayer for the Issuance of
a Writ of Preliminary Injunction and/or Temporary Restraining Order under Rule 65.

Issue:
Whether or not Penera engaged in an election campaign or partisan political activity
outside the campaign period.

Ruling:
No.

Section 79(a) of the Omnibus Election Code defines a “candidate” as “any person
aspiring for or seeking an elective public office, who has filed a certificate of
candidacy x x x x.” The second sentence, third paragraph, Section 15 of RA 8436, as
amended by Section 13 of RA 9369, provides that, “any person who files his
certificate of candidacy within the period for filing shall only be considered as a
candidate at the start of the campaign period for which he filed his certificate
of candidacy.” The immediately succeeding proviso in the same third paragraph
states that, “unlawful acts or omissions applicable to a candidate shall take effect
only upon the start of the aforesaid campaign period.” x x x x

The campaign period for local officials began on 30 March 2007 and ended on 12
May 2007. Penera filed her certificate of candidacy on 29 March 2007. Penera
was thus a candidate on 29 March 2009 only for purposes of printing the ballots. On
29 March 2007, the law still did not consider Penera a candidate for purposes
other than the printing of ballots.

The acts committed by Penera prior to 30 March 2007, the date when she
became a “candidate,” even if constituting election campaigning or partisan
political activities, are not punishable under Section 80 of the Omnibus Election
Code. Such acts are within the realm of a citizen’s protected freedom of expression. x
xxx

Under Section 11 of RA 8436, the only purpose for the early filing of certificates
of candidacy is to give ample time for the printing of official ballots. x x x x

Congress wanted to insure that no person filing a certificate of candidacy under


the early deadline required by the automated election system would be
disqualified or penalized for any partisan political act done before the start of
the campaign period. Thus, in enacting RA 9369, Congress expressly wrote the Lanot
doctrine into the second sentence, third paragraph of the amended Section 15 of RA
8436, thus:
xxxx
For this purpose, the Commission shall set the deadline for the filing of certificate of candidacy/petition
for registration/manifestation to participate in the election. Any person who files his certificate of
candidacy within this period shall only be considered as a candidate at the start of the campaign period
for which he filed his certificate of candidacy: Provided, That, unlawful acts or omissions applicable to a
candidate shall take effect only upon the start of the aforesaid campaign period: Provided, finally, That
any person holding a public appointive office or position, including active members of the armed forces,
and officers and employees in government-owned or -controlled corporations, shall be considered ipso
facto resigned from his/her office and must vacate the same at the start of the day of the filing of his/her
certificate of candidacy. (Boldfacing and underlining supplied)

In enacting RA 9369, Congress even further clarified the first proviso in the third
paragraph of Section 15 of RA 8436. The original provision in RA 8436 states —

x x x Provided, further, That, unlawful acts or omissions applicable to a candidate shall take effect upon
the start of the aforesaid campaign period, x x x.

In RA 9369, Congress inserted the word “only” so that the first proviso now reads —

x x x Provided, That, unlawful acts or omissions applicable to a candidate shall take effect only upon
the start of the aforesaid campaign period x x x x.

Page 283 of 323


Thus, Congress not only reiterated but also strengthened its mandatory directive
that election offenses can be committed by a candidate “only” upon the start of
the campaign period. This clearly means that before the start of the campaign
period, such election offenses cannot be so committed.

When the applicable provisions of RA 8436, as amended by RA 9369, are read


together, these provisions of law do not consider Penera a candidate for purposes
other than the printing of ballots, until the start of the campaign period. There is
absolutely no room for any other interpretation.

We quote with approval the Dissenting Opinion of Justice Antonio T. Carpio:

“x x x x The definition of a ‘candidate’ in Section 79(a) of the Omnibus Election Code


should be read together with the amended Section 15 of RA 8436. A “‘candidate’ refers
to any person aspiring for or seeking an elective public office, who has filed a
certificate of candidacy by himself or through an accredited political party,
aggroupment or coalition of parties.” However, it is no longer enough to merely file
a certificate of candidacy for a person to be considered a candidate because “any
person who files his certificate of candidacy within the filing period shall only be
considered a candidate at the start of the campaign period for which he filed his
certificate of candidacy.” Any person may thus file a certificate of candidacy on any
day within the prescribed period for filing a certificate of candidacy yet that person
shall be considered a candidate, for purposes of determining one’s possible violations
of election laws, only during the campaign period. Indeed, there is no ‘election
campaign’ or ‘partisan political activity’ designed to promote the election or
defeat of a particular candidate or candidates to public office simply because
there is no “candidate” to speak of prior to the start of the campaign period.
Therefore, despite the filing of her certificate of candidacy, the law does not
consider Penera a candidate at the time of the questioned motorcade which was
conducted a day before the start of the campaign period. x x x”

In layman’s language, this means that a candidate is liable for an election offense only
for acts done during the campaign period, not before. The law is clear as daylight —
any election offense that may be committed by a candidate under any election
law cannot be committed before the start of the campaign period. x x x x

The forum for examining the wisdom of the law, and enacting remedial measures, is
not this Court but the Legislature. This Court has no recourse but to apply a law
that is as clear, concise and express as the second sentence, and its immediately
succeeding proviso, as written in the third paragraph of Section 15 of RA 8436, as
amended by RA 9369.

*Amora Jr. v. Comelec, G.R. No. 192280, January 25, 2011.


(Disqualification; Notarization of COC; Oath)

Facts:
-On December 1, 2009, petitioner Sergio G. Amora, Jr. (Amora) filed his Certificate of
Candidacy (COC) for Mayor of Candijay, Bohol. At that time, Amora was the
incumbent Mayor of Candijay and had been twice elected to the post, in the years
2004 and 2007.

-To oppose Amora, the Nationalist People’s Coalition (NPC) fielded Trygve L. Olaivar
(Olaivar) for the mayoralty post. Respondent Arnielo S. Olandria (Olandria) was one of
the candidates for councilor of the NPC in the same municipality.

-On March 5, 2010, Olandria filed before the COMELEC a Petition for Disqualification
against Amora. Olandria alleged that Amora’s COC was not properly sworn contrary
to the requirements of the Omnibus Election Code (OEC) and the 2004 Rules on
Notarial Practice. Olandria pointed out that, in executing his COC, Amora merely
presented his Community Tax Certificate (CTC) to the notary public, Atty. Oriculo

Page 284 of 323


Granada (Atty. Granada), instead of presenting competent evidence of his identity.
Consequently, Amora’s COC had no force and effect and should be considered as not
filed.

-After due proceedings, the Second Division of the COMELEC granted the petition and
disqualified Amora from running for Mayor of Candijay, Bohol. Posthaste, Amora filed
a Motion for Reconsideration before the COMELEC En Banc.

-Meanwhile, on May 10, 2010, national and local elections were held. Amora won.
Subsequently, the Muncipal Board of Canvassers of Candijay, Bohol, proclaimed
Amora as the winner for the position of Municipal Mayor of Candijay, Bohol.

-Eventually, the COMELEC En Banc affirmed the ruling of the COMELEC Second
Division disqualifying Amora.

-Contention of the Comelec: Amora’s COC is rendered invalid when he only presented
his CTC to the notary public. It is apparent that a CTC, which bears no photograph, is
no longer a valid form of identification for purposes of Notarization of Legal
Documents. The COV, therefore, was not properly verified.

Issue:
Whether or not Amora should be disqualified under the circumstances.

Ruling:
No.

In this case, it was grave abuse of discretion to uphold Olandria’s claim that an
improperly sworn COC is equivalent to possession of a ground for disqualification. Not
by any stretch of the imagination can we infer this as an additional ground for
disqualification.

It is quite obvious that the Olandria petition is not based on any of the grounds for
disqualification as enumerated in the Monibus Election Code. Nowhere therein does
it specify that a defective notarization is a ground for the disqualification of a
candidate. Yet, the COMELEC would uphold that petition upon the outlandish claim
that it is a petition to disqualify a candidate “for lack of qualifications or possessing
some grounds for disqualification.”

The proper characterization of a petition as one for disqualification under the pertinent
provisions of laws cannot be made dependent on the designation, correctly or
incorrectly, of a petitioner. The absurd interpretation of Olandria, respondent herein,
is not controlling; the COMELEC should have dismissed his petition outright.

A petition for disqualification relates to the declaration of a candidate as


ineligible or lacking in quality or accomplishment fit for the position of mayor.
The distinction between a petition for disqualification and the formal requirement in
Section 73 of the OEC that a COC be under oath is not simply a question of semantics
as the statutes list the grounds for the disqualification of a candidate.

Another red flag for the COMELEC to dismiss Olandria’s petition is the fact that
Amora claims to personally know the notary public, Atty. Granada, before whom
his COC was sworn. Competent evidence of identity is not required in cases
where the affiant is personally known to the Notary Public, which is the case
herein. x x x x

Technicalities and procedural niceties in election cases should not be made to stand in
the way of the true will of the electorate. Laws governing election contests must be
liberally construed to the end that the will of the people in the choice of public
officials may not be defeated by mere technical objections.

Page 285 of 323


Election contests involve public interest, and technicalities and procedural barriers
must yield if they constitute an obstacle to the determination of the true will of the
electorate in the choice of their elective officials. The Court frowns upon any
interpretation of the law that would hinder in any way not only the free and intelligent
casting of the votes in an election but also the correct ascertainment of the results.

Our ruling herein does not do away with the formal requirement that a COC be sworn.
In fact, we emphasize that the filing of a COC is mandatory and must comply with
the requirements set forth by law. In this case, however, contrary to the
declarations of the COMELEC, Amora complied with the requirement of a sworn
COC. He readily explained that he and Atty. Granada personally knew each other; they
were not just colleagues at the League of Municipal Mayors, Bohol Chapter, but they
consider each other as distant relatives. Thus, the alleged defect in the oath was
not proven by Olandria since the presentation of a CTC turned out to be
sufficient in this instance. On the whole, the COMELEC should not have brushed
aside the affidavit of Atty. Granada and remained inflexible in the face of Amora’s
victory and proclamation as Mayor of Candijay, Bohol.

*Cerafica v. Comelec, G.R. No. 205136, December 2, 2014.


(Comelec; Cancellation of COCs; Substitution; Due Process)

Facts:
-On 1 October 2012, Kimberly filed her COC for Councilor, City of Taguig for the 2013
Elections. Her COC stated that she will be twenty (20) years of age on the day of the
elections, in contravention of the requirement that one must be at least twenty-three
(23) years of age on the day of the elections as set out in Sec. 9 (c) of Republic Act
(R.A.) No. 8487 (Charter of the City of Taguig). As such, Kimberly was summoned to a
clarificatory hearing due to the age qualification.

-Instead of attending the hearing, Kimberly opted to file a sworn Statement of


Withdrawal of COC on 17 December 2012. Simultaneously, Olivia filed her own COC
as a substitute of Kimberly. Owing to these events, the clarificatory hearing no longer
pushed through.

-In a Memorandum, Director Esmeralda Amora-Ladra (Director Amora-Ladra) of the


Comelec Law Department recommended the cancellation of Kimberly’s COC, and
consequently, the denial of the substitution of Kimberly by Olivia. Relying on
Comelec Resolution No. 9551, Director Amora-Ladra opined that it is as if no COC
was filed by Kimberly; thus, she cannot be substituted.

-In a Special En Banc Meeting of the Comelec, the Comelec adopted the
recommendation of Director Amora-Ladra, cancelled Kimberly’s COC, and denied the
substitution of Kimberly by Olivia as an effect of the cancellation of Kimberly’s COC.

-Olivia then went to the SC via Special Civil Action for Certiorari under Rule 64.

Issues:
(1) Whether or not the Comelec committed grave abuse of discretion in declaring that
Kimberly, being under age, could not be considered to have filed a valid COC and,
thus, could not be validly substituted by Olivia;

(2) Whether or not Kimberly may be validly substituted by Olivia.

Ruling:
(1) Yes.

Firstly, subject to its authority over nuisance candidates and its power to deny due
course to or cancel COCs under Sec. 78, Batas Pambansa (B.P.) Blg. 881, the
Comelec has the ministerial duty to receive and acknowledge receipt of COCs.

Page 286 of 323


In Cipriano v. Comelec, we ruled that the Comelec has no discretion to give or not
to give due course to COCs. We emphasized that the duty of the Comelec to give
due course to COCs filed in due form is ministerial in character, and that while
the Comelec may look into patent defects in the COCs, it may not go into matters not
appearing on their face. The question of eligibility or ineligibility of a candidate is
thus beyond the usual and proper cognizance of the Comelec. x x x x

The COMELEC may not, by itself, without the proper proceedings, deny due
course to or cancel a certificate of candidacy filed in due form. In Sanchez vs.
Del Rosario, the Court ruled that the question of eligibility or ineligibility of a candidate
for non-age is beyond the usual and proper cognizance of the COMELEC. x x x x

Under Section 3, Rule 23 of the 1993 COMELEC Rules of Procedure, a petition for
the denial or cancellation of a certificate of candidacy must be heard
summarily after due notice. It is thus clear that cancellation proceedings involve
the exercise of the quasi-judicial functions of the COMELEC which the COMELEC
in division should first decide. x x x x

The determination of whether a candidate is eligible for the position he is


seeking involves a determination of fact where parties must be allowed to
adduce evidence in support of their contentions. We thus caution the Comelec
against its practice of impetuous cancellation of COCs via minute resolutions adopting
the recommendations of its Law Department when the situation properly calls for
the case's referral to a Division for summary hearing.

(2) Yes.

First, there was a valid withdrawal of Kimberly’s COC after the last day for the filing of
COCs; second, Olivia belongs to and is certified to by the same political party to which
Kimberly belongs; and third, Olivia filed her COC not later than mid-day of election
day. x x x x

It would have been different if there was a petition to deny due course to or cancel
Kimberly’s certificate of candidacy. For if the COMELEC cancelled Kimberly’s
certificate of candidacy after the proper proceedings, then she is no candidate at all
and there can be no substitution of a person whose certificate of candidacy has been
cancelled and denied due course. However, Kimberly’s certificate of candidacy was
never cancelled or denied due course by the COMELEC.

Moreover, Kimberly already withdrew her certificate of candidacy before the


COMELEC could declare that she was not a valid candidate. Therefore, unless
Kimberly’s certificate of candidacy was denied due course or cancelled in accordance
with Section 78 of the Election Code, her certificate of candidacy was valid and he
may be validly substituted by Olivia.

*Talaga v. Comelec, G.R. No. 196804, October 9, 2012. [BERSAMIN CASE!]


(Substitution; COC; Succession)

Facts:
-On November 26, 2009 and December 1, 2009, Ramon Talaga (Ramon) and Philip M.
Castillo (Castillo) respectively filed their certificates of candidacy (CoCs) for the
position of Mayor of Lucena City to be contested in the scheduled May 10, 2010
national and local elections.

-Ramon, the official candidate of the Lakas-Kampi-CMD, declared in his CoC that he
was eligible for the office he was seeking to be elected to.

-Four days later, or on December 5, 2009, Castillo filed with the COMELEC a petition
denominated as In the Matter of the Petition to Deny Due Course to or Cancel
Certificate of Candidacy of Ramon Y. Talaga, Jr. as Mayor for Having Already Served
Three (3) Consecutive Terms as a City Mayor of Lucena.

Page 287 of 323


-After due proceedings, the COMELEC First Division issued a resolution
disqualifying Ramon.

-In view of Ramon’s disqualification, Barbara Ruby filed her own CoC for Mayor of
Lucena City in substitution of Ramon, attaching thereto the Certificate of Nomination
and Acceptance (CONA) issued by Lakas-Kampi-CMD, the party that had nominated
Ramon.

-On election day on May 10, 2010, the name of Ramon remained printed on the
ballots but the votes cast in his favor were counted in favor of Barbara Ruby as his
substitute candidate, resulting in Barbara Ruby being the winner with 44,099 votes
as against Castillo’s 39,615 votes.

-It was only on May 13, 2010 when the COMELEC En Banc, upon the
recommendation of its Law Department, gave due course to Barbara Ruby’s CoC and
CONA through Resolution No. 8917, thereby including her in the certified list of
candidates. Consequently, the CBOC proclaimed Barbara Ruby as the newly-elected
Mayor of Lucena City.

-On May 20, 2010, Castillo filed a Petition for Annulment of Proclamation with the
COMELEC, docketed as SPC 10-024. He alleged that Barbara Ruby could not
substitute Ramon because his CoC had been cancelled and denied due course.

-After due proceedings, the COMELEC Second Division dismissed Castillo’s petition.
Upon appeal, the COMELEC En Banc reversed the ruling of the COMELEC Second
Division.

Issues:
(1) Whether or not the substitution by Barbara Ruby as candidate for the position of
Mayor of Lucena City in lieu of Ramon was valid;
(2) Whether or not Castillo, as the second placer, must assume the position of Mayor
in view of Ramon’s disqualification and Ruby’s invalid substitution.

Ruling:
No. The substitution was invalid.

A. Existence of a valid CoC is a condition sine qua non for a valid substitution.

The filing of a CoC within the period provided by law is a mandatory requirement for
any person to be considered a candidate in a national or local election. x x x x
Considering that a cancelled CoC does not give rise to a valid candidacy, there
can be no valid substitution of the candidate under Section 77 of the Omnibus
Election Code. It should be clear, too, that a candidate who does not file a valid
CoC may not be validly substituted, because a person without a valid CoC is not
considered a candidate in much the same way as any person who has not filed a CoC
is not at all a candidate.

Likewise, a candidate who has not withdrawn his CoC in accordance with Section
73 of the Omnibus Election Code may not be substituted. A withdrawal of
candidacy can only give effect to a substitution if the substitute candidate submits
prior to the election a sworn CoC as required by Section 73 of the Omnibus Election
Code.

B. Declaration of Ramon’s disqualification rendered his CoC invalid; hence, he


was not a valid candidate to be properly substituted.

To accord with the constitutional and statutory proscriptions, Ramon was absolutely
precluded from asserting an eligibility to run as Mayor of Lucena City for the
fourth consecutive term. Resultantly, his CoC was invalid and ineffectual ab initio
for containing the incurable defect consisting in his false declaration of his

Page 288 of 323


eligibility to run. The invalidity and inefficacy of his CoC made his situation even
worse than that of a nuisance candidate because the nuisance candidate may remain
eligible despite cancellation of his CoC or despite the denial of due course to the CoC
pursuant to Section 69 of the Omnibus Election Code.

We stress that a non-candidate like Ramon had no right to pass on to his


substitute. As Miranda v. Abaya aptly put it:

Even on the most basic and fundamental principles, it is readily understood that the
concept of a substitute presupposes the existence of the person to be substituted, for
how can a person take the place of somebody who does not exist or who never was.
The Court has no other choice but to rule that in all the instances enumerated in
Section 77 of the Omnibus Election Code, the existence of a valid certificate of
candidacy seasonably filed is a requisite sine qua non.

All told, a disqualified candidate may only be substituted if he had a valid


certificate of candidacy in the first place because, if the disqualified candidate
did not have a valid and seasonably filed certificate of candidacy, he is and was
not a candidate at all. If a person was not a candidate, he cannot be substituted
under Section 77 of the Code. Besides, if we were to allow the so-called "substitute" to
file a "new" and "original" certificate of candidacy beyond the period for the filing
thereof, it would be a crystalline case of unequal protection of the law, an act abhorred
by our Constitution.

(2) No. It should be the Vice Mayor.

We cannot agree with Castillo’s assertion that with Ramon’s disqualification becoming
final prior to the May 10, 2010 elections, the ruling in Cayat was applicable in his
favor. Barbara Ruby’s filing of her CoC in substitution of Ramon significantly
differentiated this case from the factual circumstances obtaining in Cayat. Rev.
Fr. Nardo B. Cayat, the petitioner in Cayat, was disqualified on April 17, 2004, and his
disqualification became final before the May 10, 2004 elections. Considering that no
substitution of Cayat was made, Thomas R. Palileng, Sr., his rival, remained the
only candidate for the mayoralty post in Buguias, Benguet. In contrast, after
Barbara Ruby substituted Ramon, the May 10, 2010 elections proceeded with her
being regarded by the electorate of Lucena City as a bona fide candidate. To the
electorate, Barbara Ruby became a contender for the same position vied for by
Castillo, such that she stood on the same footing as Castillo. Such standing as a
candidate negated Castillo’s claim of being the candidate who obtained the
highest number of votes, and of being consequently entitled to assume the office
of Mayor.

Indeed, Castillo could not assume the office for he was only a second placer. Labo, Jr.
should be applied. There, the Court emphasized that the candidate obtaining the
second highest number of votes for the contested office could not assume the office
despite the disqualification of the first placer because the second placer was “not
the choice of the sovereign will.” Surely, the Court explained, a minority or
defeated candidate could not be deemed elected to the office. There was to be no
question that the second placer lost in the election, was repudiated by the electorate,
and could not assume the vacated position. No law imposed upon and compelled the
people of Lucena City to accept a loser to be their political leader or their
representative.

The only time that a second placer is allowed to take the place of a disqualified
winning candidate is when two requisites concur, namely: (a) the candidate who
obtained the highest number of votes is disqualified; and (b) the electorate was
fully aware in fact and in law of that candidate’s disqualification as to bring such
awareness within the realm of notoriety but the electorate still cast the plurality
of the votes in favor of the ineligible candidate. Under this sole exception, the
electorate may be said to have waived the validity and efficacy of their votes by
notoriously misapplying their franchise or throwing away their votes, in which case

Page 289 of 323


the eligible candidate with the second highest number of votes may be deemed elected.
But the exception did not apply in favor of Castillo simply because the second
element was absent. The electorate of Lucena City were not the least aware of
the fact of Barbara Ruby’s ineligibility as the substitute. In fact, the COMELEC En
Banc issued the Resolution finding her substitution invalid only on May 20, 2011, or a
full year after the decisions.

*Tagolino v. HRET, G.R. No. 202202, March 19, 2013. [SEVERAL PRINCIPLES
HERE!!!]
(Substitution; Disqualification; Residency; Notarization; Quo Warranto; Succession)

***LOCAL GOVERNMENT***

*Villagracia v. Fifth Shari’a District Court, G.R. No. 188832, April 23, 2014.
(Jurisdiction; Muslims)
Facts:
-Roldan filed an action to recover the possession of the parcel of land with
respondent Fifth Shari’a District Court against Vivencio.

-In his petition, Roldan alleged that he is a Filipino Muslim; that he is the registered
owner of the lot covered by Transfer Certificate of Title No. 15633; and that Vivencio
occupied his property, depriving him of the right to use, possess, and enjoy it. He
prayed that respondent Fifth Shari’a District Court order Vivencio to vacate his
property.

-Respondent court took cognizance of the case and caused service of summons on
Vivencio. However, despite service of summons, Vivencio failed to file his answer.

-Roldan then moved that he be allowed to present evidence ex parte, which respondent
Fifth Shari’a District Court granted.

-In its decision, respondent Fifth Shari’a District Court ruled that Roldan, as
registered owner, had the better right to possess the parcel of land. It ordered Vivencio
to vacate the property, turn it over to Roldan, and pay ₱10,000.00 as moderate
damages and ₱5,000.00 as attorney’s fees.

-Vivencio filed a petition for relief from judgment with prayer for issuance of writ of
preliminary injunction, stating that he is a Christian, and therefore, respondent Fifth
Shari’a District Court had no jurisdiction to take cognizance of Roldan’s action for
recovery of possession of a parcel of land.

-Respondent Fifth Shari’a District Court denied Vivencio’s petition for relief from
judgment for lack of merit.

-Vivencio filed with the CS a petition for certiorari with prayer for issuance of
temporary restraining order.

-Roldan’s contention: Vivencio waived his right to defend himself when he failed to file
any pleading despite having been duly served with summons and notices of Roldan’s
motion to present evidence ex parte, respondent Fifth Shari’a District Court’s decision
and the writ of execution.

Issues:
(1) Whether or not a Shari’a District Court may validly hear, try, and decide a real
action where one of the parties is a non-Muslim;

(2) Whether or not a Shari’a District Court may validly hear, try, and decide a real
action filed by a Muslim against a non-Muslim if the non-Muslim defendant was
served with summons.

Page 290 of 323


Ruling:
(1) No.

The law conferring the jurisdiction of Shari’a District Courts is the Code of the Muslim
Personal Laws of the Philippines. Under Article 143 of the Muslim Code, Shari’a
District Courts have concurrent original jurisdiction with “existing civil courts” over
real actions not arising from customary contracts wherein the parties involved are
Muslims:

“ART 143. Original jurisdiction. – x x x x

(2) Concurrently with existing civil courts, the Shari’a District Court shall have original jurisdiction
over:
xxxx
(b) All other personal and real actions not mentioned in paragraph 1(d)42 wherein the parties involved
are Muslims except those for forcible entry and unlawful detainer, which shall fall under the exclusive
original jurisdiction of the Municipal Circuit Court; and x x x x”

When ownership is acquired over a particular property, the owner has the right to
possess and enjoy it. If the owner is dispossessed of his or her property, he or she has
a right of action to recover its possession from the dispossessor. When the property
involved is real, such as land, the action to recover it is a real action; otherwise, the
action is a personal action. In such actions, the parties involved must be Muslims
for Shari’a District Courts to validly take cognizance of them.

In this case, the allegations in Roldan’s petition for recovery of possession did not state
that Vivencio is a Muslim. When Vivencio stated in his petition for relief from
judgment that he is not a Muslim, Roldan did not dispute this claim.

When it became apparent that Vivencio is not a Muslim, respondent Fifth Shari’a
District Court should have motu proprio dismissed the case.

Respondent Fifth Shari’a District Court had no authority under the law to decide
Roldan’s action because not all of the parties involved in the action are Muslims.
Thus, it had no jurisdiction over Roldan’s action for recovery of possession. All
its proceedings in SDC Special Proceedings Case No. 07-200 are void. x x x x

Under Article 143 of the Muslim Code, the jurisdiction of Shari’a District Courts over
real actions not arising from customary contracts is concurrent with that of existing
civil courts. However, this concurrent jurisdiction over real actions “is applicable solely
when both parties are Muslims” as this court ruled in Tomawis v. Hon. Balindong.
When one of the parties is not a Muslim, the action must be filed before the
regular courts.

(2) No.

Fifth Shari’a District Court has no jurisdiction over the subject matter of the action,
with Vivencio not being a Muslim. Therefore, all the proceedings before respondent
Shari’a District Court, including the service of summons on Vivencio, are void.

*Miranda v. Aguirre, G.R. No. 133064, September 16, 1999.


(Plebiscite; Conversion: From independent component city to component city)

-On May 5, 1994, Republic Act No. 7720 which converted the municipality of
Santiago, Isabela into an independent component city was signed into law.

-On July 4, 1994, the people of Santiago ratified R.A. No. 7720 in a plebiscite.

Page 291 of 323


-On February 14, 1998, R.A. No. 8528 was enacted. It amended R.A. No. 7720. Among
others, it changed the status of Santiago from an independent component city into a
component city.

-Petitioners filed a petition challenging the validity of R.A. No. 8528 on the ground that
it has no provision submitting the law for ratification by the people of Santiago City in
a proper plebiscite.

-Contention of respondents: R.A. No. 8528 merely reclassified Santiago City from an
independent component city into a component city. It did not involve any “creation,
division, merger, abolition, or substantial alteration of boundaries of local government
units,” and hence, a plebiscite of the people of Santiago is unnecessary.

Issue:
Whether or not R.A. No. 8528 is valid despite its lack of provision regarding the
conduct of plebiscite to ratify the reclassification of Santiago City.

Ruling:
No. It is unconstitutional.

Section 10, Article X of the 1987 Constitution provides:

“No province, city, municipality, or barangay may be created, or divided, merged, abolished, or its
boundary substantially altered except in accordance with the criteria established in the local government
code and subject to approval by a majority of the votes cast in a plebiscite in the political units directly
affected.”

This constitutional requirement is reiterrated in Section 10, Chapter 2 of the Local


Government Code (R.A. No. 7160), thus:

“Sec. 10. No province, city, municipality, or barangay may be created, divided, merged, abolished,
or its boundary substantially altered except in accordance with the criteria established in the local
government code and subject to approval by a majority of the votes cast in a plebiscite in the political
units directly affected.”

The power to create, divide, merge, abolish or substantially alter boundaries of local
government units belongs to Congress. This power is part of the larger power to enact
laws which the Constitution vested in Congress. The exercise of the power must be in
accord with the mandate of the Constitution.

In the case at bar, the issue is whether the downgrading of Santiago City from an
independent component city to a mere component city requires the approval of the
people of Santiago City in a plebiscite. The resolution of the issue depends on
whether or not the downgrading falls within the meaning of creation, division,
merger, abolition or substantial alteration of boundaries of municipalities per
Section 10, Article X of the Constitution. A close analysis of the said
constitutional provision will reveal that the creation, division, merger, abolition
or substantial alteration of boundaries of local government units involve a
common denominator — material change in the political and economic rights of
the local government units directly affected as well as the people therein. It is
precisely for this reason that the Constitution requires the approval of the people “in
the political units directly affected.” It is not difficult to appreciate the rationale of this
constitutional requirement. The 1987 Constitution, more than any of our previous
Constitutions, gave more reality to the sovereignty of our people for it was borne out of
the people power in the 1986 EDSA revolution. Its Section 10, Article X addressed
the undesirable practice in the past whereby local government units were
created, abolished, merged or divided on the basis of the vagaries of politics and
not of the welfare of the people. Thus, the consent of the people of the local
government unit directly affected was required to serve as a checking mechanism to
any exercise of legislative power creating, dividing, abolishing, merging or altering the
boundaries of local government units. It is one instance where the people in their
sovereign capacity decide on a matter that affects them — direct democracy of the

Page 292 of 323


people as opposed to democracy thru people's representatives. This plebiscite
requirement is also in accord with the philosophy of the Constitution granting more
autonomy to local government units.

The changes that will result from the downgrading of the city of Santiago from
an independent component city to a component city are many and cannot be
characterized as insubstantial. For one, the independence of the city as a
political unit will be diminished. The city mayor will be placed under the
administrative supervision of the provincial governor. The resolutions and
ordinances of the city council of Santiago will have to be reviewed by the
Provincial Board of Isabela. Taxes that will be collected by the city will now have
to be shared with the province. x x x x

It is markworthy that when R.A. No. 7720 upgraded the status of Santiago City from a
municipality to an independent component city, it required the approval of its people
thru a plebiscite called for the purpose. There is neither rhyme nor reason why this
plebiscite should not be called to determine the will of the people of Santiago
City when R.A. No. 8528 downgrades the status of their city. Indeed, there is
more reason to consult the people when a law substantially diminishes their
right.

Rule II, Article 6, paragraph (f) (1) of the Implementing Rules and Regulations of the
Local Government Code is in accord with the Constitution when it provides that:

(f) Plebiscite — (1) no creation, conversion, division, merger, abolition, or substantial alteration of
boundaries of LGUS shall take effect unless approved by a majority of the votes cast in a plebiscite called
for the purpose in the LGU or LGUs affected. The plebiscite shall be conducted by the Commission on
Elections (COMELEC) within one hundred twenty (120) days from the effectivity of the law or ordinance
prescribing such action, unless said law or ordinance fixes another date. x x x x”

The rules cover all conversions, whether upward or downward in character, so


long as they result in a material change in the local government unit directly
affected, especially a change in the political and economic rights of its people.

*Tan v. Comelec, G.R. No. 73155, July 11, 1986.


(Plebiscite; Creation of Province)

-Congress enacted Batas Pambansa Blg. 885, “An Act Creating a New Province in the
Island of Negros to be known as the Province of Negros del Norte.”

-Contained therein is a provision regarding conduct of plebiscite involving the


inhabitants of Negros del Norte only, to the exclusion of the voters from the rest of
the province of Negros Occidental.

-Petitioners—who are residents of the Province of Negros Occidental, in the various


cities and municipalities therein—went to the Comelec via prohibition, seeking to stop
the conduct of the said plebiscite.

Contentions of respondents: (1) The remaining cities and municipalities of the Province
of Negros Occidental not included in the area of the new Province of Negros del Norte,
do not fall within the meaning and scope of the term “unit or units affected,” as
referred to in Section 3 of Art. XI of our Constitution; (2) The petition should be
dismissed because the motive and wisdom of B.P. Blg. 885 is a political question that
cannot be challenged by petitioners.

Issues:
(1) Whether or not the term “unit or units affected” includes the voters of the mother
province (Negros Occidental) and not only the voters of the newly created province of
Negros del Norte, for purposes of the plebiscite.
(2) Whether or not the petition should be dismissed because the wisdom of B.P. Blg.
885 cannot be challenged by petitioners.

Page 293 of 323


Ruling:
Yes.

Article XI, Section 3 of the Constitution provides:

“SEC. 3. No province, city, municipality or barrio may be created, divided, merged abolished, or its
boundary substantially altered, except in accordance with the criteria established in the local government
code, and subject to the approval by a majority of the votes in a plebiscite in the unit or units
affected.”

It can be plainly seen that the aforecited constitutional provision makes it imperative
that there be first obtained "the approval of a majority of votes in the plebiscite in the
unit or units affected" whenever a province is created, divided or merged and there is
substantial alteration of the boundaries. It is thus inescapable to conclude that the
boundaries of the existing province of Negros Occidental would necessarily be
substantially altered by the division of its existing boundaries in order that there
can be created the proposed new province of Negros del Norte. Plain and simple
logic will demonstrate than that two political units would be affected. The first
would be the parent province of Negros Occidental because its boundaries would
be substantially altered. The other affected entity would be composed of those in
the area subtracted from the mother province to constitute the proposed
province of Negros del Norte.

We find no way to reconcile the holding of a plebiscite that should conform to said
constitutional requirement but eliminates the participation of either of these two
component political units. No amount of rhetorical flourishes can justify exclusion
of the parent province in the plebiscite because of an alleged intent on the part of
the authors and implementors of the challenged statute to carry out what is claimed to
be a mandate to guarantee and promote autonomy of local government units. The
alleged good intentions cannot prevail and overrule the cardinal precept that what our
Constitution categorically directs to be done or imposes as a requirement must first be
observed, respected and complied with. No one should be allowed to pay homage to a
supposed fundamental policy intended to guarantee and promote autonomy of local
government units but at the same time transgress, ignore and disregard what the
Constitution commands in Article XI Section 3 thereof. Respondents would be no
different from one who hurries to pray at the temple but then spits at the Idol therein.

(2) No.

We find no merit in the submission of the respondents that the petition should be
dismissed because the motive and wisdom in enacting the law may not be challenged
by petitioners. The principal point raised by the petitioners is not the wisdom and
motive in enacting the law but the infringement of the Constitution which is a
proper subject of judicial inquiry.

*Umali v. Comelec, G.R. No. 203974, April 22, 2014.


(Plebiscite; Conversion: From component city to highly urbanized city)

-On July 11, 2011, the Sangguniang Panglungsod of Cabanatuan City passed
Resolution No. 183-2011, requesting the President to declare the conversion of
Cabanatuan City from a component city of the province of Nueva Ecija into a highly
urbanized city (HUC).

-Acceding to the request, the President issued Presidential Proclamation No. 418,
Series of 2012, proclaiming the City of Cabanatuan as an HUC subject to “ratification
in a plebiscite by the qualified voters therein, as provided for in Section 453 of the
Local Government Code of 1991.”

-Respondent COMELEC, acting on the proclamation, issued Minute Resolution No.


12-0797 which provides that, “for purposes of the plebiscite for the conversion of
Cabanatuan City from component city to highly-urbanized city, only those registered
residents of Cabanatuan City should participate in the said plebiscite.”

Page 294 of 323


-Eventually, petitioner went to the SC via petition for certiorari, questioning the
validity of said Comelec’s resolution.

-Comelec’s contention: Cabanatuan City is merely being converted from a component


city into an HUC and that the political unit directly affected by the conversion will only
be the city itself. No political unit will be created, merged with another, or will be
removed from another LGU, and no boundaries will be altered. Hence, the
inclusion of the entire Nueva Ecija province is not necessary for purposes of plebiscite.

-The Comelec cites Sec. 453 of the Local Government Code to support its claim, to wit:

“Section 453. Duty to Declare Highly Urbanized Status. – It shall be the duty of the President to declare a
city as highly urbanized within thirty (30) days after it shall have met the minimum requirements
prescribed in the immediately preceding Section, upon proper application therefor and ratification in a
plebiscite by the qualified voters therein.”

Issue:
Whether or not the plebiscite should include the participation of the registered voters
of the mother province of Nueva Ecija.

Ruling:
Yes.

While conversion to an HUC is not explicitly provided in Sec. 10, Art. X of the
Constitution, we nevertheless observe that the conversion of a component city into an
HUC is substantial alteration of boundaries.

As the phrase implies, “substantial alteration of boundaries” involves and


necessarily entails a change in the geographical configuration of a local
government unit or units. However, the phrase “boundaries” should not be
limited to the mere physical one, referring to the metes and bounds of the LGU,
but also to its political boundaries. It also connotes a modification of the
demarcation lines between political subdivisions, where the LGU’s exercise of
corporate power ends and that of the other begins. And as a qualifier, the alteration
must be “substantial” for it to be within the ambit of the constitutional provision.

Pertinent is Art. 12(c) of the LGC’s Implementing Rules and Regulations, which reads:

“Art. 12. Conversion of a Component City into a Highly Urbanized City. –


xxxx
(c) Effect of Conversion – The conversion of a component city into a highly-urbanized city shall make it
independent of the province where it is geographically located.”

Verily, the upward conversion of a component city, in this case Cabanatuan City, into
an HUC will come at a steep price. It can be gleaned from the above-cited rule that the
province will inevitably suffer a corresponding decrease in territory brought
about by Cabanatuan City’s gain of independence. With the city’s newfound
autonomy, it will be free from the oversight powers of the province, which, in
effect, reduces the territorial jurisdiction of the latter. What once formed part of
Nueva Ecija will no longer be subject to supervision by the province. In more concrete
terms, Nueva Ecija stands to lose 282.75 sq. km. of its territorial jurisdiction
with Cabanatuan City’s severance from its mother province. This is equivalent to
carving out almost 5% of Nueva Ecija’s 5,751.3 sq. km. area. This sufficiently satisfies
the requirement that the alteration be “substantial.”

Taking this decrease in territory and population in connection with the above formula,
it is conceded that Nueva Ecija will indeed suffer a reduction in IRA given the
decrease of its multipliers’ values.

Page 295 of 323


Likewise, once converted, the taxes imposed by the HUC will accrue to itself. Prior to
this, the province enjoys the prerogative to impose and collect taxes such as those on
sand, gravel and other quarry resources, professional taxes, and amusement taxes
over the component city. While, it may be argued that this is not a derogation of the
province’s taxing power because it is in no way deprived of its right to collect the
mentioned taxes from the rest of its territory, the conversion will still reduce the
province’s taxing jurisdiction, and corollary to this, it will experience a
corresponding decrease in shares in local tax collections. This reduction in both
taxing jurisdiction and shares poses a material and substantial change to the
province’s economic rights, warranting its participation in the plebiscite. x x x x

Needless to stress, the alteration of boundaries would necessarily follow Cabanatuan


City’s conversion in the same way that creations, divisions, mergers, and abolitions
generally cannot take place without entailing the alteration. The enumerated acts,
after all, are not mutually exclusive, and more often than not, a combination of these
acts attends the reconfiguration of LGUs.

In light of the foregoing disquisitions, the Court rules that conversion to an HUC is
substantial alternation of boundaries governed by Sec. 10, Art. X and resultantly,
said provision applies, governs and prevails over Sec. 453 of the LGC. x x x x

In view of these changes in the economic and political rights of the province of
Nueva Ecija and its residents, the entire province certainly stands to be directly
affected by the conversion of Cabanatuan City into an HUC. Following the
doctrines in Tan and Padilla, all the qualified registered voters of Nueva Ecija should
then be allowed to participate in the plebiscite called for that purpose.

*Alta Vista Golf and Country Club v. Cebu City, G.R. No. 180235, January 20, 2016.
(Local Government Code; Ejusdem Generis; Statutory Construction; Taxation Law:
Amusement Tax)

Facts:
-Petitioner is a non-stock and non-profit corporation operating a golf course in Cebu
City.

-On June 21, 1993, the Sangguniang Panlungsod of Cebu City enacted City Tax
Ordinance No. LXIX, Section 42 of which imposed an amusement tax on golf
courses and polo grounds at the rate of twenty percent (20%).

-In an Assessment Sheet dated August 6, 1998, prepared by the Cebu City Assessor,
petitioner was originally assessed deficiency business taxes, fees, and other charges
for the year 1998, in the total amount of P3,820,095.68, which included amusement
tax on its golf course amounting to P2,612,961.24 based on gross receipts of
P13,064,806.20.

-Through the succeeding years, respondent Cebu City repeatedly attempted to collect
from petitioner its deficiency business taxes, fees, and charges for 1998, a substantial
portion of which consisted of the amusement tax on the golf course.

-Through a letter dated October 11, 2005, respondent Camarillo sought to collect once
more from petitioner deficiency business taxes, fees, and charges for the year 1998,
totaling P2,981,441.52.

-Petitioner, through counsel, wrote respondent Camarillo a letter11 dated October 17,
2005 still disputing the amusement tax assessment on its golf course for 1998 for
being illegal.

-On January 12, 2006, petitioner was served with a Closure Order dated December
28, 2005 issued by respondent City Mayor Osmeña.

Page 296 of 323


-According to the Closure Order, petitioner committed blatant violations of the laws
and Cebu City Ordinances including: Nonpayment of deficiency on Amusement Tax
and the penalties relative therewith totaling Php2,953,586.86.

-The said closure order prompted petitioner to file with the RTC a Petition for
Injunction, Prohibition, Mandamus, Declaration of Nullity of Closure Order,
Declaration of Nullity of Assessment, and Declaration of Nullity of Section 42 of Cebu
City Tax Ordinance, with Prayer for Temporary Restraining Order and Writ of
Preliminary Injunction, against respondents.

-After due proceedings, the RTC denied the prayer of petitioner for issuance of a
preliminary TRO, affirming respondent Mayor Osmeña's authority to issue or grant
business licenses and permits pursuant to the police power inherent in his office.

-On March 20, 2006, petitioner paid under protest to respondent Cebu City, through
respondent Camarillo, the assessed amusement tax, plus penalties, interest, and
surcharges, in the total amount of P2,750,249.17.

-Eventually, the RTC dismissed the injunction case filed by petitioner.

-Petitioner then went to the SC via Rule 45 on pure questions of law.

Issue:
Whether or not the subject ordinance is valid.

Ruling:
No.

Section 42 of the Revised Omnibus Tax Ordinance, as amended, imposing amusement


tax on golf courses is null and void as it is beyond the authority of respondent Cebu
City to enact under the Local Government Code.

The Local Government Code authorizes the imposition by local government units of
amusement tax under Section 140, which provides:

“Sec. 140. Amusement Tax. - (a) The province may levy an amusement tax to be collected from the
proprietors, lessees, or operators of theaters, cinemas, concert halls, circuses, boxing stadia, and
other places of amusement at a rate of not more than thirty percent (30%) of the gross receipts from
admission fees.

(b) In the case of theaters or cinemas, the tax shall first be deducted and withheld by their proprietors,
lessees, or operators and paid to the provincial treasurer before the gross receipts are divided between
said proprietors, lessees, or operators and the distributors of the cinematographic films. x x x x”

“Amusement places,” as defined in Section 13l(c) of the Local Government Code,


“include theaters, cinemas, concert halls, circuses and other places of
amusement where one seeks admission to entertain oneself by seeing or viewing
the show or performance.”

The pronouncements of the Court in Pelizloy Realty Corporation v. The Province of


Benguet are of particular significance to this case. The Court, in Pelizloy Realty,
declared null and void the second paragraph of Article X, Section 59 of the Benguet
Provincial Code, in so far as it imposes amusement taxes on admission fees to resorts,
swimming pools, bath houses, hot springs, and tourist spots. Applying the principle of
ejusdem generis, as well as the ruling in the PBA case, the Court expounded on the
authority of local government units to impose amusement tax under Section 140,
in relation to Section 131(c), of the Local Government Code, as follows:

Under the principle of ejusdem generis, "where a general word or phrase follows
an enumeration of particular and specific words of the same class or where the
latter follow the former, the general word or phrase is to be construed to include,

Page 297 of 323


or to be restricted to persons, things or cases akin to, resembling, or of the same
kind or class as those specifically mentioned." x x x x

Indeed, theaters, cinemas, concert halls, circuses, and boxing stadia are bound by a
common typifying characteristic in that they are all venues primarily for the staging of
spectacles or the holding of public shows, exhibitions, performances, and other events
meant to be viewed by an audience. Accordingly, 'other places of amusement' must
be interpreted in light of the typifying characteristic of being venues "where one
seeks admission to entertain oneself by seeing or viewing the show or
performances" or being venues primarily used to stage spectacles or hold public
shows, exhibitions, performances, and other events meant to be viewed by an
audience. x x x x

In light of Pelizloy Realty, a golf course cannot be considered a place of


amusement. As petitioner asserted, people do not enter a golf course to see or
view a show or performance. Petitioner also, as proprietor or operator of the golf
course, does not actively display, stage, or present a show or performance. People go
to a golf course to engage themselves in a physical sport activity, i.e., to play golf;
the same reason why people go to a gym or court to play badminton or tennis or to a
shooting range for target practice, yet there is no showing herein that such gym, court,
or shooting range is similarly considered an amusement place subject to amusement
tax. There is no basis for singling out golf courses for amusement tax purposes
from other places where people go to play sports. This is in contravention of one of
the fundamental principles of local taxation: that the “taxation shall be uniform in
each local government unit.” Uniformity of taxation, like the kindred concept of equal
protection, requires that all subjects or objects of taxation, similarly situated, are to be
treated alike both in privileges and liabilities.”

[NOTE: Here, the closure order was set aside by the SC.]

*Hon. Cayabyab v. Dimson, G.R. No. 223862, July 10, 2017.


(Closure Order; Mayor’s Permit; Business Permit; Civil Law: Nuisance; Due Process)

Facts:
-Respondent Jaime C. Dimson (Dimson) is the owner of a poultry farm located in
Barangay Prado Siongco, Lubao, Pampanga (subject poultry farm), which had been
operating for more than 30 years.

-In January 2014, he applied for a barangay clearance with the office of petitioner
barangay chairman, preparatory to his application for a business permit, and was
informed that the issuance thereof is conditioned on a prior ocular inspection of the
subject poultry farm by the Office of the Mayor of Lubao, Pampanga, Mayor Cayabyab.

-However, despite the conduct of an ocular inspection, Chairman David refused to


issue the clearance; hence, no business permit was issued in favor of Dimson.

-On April 29, 2014, Dimson received a Cease and Desist Order (CDO) dated April 28,
2014 from the Office of Mayor Cayabyab, directing him to desist from further
conducting any poultry farming on the grounds of: (a) lack of a Barangay Business
Permit and a Mayor's Permit; (b) foul odor being emitted by the subject poultry farm
that offended passing motorists, and for which complaints were filed by those affected.

-Thereafter, a Closure Order dated June 20, 2014 was issued by Mayor Cayabyab
effectively shutting down the subject poultry farm.

-Aggrieved, Dimson filed a Petition for Certiorari, Mandamus, Prohibition (With


Application for Preliminary Mandatory Injunction) and prayed for the issuance of a
TRO against Mayor Cayabyab and Chairman David (petitioners) before the RTC of
Guagua, Pampanga.

Page 298 of 323


-Dimson’s contention: His poultry farm is not a nuisance per se that can be
summarily abated without the intervention of the courts.

Issue:
Whether or not the CDO and Closure Order issued by the Mayor were valid even when
without the intervention of the courts.

Ruling:
Yes.

A business permit must be secured from the municipal business permits and licensing
office in order for the business to legally operate in the locality. While poultry farming
is admittedly a legitimate business, it cannot operate without a business permit,
which expires on the 31st of December of every year and must be renewed before the
end of January of the following year.

In the present case, there is no showing that Dimson filed any application for renewal
of his business permit to operate the subject poultry farm in 2014, apparently due to
his failure to secure the necessary barangay clearance which was not issued based
on complaints of foul odor being emitted by the said farm. Records show that
complaints from neighboring barangays were received by the office of Mayor
Cayabyab bewailing the foul odor coming from the said farm, which was
confirmed upon ocular inspection conducted by the Health and Sanitation Office
of the Municipality of Lubao, Pampanga. Settled is the rule that acts of public
officers are presumed to be regular and valid, unless sufficiently shown to be
otherwise. In this case, Dimson was unable to refute the finding that foul odor is
being emitted by his farm, having failed to present the inspection report of the
sanitary officer who purportedly did not note any such foul smell in the farm. Not
having passed the necessary sanitation standard, there was, therefore, a prima
facie valid reason for the withholding of the required barangay clearance, which
is a prerequisite to the renewal of Dimson's business permit to operate.

Having failed to apply for and secure the necessary business permit to operate in
2014 on account of his inability to obtain the required barangay clearance due to
non-compliance with a requirement standard, Dimson may not legally operate in
the Municipality of Lubao, Pampanga, thereby, warranting the issuance by Mayor
Cayabyab of the CDO and the Closure Order. Accordingly, no error, much less grave
abuse of discretion can be ascribed on the RTC in denying Dimson's application for
the issuance of a TRO against the said orders. In the absence of a business permit,
Dimson has no clear legal right to resume his operations pending final
determination by the RTC of the merits of the main case for certiorari,
mandamus, and prohibition. A clear legal right means one clearly founded in or
granted by law or is enforceable as a matter of law, which is not extant in the
present case. It is settled that the possibility of irreparable damage without proof of
an actual existing right is not a ground for the issuance of an injunctive relief.

*Matalin Coconut v. Municipal Council of Malabang, Lanao del Sur, G.R. No. L-28138,
August 13, 1986.
(Local Government Code; License Fee; Ordinance; Tax)

Facts:
-The Municipal Council of Malabang, Lanao del Sur enacted Municipal Ordinance No.
45-46, imposing a “police inspection fee” of P.30 per sack of cassava starch or flour,
which shall be paid by the shipper before the same is transported or shipped outside
the municipality.

-The validity of the ordinance was challenged by the Matalin Coconut, Inc. in a
petition for declaratory relief filed with the RTC of Lanao del Sur against the Municipal
Council, the Municipal Mayor and the Municipal Treasurer of Malabang, Lanao del
Sur.

Page 299 of 323


-After due proceedings, the RTC declared the ordinance void. The municipality
appealed.

-Contention of the municipality: The “police inspection fee” is a license fee that can be
validly imposed by the municipality pursuant to its police power to regulate the
shipping out of flour.

Issue:
Whether or not the subject ordinance is void.

Ruling:
Yes.

We agree with the finding of the trial court that the amount collected under the
ordinance in question is not a license fee, and partakes of the nature of a tax,
although denominated as “police inspection fee” since its undeniable purpose is
to raise revenue.

According to Section 2 of the above-mentioned Act, the tax levied must be “for public
purposes, just and uniform.” As correctly held by the trial court, the so-called “police
inspection fee” levied by the ordinance is “unjust and unreasonable.”

It has been proven that the only service rendered by the Municipality of Malabang, by
way of inspection, is for the policeman to verify from the driver of the trucks of the
petitioner passing by at the police checkpoint the number of bags loaded per trip
which are to be shipped out of the municipality based on the trip tickets for the
purpose of computing the total amount of tax to be collected and for no other
purpose.

The pretention of respondents that the police, aside from counting the number of bags
shipped out, is also inspecting the cassava flour starch contained in the bags to find
out if the said cassava flour starch is fit for human consumption could not be given
credence by the Court because, aside from the fact that said purpose is not so
stated in the ordinance in question, the policemen of said municipality are not
competent to determine if the cassava flour starch are fit for human
consumption. The further pretention of respondents that the trucks of the petitioner
hauling the bags of cassava flour starch from the mill to the bodega at the beach of
Malabang are escorted by a policeman from the police checkpoint to the beach for the
purpose of protecting the truck and its cargoes from molestation by undesirable
elements could not also be given credence by the Court because it has been shown,
beyond doubt, that the petitioner has not asked for the said police protection
because there has been no occasion where its trucks have been molested, even
for once, by bad elements from the police checkpoint to the bodega at the beach, it is
solely for the purpose of verifying the correct number of bags of cassava flour starch
loaded on the trucks of the petitioner as stated in the trip tickets, when unloaded at
its bodega at the beach. The imposition, therefore, of a police inspection fee of P.30 per
bag, imposed by said ordinance is unjust and unreasonable. x x x x

The Court finally finds the inspection fee of P0.30 per bag, imposed by the ordinance
in question to be excessive and confiscatory. It has been shown by the petitioner,
Matalin Coconut Company, Inc., that it is merely realizing a marginal average
profit of P0.40, per bag, of cassava flour starch shipped out from the Municipality
of Malabang because the average production is P15.60 per bag, including
transportation costs, while the prevailing market price is P16.00 per bag. The further
imposition, therefore, of the tax of P0.30 per bag, by the ordinance in question
would force the petitioner to close or stop its cassava flour starch milling
business considering that it is maintaining a big labor force in its operation, including
a force of security guards to guard its properties. The ordinance, therefore, has an
adverse effect on the economic growth of the Municipality of Malabang, in
particular, and of the nation, in general, and is contrary to the economic policy of the
government.

Page 300 of 323


*Rimando v. Naguilian Emission Testing Center, G.R. No. 198860, July 23,
2012.
(Local Government Code; Business Permit; Police Power; Remedial Law: Mandamus)

Facts:
-The Municipality of Naguilian (petitioner) is the declared owner of the subject parcel
of land by virtue of Tax Declaration No. 002-01197.

-However, Naguilian Emission Testing Center, Inc., represented by its President,


Rosemarie Llarenas (respondent) insists that its business is being conducted on a
parcel of land which formerly belonged to the national government but later on
certified by the Department of Environment and Natural Resources (DENR) as an
alienable and disposable land of the public domain.

-On January 18, 2008, the respondent filed an application for the renewal of its
business permit and paid the corresponding fees therefor.

-The petitioner refused to issue a business permit unless and until the respondent
executes a contract of lease with the Municipality of Naguilian.

-Eventually, respondent filed a petition for mandamus before the RTC to compel the
petitioner to issue a business permit.

-The RTC denied the petition for lack of merit based on the ratiocinations that, among
others, under Section 6A.01 of the Revenue Code of the Municipality of Naguilian, the
municipality has the right to require the petitioner to sign a contract of lease because
its business operation is being conducted on a real property owned by the
municipality.

Issue:
Whether or not mandamus shall prosper under the circumstances.

Ruling:
No.

A mayor cannot be compelled by mandamus to issue a business permit since the


exercise of the same is a delegated police power hence, discretionary in nature.
This was the pronouncement of this Court in Roble Arrastre, Inc. v. Hon. Villaflor where
a determination was made on the nature of the power of a mayor to grant business
permits under the Local Government Code, viz:

Central to the resolution of the case at bar is a reading of Section 444(b)(3)(iv) of the
Local Government Code of 1991, which provides, thus:

SEC. 444. The Chief Executive: Powers, Duties, Functions and Compensation.

(b) For efficient, effective and economical governance the purpose of which is the general welfare of the
municipality and its inhabitants pursuant to Section 16 of this Code, the municipal mayor shall:
xxxx
3) Initiate and maximize the generation of resources and revenues, and apply the same to the
implementation of development plans, program objectives and priorities as provided for under Section 18
of this Code, particularly those resources and revenues programmed for agro-industrial development and
country-wide growth and progress, and relative thereto, shall:

xxxx

(iv) Issue licenses and permits and suspend or revoke the same for any violation of the conditions
upon which said licenses or permits had been issued, pursuant to law or ordinance.

As Section 444(b)(3)(iv) so states, the power of the municipal mayor to issue licenses is
pursuant to Section 16 of the Local Government Code of 1991, which declares:

Page 301 of 323


SEC. 16. General Welfare. – Every local government unit shall exercise the powers expressly
granted, those necessarily implied therefrom, as well as powers necessary, appropriate, or
incidental for its efficient and effective governance, and those which are essential to the
promotion of the general welfare. Within their respective territorial jurisdictions, local government units
shall ensure and support, among other things, the preservation and enrichment of culture, promote
health and safety, enhance the right of the people to a balanced ecology, encourage and support the
development of appropriate and self-reliant scientific and technological capabilities, improve public
morals, enhance economic prosperity and social justice, promote full employment among their residents,
maintain peace and order, and preserve the comfort and convenience of their inhabitants.

Section 16, known as the general welfare clause, encapsulates the delegated
police power to local governments. Local government units exercise police power
through their respective legislative bodies. Evidently, the Local Government Code of
1991 is unequivocal that the municipal mayor has the power to issue licenses and
permits and suspend or revoke the same for any violation of the conditions upon
which said licenses or permits had been issued, pursuant to law or ordinance. x x x x

Section 444(b)(3)(iv) of the Local Government Code of 1991, whereby the power of the
respondent mayor to issue license and permits is circumscribed, is a manifestation
of the delegated police power of a municipal corporation. Necessarily, the
exercise thereof cannot be deemed ministerial. As to the question of whether the
power is validly exercised, the matter is within the province of a writ of
certiorari, but certainly, not of mandamus.

Indeed, as correctly ruled by the RTC, the petition for mandamus filed by the
respondent is incompetent to compel the exercise of a mayor’s discretionary
duty to issue business permits.

*Demaala v. COA, G.R. No. 199752, February 17, 2015.


(LGU; Real Property Tax; Local Autonomy; COA)

Facts:
-The Sangguniang Panlalawigan of Palawan enacted Provincial Ordinance No. 332-A,
Series of 1995, entitled “An Ordinance Approving and Adopting the Code Governing
the Revision of Assessments, Classification and Valuation of Real Properties in the
Province of Palawan” (Ordinance).

-Chapter 5, Section 48 of the Ordinance provides for an additional levy on real


property tax for the special education fund (SEF) at the rate of one-half percent or
0.5%.

-In conformity with Section 48 of the Ordinance, the Municipality of Narra, Palawan,
with Demaala as mayor, collected from owners of real properties located within its
territory an annual tax as special education fund at the rate of 0.5% of the assessed
value of the property subject to tax. This collection was effected through the municipal
treasurer.

-On post-audit, the Audit Team Leader of the Commission on Audit (COA) issued Audit
Observation Memorandum (AOM) No. 03-005 in which he questioned the levy of the
special education fund at the rate of only 0.5% rather than at 1%, the rate stated
in Section 23511 of Republic Act No. 7160, otherwise known as the Local
Government Code of 1991 (Local Government Code).

-After evaluating AOM No. 03-005, the Regional Cluster Director issued NC No. 2004-
04-101 in the amount of ₱1,125,416.56. He held Demaala, the municipal treasurer of
Narra, and all special education fund payors liable for the deficiency in special
education fund collections.

-After long proceedings before the COA and getting an adverse ruling, petitioner went
to the SC via certiorari to question the same.

Issue:

Page 302 of 323


Whether or not the imposition of the rate of only 0.5% rather than at 1%, the rate
stated in Section 23511 of Republic Act No. 7160, is valid.

Ruling:
Yes.

Setting the rate of the additional levy for the special education fund at less than
1% is within the taxing power of local government units. It is consistent with the
guiding constitutional principle of local autonomy. x x x x

Consistent with the 1987 Constitution’s declared preference, the taxing powers of local
government units must be resolved in favor of their local fiscal autonomy. In City
Government of San Pablo v. Reyes:

The power to tax is primarily vested in Congress. However, in our jurisdiction, it may
be exercised by local legislative bodies, no longer merely by virtue of a valid delegation
as before, but pursuant to direct authority conferred by Section 5, Article X of the
Constitution. Thus Article X, Section 5 of the Constitution reads:

Sec. 5 — Each Local Government unit shall have the power to create its own sources of revenue and to
levy taxes, fees and charges subject to such guidelines and limitations as the Congress may provide,
consistent with the basic policy of local autonomy. Such taxes, fees and charges shall accrue exclusively
to the Local Governments.

The important legal effect of Section 5 is that henceforth, in interpreting


statutory provision on municipal fiscal powers, doubts will have to be resolved in
favor of municipal corporations. x x x x

The limits on the level of additional levy for the special education fund under
Section 235 of the Local Government Code should be read as granting fiscal
flexibility to local government units.

Book II of the Local Government Code governs local taxation and fiscal matters. Title II
of Book II governs real property taxation.

Section 235 of the Local Government Code allows provinces and cities, as well as
municipalities in Metro Manila, to collect, on top of the basic annual real property tax,
an additional levy which shall exclusively accrue to the special education fund, to wit:

Section 235. Additional Levy on Real Property for the Special Education Fund. - A province or city, or a
municipality within the Metropolitan Manila Area, may levy and collect an annual tax of one
percent (1%) on the assessed value of real property which shall be in addition to the basic real
property tax. The proceeds thereof shall exclusively accrue to the Special Education Fund (SEF).

The special education fund is not an original creation of the Local Government Code. It
was initially devised by Republic Act No. 5447.45 The rate of 1% is also not a detail
that is original to the Local Government Code. As discussed in Commission on Audit v.
Province of Cebu: The Special Education Fund was created by virtue of R. A. No. 5447,
which is an act creating a special education fund to be constituted from the proceeds
of an additional real property tax and a certain portion of the taxes on Virginia-type
cigarettes and duties on imported leaf tobacco, defining the activities to be financed,
creating school boards for the purpose, and appropriating funds therefrom, which took
effect on January 1, 1969. Pursuant thereto, P.D. No. 464, also known as the Real
Property Tax Code of the Philippines, imposed an annual tax of 1% on real property
which shall accrue to the SEF.

The operative phrase in Section 235’s grant to municipalities in Metro Manila,


cities, and provinces of the power to impose an additional levy for the special
education fund is prefixed with "may," thus, "may levy and collect an annual tax
of one percent (1%)."

Page 303 of 323


Where the provision reads "may," this word shows that it is not mandatory but
discretionary. It is an auxiliary verb indicating liberty, opportunity, permission
and possibility. The use of the word "may" in a statute denotes that it is directory in
nature and generally permissive only. Respondent concedes that Section 235’s grant
to municipalities in Metro Manila, to cities, and to provinces of the power to impose an
additional levy for the special education fund makes its collection optional. It is not
mandatory that the levy be imposed and collected. The controversy which the
Commission on Audit created is not whether these local government units have
discretion to collect but whether they have discretion on the rate at which they are to
collect. x x x x

Section 235’s permissive language is unqualified. Moreover, there is no limiting


qualifier to the articulated rate of 1% which unequivocally indicates that any
and all special education fund collections must be at such rate.

At most, there is a seeming ambiguity in Section 235. Consistent with what has earlier
been discussed however, any such ambiguity must be read in favor of local fiscal
autonomy. As in San Juan v. Civil Service Commission, the scales must weigh in
favor of the local government unit.

*Smart Communications v. Malvar, Batangas, G.R. No. 204429, February 18,


2014.
(Tax; Remedial Law: Jurisdiction of Court of Tax Appeals; Fees; Local Government
Code; Police Power)

Facts:
-Petitioner Smart Communications, Inc. (Smart) is a domestic corporation engaged in
the business of providing telecommunications services to the general public while
respondent Municipality of Malvar, Batangas (Municipality) is a local government unit
created by law.

-In the course of its business, Smart constructed a telecommunications tower within
the territorial jurisdiction of the Municipality.

-On 30 July 2003, the Municipality passed Ordinance No. 18, series of 2003, entitled
"An Ordinance Regulating the Establishment of Special Projects."

-On 24 August 2004, Smart received from the Permit and Licensing Division of the
Office of the Mayor of the Municipality an assessment letter with a schedule of
payment for the total amount of ₱389,950.00 for Smart’s telecommunications tower.

-On 9 September 2004, Smart filed a protest, claiming lack of due process in the
issuance of the assessment. In the same protest, Smart challenged the validity of
Ordinance No. 18 on which the assessment was based.

-In a letter dated 28 September 2004, the Municipality denied Smart’s protest.

-On 17 November 2004, Smart filed with Regional Trial Court of Tanauan City,
Batangas, Branch 6, an "Appeal/Petition" assailing the validity of Ordinance No. 18.

-The RTC sustained the validity of the ordinance.

-Aggrieved, Smart filed a petition for review with the CTA First Division, which
affirmed the ruling of the RTC. Upon petition for review by Smart before the CTA En
Banc, the said court dismissed the petition on the ground of lack of jurisdiction,
ratiocinating that Republic Act No. 9282 (AN ACT EXPANDING THE JURISDICTION
OF THE COURT OF TAX APPEALS) does not confer on the CTA jurisdiction to resolve
cases where the constitutionality of a law or rule is challenged.

-Contentions of Smart: (1) The CTA has jurisdiction over the case considering that
imposition of taxes is involved; (2) the Municipality is encroaching on the regulatory

Page 304 of 323


powers of the National Telecommunications Commission (NTC), as the regulation of
telecommunications entities and all aspects of its operations is specifically lodged by
law on the NTC; (3) the subject fees are unjust, excessive, oppressive and confiscatory.

-Ruling of the CTA:

Issues:
(1) Whether or not the CTA has jurisdiction over the case;
(2) Whether or not the Municipality encroaches on the regulatory powers of the
National Telecommunications Commission (NTC);
(3) Whether or not the subject fees are unjust, excessive, oppressive and confiscatory.

Ruling:
(1) No.

Jurisdiction is conferred by law. Republic Act No. 1125, as amended by Republic Act
No. 9282, created the Court of Tax Appeals. Section 7, paragraph (a), sub-paragraph
(3)15 of the law vests the CTA with the exclusive appellate jurisdiction over "decisions,
orders or resolutions of the Regional Trial Courts in local tax cases originally decided
or resolved by them in the exercise of their original or appellate jurisdiction." x x x x

Section 5, Article X of the 1987 Constitution provides that "each local government unit
shall have the power to create its own sources of revenues and to levy taxes, fees, and
charges subject to such guidelines and limitations as the Congress may provide,
consistent with the basic policy of local autonomy. Such taxes, fees, and charges shall
accrue exclusively to the local government."

The LGC defines the term "charges" as referring to pecuniary liability, as rents or fees
against persons or property, while the term "fee" means "a charge fixed by law or
ordinance for the regulation or inspection of a business or activity."

The Court finds that the fees imposed under Ordinance No. 18 are not taxes.

In this case, the Municipality issued Ordinance No. 18, which is entitled "An
Ordinance Regulating the Establishment of Special Projects," to regulate the
"placing, stringing, attaching, installing, repair and construction of all gas
mains, electric, telegraph and telephone wires, conduits, meters and other
apparatus, and provide for the correction, condemnation or removal of the same
when found to be dangerous, defective or otherwise hazardous to the welfare of
the inhabitants.” It was also envisioned to address the foreseen "environmental
depredation" to be brought about by these "special projects" to the Municipality.
Pursuant to these objectives, the Municipality imposed fees on various structures,
which included telecommunications towers.

As clearly stated in its whereas clauses, the primary purpose of Ordinance No. 18 is
to regulate the "placing, stringing, attaching, installing, repair and construction of all
gas mains, electric, telegraph and telephone wires, conduits, meters and other
apparatus" listed therein, which included Smart’s telecommunications tower. Clearly,
the purpose of the assailed Ordinance is to regulate the enumerated activities
particularly related to the construction and maintenance of various structures.
The fees in Ordinance No. 18 are not impositions on the building or structure
itself; rather, they are impositions on the activity subject of government
regulation, such as the installation and construction of the structures.

Since the main purpose of Ordinance No. 18 is to regulate certain construction


activities of the identified special projects, which included "cell sites" or
telecommunications towers, the fees imposed in Ordinance No. 18 are primarily
regulatory in nature, and not primarily revenue-raising. While the fees may
contribute to the revenues of the Municipality, this effect is merely incidental.
Thus, the fees imposed in Ordinance No. 18 are not taxes.

Page 305 of 323


In Progressive Development Corporation v. Quezon City, the Court declared that "if the
generating of revenue is the primary purpose and regulation is merely incidental, the
imposition is a tax; but if regulation is the primary purpose, the fact that
incidentally revenue is also obtained does not make the imposition a tax."

We accordingly say that the designation given by the municipal authorities does
not decide whether the imposition is properly a license tax or a license fee. The
determining factors are the purpose and effect of the imposition as may be
apparent from the provisions of the ordinance.

Considering that the fees in Ordinance No. 18 are not in the nature of local
taxes, and Smart is questioning the constitutionality of the ordinance, the CTA
correctly dismissed the petition for lack of jurisdiction.

(2) No.

To repeat, Ordinance No. 18 aims to regulate the “placing, stringing, attaching,


installing, repair and construction of all gas mains, electric, telegraph and telephone
wires, conduits, meters and other apparatus” within the Municipality. The fees are
not imposed to regulate the administrative, technical, financial, or marketing
operations of telecommunications entities, such as Smart’s; rather, to regulate the
installation and maintenance of physical structures – Smart’s cell sites or
telecommunications tower. The regulation of the installation and maintenance of
such physical structures is an exercise of the police power of the Municipality.
Clearly, the Municipality does not encroach on NTC’s regulatory powers.

(3) No.

Aside from this bare allegation, Smart did not present any evidence substantiating
its claim that the subject fees are unjust, excessive, oppressive and confiscatory.

Smart merely pleaded for the declaration of unconstitutionality of Ordinance No.


18 in the Prayer of the Petition, without any argument or evidence to support its
plea. Nowhere in the body of the Petition was this issue specifically raised and
discussed. Significantly, Smart failed to cite any constitutional provision allegedly
violated by respondent when it issued Ordinance No. 18.

Settled is the rule that every law, in this case an ordinance, is presumed valid.
To strike down a law as unconstitutional, Smart has the burden to prove a clear
and unequivocal breach of the Constitution, which Smart miserably failed to do.
In Lawyers Against Monopoly and Poverty (LAMP) v. Secretary of Budget and
Management, the Court held, thus:

To justify the nullification of the law or its implementation, there must be a clear and
unequivocal, not a doubtful, breach of the Constitution. In case of doubt in the
sufficiency of proof establishing unconstitutionality, the Court must sustain legislation
because "to invalidate [a law] based on xx x baseless supposition is an affront to the
wisdom not only of the legislature that passed it but also of the executive which
approved it." This presumption of constitutionality can be overcome only by the
clearest showing that there was indeed an infraction of the Constitution, and only
when such a conclusion is reached by the required majority may the Court pronounce,
in the discharge of the duty it cannot escape, that the challenged act must be struck
down.

*City of Lapu-Lapu v. PEZA, G.R. No. 184203, November 26, 2014.


(Real Property Tax; Exemptions; Local Government Code; Remedial Law: Declaratory
Relief)

Facts:
-In the letter dated March 25, 1998, the City of Lapu-Lapu, through the Office of the
Treasurer, demanded from the PEZA PhP32,912,350.08 in real property taxes for

Page 306 of 323


the period from 1992 to 1998 on the PEZA’s properties located in the Mactan
Economic Zone.

-The City made subsequent demands on the PEZA. In its last reminder dated May 13,
2002, the City assessed the PEZA PhP86,843,503.48 as real property taxes for the
period from 1992 to 2002.

-On September 11, 2002, the PEZA filed a petition for Declaratory Relief with the
Regional Trial Court of Pasay City, praying that the trial court declare it exempt from
payment of real property taxes.

-Contention of the City: No provision in the Special Economic Zone Act of 1995
specifically exempted the PEZA from payment of real property taxes, unlike Section 21
of Presidential Decree No. 66 that explicitly provided for EPZA’s exemption. Since no
legal provision explicitly exempted the PEZA from payment of real property taxes, the
City argued that it can tax the PEZA.

Issues:
(1) Whether or not an action for declaratory relief is the proper action under the
circumstances;
(2) Whether or not PEZA is exempt from payment of real property taxes.

Ruling:
(1) No.

A special civil action for declaratory relief is filed for a judicial determination of any
question of construction or validity arising from, and for a declaration of rights and
duties, under any of the following subject matters: a deed, will, contract or other
written instrument, statute, executive order or regulation, ordinance, orany other
governmental regulation. However, a declaratory judgment may issue only if there
has been "no breach of the documents in question." If the contract or statute
subject matter of the action has already been breached, the appropriate ordinary civil
action must be filed. If adequate relief is available through another form of action or
proceeding, the other action must be preferred over an action for declaratory relief. x x
xx

We rule that the PEZA erred in availing itself of a petition for declaratory relief against
the City. The City had already issued demand letters and real property tax
assessment against the PEZA, in violation of the PEZA’s alleged tax-exempt
status under its charter. The Special Economic Zone Act of 1995, the subject matter
of PEZA’s petition for declaratory relief, had already been breached. The trial court,
therefore, had no jurisdiction over the petition for declaratory relief. x x x x

In the present case, the Regional Trial Court had no jurisdiction over the subject
matter of the action, specifically, over the remedy sought. As this court explained in
Malana v. Tappa:

“An action for declaratory relief presupposes that there has been no actual
breach of the instruments involved or of rights arising thereunder. Since the
purpose of an action for declaratory relief is to secure an authoritative statement of
the rights and obligations of the parties under a statute, deed, or contract for their
guidance in the enforcement thereof, or compliance therewith, and not to settle
issues arising from an alleged breach thereof, it may be entertained only before the
breach or violation of the statute, deed, or contract to which it refers. A petition for
declaratory relief gives a practical remedy for ending controversies that have not
reached the state where another relief is immediately available; and supplies the need
for a form of action that will set controversies at rest before they lead to a repudiation
of obligations, an invasion of rights, and a commission of wrongs.

Where the law or contract has already been contravened prior to the filing of an
action for declaratory relief, the courts can no longer assume jurisdiction over

Page 307 of 323


the action. In other words, a court has no more jurisdiction over an action for
declaratory relief if its subject has already been infringed or transgressed before the
institution of the action.”

In the present case, the PEZA did not avail itself of any of the remedies against a
notice of assessment. A petition for declaratory relief is not the proper remedy
once a notice of assessment was already issued.

Instead of a petition for declaratory relief, the PEZA should have directly resorted to a
judicial action. The PEZA should have filed a complaint for injunction, the
“appropriate ordinary civil action” to enjoin the City from enforcing its demand
and collecting the assessed taxes from the PEZA. After all, a declaratory judgment
as to the PEZA’s tax-exempt status is useless unless the City is enjoined from
enforcing its demand.

Injunction is a judicial writ, process or proceeding whereby a party is ordered to


do or refrain from doing a certain act. It may be the main action or merely a
provisional remedy for and as incident in the main action. The essential requisites of a
writ of injunction are: “(1) there must be a right in esse or the existence of a right to be
protected; and (2) the act against which the injunction is directed to constitute a
violation of such right.”

(2) Yes.

A. The PEZA is an instrumentality of the national government

An instrumentality is "any agency of the National Government, not integrated within


the department framework, vested with special functions or jurisdiction by law,
endowed with some if not all corporate powers, administering special funds, and
enjoying operational autonomy, usually through a charter."

Examples of instrumentalities of the national government are the Manila International


Airport Authority, the Philippine Fisheries Development Authority, the Government
Service Insurance System, and the Philippine Reclamation Authority. These entities
are not integrated within the department framework but are nevertheless vested with
special functions to carry out a declared policy of the national government.

Similarly, the PEZA is an instrumentality of the national government. It is not


integrated within the department framework but is an agency attached to the
Department of Trade and Industry. Book IV, Chapter 7, Section 38(3)(a) of the
Administrative Code of 1987 defines "attachment":

“SEC. 38. Definition of Administrative Relationship.– Unless otherwise expressly stated in the Code or in
other laws defining the special relationships of particular agencies, administrative relationships shall be
categorized and defined as follows:

xxxx

(3) Attachment.– (a) This refers to the lateral relationship between the department or its equivalent and
the attached agency or corporation for purposes of policy and program coordination. The coordination
may be accomplished by having the department represented in the governing board of the attached
agency or corporation, either as chairman or as a member, with or without voting rights, if this is
permitted by the charter; having the attached corporation or agency comply with a system of periodic
reporting which shall reflect the progress of the programs and projects; and having the department or its
equivalent provide general policies through its representative in the board, which shall serve as the
framework for the internal policies of the attached corporation or agency.”

Attachment, which enjoys "a larger measure of independence" compared with other
administrative relationships such as supervision and control, is further explained in
Beja, Sr. v. Court of Appeals:

Page 308 of 323


An attached agency has a larger measure of independence from the Department to
which it is attached than one which is under departmental supervision and control or
administrative supervision. This is borne out by the "lateral relationship" between the
Department and the attached agency. The attachment is merely for "policy and
program coordination." With respect to administrative matters, the independence of an
attached agency from Departmental control and supervision is further reinforced by
the fact that even an agency under a Department’s administrative supervision is free
from Departmental interference with respect to appointments and other personnel
actions "in accordance with the decentralization of personnel functions" under the
Administrative Code of 1987. Moreover, the Administrative Code explicitly provides
that Chapter 8 of Book IV on supervision and control shall not apply to chartered
institutions attached to a Department.

With the PEZA as an attached agency to the Department of Trade and Industry,
the 13-person PEZA Board is chaired by the Department Secretary. Among the
powers and functions of the PEZA is its ability to coordinate with the Department of
Trade and Industry for policy and program formulation and implementation. In
strategizing and prioritizing the development of special economic zones, the PEZA
coordinates with the Department of Trade and Industry. x x x x

Being an instrumentality of the national government, the PEZA cannot be taxed by


local government units. Although a body corporate vested with some corporate powers,
the PEZA is not a government-owned or controlled corporation taxable for real
property taxes. x x x x

B. The PEZA assumed the non-profit character, including the tax exempt status,
of the EPZA

The PEZA’s predecessor, the EPZA, was declared non-profit in character with all its
revenues devoted for its development, improvement, and maintenance. Consistent
with this non-profit character, the EPZA was explicitly declared exempt from real
property taxes under its charter. Section 21 of Presidential Decree No. 66 provides:

Section 21. Non-profit Character of the Authority; Exemption from Taxes. The Authority shall be non-
profit and shall devote and use all its returns from its capital investment, as well as excess revenues from
its operations, for the development, improvement and maintenance and other related expenditures of the
Authority to pay its indebtedness and obligations and in furtherance and effective implementation of the
policy enunciated in Section 1 of this Decree. In consonance therewith, the Authority is hereby declared
exempt:
xxxx
(b) From all income taxes, franchise taxes, realty taxes and all other kinds of taxes and licenses to be paid
to the National Government, its provinces, cities, municipalities and other government agencies and
instrumentalities.

The Special Economic Zone Act of 1995, on the other hand, does not specifically
exempt the PEZA from payment of real property taxes.

Nevertheless, we rule that the PEZA is exempt from real property taxes by virtue of its
charter. A provision in the Special Economic Zone Act of 1995 explicitly
exempting the PEZA is unnecessary. The PEZA assumed the real property
exemption of the EPZA under Presidential Decree No. 66.

C. Real properties under the PEZA’s title are owned by the Republic of the
Philippines.

Under Section 234(a) of the Local Government Code, real properties owned by the
Republic of the Philippines are exempt from real property taxes:

SEC. 234. Exemptions from Real Property Tax. – The following are exempted from payment of real
property tax:

(a) Real property owned by the Republic of the Philippines or any of its political subdivisions except when
the beneficial use thereof has been granted, for consideration or otherwise, to a taxable person.

Page 309 of 323


Properties owned by the state are either property of public dominion or patrimonial
property. Article 420 of the Civil Code of the Philippines enumerates property of public
dominion:

Art. 420. The following things are property of public dominion:

(1) Those intended for public use, such as roads, canals, rivers, torrents, ports and bridges constructed
by the State, banks, shores, roadsteads, and others of similar character;

(2) Those which belong to the State, without belonging for public use, and are intended for some public
service or for the development of the national wealth. x x x x

Properties of public dominion are outside the commerce of man. These


properties are exempt from "levy, encumbrance or disposition through public or
private sale." x x x x

In this case, the properties sought to be taxed are located in publicly owned
economic zones. These economic zones are property of public dominion. The City
seeks to tax properties located within the Mactan Economic Zone, the site of which
was reserved by President Marcos under Proclamation No. 1811, Series of 1979.
Reserved lands are lands of the public domain set aside for settlement or public use,
and for specific public purposes by virtue of a presidential proclamation. Reserved
lands are inalienable and outside the commerce of man, and remain property of
the Republic until withdrawn from public use either by law or presidential
proclamation. Since no law or presidential proclamation has been issued
withdrawing the site of the Mactan Economic Zone from public use, the property
remains reserved land.

*Manila v. Teotico, G.R. No. L-23052, January 29, 1968.


(Municipal Liability; General Law; Special Law; City Charter; Civil Law: Damages)

Facts:
-Respondent Genaro N. Teotico, while attempting to board a jeepney, fell inside an
uncovered and unlighted manhole on P. Burgos Avenue.

-Due to the fall, his head hit the rim of the manhole breaking his eyeglasses and
causing broken pieces thereof to pierce his left eyelid. As blood flowed therefrom,
impairing his vision, several persons came to his assistance and pulled him out of the
manhole. One of them brought Teotico to the Philippine General Hospital, where his
injuries were treated, after which he was taken home.

-In addition to the lacerated wound in his left upper eyelid, Teotico suffered
contusions on the left thigh, the left upper arm, the right leg and the upper lip apart
from an abrasion on the right infra-patella region. These injuries and the allergic
eruption caused by anti-tetanus injections administered to him in the hospital,
required further medical treatment by a private practitioner who charged therefor
P1,400.00.

-As a consequence of the foregoing occurrence, Teotico filed with the RTC of Manila, a
complaint for damages against the City of Manila, its mayor, city engineer, city health
officer, city treasurer and chief of police.

-After due proceedings, the RTC dismissed the complaint.

-Upon appeal, the CA reversed the RTC. The CA sentenced the City of Manila to pay
damages in the aggregate sum of P6,750.00.

-Aggrieved, the City of Manila then went to the SC via Rule 45.

-Section 4 of Republic Act No. 409, or the Charter of the City of Manila, reads:

Page 310 of 323


“The city shall not be liable or held for damages or injuries to persons or property arising from the
failure of the Mayor, the Municipal Board, or any other city officer, to enforce the provisions of
this chapter, or any other law or ordinance, or from negligence of said Mayor, Municipal Board, or other
officers while enforcing or attempting to enforce said provisions.”

-On the other hand, Article 2189 of the Civil Code of the Philippines provides:

“Provinces, cities and municipalities shall be liable for damages for the death of, or injuries
suffered by, any person by reason of defective conditions of road, streets, bridges, public buildings,
and other public works under their control or supervision.”

-Contention of the City of Manila: The provision of the Charter of the City of Manila
should prevail over that of the Civil Code because Republic Act 409, is a special law,
intended exclusively for the City of Manila, whereas the Civil Code is a general law,
applicable to the entire Philippines.

Issue:
Whether or not the City of Manila should be held liable for Teotico’s injuries.

Ruling:
Yes.

It is true that, insofar as its territorial application is concerned, Republic Act No. 409
is a special law and the Civil Code a general legislation; but, as regards the subject-
matter of the provisions above quoted, Section 4 of Republic Act 409 establishes a
general rule regulating the liability of the City of Manila for: "damages or injury
to persons or property arising from the failure of "city officers to enforce the
provisions of" said Act "or any other law or ordinance, or from negligence" of the
city "Mayor, Municipal Board, or other officers while enforcing or attempting to
enforce said provisions."

Upon the other hand, Article 2189 of the Civil Code constitutes a particular
prescription making "provinces, cities and municipalities . . . liable for damages
for the death of, or injury suffered by any person by reason" — specifically — "of
the defective condition of roads, streets, bridges, public buildings, and other-
public works under their control or supervision."

In other words, said Section 4 refers to liability arising from negligence, in


general, regardless of the object thereof, whereas Article 2189 governs liability
due to “defective streets,” in particular. Since the present action is based upon
the alleged defective condition of a road, said Article 2189 is decisive thereon.

[NOTE: The SC affirmed the ruling of the CA.]

*Guilatco v. Dagupan, G.R. No. 61516, March 21, 1989.


(Municipal Liability; Civil Law; Control and Supervision)

Facts:
-Petitioner Florentina A. Guilatco while she was about to board a motorized tricycle at
a sidewalk located at Perez Boulevard (a National Road, under the control and
supervision of the City of Dagupan) accidentally fell into a manhole located on said
sidewalk, thereby causing her right leg to be fractured.

-As a result thereof, she had to be hospitalized, operated on, confined, at first at the
Pangasinan Provincial Hospital, from July 25 to August 3, 1978 (or for a period of 16
days). She also incurred hospitalization, medication and other expenses to the tune of
P 8,053.65.

-In view of this, Guilatco filed a complaint for damages before the RTC. After due
proceedings, the RTC ordered the City of Dagupan to pay Guilatco in the amount of
P15,924.00 as actual damages, P150,000.00 as moral damages, P50,000.00 as

Page 311 of 323


exemplary damages, and P3,000.00 as attorney's fees, and litigation expenses, plus
costs.

-On appeal by the respondent City of Dagupan, the CA reversed the lower court
findings on the ground that no evidence was presented by the Guilatco to prove that
the City of Dagupan had “control or supervision” over Perez Boulevard.

-Aggrieved, Guilatco went to the SC.

-Contention of the City of Dagupan: Perez Boulevard, where the fatal drainage hole is
located, is a national road that is not under the control or supervision of the City
of Dagupan. Hence, no liability should attach to the city. It is actually the Secretary
of Public Highways that has control or supervision of the national road.

Issue:
Whether or not the City of Dagupan is laible for Guilatco’s injurires.

Ruling:
Yes.

The liability of public corporations for damages arising from injuries suffered by
pedestrians from the defective condition of roads is expressed in the Civil Code as
follows:

Article 2189. Provinces, cities and municipalities shall be liable for damages for the death of, or
injuries suffered by, any person by reason of the defective condition of roads, streets, bridges,
public buildings, and other public works under their control or supervision.

It is not even necessary for the defective road or street to belong to the
province, city or municipality for liability to attach. The article only requires
that either control or supervision is exercised over the defective road or street.

In the case at bar, this control or supervision is provided for in the charter of
Dagupan and is exercised through the City Engineer who has the following duties:

Sec. 22. The City Engineer--His powers, duties and compensation-There shall be a city engineer, who
shall be in charge of the department of Engineering and Public Works. He shall receive a salary of not
exceeding three thousand pesos per annum. He shall have the following duties:
xxxx
(j) He shall have the care and custody of the public system of waterworks and sewers, and all
sources of water supply, and shall control, maintain and regulate the use of the same, in accordance
with the ordinance relating thereto; shall inspect and regulate the use of all private systems for supplying
water to the city and its inhabitants, and all private sewers, and their connection with the public sewer
system.
xxxx

The same charter of Dagupan also provides that the laying out, construction and
improvement of streets, avenues and alleys and sidewalks, and regulation of the use
thereof, may be legislated by the Municipal Board. Thus the charter clearly
indicates that the city indeed has supervision and control over the sidewalk
where the open drainage hole is located.

The express provision in the charter holding the city not liable for damages or injuries
sustained by persons or property due to the failure of any city officer to enforce the
provisions of the charter, can not be used to exempt the city, as in the case at bar.

The charter only lays down general rules regulating the liability of the city. On
the other hand article 2189 applies in particular to the liability arising from
"defective streets, public buildings and other public works."

The City Engineer, Mr. Alfredo G. Tangco, admits that he exercises control or
supervision over the said road. But the city cannot be excused from liability by the

Page 312 of 323


argument that the duty of the City Engineer to supervise or control the said provincial
road belongs more to his functions as an ex-officio Highway Engineer of the
Department of Public Highway than as a city officer. This is because while he is
entitled to an honorarium from the Department of Public Highways, his salary from
the city government substantially exceeds the honorarium.

Alfredo G. Tangco "in his official capacity as City Engineer of Dagupan, as Ex-Officio
Highway Engineer, as Ex-Officio City Engineer of the Bureau of Public Works, and,
last but not the least, as Building Official for Dagupan City, receives the following
monthly compensation: P1,810.66 from Dagupan City; P200.00 from the Department
of Public Highways; P100.00 from the Bureau of Public Works and P500.00 by virtue
of P.D. 1096, respectively." This function of supervision over streets, public buildings,
and other public works pertaining to the City Engineer is coursed through a
Maintenance Foreman and a Maintenance Engineer. Although these last two
officials are employees of the National Government, they are detailed with the
City of Dagupan and hence receive instruction and supervision from the city
through the City Engineer.

There is, therefore, no doubt that the City Engineer exercises control or
supervision over the public works in question. Hence, the liability of the city to
the petitioner under Article 2198 of the Civil Code is clear.

*Municipality of San Fernando, La Union v. Hon. Firme, G.R. No. L-52179, April 8, 1991.
(Municipal Liability; Governmental Functions; Regular Employee)

Facts:
-A collision occurred involving: (1) a passenger jeepney driven by Bernardo Balagot
and owned by the Estate of Macario Nieveras; (2) a gravel and sand truck driven by
Jose Manandeg and owned by Tanquilino Velasquez; and (3) a dump truck of the
Municipality of San Fernando, La Union and driven by Alfredo Bislig.

-Due to the impact, several passengers of the jeepney including Laureano Baniña Sr.
died as a result of the injuries they sustained and four (4) others suffered varying
degrees of physical injuries.

-The heirs of Baniña (respondents) instituted a compliant for damages against the
Estate of Macario Nieveras and Bernardo Balagot, owner and driver, respectively, of
the passenger jeepney. However, the aforesaid defendants filed a Third-Party
Complaint against the Municipality of San Fernando (petitioner) and the driver of a
dump truck of petitioner.

-After due proceedings, the RTC ordered the Municipality of San Fernando, La Union
and Alfredo Bislig to pay jointly and severally, the heirs of Baniña the sums of
P1,500.00 as funeral expenses and P24,744.24 as the lost expected earnings of the
late Laureano Baniña Sr., P30,000.00 as moral damages, and P2,500.00 as
attorney's fees. Meanwhile, the Complaint was dismissed as to the Estate of Macario
Nieveras and Bernardo Balagot.

Issue:
Whether or not the Municipality of San Fernando, La Union is liable for damages for
the death of Baniña.

Ruling:
No.

In the case at bar, the driver of the dump truck of the municipality insists that, “he
was on his way to the Naguilian river to get a load of sand and gravel for the repair of
San Fernando's municipal streets.”

In the absence of any evidence to the contrary, the regularity of the performance
of official duty is presumed pursuant to Section 3(m) of Rule 131 of the Revised

Page 313 of 323


Rules of Court. Hence, We rule that the driver of the dump truck was performing
duties or tasks pertaining to his office.

We already stressed in the case of Palafox, et. al. vs. Province of Ilocos Norte that “the
construction or maintenance of roads in which the truck and the driver worked
at the time of the accident are admittedly governmental activities.”

After a careful examination of existing laws and jurisprudence, We arrive at the


conclusion that the municipality cannot be held liable for the torts committed by
its regular employee, who was then engaged in the discharge of governmental
functions. Hence, the death of the passenger –– tragic and deplorable though it
may be –– imposes on the municipality no duty to pay monetary compensation.

[NOTE: The SC absolved the Municipality of San Fernando, La Union from any
liability in favor of the heirs of Baniña.]

*Torio v. Fontanilla, G.R. No. L-29993, October 23, 1978.


(Municipal Liability; Proprietary Functions; Negligence)

Facts:
-On October 21, 1958, the Municipal Council of Malasiqui, Pangasinan, passed
Resolution No. 159 whereby it resolved to manage the 1959 Malasiqui town fiesta
celebration on January 21, 22, and 23, 1959.

-The council appropriated the amount of P100.00 for the construction of two stages,
which were 5-½ meters by 8 meters in size, had a wooden floor high at the rear and
were supported by 24 bamboo posts — 4 in a row in front, 4 in the rear and 5 on each
side — with bamboo braces.

-Unfortunately, during the “zarzuela,” the stage collapsed and Vicente Fontanilla who
was at the rear of the stage was pinned underneath, causing his death.

-The heirs of Vicente Fontanilia filed a complaint for damages before the RTC against
the Municipality of Malasiqui, the Municipal Council of Malasiqui and all the
individual members of the Municipal Council.

-Contention of the municipality: The municipality performs sovereign functions and


the holding of a town fiesta was an exercise of its governmental functions from which
no liability can arise to answer for the negligence of any of its agents.

-After due proceedings, the RTC dismissed the complaint, ratiocinating that, “the
municipal council had exercised due diligence and care like a good father of the
family in selecting a competent man to construct a stage strong enough for the
occasion, and that if it collapsed, the same was due to forces beyond the control of the
committee on entertainment.”

-Aggrieved, the heirs of Fontanilla went to the CA, which reversed the trial court's
decision and orderedthe municipality to pay jointly and severally the heirs of Vicente
Fontanilla the sums of P12,000.00 by way of moral and actual damages, P1200.00 as
attorney's fees; and the costs.

Issue:
Whether or not the municipality is liable for damages for the death of Fontanilla.

Ruling:
Yes.

The holding of the town fiesta in 1959 by the municipality of Malsiqui


Pangasinan was an exercise of a private or proprietary function of the
municipality. Section 2282 of the Chatter on Municipal Law of the Revised
Administrative Code provides:

Page 314 of 323


“Section 2282. Celebration of fiesta. — Fiesta may be held in each municipality not oftener than once
a year upon a date fixed by the municipal council. A fiesta shall not be held upon any other date than
that lawfully fixed therefor, except when, for weighty reasons, such as typhoons, foundations,
earthquakes, epidemics, or other public ties, the fiesta cannot be held in the date fixed in which case it
may be held at a later date in the same year, by resolution of the council.”

This provision simply gives authority to the municipality to celebrate a yearly


fiesta but it does not impose upon it a duty to observe one. Holding a fiesta even if
the purpose is to commemorate a religious or historical event of the town is in essence
an act for the special benefit of the community and not for the general welfare of the
public performed in pursuance of a policy of the state. The mere fact that the
celebration, as claimed, was not to secure profit or gain but merely to provide
entertainment to the town inhabitants is not a conclusive test. For instance, the
maintenance of parks is not a source of income for the nonetheless it is private
undertaking as distinguished from the maintenance of public schools, jails, and the
like which are for public service. x x x x

The Municipality cannot evade liability under the claim that it was Jose
Macaraeg who constructed the stage. The municipality acting through its
municipal council appointed Macaraeg as chairman of the sub-committee on
entertainment and in charge of the construction of the "zarzuela" stage.
Macaraeg acted merely as an agent of the Municipality. Under the doctrine of
respondent superior mentioned earlier, petitioner is responsible or liable for the
negligence of its agent acting within his assigned tasks.

When it is sought to render a municipal corporation liable for the act of servants or
agents, a cardinal inquiry is, whether they are the servants or agents of the
corporation. If the corporation appoints or elects them, can control them in the
discharge of their duties, can continue or remove the can hold them responsible for
the manner in which they discharge their trust, and if those duties relate to the
exercise of corporate powers, and are for the benefit of the corporation in its local or
special interest, they may justly be regarded as its agents or servants, and the
maxim of respondent superior applies.

*Borja v. Comelec, G.R. No. 133495, September 3, 1998.


(Elective Local Officials; Disqualification; Three-Term Limit; Succession)

Facts:
-Private respondent Jose T. Capco, Jr. was elected vice-mayor of Pateros on January
18, 1988 for a term ending June 30, 1992.

-On September 2, 1989, he became mayor, by operation of law, upon the death of
the incumbent, Cesar Borja.

-On May 11, 1992, he ran and was elected mayor for a term of three years which
ended on June 30, 1995. On May 8, 1995, he was reelected mayor for another term
of three years ending June 30, 1998.

-On March 27, 1998, private respondent Capco filed a certificate of candidacy for
mayor of Pateros relative to the May 11, 1998 elections. Benjamin U. Borja Jr.
(petitioner), who was also a candidate for mayor, sought Capco's disqualification.

-Borja’s contention: Capco would have already served as mayor for three consecutive
terms by June 30, 1998 and would therefore be ineligible to serve for another term
after that.

-The Comelec Second Division ruled in favor of petitioner and declared private
respondent Capco disqualified from running for reelection as mayor of Pateros.

Page 315 of 323


-However, on motion of private respondent the COMELEC en banc, voting 5-2,
reversed the decision and declared Capco eligible to run for mayor in the May 11,
1998 elections.

-During the election proper, Capco received 16,558 votes against petitioner's 7,773
votes and was proclaimed elected by the Municipal Board of Canvassers.

-Aggrieved, Borja Jr went tot the SC via petition for certiorari.

Issue:
Whether or not a vice-mayor who succeeds to the office of mayor by operation of law
and serves the remainder of the term is considered to have served a term in that office
for the purpose of the three-term limit.

Ruling:
No.

Article X, §8 of the Constitution provides:

“Sec. 8. The term of office of elective local officials, except barangay officials, which shall be determined by
law, shall be three years and no such official shall serve for more than three consecutive terms.
Voluntary renunciation of the office for any length of time shall not be considered as an interruption in
the continuity of his service for the full term for which he was elected.”

This provision is restated in §43(b) of the Local Government Code (R.A. No. 7160):

“Sec. 43. Term of Office. —

(b) No local elective official shall serve for more than three (3) consecutive terms in the same
position. Voluntary renunciation of the office for any length of time shall not be considered as an
interruption in the continuity of service for the full term for which the elective official concerned was
elected.”

Textual analysis supports the ruling of the COMELEC that Art. X, §8 contemplates
service by local officials for three consecutive terms as a result of election. The first
sentence speaks of "the term of office of elective local officials" and bars "such
officials" from serving for more than three consecutive terms.

The second sentence, in explaining when an elective local official may be deemed to
have served his full term of office, states that "voluntary renunciation of the office for
any length of time shall not be considered as an interruption in the continuity of his
service for the full term for which he was elected." The term served must therefore
be one "for which [the official concerned] was elected." The purpose of this
provision is to prevent a circumvention of the limitation on the number of terms an
elective local official may serve. Conversely, if he is not serving a term for which he
was elected because he is simply continuing the service of the official he
succeeds, such official cannot be considered to have fully served the term
notwithstanding his voluntary renunciation of office prior to its expiration.

The term limit for elective local officials must be taken to refer to the right to be
elected as well as the right to serve in the same elective position. Consequently,
it is not enough that an individual has served three consecutive terms in an
elective local office; he must also have been elected to the same position for the
same number of times before the disqualification can apply.

This point can be made clearer by considering the following cases or situations:

Case No. 1. Suppose A is a vice-mayor who becomes mayor by reason of the death of
the incumbent. Six months before the next election, he resigns and is twice elected
thereafter. Can he run again for mayor in the next election?

Yes, because although he has already first served as mayor by succession and
subsequently resigned from office before the full term expired, he has not

Page 316 of 323


actually served three full terms in all for the purpose of applying the term limit.
Under Art. X, §8, voluntary renunciation of the office is not considered as an
interruption in the continuity of his service for the full term only if the term is one
"for which he was elected." Since A is only completing the service of the term for
which the deceased and not he was elected, A cannot be considered to have
completed one term. His resignation constitutes an interruption of the full term.

Case No. 2. Suppose B is elected mayor and, during his first term, he is twice
suspended for misconduct for a total of 1 year. If he is twice reelected after that,
can he run for one more term in the next election?

Yes, because he has served only two full terms successively.

In both cases, the mayor is entitled to run for reelection because the two conditions for
the application of the disqualification provisions have not concurred, namely, that the
local official concerned has been elected three consecutive times and that he has
fully served three consecutive terms. In the first case, even if the local official is
considered to have served three full terms notwithstanding his resignation before the
end of the first term, the fact remains that he has not been elected three times. In the
second case, the local official has been elected three consecutive times, but he has not
fully served three consecutive terms.

Case No. 3. The case of vice-mayor C who becomes mayor by succession involves a
total failure of the two conditions to concur for the purpose of applying Art. X, §8.
Suppose he is twice elected after that term, is he qualified to run again in the next
election?

Yes, because he was not elected to the office of mayor in the first term but simply
found himself thrust into it by operation of law. Neither had he served the full term
because he only continued the service, interrupted by the death, of the deceased
mayor.

To consider C in the third case to have served the first term in full and therefore
ineligible to run a third time for reelection would be not only to falsify reality but also
to unduly restrict the right of the people to choose whom they wish to govern them. If
the vice-mayor turns out to be a bad mayor, the people can remedy the situation
by simply not reelecting him for another term. But if, on the other hand, he proves
to be a good mayor, there will be no way the people can return him to office (even if it
is just the third time he is standing for reelection) if his service of the first term is
counted as one for the purpose of applying the term limit.

To consider C as eligible for reelection would be in accord with the


understanding of the Constitutional Commission that while the people should be
protected from the evils that a monopoly of political power may bring about,
care should be taken that their freedom of choice is not unduly curtailed.

[IMPORTANT NOTE:
Do not apply this jurisprudence (which involves succession) if the case involves
preventive suspension. Apply Aldovino Jr. v. Comelec, G.R. No. 184836, December
23, 2009 instead.]

*Aldovino Jr. v. Comelec, G.R. No. 184836, December 23, 2009. [IMPORTANT!!!
DOCTRINAL!!!]
(Disqualification; Three-Term Limit; Preventive Suspension; Renunciation)

Facts:
-Respondent Wilfredo F. Asilo (Asilo) was elected councilor of Lucena City for three
consecutive terms: for the 1998-2001, 2001-2004, and 2004-2007 terms,
respectively.

Page 317 of 323


-In September 2005 or during his 2004-2007 term of office, the Sandiganbayan
preventively suspended him for 90 days in relation with a criminal case he then
faced. The SC, however, subsequently lifted the Sandiganbayan’s suspension order;
hence, he resumed performing the functions of his office and finished his term.

-In the 2007 election, Asilo filed his certificate of candidacy for the same position.

-Simon B. Aldovino, Jr., Danilo B. Faller, and Ferdinand N. Talabong (petitioners)


sought to deny due course to Asilo’s certificate of candidacy or to cancel it on the
ground that he had been elected and had served for three terms; his candidacy for a
fourth term therefore violated the three-term limit rule under Section 8, Article X of
the Constitution and Section 43(b) of RA 7160.

-The COMELEC’s Second Division ruled against the petitioners and in Asilo’s favor,
ratiocinating that the three-term limit rule did not apply, as Asilo failed to render
complete service for the 2004-2007 term because of the suspension the
Sandiganbayan had ordered. Upon appeal, the Comelec En Banc affirmed the Second
Division’s ruling.

-Petitioners then went to the SC via Rule 45.

Issue:
Whether or not the preventive suspension of an elected public official constitutes an
interruption of his term of office for purposes of the three-term limit rule.

Ruling:
No.

Section 8, Article X of the Constitution states:

“Section 8. The term of office of elective local officials, except barangay officials, which shall be
determined by law, shall be three years and no such official shall serve for more than three
consecutive terms. Voluntary renunciation of the office for any length of time shall not be
considered as an interruption in the continuity of his service for the full term for which he was
elected.”

As worded, the constitutional provision fixes the term of a local elective office and
limits an elective official’s stay in office to no more than three consecutive terms. This
is the first branch of the rule embodied in Section 8, Article X. Significantly, this
provision refers to a "term" as a period of time – three years – during which an official
has title to office and can serve.

The "limitation" under this first branch of the provision is expressed in the negative –
"no such official shall serve for more than three consecutive terms." This formulation –
no more than three consecutive terms – is a clear command suggesting the existence
of an inflexible rule. While it gives no exact indication of what to "serve. . . three
consecutive terms" exactly connotes, the meaning is clear – reference is to the term,
not to the service that a public official may render. In other words, the limitation
refers to the term.

The second branch relates to the provision’s express initiative to prevent any
circumvention of the limitation through voluntary severance of ties with the public
office; it expressly states that voluntary renunciation of office "shall not be considered
as an interruption in the continuity of his service for the full term for which he was
elected." This declaration complements the term limitation mandated by the first
branch.

A notable feature of the second branch is that it does not textually state that voluntary
renunciation is the only actual interruption of service that does not affect “continuity
of service for a full term” for purposes of the three-term limit rule. It is a pure
declaratory statement of what does not serve as an interruption of service for a full

Page 318 of 323


term, but the phrase “voluntary renunciation,” by itself, is not without significance in
determining constitutional intent.

The word “renunciation” carries the dictionary meaning of abandonment. To


renounce is to give up, abandon, decline, or resign. It is an act that emanates from its
author, as contrasted to an act that operates from the outside. Read with the
definition of a "term" in mind, renunciation, as mentioned under the second branch of
the constitutional provision, cannot but mean an act that results in cutting short
the term, i.e., the loss of title to office. The descriptive word "voluntary" linked
together with "renunciation" signifies an act of surrender based on the surenderee’s
own freely exercised will; in other words, a loss of title to office by conscious
choice. In the context of the three-term limit rule, such loss of title is not considered
an interruption because it is presumed to be purposely sought to avoid the application
of the term limitation. x x x x

From all the above, we conclude that the “interruption” of a term exempting an
elective official from the three-term limit rule is one that involves no less than
the involuntary loss of title to office. The elective official must have
involuntarily left his office for a length of time, however short, for an effective
interruption to occur. This has to be the case if the thrust of Section 8, Article X and
its strict intent are to be faithfully served, i.e., to limit an elective official’s continuous
stay in office to no more than three consecutive terms, using "voluntary renunciation"
as an example and standard of what does not constitute an interruption.

Thus, based on this standard, loss of office by operation of law, being involuntary,
is an effective interruption of service within a term, as we held in Montebon. On
the other hand, temporary inability or disqualification to exercise the functions
of an elective post, even if involuntary, should not be considered an effective
interruption of a term because it does not involve the loss of title to office or at
least an effective break from holding office; the office holder, while retaining
title, is simply barred from exercising the functions of his office for a reason
provided by law.

An interruption occurs when the term is broken because the office holder lost
the right to hold on to his office, and cannot be equated with the failure to
render service. The latter occurs during an office holder’s term when he retains title
to the office but cannot exercise his functions for reasons established by law. Of
course, the term "failure to serve" cannot be used once the right to office is lost;
without the right to hold office or to serve, then no service can be rendered so that
none is really lost.

In all cases of preventive suspension, the suspended official is barred from


performing the functions of his office and does not receive salary in the
meanwhile, but does not vacate and lose title to his office; loss of office is a
consequence that only results upon an eventual finding of guilt or liability. x x x x
Thus, while a temporary incapacity in the exercise of power results, no position
is vacated when a public official is preventively suspended. This was what
exactly happened to Asilo. x x x x

Strict adherence to the intent of the three-term limit rule demands that preventive
suspension should not be considered an interruption that allows an elective official’s
stay in office beyond three terms. A preventive suspension cannot simply be a term
interruption because the suspended official continues to stay in office although
he is barred from exercising the functions and prerogatives of the office within
the suspension period. The best indicator of the suspended official’s continuity in
office is the absence of a permanent replacement and the lack of the authority to
appoint one since no vacancy exists.

*Sangguniang Barangay of Don Mariano Marcos v. Martinez, G.R. No. 170626, March 3,
2008.
(Local Government Code; Discipline; Removal)

Page 319 of 323


Facts:
-Petitioner Sangguniang Barangay is the legislative body of Barangay Don Mariano
Marcos, Bayombong, Nueva Vizcaya. Respondent Martinez is the incumbent Punong
Barangay of the said local government unit.

-On 5 November 2004, Martinez was administratively charged with for Dishonesty,
Misconduct in Office and Violation of the Anti-Graft and Corrupt Practices Act,
by petitioner through the filing of a verified complaint before the Sangguniang
Bayan as the disciplining authority over elective barangay officials pursuant to Section
614 of Rep. Act No. 7160, otherwise known as the Local Government Code.

-Upon his failure to file an Answer to the Amended Administrative Complaint dated 6
December 2004, Martinez was declared by the Sangguniang Bayan as in default.
Pending the administrative proceedings, Martinez was placed under preventive
suspension for 60 days or until 8 August 2005.

-On 28 July 2005, the Sangguniang Bayan rendered its Decision which imposed upon
Martinez the penalty of removal from office.

-Martinez filed a Special Civil Action for Certiorari with a prayer for Temporary
Restraining Order and Preliminary Injunction before the trial court against petitioner.

-After due proceedings, the RTC declared the Decision of the Sangguniang Bayan void,
maintaining that the proper courts, and not the petitioner, are empowered to remove
an elective local official from office.

-Contention of Sangguniang Barangay: Administrative cases involving elective


barangay officials may be filed with, heard and decided by the Sangguniang
Panlungsod or Sangguniang Bayan concerned, which can, thereafter, impose a penalty
of removal from office. The courts are merely tasked with issuing the order of removal,
after the Sangguniang Panlungsod or Sangguniang Bayan finds that a penalty of
removal is warranted.

Issue:
Whether or not the Sangguniang Bayan may remove Martinez, an elective local official,
from office.

Ruling:
No.

Section 60 of the Local Government Code conferred upon the courts the power
to remove elective local officials from office:

“Section 60. Grounds for Disciplinary Actions.—An elective local official may be disciplined,
suspended, or removed from office on any of the following grounds:

x x x x.

An elective local official may be removed from office on the grounds enumerated above by order of
the proper court.”

In Salalima v. Guingona, Jr., the Court en banc categorically ruled that the Office of the
President is without any power to remove elected officials, since the power is
exclusively vested in the proper courts as expressly provided for in the last paragraph
of Section 60 of the Local Government Code. It further invalidated Article 125, Rule
XIX of the Rules and Regulations Implementing the Local Government Code of 1991,
which provided that:

Article 125. Grounds for Disciplinary Actions. x x x.


x x x x.

Page 320 of 323


(b) An elective local official may be removed from office on the grounds enumerated in paragraph (a) of
this Article by order of the proper court or the disciplining authority whichever first acquires
jurisdiction to the exclusion of the other.

The Court nullified the aforequoted rule since the Oversight Committee that prepared
the Rules and Regulations of the Local Government Code exceeded its authority when
it granted to the "disciplining authority" the power to remove elective officials, a power
which the law itself granted only to the proper courts. Thus, it is clear that under
the law, the Sangguniang Bayan is not vested with the power to remove
Martinez. x x x x

The rule which confers to the proper courts the power to remove an elective
local official from office is intended as a check against any capriciousness or
partisan activity by the disciplining authority. Vesting the local legislative body
with the power to decide whether or not a local chief executive may be removed from
office, and only relegating to the courts a mandatory duty to implement the decision,
would still not free the resolution of the case from the capriciousness or partisanship
of the disciplining authority. Thus, the petitioner’s interpretation would defeat the
clear intent of the law.

Congress clearly meant that the removal of an elective local official be done only
after a trial before the appropriate court, where court rules of procedure and
evidence can ensure impartiality and fairness and protect against political
maneuverings. Elevating the removal of an elective local official from office from an
administrative case to a court case may be justified by the fact that such removal not
only punishes the official concerned but also, in effect, deprives the electorate of
the services of the official for whom they voted.

As the law stands, Section 61 of the Local Government Code provides for the
procedure for the filing of an administrative case against an erring elective barangay
official before the Sangguniang Panlungsod or Sangguniang Bayan. However, the
Sangguniang Panlungsod or Sangguniang Bayan cannot order the removal of an
erring elective barangay official from office, as the courts are exclusively vested
with this power under Section 60 of the Local Government Code. Thus, if the
acts allegedly committed by the barangay official are of a grave nature and, if
found guilty, would merit the penalty of removal from office, the case should be
filed with the regional trial court. Once the court assumes jurisdiction, it retains
jurisdiction over the case even if it would be subsequently apparent during the trial
that a penalty less than removal from office is appropriate. On the other hand, the
most extreme penalty that the Sangguniang Panlungsod or Sangguniang Bayan
may impose on the erring elective barangay official is suspension; if it deems
that the removal of the official from service is warranted, then it can resolve
that the proper charges be filed in court.

*Nazareno v. City of Dumaguete, G.R. No. 181559, October 2, 2009.


(“Midnight Appointments”; Appointive Local Officials; CSC Rules; Exemptions)

Facts:
-Then Dumaguete City Mayor Felipe Antonio B. Remollo sought re-election in the May
14, 2001 elections, but lost to respondent Mayor Agustin R. Perdices.

-Thereafter, on June 5, 7, and 11, 2001, outgoing Mayor Remollo promoted 15 city
hall employees, and regularized another 74 city hall employees, including the herein
52 petitioners.

-On July 2, 2001, Mayor Perdices publicly announced at the flag raising ceremony at
the Dumaguete City Hall grounds that he would not honor the appointments made by
former Mayor Remollo. On the same day, he instructed the City Administrator to direct
respondent the City Assistant Treasurer to refrain from making any cash
disbursements for payments of petitioners’ salary differentials based on their new
positions.

Page 321 of 323


-On August 1, 2001, the CSC Field Office in Dumaguete City, through Director II
Fabio R. Abucejo, revoked and invalidated the appointments of the petitioners based,
among others, on: (1) The appointing officer of the 89 appointments was an outgoing
local official who lost during the 14 May 2001 elections for City Mayor of Dumaguete
City; (2) The 89 appointments were all issued after the elections and when the
new city mayor was about to assume office.

-Director Abucejo invalidated the appointments as the same were done in violation of
CSC Resolution No. 010988 dated June 4, 2001, the pertinent portions of which
provide:

“3. All appointments, whether original, transfer, reemployment, reappointment, promotion or


demotion, except in cases of renewal and reinstatement, regardless of status, which are issued AFTER
the elections, regardless of their dates of effectivity and/or date of receipt by the Commission, including
its Regional or Field Offices, of said appointments or the Report of Personnel Actions (ROPA) as the case
may be, shall be disapproved unless the following requisites concur relative to their issuance:

a) The appointment has gone through the regular screening by the Personnel Selection Board (PSB) before
the prohibited period on the issuance of appointments as shown by the PSB report or minutes of its
meeting;
b) That the appointee is qualified;
c) There is a need to fill up the vacancy immediately in order not to prejudice public service and/or
endanger public safety;
d) That the appointment is not one of those mass appointments issued after the elections.”

-The CSC en banc affirmed Director Abucejo’s order. Upon petition for review via Rule
43, the CA affirmed the CSC en banc.

-Petitioners’ contention: CSC Resolution No. 010988 is invalid because the


Commission is without authority to issue regulations prohibiting mass appointments
at the local government level. Petitioners cite De Rama v. Court of Appeals which held
that Section 15, Article VII of the Constitution involving “midnight appointments” is
only applicable to the President or Acting President, but not to local officials.

Issue:
Whether or not the invalidation of petitioners’ appointments was proper.

Ruling:
Yes.

The CSC, as the central personnel agency of the government, has statutory authority
to establish rules and regulations to promote efficiency and professionalism in the civil
service. x x x x

We find that there was substantial reason behind the issuance of CSC Resolution
No. 010988. It is true that there is no constitutional prohibition against the issuance
of "mass appointments" by defeated local government officials prior to the expiration of
their terms. Clearly, this is not the same as a "midnight appointment," proscribed by
the Constitution, which refers to those appointments made within two months
immediately prior to the next presidential election.

As we ruled in De Rama v. Court of Appeals:

The records reveal that when the petitioner brought the matter of recalling the
appointments of the fourteen (14) private respondents before the CSC, the only reason
he cited to justify his action was that these were midnight appointments that are
forbidden under Article VII, Section 15 of the Constitution. However, the CSC ruled,
and correctly so, that the said prohibition applies only to presidential
appointments. In truth and in fact, there is no law that prohibits local elective
officials from making appointments during the last days of his or her tenure.

However, even while affirming De Rama, we explained in Quirog v. Aumentado, that:

Page 322 of 323


We, however, hasten to add that the aforementioned ruling does not mean that
the raison d' etre behind the prohibition against midnight appointments may not
be applied to those made by chief executives of local government units, as here.
Indeed, the prohibition is precisely designed to discourage, nay, even preclude,
losing candidates from issuing appointments merely for partisan purposes
thereby depriving the incoming administration of the opportunity to make the
corresponding appointments in line with its new policies.

Quirog also involved the disapproval of an appointment for non-compliance with CSC
Resolution No. 010988. However, we found that Quirog’s appointment was made on
June 1, 2001, or three days prior to the issuance of CSC Resolution No. 010988. As
such, we ruled that the retroactive application of the law was not warranted. x x x x

This case is a typical example of the practice of outgoing local chief executives to issue
“midnight appointments,” especially after their successors have been proclaimed. It
does not only cause animosities between the outgoing and the incoming officials, but
also affects efficiency in local governance. Those appointed tend to devote their time
and energy in defending their appointments instead of attending to their functions.

It is not difficult to see the reasons behind the prohibition on appointments


before and after the elections. Appointments are banned prior to the elections to
ensure that partisan loyalties will not be a factor in the appointment process, and to
prevent incumbents from gaining any undue advantage during the elections. To this
end, appointments within a certain period of time are proscribed by the Omnibus
Election Code and related issuances. After the elections, appointments by defeated
candidates are prohibited, except under the circumstances mentioned in CSC
Resolution No. 010988, to avoid animosities between outgoing and incoming officials,
to allow the incoming administration a free hand in implementing its policies, and to
ensure that appointments and promotions are not used as a tool for political
patronage or as a reward for services rendered to the outgoing local officials.

-o-0-o-

Atty. Ced A. Jimenez

Page 323 of 323

You might also like